You are on page 1of 239

A Must for Civil Services (Pre) Examination,

State PCS & Other Competitive Exams

INDIAN
ECONOMY
Topical Coverage of Syllabus and Previous Years’ Questions
with more than 3000 MCQs and 5 Practice Sets
A Must for Civil Services (Pre) Examination,
State PCS & Other Competitive Exams

INDIAN
ECONOMY
Topical Coverage of Syllabus and Previous Years’ Questions
with more than 3000 MCQs and 5 Practice Sets

Authored By
Rakesh Kumar Roshan

Arihant Publications (India) Ltd.


Arihant Publications (India) Ltd.
All Rights Reserved

© Publishers
No part of this publication may be re-produced, stored in a retrieval system or distributed in
any form or by any means, electronic, mechanical, photocopying, recording, scanning, web
or otherwise without the written permission of the publisher. Arihant has obtained all the
information in this book from the sources believed to be reliable and true. However, Arihant
or its editors or authors or illustrators don’t take any responsibility for the absolute
accuracy of any information published and the damages or loss suffered there upon.
All disputes subject to Meerut (UP) jurisdiction only.

ADMINISTRATIVE & PRODUCTION OFFICES


Regd. Office
‘Ramchhaya’ 4577/15, Agarwal Road, Darya Ganj, New Delhi -110002
Tele: 011- 47630600, 43518550; Fax: 011- 23280316

Sales & Support Offices


Agra, Ahmedabad, Bengaluru, Bareilly, Chennai, Delhi, Guwahati, Hyderabad,
Jaipur, Jhansi, Kolkata, Lucknow, Meerut, Nagpur & Pune

Head Office
Kalindi, TP Nagar, Meerut (UP) - 250002
Tele: 0121-2401479, 2512970, 4004199; Fax: 0121-2401648

ISBN : 978-93-13197-42-3
Price : `230.00

PUBLISHED BY : ARIHANT PUBLICATIONS (I) LTD.

PRODUCTION TEAM
Publishing Manager : Amit Verma Inner Designer : Pradeep Kumar
Project Head : Karishma Yadav Page Layout Designer : Vinay Sharma
Project Coordinator : Aleena Zaidi Proof Reader : Soniya Rawat
Cover Designer : Shanu Mansoori

For further information about books Publish by Arihant


log on to www.arihantbooks.com or email to info@arihantbooks.com
CONTENTS
1. Introduction to Economics 1-4 Over The Counter Exchange of India (OTCEI)
Meaning of Economics Other Investment Agents
Types of Economies Banking in India
Sectors of an Economy Reserve Bank of India (RBI)
Other Sectors of Economy Methods of Credit Control
Scheduled Commercial Banks
2. National Income 5-9 State Bank of India
National Income Private Banks
Real National Income (RNI) Foreign Banks
Calculating National Income Regional Rural Banks
Gross Fixed Capital Formation (GFCF) Scheduled Co-operative Banks
Indian Organisations Related to National Types of Banking System
Income Accounts Inter-Bank Transfer
Banking Sector Reforms
3. Economic Growth and Development 10-16 Basel Norms
Economic Growth Banking Ombudsman
Economic Development Development Financial Institutions
Measurement of Economic Development Insurance Companies
Physical Quality of Life Index (PQLI)
Millennium Development Goals (MDGs) 6. Inflation 49-53
Different Levels of Economic Development Causes of Inflation
Economy and Environment Effects of Inflation
Environmental Taxes Measures of Inflation
12th Five Year Plan and Sustainability Change in Reporting of Inflation
Measures to Check Inflation
4. Economic Planning in India 17-26
Meaning of Economic Planning 7. Public Finance 54-65
Strategies of Planning Public Revenue
History of Planning in India Types of Tax
National Institution for Transforming India (NITI) Aayog Direct Tax Code (DTC)
Planning Commission Sources of Revenue
National Development Council (NDC) Public Expenditure
Five Year Plans in India Debt Management Strategy
Twelfth Five Year Plan (2012-2017) Public Debt
Economic Reform Finance Commission
India Vision 2020 Fiscal Policy
Fiscal Responsibility and Budget Management Act,
5. Money and Banking 27-48 (FRBM) 2003
Money Union Budget
Measures of Money Supply in India Deficit
Financial Sector
Financial Markets 8. India’s Balance of Payments 66-72
Regulatory Framework Balance of Payments (BoP)
Reforms in Capital Market of India Foreign Capital
Qualified Foreign Investor (QFI) Foreign Exchange
Stock Exchanges in India Capital Account Convertibility in India
National Stock Exchange (NSE) India’s External Debt
Bombay Stock Exchange (BSE) NRI Deposits
MCX-SX (MCX Stock Exchange Limited)
9. India’s Foreign Trade 73-77 14. Infrastructure 124-134
Foreign Trade Policy (FTP), 2015-20 Definition of Infrastructure
India's Trading Infrastructure in 12th Plan
Import/Export Controls Transport System in India
Trade Composition Major Schemes
Import Cover
Export Promotion 15. Poverty and Unemployment 135-144
General Agreement on Tariffs and Trade Poverty
Poverty in India
10. Demographic Profile of India 78-84 Human Poverty Index (HPI)
Demography Expert Groups for Estimating Poverty
Birth and Death Rates Inequality
Density of Population Employment and Unemployment
Literacy Poverty Eradication and
Employment Related Programme
11. Agriculture 85-99 Mahatma Gandi National Rural Employment
Agricultural Sector Guarantee Scheme (MGNREGS)
Five Year Plan and Agriculture
Second Green Revolution in India 16. Government Schemes and Programmes 145-161
Evergreen Revolution Social Welfare Schemes
National Commission on Farmers Skill Development and Employment Scheme
Agricultural Inputs Rural Infrastructure and Development
Food Security Housing and Sanitation Scheme
Food Processing Industry Education
Land Reforms Financial Inclusion Mission
Agricultural Finance and Credit Facilities Health
Agricultural Marketing and Extension Services Women Empowerment Programmes
Agriculture Insurance Social Defence
Major Schemes of Agriculture Other Social Protection Programmes
Animal Husbandry, Dairying and Fisheries
17. International Financial and Economic
12. Indian Industry 100-117 Organisation 162-171
Industries in Indian Economy International Organisations
Industrial Policy International Monetary Fund (IMF)
An Evaluation of Industrial Policy Before 1991 IMF Quota
Companies Act, 2013 Special Drawing Rights (SDRs)
Public Sector IMF Reforms
Policy of Navratnas World Bank Group
Disinvestment The United Nations Conference on Trade and
The Micro, Small and Medium Enterprises Development (UNCTAD)
(MSMEs) World Trade Organisation (WTO)
Schemes/Programmes for Small Industries Other Important International Organisations
Industrial Sickness Organisation of Petroleum Exporting Countries (OPEC)
Revival of Sick Units Organisation for Economic Co-operation and
Economic Census Development (OECD)
India’s Manufacturing Sector
Eurasian Economic Union (EEU)
Some Large-Scale Industries
Regional Financial Institutions
Economic Reforms
Make in India Economic Survey 2018-19 172-173
Digital India
Union Budget 2019-20 174-180
13. Services Sector 118-123
Practice Sets (1-5) 181-201
Current Scenario
Composition of Service Sector Previous Year’s Solved Papers 1 205-222
Services Sectors of India
Some Major Services of India Previous Year’s Solved Papers 2 223-230
Challenges of Service Sector
TOPICS FOCUS &
TREND OF QUESTIONS

INTRODUCTION AND NATIONAL INCOME DEMOGRAPHIC PROFILE


This chapter provides an overview of various important This chapter assumes greater significance in the background
sectors of Indian economy and its specific features. Important of demographic dividend in India. Key concepts such as
ceoncepts of this chapter are GDP, GNP, GDP deflator and demographic transition theory, birth and death rates, age
other such statistical tools as a measure of economic growth. structure, population density, sex ratio and literacy are
The changed pattern of UPSC demands a deeper pertinent from examination point of view. Further questions
understanding of their meaning and importance as well as can also be asked from census data, Population Policy and
how they are calculated and their trends in recent years, family welfare programmes. Earlier questions asked in
which has also been asked in previous exams. competitive exams include how demographic dividend can
be reaped, age structure-composition of India etc.
INFLATION
Important topics from examination point of view are causes of MONEY AND BANKING
supply side and demand side inflation, role of Monetary Policy This is the most important chapter in economy section. The
and Fiscal Policy in inflation management, its ill effects on revised pattern of examination has asked analytical type of
economy and other related concepts such as deflation, hyper- questions from topics such as money and capital market and
inflation and stagflation. Questions asked in previous year reforms measure therein, banking in India, Monetary Policy
exams include causes of inflation, effect of liquidity on and RBI, interest rates and money supply, insurance and
inflation, impact of inflation on bond yield and debtors etc. pension sector. Most questions asked in the exams are about
RBI’s regulatory role, repo and reverse repo rate, money
ECONOMIC GROWTH AND DEVELOPMENT supply and corrosponding order of liquidity, priority sector
This chapter covers the important and relevant concepts and lending, lead bank schemes and measures of financial
issues of growth and development. Other important topics inclusion.
dealt here are difference between growth and development,
various measures of development, human development and INDIAN PUBLIC FINANCE
HDI, sustainable development, MDGs etc. Most of the Most essential topics of this chapter are Fiscal Policy, public
questions asked in previous exams from this section are revenue and expenditure and trends therein, taxation system,
regarding factors contributing to economic growth,
GST, fiscal consolidation, FRBM Act, Finance Commission and
constituents of HDI etc. Trends in savings and investment,
various types of budgets like gender budgeting outcome and
capital-output ratio, etc.
performance budgeting etc. Questions asked in examination
test the conceptual understanding of the above topics such as
ECONOMIC PLANNING IN INDIA what is the impact of deficit financing, what are various ways
This chapter discusses meaning, evolution and strategies of to achieve fiscal consolidation constituents of GST, VAT,
planning. From examination perspective, the important focus CENVAT and related concepts, fiscal federalism etc.
areas are role of Planning Commission and NDC, objectives
and achievements of various Five Year Plans and planning in INDIA’S BOP
liberalised economy. Previous years’ questions asked in
This chapter deals with India’s external sector and important
examination includes topics such as mobilisation of resources,
topics here are meaning and constituents of BoP, current and
approach of planning in India, structure and functions of NDC
and Planning Commission. capital account convertibility, FDI and FII, value of rupee in
forex market,
INTERNATIONAL FINANCIAL ORGANISATION INFRASTRUCTURE
This chapter offers an in-depth understanding of structures, Important topics in this chapter are classification and
functions, and way of lending of important international financial significance of infrastructure, problems in infrastructure
organisations such as IMF, World Bank group, development, issue of investment and financing, PPPs and
WTO, ADB etc. Questions have been asked about lending their advantages, performance of energy sector and
programme of IMF, Agreement on Agriculture and NAMA reforms therein, urban infrastructure and transport, critical
of WTO , TRIPS, TRIMS and DOHA sound of WTO, agencies of infrastructure.
World Bank and terms of lending, SDRs and Quota reforms etc.
NATURAL RESOURCES
AGRICULTURE This chapter deals with minerals, water, land and
It is the most important sector of Indian economy. Essential forest resources. Essential topics in this chapter are
topics to be read in this chapter are crops and cropping concerned with mineral resource’s distribution, use,
pattern, Second Green Revolution, Agricultural Price Policy , conservation and management. Pertinent to this is issue of
finance and marketing, extension services, farm credit,
environment management and climate change. Trends of
problems in productivity and land reforms, food processing and
previous Years exams suggests questions about watershed
related schemes. Questions asked in previous years’ exams
management, land use policy, national mining policy,
include production trends and patterns, various agricultural
institutions and their functions, government schemes social and agro forestry, application of S and T in
related to agriculture, issues and trends in agricultral credit, conservation and management climate change
objectives of Mega food Parks and related food processing negotiations etc.
schemes etc.
POVERTY AND UNEMPLOYMENT
INDUSTRIAL SECTOR Important topics in this chapter are concepts of absolute
From an examination perspective, important topics are New and relative poverty, methodology of poverty calculation,
Industrial Policy, MSMEs, PSEs, industrial sickness, Disinvestment various committees and their recommendations, poverty
Policy, role of CCI, National Manufacturing Policy, NIMZs, and alleviation schemes, types of unemployment and
recent trends in industrial growth. Questions have been asked in methodology of calculating unemployment, MNREGA and
previous exams regarding skill development, labour laws and other employment generation schemes.
reforms, measures to promote MSMEs, Index of Industrial
Production, core sectors of manufacturing etc. INCLUSIVE DEVELOPMENT
This chapter is a recent addition in economy section and
SERVICES SECTOR significant weightage has been given in the examination.
Significant topics in this chapter are performance and trends of The important topics are related to concept, need and
various services sectors such as IT and ITeS, real estate, strategies of inclusive growth. Important government
communications, construction, tourism sectors etc, issue of FDI policies and schemes relating to health and sanitation,
and liberalisation of services sector and services employment in education, skill development, rural development, women
India. East years examination have focused on Contitution and and child development etc are important from
performance of services sector in economic growth. examination point of view.
Chapter one
Introduction to Economics

Meaning of Economics
“Economics is a social — The term ‘economics’ comes from the Greek term Oikonomos, which is composed of
oikos (house) and nomos (custom or law), meaning Rules of the Household.
science, which tries to
Economics is the social science that studies economic activities to gain an
study how to achieve the understanding of the processes that govern the production, distribution and
maximum benefits using consumption of goods and services in an economy.
limited resources. — Initially, the study of economics concentrated mainly on wealth by concentrating on
factors of production and consumption. This emphasis on wealth excluded from its
Understanding
study, those who were not directly connected with the formal economic system. Thus,
economics is important the needs of poor, senior citizens, children etc. were neglected. It was corrected with
to provide for the the emergence of welfare economics, which focused on welfare needs of the whole
maximum welfare of society instead of just the production of wealth.

society using the Branches of Economics


resources available.” Traditionally, economics has been divided into the two main branches :

Micro Economics
— It is concerned with how supply and demand interact in individual market and how
these interactions determine the price level of goods and services.
— It examines the economic behaviour of individual actor at the level of the individual
economic entity — the individual firm, the individual consumer and the individual
worker.

Macro Economics
— It is concerned with how the overall economy works. It studies such things as
employment, Gross Domestic Product (GDP) and inflation; the stuff of news stories
and government policy debates.
— It studies the economy as a whole and its features like national income, employment,
poverty, balance of payments and inflation.

Some Other Branches of Economics


Development Economics
— It is a branch of economics, which deals with the economic aspects of the
development process in low income countries.
— The goal of development economics is to determine how poor countries can be
transformed into prosperous ones.

Behavioral Economics
— This branch studies the effects of social, cognitive and emotional factors on the
economic decisions of individuals and their consequences for market prices, returns
and resource allocation.
2 Magbook ~ Indian Economy

— It is mainly exploring why people sometimes make irrational Command Economy


decisions and why and how their behaviour does not follow — Here, the resources of production are completely under
the predictions of economic models. government control. The functioning of these economies
Environmental Economics is based on control planning. Due to lack of competition,
resource allocation is inefficient and consumers have very
— This branch studies the economic impact of environmental
little choices. Examples of this type are the former Soviet
policies. Its goal is to balance the economic activity and the
Union, Cuba, North Korea etc.
environmental impacts, by taking into account all the costs
and benefits. Mixed Economy
— This type of economy consists of a combination of public
International Economics sector and private sector units. Here, the government is
— It is a branch of economics, which studies economic inter- the decision-maker for the public sector and individuals,
actions among different countries, including foreign trade, and businessmen make decisions for the private sector.
foreign exchange, balance of payments and balance of trade. — It basically incorporates governmental involvement in a
— The guiding principle in the study of international economics is market based economy. Examples of this type are India,
comparative advantage, which indicates that every country, no Russia and UK etc.
matter what their level of development, can find something that
it can produce cheaper than others. Open Economy
— An economy is said to be open, if it has trade with other
Information Economics economies. In this economy, market is mostly free from
— It is a branch, which studies how information and information trade barriers and where exports and imports form a large
technology influence the economy. percentage of the GDP.
Information economics has certain characteristics, like: — The degrees of the openness of an economy determines
(i) It is easy to create, but hard to trust. government’s freedom to pursue economic policies of its
(ii) It is easy to spread, but hard to control. choice and the susceptibility of the country to
international economic cycles.
Demographic Economics
Closed Economy
— Demographic economics or Population economics is the
application of economics to demography; the study of human — An economy is said to be closed, if it has no trade or
population, including size, growth, density, distribution and vital trade area with other economies. In this economy, the
statistics. consumer get everything within the economic borders
and government act as the arbitrator, articulator and
facilitator.
Types of Economies
— Typically, economies are divided into different types based on Capitalist Economy
the extent of government involvement in economic — Capitalism is the economic system based on private or
decision-making. corporate ownership, production and distribution of
Based on the above criteria, the following are the major types of goods. Capitalists favour a system of free enterprise which
economies: means the government does not interfere in the economy
that the laws of supply and demand will make sure that
Traditional Economy the economy runs most efficiently in meeting people’s
— In this type of economy, there is very little government needs. Capitalism is characterised by competition in
involvement. Allocation of resources here is based on rituals, which there is rivalry in supplying or getting an economic
habits or customs. service or goods.
— Economic roles are defined by the family and people work Socialist Economy
together for the common good. There is also very little individual — Socialism is an economic system in which the means of
choice in this system. Examples of this type exist in tribes in production are socially owned and used to meet human
Amazon, Aborigines in Australia etc. needs instead of to create profits.
Free Market Economy — Socialism tends to favour cooperation whereas capitalism
— This type of economy also has very little governmental is characterised by competitions. A form of socialism
interference or control. Economic decisions here are made called communism sprang up based on the writings of
based on market principles. There is a lot of competition Karl Marx and Friedrich Engels.
between firms, which provides many choices to consumers. — Communism advocates class struggle and revolution to
— Resources for production are under private ownership and they establish a society of cooperation with strong government
make their decisions with the desire to maximise profits. control. Communism predominated in the former Soviet
Although, there are no pure free market economies. United Union and much of Eastern Europe at one time. Today, it
States and Australia come close to this type. predominates in China and Cuba, but its influence has
lessened.
Magbook ~ Introduction to Economics 3

Sectors of an Economy Other Sectors of Economy


— A nation’s economy can be broadly divided into various Quaternary Sector
sectors to define the proportion of people engaged in a — This sector consists of the intellectual and knowledge based
particular sector. This categorisation is generally seen as a
activities. Examples of activities associated with this sector
continuum of the distance from the natural environment.
are research and development, culture, information
— Traditionally, economies are divided into the following three
technology, consulting, financial planning, education etc.
sectors :
Quinary Sector
(i) Primary Sector
— This sector consists of the highest levels of decision-making
— This sector is involved in the extraction or harvesting of
in a country. This includes the top officials of government,
products from the Earth. It includes the production of raw
media, universities etc.
materials and basic foods. Some of the activities included in
this sector are agriculture, mining, forestry, fishing, quarrying
etc. Classification of Countries
— The packaging and processing of raw materials is also by the World Bank
considered as a part of this sector. As an economy develops The World Bank prepares the World Development Report (WDR).
the share of primary sector in total production and
The WDR as of July, 2018, classified the different countries on the
employment goes down.
basis of their per capita income.
(ii) Secondary Sector Categories Based on Per Capita Income
— The secondary sector of the economy is involved in the ◆
High income countries — $ 12,055 and above
production of finished goods. All manufacturing, processing
and construction activities lie in this sector (in India,

Upper middle income countries — Above $ 3895 to $ 12,055
construction sometimes considered as part of the services or ◆
Lower middle income countries — Above $ 996 to $ 3895
tertiary sector). ◆
Low income countries — $ 995 and less.
— Some of the activities in this sector are metal working, Source World Bank Report, 2018
automobile manufacturing, textile, production, shipbuilding
etc. Most economies in their process of development go
through the middle phase, where the secondary sector Classification of Countries Based
becomes the largest sector of the economy in terms of
production and employment with the reduction in importance on Economical Activities
of the primary sector. Developed Country
— India is an exception, where we have directly moved to — It refers to a country with a relatively high level of economic
services sector development, without first improving the
growth and security. A country’s degree of development is
manufacturing capabilities.
evaluated on the basis of per capita income or GDP, level of
(iii) Tertiary Sector industrialisation, general standard of living and the amount
— The tertiary sector of the economy is also called as the of widespread infrastructure.
services sector. This sector provides services to the general Developing Country
population and to business. Some of the activities which are — It is also called a less developed country with lower living
part of this sector are retail, transportation, entertainment, standard, underdeveloped industrial base and low Human
tourism and banking etc. In the advanced developed Development Index (HDI).
economies, the tertiary sector is the largest in terms of
production and employment. Least Developed Country
— Sometimes, two more sectors, i.e. quaternary and quinary are
— According to the United Nations, countries having lowest
defined separately, even though these can also be considered indicator of socio-economic development with the lowest
as part of the services sector itself. HDI ratings are called as least developed countries.
Self Check
Build Your Confidence
1. A ‘closed economy’ is an economy, in which [UPSC 2011] 6. Which one of the following statements regarding
(a) the money supply is fully controlled socialist economy is not correct?
(b) deficit financing takes place (a) In this system, means of production is socially
(c) only export takes place owned
(d) neither export nor import take place (b) Socialist economy favours cooperative society
(c) It predominates in USA and Cuba
2. Mixed economy means an economy where [UPSC 2009]
(d) It emphasised on meeting human needs instead of
(a) both agriculture and industry are equally promoted by the state to create profits
(b) there is co-existence of private sector alongwith the public
sector 7. Consider the following statements regarding
(c) there is importance of small scale industries alongwith heavy macro economics
industries 1. It is concerned with the overall economy works.
(d) economy is controlled by military as well as civilian rules 2. It studies employments, gross domestic product
and inflation.
3. Consider the following statements regarding secondary
sector Which of the statement(s) given above is/are
correct?
1. Most economies in their process of development go through
(a) Only 1 (b) Only 2
the middle phase, where the secondary sector becomes the
(c) Both 1 and 2 (d) Neither 1 nor 2
largest sector of the economy.
2. India is an exception, where we have directly moved to service 8. In which of the following types of economy, the
sector development without first improving the manufacturing resources of production are completely under
capabilities. control?
Which of the statement(s) given above is/are correct? (a) Mixed economy
(a) Only 1 (b) Open economy
(b) Only 2 (c) Closed economy
(c) Both 1 and 2 (d) Command economy
(d) Neither 1 nor 2
9. In which type of economy, resources for
4. Consider the following statements regarding tertiary sector production are under private ownership and they
1. In the advanced developed economies, the tertiary sector is make their decisions with the desire to maximise
the largest in terms of production and employment. profits?
2. Quaternary and quinary are not a parts of service sector. (a) Command economy (b) Capitalist economy
Which of the statement(s) given above is/are correct? (c) Free Market economy (d) Mixed economy
(a) Only 1 10. In which type of economy, the consumers get
(b) Only 2 everything within the economic borders and
(c) Both 1 and 2 government acts as the arbitrator?
(d) Neither 1 nor 2 (a) Closed economy (b) Open economy
5. Which one of the following statements regarding capitalist (c) Capitalist economy (d) Mixed economy
economy is not true? 11. An economy is said to be open economy if
(a) Capitalism is the economic system based on private or (a) it has trade with other economies.
corporate ownership (b) it has no trade area with other economies.
(b) Capitalists favour the government interference in the economy (c) it has basically incorporates governmental
(c) Capitalism is characterised by competition in the market involvement.
(d) In capitalist economy, market is free (d) None of these

1. (d) 2. (b) 3. (c) 4. (a) 5. (b) 6. (c) 7. (c) 8. (d) 9. (c) 10. (a)
11. (a)
Magbook ~ National Income 5

Chapter two
National Income

National Income Market Price


National Income is defined as the total
— Market price is the price that customers
National Income is an —
actually pay. It includes the component
net earnings from the production of
uncertain term which is goods and services in a country over a of indirect taxes and of subsidies.
used interchangeably with period of time, usually one year and Accordingly, when indirect taxes are
consisting essentially of wages, salaries, deducted and subside added to the
national dividend, national market price, we get values of National
rent, profits and interest.
output and national Income at factor cost
National Income = C + I + G + (X − M)
expenditure. On this basis, MP = FC + Indirect taxes – Subsidies
national income National income is considered as NNP at Or MP = FC + Net indirect taxes
factor cost.
accounting records the
Where,
GDP (Gross Domestic Product)
leel of activity in accounts
C = Total Consumption Expenditure — It is the monetary value of all final goods
such as total revenues
I = Total Investment Expenditure and services produced with a country’s
earned by domestic G = Total Government Expenditure border in a specific time.
corporation, wages paid to X = Export —GDP = C + I + G + NX
foreign works and amount M = Import Where C = Consumption
spend on sales and — It can be measured by Gross National I = Investment
Product (GNP), Gross Domestic G = Government expenditure
income taxes by Product (GDP), Gross National Income NX = Net Export
corporations and (GNI), Net National Product (NNP), Net
GDP at FC and MP
individuals outsiding in the National Income (NNI) and Per-Capita
Income (PCI). GDPMP = GNPMP − (X −M)
country.
GDPFC = GNPFC − (X − M)
National Income and Where, X is the export and M is import of a
country.
its Related Aggregates
— Various National Income aggregates are
Nominal GDP
estimated either at factor cost or at — It is the market value (money-value) of all
market price. final goods and services produced within
the country.
Factor Cost
— Factor cost refers to cost of factors of
Real GDP
production viz., rent of land, interest of — The adjustment transforms the nominal
land, interest of capital wages for GDP into an index for quantity of total
compensation of employees for labour output. It is a measurement of the value
and profit for entrepreneurship of output economy, adjusted for price
FC = MP − Indirect taxes + Subsidies changes.
6 Magbook ~ Indian Economy

India Changes Base Year for GDP Calculation Personal Income (PI)
— Choosing a base year is the first step while counting the real GDP. — It is the income of the residents (individuals) of a
— A real GDP growth rate removes any effects that have arisen due country. To calculate personal income, transfer
to inflation to give us a true picture of economic reality. payments to individuals are added to National
— For the revised GDP calculations the Indian statisticians have Income, while social security contributions,
changed the base year from 2004-05 to 2011-12. corporate tax and undistributed profits are
subtracted.
— This mean that the real GDP will be counted by keeping the
prices of 2011-12 as the base prices instead of referring to the Personal Income = National Income
prices of 2004-05. + Transfer payments
— This changed alone has played can important part in shooting up – Social security contributions
the GDP and related numbers. – Corporate tax
— The change in base year is not an unusual phenomena as base – Undistributed profits
year is regularly updated.
Disposable Income (DI)
— The government has proposed the new base year for GDP and
IIP (Index of Industrial Production) as 2017-18 while for CPI it — It is the income of individuals at their disposal after
will be 2018. paying direct tax liabilities.
Disposable income = Personal income
GNP (Gross National Product) – Direct taxes (e.g. Income tax)
— It is the market value of all products and services produced in Green Economy
one year of a country (i.e. by labour and property).
— In this economy, which deals with the
GNP = GDP + X − M . environmental risks and ecological scarcity and
also an economy that aims for sustainable
development without degrading the environment.
Difference Between GDP and GNP
In GDP, goods and services produced in a country is Green GDP
added, whether it is produced by residents of the — It is the calculation of net natural consumption (i.e.
country or foreigners. In GNP, the production of resource depletion, environmental degradation,
foreigners in the country is not included, while the protective and restorative environmental initiatives).
production of nationals outside the country is included.
Green GNP
— GNP means that there has to be an adjustment for
Net National Product (NNP) the depletion of the country’s physical assets.
— It is the value of GNP after deducting depreciation of plant and Calculating National Income
machinery.
NNP = GNP – Depreciation According to Simon Kuznets, National Income can be
calculated by three method are as follows:
National Income (NI) = NNP − Indirect taxes + Subsidies
(i) Product Method In this method, net value of final
Real National Income (RNI) goods and services produced in a country, during
a year is obtained, which is called Total Final
— It is the value of National Income adjusted for inflation and
Product.
calculated from some reference point (base year).
(ii) Income Method In this method, a total of net
Real National Income
income earned by working people in different
= NNP at current prices × 100/Price index sector and commercial enterprises is obtained. By
Per-Capita Income (PCI) this method, NI is obtained by adding receipts as
— It is the measure of the amount of money that is being earned total rent, total wages, total interest and total profit.
per person in a certain area. (iii) Consumption Method It is also called Expenditure
— Per-Capita Income of a Country Method. Income is either spent on consumption
National Income or saved. Hence, NI is the additional of total
= consumption and total saving.
Population of the Country
Magbook ~ National Income 7

National Income understates social welfare, non-market


Problems in Calculating of —
transactions like home-makers service and do-it-yourself
National Income projects are not counted.
— National Income does not measure an increase in leisure or
Black Money work satisfaction changes in product quality.
— Illegal activities like smuggling and unreported income due — National Income does not accurately reflect changes in
to tax evasion and corruption are keeping outside the GDP environment like oil spills clean-up is measured as positive
estimates. Thus, parallel economy poses a serious hurdle output, but increased in pollution is not measured as
to accurate GDP estimates. negative.
— Per-Capital Income is a more meaningful measure of living
Non-Monetisation standards than total National Income.
— In most of rural economy considerable portion of — Problem of double counting, however, problem of double
transaction occurs informally and, they are called as counting could be avoided by utilising the value added
Non-Monetised Economy. This keeps the GDP estimates at approach.
lower level than the actual. — Problems of depreciation estimation.
Different methods of calculating or estimating depreciation.
Growing Service Sector —
— Arbitrary definition.
— Many services like BPO, value addition in legal — Inclusion or exclusion of certain items in National Income
consultancy, health services, financial and business accounting can cause confusion.
services and service sector as a whole is not based on — Challenges like difficulties in getting information, especially
accurate reporting and hence, national income is those related to underground economy.
underestimated.
Double Counting Gross Fixed Capital Formation
— It is also a hurdle to accurate GDP estimates. Though, (GFCF)
there are some corrective measures, but it is difficult to — It refers to net additions of capital stock such as equipment,
eliminate it. buildings and other intermediate goods. A nation uses
capital stock in combination with labour to provide services
National Incomes First and produce goods.
Estimations in India — To grow at a faster rate, a nation needs high levels of
— The first attempt to calculate National Income of India was capital formation, so that, it can grow its aggregate
made by Dadabhai Naoroji in 1867-68, who estimated income as well as Per-Capital Income. This is because
Per-Capita Income to be ` 20. higher levels of capital stock enable an economy to
— The first scientific method was made by Professor VKRV produce more goods and services.
Rao in 1931-32, but was not very satisfactory. — To achieve a high level of capital formation, a nation should
also achieve high levels of domestic savings (both
— The first official attempt was made by National Income
households and firms), so that capital formation can be
Committee headed by Professor PC Mahalanobis in 1949.
funded without relying on external debt.
— According to the National Income Committee Report — In GFCF, the term gross signifies that adjustments due to
(1954), National Income of India was ` 8710 crore and depreciation of capital stock (e.g. machinery) are not
Per-Capita Income was ` 225 in 1948-49. made. When such an adjustment is made, it is called Net
— In India, Central Statistical Organisation (1949) now Fixed Capital Formation.
renamed as Central Statistical Office (CSO) has been — The term fixed signifies that only fixed capital is counted
formulating National Income. and financial assets, stocks of inventories etc are
excluded.
Limitations in the Measurement — GFCF also excludes land sales and purchases.
of National Incomes
Incremental Capital Output Ratio (ICOR)
— Whilst measuring National Income, we need to be aware of
some of the following limitations, challenges, problems
— ICOR is used to assess a country’s level of production
which are discussed below efficiency. ICOR equals Annual Investment or Annual
Increase in GDP. Higher levels of ICOR means that capital
— National Income measures domestic economic performance,
is not being used efficiently to increase production.
not social welfare, but there should be a strong positive
correlation.
Generally, for most countries ICOR is at around 3.
8 Magbook ~ Indian Economy

Indian Organisations Related to — NSSO data on socio-economic surveys are regularly


released through the quarterly publication ‘Sarvekshana’
National Income Accounts issued by the Department of Statistics.
Central Statistical Office (CSO)
— Central Statistical Office (CSO), was set-up on 2nd May,
National Income’s Trend in
1951. It is one of the two wings of the National Statistical Growth and Structure
Organisation (NSO), along with National Sample Survey — During 2004-05 and 2010-11, the growth rate in
Office (NSSO), responsible for coordination of statistical National Income (NI) further accelerated to 8.4% (at
activities in the country and for evolving and maintaining 2004-05 price). The CSO has released the revised GDP
statistical standards. numbers on 31st May, 2019. As per new series of base
— Its activities include compilation of national accounts; year (2011-12), the GDP of India, at constant price, for
conduct of annual survey of industries and economic the year of 2018-19 was 6.6%.
censuses, compilation of index of industrial production, as — The per capita income in real terms (at 2011-12 prices)
well as consumer price indices. during 2017-18 is likely to attain a level of ` 86668 as
— It also deals with various social statistics, training, compared to ` 82269 for the year 2016-17. The per
international cooperation, industrial classification etc. capita income at current price during 2017-18 was
estimated ` 112835. The per capita Income has been
National Sample Survey Office (NSSO)
estimated at ` 92565 for the year 2018-19.
— The NSSO was set-up in 1950, for conducting large scale — As industrialisation spreads, there is an improvement in
sample surveys to meet the data needs of the country, for the share of industry and services. However, the evident
the estimation of National Income and other aggregates. change in Indian economy is slow, because of the slow
— It was recognised in 1970, by bringing together all aspects rate of growth of manufacturing.
of survey work under a single agency known as NSSO.
— The NSSO undertakes the fieldwork of Annual Survey of
Growth (in GVA) in Major Sectors (in percentage)
Industries in the whole country except Jammu and Kashmir.
(At 2017-19 prices)
— All India household consumer expenditure survey, which is Sector 2017-18 2018-19
the main source of data on the level of living of the Indian Agriculture, forestry and 5.0 2.9
population is also carried out by NSSO. fishing
— The data from this survey is used by the Planning Mining and quarrying 5.1 1.3
Commission to carry out all its poverty calculations,
including the setting up of poverty line and estimating the Manufacturing 5.9 6.9
number and proportion of poor in different regions of the Electricity, gas, water supply, 8.6 7.0
country. etc.
— NSSO survey on employment and unemployment helps to Construction 5.6 8.7
assess the volume and structure of employment and Trade, hotels, transport and 7.8 6.9
unemployment. It also conducts land holding surveys communication and service
alongwith crop estimation surveys of the principal crops. related to broadcasting
— NSSO is the only source that provides periodic estimates of Financing, real estate, 6.2 7.4
literacy, school enrolment, utilisation of educational professional services, etc
services, general morbidity, maternity and child care and Public administration, defence 11.9 8.6
utilisation of medical services. It also conducts survey to and other services
estimate the total number of physically disabled persons in
GVA at constant basic prices 6.9 6.6
the country and utilisation of Public Distribution System
(PDS). (Based on the data from the CSO)
Self Check
Build Your Confidence
1. The term National Income represents [IAS 2001] 6. Which one of the following institution prepares the
(a) Gross National Product (GNP) at market prices minus National Income estimates in India? [UPPCS 2006]
depreciation (a) Planning Commission
(b) Gross National Product (GNP) at market prices minus (b) Reserve Bank of India
depreciation plus net factor income from abroad (c) Central Statistical Organisation
(c) Gross National Product (GNP) at market prices minus (d) India Statistics Institute
depreciation and indirect taxes plus subsidies
7. Which one of the following sectors is the major
(d) Gross National Product (GNP) at market prices minus net
contribution towards the Gross Domestic Saving in
factor income from abroad India in recent time?
2. Consider the following statements (a) Public Sector (b) Private Sector
1. GDP is a better measure of national income than GNP. (c) Corporate Sector (d) Household Sector
2. GNP is always higher than GDP. 8. Which one of the following the most appropriate
Which of the statement(s) given above is/are correct? method to measure the economic growth of a country?
(a) Only 1 (b) Only 2 (a) National Income (b) Net National Product
(c) Both 1 and 2 (d) Neither 1 nor 2 (c) Gross Capital Formation (d) Gross Domestic Product
3. Which of the following is not included in the estimates of 9. Consider the following statements with reference to
National Income? Indian economy. [UPSC 2015]
(a) Sale of collector’s item
1. The rate of growth of Real Gross Domestic Product has
(b) Addition to inventory, but not sale of the company’s
steadily increased in the last decade.
products
2. The Gross Domestic Product at Market Price (in
(c) Market rent of self owned house
rupees) has steadily increased in the last decade.
(d) Cost of government services
Which of the statement(s) given above is/are correct?
4. Consider the following statements with reference to
(a) Only 1 (b) Both 1 and 2
Indian economy. [IAS 2010]
(c) Only 2 (d) Neither 1 nor 2
1. The GDP has increased by four times in the last 10 years.
10. The National Income of a country for a given period is
2. The percentage share of public sector in GDP has
equal to the [UPSC 2013]
declined in last 10 years.
(a) total value of goods and services produced by the
Which of the statement(s) given above is/are correct?
nationals
(a) Only 1 (b) Only 2
(b) sum of the total consumption and investment
(c) Both 1 and 2 (d) Neither 1 nor 2
expenditure
5. Consider the following statements (c) sum of personal income of all individuals
1. National Income is same as Net National Product at (d) money value of final goods and services produced
factor price. 11. In which of the following sectors, growth in GDP
2. The National Income of India is estimated mainly through continuously decreasing from 2013-14?
production and income methods. (a) Agriculture, forestry and fishing
Which of the statement (s) given above is/are correct? (b) Industry
(a) Only 1 (b) Only 2 (c) Manufacturing
(c) Both 1 and 2 (d) Neither 1 nor 2 (d) Financing, real estate, professional services etc.

1. (c) 2. (d) 3. (a) 4. (b) 5. (c) 6. (c) 7. (d) 8. (d) 9. (b) 10. (d)
11. (a)
Chapter three
Economic Growth and
Development
Economic Growth
Economic growth is — Economic growth is an increased economic capacity to produce goods and services,
an indicator of wealth, compared from one period of time to another which is conventionally measured by
increased in a country’s GDP (Gross Domestic Product) or GNP (Gross National Product)
reflecting the quantity or per capita Net Domestic Product (NDP). Per capita NDP is the most appropriate
of resources available measure of economic growth.
to a society. But it — Economic growth comes in two forms:

provides no (i) An economy can either grow extensively by using more resources (i.e. physical, human or
natural capital).
information about the
(ii) Intensively by using the same amount of resources more efficiently (productively).
allocation of these
resources. Economic
development is a
Economic Development
— According to Michael Todaro ‘‘Economic development is an increase in living standards
normative concept. It improvement in self-esteem needs and freedom from oppression as well as a greater choice.’’
applies in the context — It is referred to as the quantitative and qualitative changes in economy such as

of people’s sense of development of human capital, critical infrastructure, regional competitiveness,


environmental sustainability, social inclusion, health, safety, literacy etc.
morality.
— Human Development Index (HDI) is the most appropriate measure of economic
development.
Economic development in all societies must have atleast the following objectives:
— To increase the availability and widen the distribution of basic life sustaining goods.
— To raise levels of living by ensuring higher incomes, more jobs and greater attention to culture.
— To expand the range of economic and social choices available to both individuals and nations.

Difference Between Economic Growth and Economic Development


Economic Growth Economic Development
Economic growth refers to an increase Economic development implies an upward movement
over time in a country’s real output of of the entire social system in terms of income,
goods and services (GNP). savings and investment alongwith progressive
changes in socio-economic structure of country.
Growth relates to a gradual increase in Development relates to growth of human capital
one of the components of GDP : index, a decrease in inequality figures and structural
consumption, government spending changes that improve the general population’s quality
investment, net exports. of life.
It is qualitative in nature and measured It is qualitative and measured through HDI, GDI, HPI
through increase in real GDP. etc.
It is concerned with increase in the It is concerned with structural changes in the
economy’s output. economy.
Magbook ~ Economic Growth and Development 11

Measurement of Economic Human Development Index


(HDI)
Development — The United Nations Development Programme
— To measure economic development is a complex process. (UNDP) introduced the HDI in its first Human
Economists have used various yardsticks for measuring economic Development Report (HDR) prepared under the
development. stewardship of Mahbub-ul-Haq in 1990.
Life Expectancy Index (LEI) Educational
National Income and Per Capita —
Attainment Index (EAI) and Standard of Living
Income Index (SLI) are the indicators of HDI.
— This is the traditional approach to measure economic — Life expectancy refers to life expectancy at birth,
development. World Bank uses the concept of per capita Gross not at age 1.
National Income (GNI) as a measure for comparing and classifying — Educational Attainment Index (EAI) is a
countries based on their stage of economic development. combination of adult literacy rate and combined
— World Development Report (WDR), 2015 (sub-titled Gender enrolment ratio.
Equality and Development) classifies countries category wise — Standard of Living Index (SLI) is represented
based on per capita GNI. here by the concept of Purchasing Power Parity
— Since official exchange rate is used in the international comparison (PPP). Per capita income is converted into PPP
of GNI, therefore, they do not give a correct picture for two reasons in terms of US dollar.
They are :
(i) Purchasing power capacity of a country ignored. Human Development Report
(ii) Official exchange rate does not reflect the value of non-traded (HDR), 2018
goods. — The United Nations Human Development Report,
— In order to overcome this problem, following the work of IB Kravis 2018 was released on 14th September, 2018
and others ‘‘International comparisons of real product and with the title ‘Human Development for Everyone’.
purchasing power’’ (1978), the UN International Comparison — In the 2018 report, India with a score of 0.640
Programme gave the Purchasing Power Parity (PPP) method. has been ranked 130 out of 189 countries in
terms of HDI. In the 2017 report, India was
Purchasing Power Parity (PPP) ranked 131 out of 188 countries.
Inequality-adjusted Human Development Index,
PPP approach was given by economist Gustav Casell in 1918. The
Gender Inequality Index (GII) and
concept is based on the law of one price, wherein the absence of trade
Multi–dimensional Poverty Index (MPI) were
and non-trade barriers, identical goods will have same price in different
introduced in HDR from the year 2010. Gender
countries, when the prices are expressed in the same currency. PPP
Development Index is introduced first time in
exchange rates are calculated by comparing the prices of the same basket
HDR in 2014. The following indexes have been
of goods are services indifferent countries.
issued by Human Development Report
PPP was first used by International Monetary fund (IMF) in 1988 for
measuring standard of living in different countries. Indian Economy is the Human Development Group
third largest economy in term of PPP. Group Index Value
The PPP is defined as the number of units of a country’s currency required Very high human development 0.800 and above
to buy the same amount of goods and services in the domestic market as High human development 0.700 and above
$ 1 would buy in the United States. e.g. if we have to spend ` 30 for Medium human development 0.550 and above
purchasing the same amount of goods and services as are purchased in Low human development 0.352 and above
spending $ 1 in USA, then the exchange rate in PPP approach is $1= ` 30.
Inequality-Adjusted Human
Physical Quality of Life Index (PQLI) Development Index (IHDI)
— PQLI was the first attempt towards providing comprehensive — IHDI adjusts the Human Development Index
measure of economic development. It was developed by Morris (HDI) for inequality in distribution of each
David Morris in the mid-1970s. dimension across the population. The IHDI
— PQLI is the average of three values, viz, life expectancy, basic literacy accounts for inequalities in HDI dimensions by
rate and infant mortality rate. Each value was scaled on 1 to 100, ‘discounting’ each dimension’s average value
where 1 represents the worst and 100 represents the best. according to its level of inequality.
12 Magbook ~ Indian Economy

— The IHDI equals the HDI, when there is no inequality — A second-generation GNH concept, treating happiness as
across people, but is less than the HDI as inequality rises. a socio- economic development metric was proposed in
In this sense, the IHDI is the actual level of human 2006, by Med Jones.
development (accounting for this inequality), while the GNH value is proposed to be an index function of the total
HDI can be viewed as an index of potential human average per capita of the following measures:
development (or the maximum level of HDI) that could be — Economic wellness — Environmental Wellness
achieved, if there was no inequality. — Physical wellness — Mental wellness
— The loss in potential human development due to inequality — Workplace wellness — Social wellness
is given by the difference between the HDI and the IHDI — Political wellness
and can be expressed as a percentage.
Genuine Progress Indicator (GPI)
Gender Inequality Index (GII) — The GPI is a concept in green economics and welfare
— GII reflects women’s disadvantage in three dimensions: economics. A GPI attempts to measure whether or not a
reproductive health, empowerment and the labour market country’s increased production of goods and expanding
for as many countries as data of reasonable quality allow. services have actually resulted in the improvement of
— The index shows the loss in human development due to welfare of the people of the country. Genuine Progress
inequality between female and male achievements in these Indicator refers to the concept of a quantitative
dimensions. It ranges from 0, which indicates that women measurement of well-being and happiness.
and men fare equal to 1, which indicates that women fare — The two measures of GPI and GNH are both motivated by
as poorly as possible in all measured dimensions. the notion that subjective measures like well-being, are
more relevant and important than more objective
Gender Development Index (GDI) measures like consumption.
— The new GDI measures gender gap in human development
achievements in three basic dimensions of human Multidimensional Poverty Index (MPI)
development health, measured by male and female life — MPI was developed in 2010, by Oxford Poverty and
expectancy at birth, education and command over Human Development Initiative and UNDP and different
economic resources. factors to determine poverty beyond income based list
HDR 2018 were used.
— The MPI is an index of acute multidimensional poverty. It
HDR, 2018 Top 5 Lowest 5
India shows the number of the people, who are
Index Countries Countries
multidimensionally poor (suffering deprivation in 33% of
HDI (Human Norway, Niger, Central With a score of weighted indicators) and the number of deprivation with
Development Switzerland, African 0.640 (India which poor households typically contend. It reflects
Index) Australia, Republic, South ranked 130 out
deprivation in very rudimentary services and core human
Ireland, Sudan, Chad, of 189 countries)
Germany Burundi functioning for people across 104 countries. The index
uses same three dimensions as the Human Development
IHDI Iceland, Central African With a score of
(Inequality Japan, Republic, South 0.468 (India
Index such as health, education and standard of living.
adjusted Human Norway, Sudan, Chad, ranked 101 out — These are measured using 10 indicators:
Development Switzerland, Niger, Sierra of 151 countries)
Index) Finland Leone
Dimensions and their 10 Indicators
GII (Gender Switzerland, With a score of Dimensions Indicators
Inequality Denmark, 0.524 (India Health 1. Child mortality
Index) (2013) Netherlands, ranked 127 out 2. Nutrition
Sweden, of 159 countries) Education 3. Years of schooling
Norway 4. Children enrolled
Living Standards 5. Cooking fuel
Gross National Happiness (GNH) 6.
7.
Toilet
Water
— The term ‘Gross National Happiness’ was coined in 1972, 8. Electricity
by Bhutan’s then King Jigme Singye Wangchuck. 9. Floor
10. Assets
— GNH was designed in on attempt to define an indicator
that measures quality of life or social progress in more — Each dimension and each indicator within a dimension is
holistic and psychological terms than the economic equally weighted.
indicator of GDP. It is not measured directly, but only by — The lower and the index value of lesser the
the factors, which are believed to lead to it. multidimensional poverty.
Magbook ~ Economic Growth and Development 13

MPI in Some Asian Countries Human Sustainable Development


Countries MPI Value Index (HSDI)
Bangladesh 0.194 — HSDI was developed in 2010, by independent economists.
Bhutan 0.175 HSDI attempts to measure the overall quality of life by
China 0.023 factoring in a fourth parameter-per capita carbon emission
India 0.121 to the existing HDI. According to HSDI, the top five
Nepal 0.154 countries are Norway, followed by New Zealand, Sweden,
Pakistan 0.228 Switzerland and France.

Global Hunger Index (GHI) National Prosperity Index (NPI)


— GHI is designed to measure and track hunger globally NPI is a standard measure of social and economic
and by country and region. It is calculated each year by development. Its three components are as follows
the International Food Policy Research Institute (IFPRI). (i) GDP growth rate
— The Global Hunger Index (GHI) was first released by (ii) Improvement in the quality of life.
IFPRI in 2006. (iii) Use of cultural heritage based on a value system in each
area of life.
GHI Report, 2018 Former President APJ Abdul Kalam said that the country’s
GHI 2018 published on 10 October, 2018. The GHI 2018 report economic growth should always be guided by ‘National
was prepared jointly by global NGOs namely, concern world Prosperity Index’ that includes components like improvement
wide (Ireland) and Welth-ungerhilfe (Germany). of quality of life and adoption of a value system derived from
our ancient civilisation besides GDP. SAARC prosperity Index
To reflect the multidimensional nature of hunger, the GHI 2018
idea is given by Abdul Kalam on the basis of NPI.
combines the following four component indicators into one index:
1. Under nourishment 2. Child wasting Millennium Development
3. Child stunting 4. Child mortality Goals (MDGs)
The GHI ranks countries on a 100-point scale. Zero is the best
score (no hunger) and 100 is the worst, although neither of MDGs were eventuated at the UNs Millennium Summit, 2000,
where the world leaders of 189 UN Member States
these extremes is reached in practice.
(193 currently) agreed on a set of quantifiable and monitorable
As per 2018 Report, India stands 103rd position out of goals for development and poverty eradication to be achieved
119 countries. by 2015.

Green Gross Domestic Product (Green GDP) MDGs Report, 2015


— Green GDP is an index of economic growth with the UN released the Global Report on MDGs. NITI Aayog released
environmental consequences of that growth factored in. the Asia-Pacific MDGs Report, 2015. India has managed to
reduce its extreme poverty incidence to a half from 49.4% in
— Green GDP monetises the loss of biodiversity and
1994 to 24.7% in 2011. The report set the limit for extreme
accounts for costs caused by climate change. Some
poverty as those living on $ 1.25 or less a day.
environmental experts prefer physical indicators (such as
waste per capita or carbon dioxide emissions per year), New Global Goals
which may be aggregated to indices such as the The United Nations General Assembly (UNGA) formally
Sustainable Development Index. adopted the 2030 Agenda for sustainable development on 26th
September, 2015, along with a set of bold new Global Goals.
Sustainable Development The 193 members GA adopted the new framework,
Index (SDI) ‘‘Transforming Our World: the 2030 Agenda for Sustainable
Development,’’ composed of 17Goals and 169 targets to wipe
— The sustainable development is defined as the
out poverty, fight inequality and tackle climate change over the
development to achieve the needs of present generation
next 15 years. Ambitious 17 goals are–no poverty, zero hunger,
without compromising future generations need.
good health and well-being, quality education, gender equality,
— Sustainable development ensures the well-being of individual clean water and sanitation, affordable and clean energy,
by integrating social development, economic development decent work and economic growth, industry, innovation and
and environmental conservation and protection. infrastructure, reduced inequality.
— The challenges of sustainable development are Sustainable cities and communities, responsible consumption
population, poverty, inequality, the shortage of drinking and production, climate action, life below water, life on land,
water, human health, consumption of energy, peace, justice and strong institution and partnership for the
deforestation and petrol consumption. goals.
14 Magbook ~ Indian Economy

Sustainable development was defined by the Brundtland


Different Levels of Economic —
Commission (1987) as the meeting of the needs of the
Development present without compromising the ability of future
generations to meet their own needs.’
Developing Country
— A developing country is a country, that has not reached Need of Sustainable Development
the Western style standard of democratic government, — The concept of sustainable development aims at
free market economy, industrialisation, social maximising the net benefits of economic activities, subject
programmes’ and human rights guarantees for their to maintaining the stock of productive assets (physical,
citizens. human and environmental) over time and providing a social
safety net to meet the basic needs of the poor.
Newly Industrialised Countries (NIC)
— Sustainable development, therefore, attempts to accelerate
— The word NIC is used for those countries, which have
development in an environmentally responsible manner
more advanced economy than other developing nations,
keeping in mind the intergenerational equity requirements.
but which have not yet fully demonstrated the signs of a
developed country. — The spirit of the conference was captured by the
expression Harmony with Nature, brought into the fore
Emerging Markets with the first principle of the Rio Declaration: ‘Human
— These are countries with social or business activity in the beings are at the centre of concerns for sustainable
process of rapid growth and industrialisation. Many of development. They are entitled to a healthy and productive
the countries like India and China, earlier considered to life in harmony with nature.’
be emerging are now said to have emerged and the term — In 1993, UNCED instituted the Commission on Sustainable
is considered to be outdated by many. Development (CSD) to follow-up on the implementation of
Agenda 21.
Economy and Environment — In 2002, 10 years after the Rio Declaration, a follow-up
conference, the World Summit on Sustainable
— Natural and environmental resources have three Development (WSSD) was convened in Johannesburg, to
economic roles: waste disposal services related to the renew the global commitment to sustainable development.
environment’s assimilative capacity, natural resource The conference agreed on the Johannesburg Plan of
inputs into production and directly consumed life Implementation (JPoI) and further tasked the CSD to
support services and aesthetic amenities. follow-up on the implementation of sustainable
— The natural and environmental resource input function is development.
central to understanding the relationship between — In December, 2009, the UN General Assembly adopted a
economic growth and environment. resolution agreeing to hold the United Nations Conference
on Sustainable Development (UNCSD), also referred to as
Sustainable Development ‘Rio+20 or Rio 20’.
— Economic development without environmental — In 2012, the member States have agreed on the following
considerations can cause serious environmental two themes for the conference : Green economy within the
damage, in turn, impairing the quality of life of context of sustainable development and poverty eradication
present and future generations. Sustainable and Institutional framework for sustainable development.
development attempts to strike a balance between the * Strong Sustainability Rule is a rule, which
demands of the economic development and the need
requires a separate preservation of each category of
for protection of the environment.
critical asset, assuming these to be complements
— It seeks to combine the elements of economic efficiency, rather than substitutes.
intergenerational equity, social concerns and * Weak Sustainability Rule is a rule, which seeks to
environmental protection. Although, the term maintain the aggregate monetary value of the total
‘sustainable development’ has many interpretations, it stock of assets, assuming a high degree of
generally refers to non-declining human wellbeing over substitutability among the various asset types.
time.
Magbook ~ Economic Growth and Development 15

increased petrol and diesel price while reducing annual


Measuring India’s Environmental CO2 emissions. Excise duties on petrol or diesel also act as
Performance an implicit carbon tax by putting an effecting price on
— In a recent ranking of Environmental Performance Index emissions. Recently, the Government of India revised its
EPI 2018 was placed 177 out of 180 countries. coal cess from ` 50 per ton to ` 100 per ton.
— The EPI exercise is nevertheless, useful in flagging
some areas of concern. India should do better on 12th Five Year Plan and
public health and environmentally preventable child
mortality, as has Bangladesh. Sustainability
— India scored 5.75 out of 100 air quality. India’s — The 12th Five Year Plan is the first time Five Year Plan has
performance in air quality is disappointing. Delhi is sustainability as a prominent focus and appreciates its
always in the news for its poor air quality. India’s total emphasis on low carbon growth strategies.
forest cover has registered a 0.2% increase between
2015 and 2017. Monitorable Targets
for the 12th Plan
Environmental Taxes — To increase forest and free cover by 5% points.
— As against the command and control approach to — To improve forest production and maintain biodiversity.
management of the environment, the economic or — To clean all critically polluted rivers by 2020.
Market Based Instruments (MBIs) approach sends — To reduce 20-25% energy use per unit of GDP by 2020.
economic signals to the polluters to modify their
— To reclaim wetlands/island lakes/ponds by 2017.
behaviour.
— The approach normally involves financial transfers
between polluters and the community and affects relative Green Accounting
prices. But the polluters have freedom to respond and
Green accounting is focused on addressing deficiencies in
adjust, in the manner they want. They can, thus, choose
conventional accounts with respect to the environment.
the least cost option to meet the requirements.
Integrated environment and economic (green) accounting,
— Hence, it is considered to be an efficient approach therefore, attempts at accounting for both socio-economic
compared to the approach based on standards and performance and its environmental effects and integrating
regulations. The MBIs, therefore, have the benefit of environmental concerns into mainstream economic planning and
being flexible and cost effective providing incentives for policies.
dynamic efficiency and resource transfer 12th Five Year
Such integrated accounts can be useful in assessing the
Plan and sustainability.
sustainability of economic growth and also the structural
distortion of the economy by environmentally unsound production
Carbon Tax and consumption patterns.
— India has cut subsidies and increased taxes on fossil
fuels (Petrol and Diesel) turning a carbon subsidy regime
into one of carbon taxation. This has significantly
Self Check
Build Your Confidence
1. Consider the following statements 6. Consider the following statements
1. Economic development is a broader and normative 1. The term ‘Trickle Down Effect’ was coined by Ronald
concept. It concerns with structural change in economy. Regan.
2. Economic growth is a narrow concept. It concerns with 2. Kyznets Curve shows relations between inequality and
increase in the economy’s output. capita income.
Which of the statement(s) given above is/are correct? Which of the statement(s) given above is/are correct?
(a) Only 1 (b) Only 2 (a) Only 1 (b) Only 2
(c) Both 1 and 2 (d) None of these (c) Both 1 and 2 (d) None of these

2. Consider the following statements about Human 7. Consider the following statements
Development Index 1. More than one-third of world population lives in low
1. World Bank introduced the Human Development Index. income countries.
2. India is a medium human development country. 2. More than three quarters of the Gross National Income
3. Among the BRICS countries, Russia stands first in the of the world is accounted for by the high income
HDI rank. economy countries.
Which of the statement(s) given above is/are correct? Which of the statement(s) given above is/are correct?
(a) Only 1 (b) 1 and 2 (a) Only 1 (b) Only 2
(c) 2 and 3 (d) All of these (c) Both 1 and 2 (d) Neither 1 nor 2

3. Which of the following could to have prevented the 8. Human Poverty Index (HPI) developed by UNDP is based
‘trickle down’ effects in Indian economy? on which of the following deprivations?
1. Increased dependence of agriculture on purchased 1. Income deprivation 2. Literacy deprivation
inputs and privately managed irrigation. 3. Social services deprivation 4. Employment deprivation
2. More employment of labour by larger landholding Select the correct answer using the codes given below
farmers. (a) 2 and 3 (b) 1, 3 and 4
3. Lowered participation women in agricultural workforce (c) 1, 2 and 4 (d) All of these
due to new technology.
9. Consider the following statements
4. The failure of the Green Revolution.
1. The term ‘Gross National Happiness’ (GNH) was coined
Select the correct answer using the codes given below
in 1972, by Bhutan’s then King Jigme Singye
(a) 1 and 2 (b) 2 and 3 Wangchuck.
(c) 1 and 3 (d) 2 and 4
2. A second-generation GNH concept, treating happiness
4. Consider the following statements as a socio-economic development metric was proposed
1. Gender Inequality Index contains three dimensions in 2006, by Med Jones.
reproductive health, empowerment and labour market. Which of the statement(s) given above is/are correct?
2. It reflects women’s disadvantage in health, (a) Only 1 (b) Only 2
empowerment and labour market. (c) 1 and 2 (d) None of these
Which of the statement(s) given above is/are correct? 10. Which one of the following indicators is not used in the
(a) Only 1 (b) Only 2 determination of Gender Development Index (GDI) in the
(c) Both 1 and 2 (d) None of these Human Development Report (HDR)? [UPPCS 2008]
5. Consider the following statements (a) Life expectancy of female
1. Global Peace Index is a brain child of Steve Kilelea. (b) Female adult literacy and gross enrollment
2. Gross National Happiness is a brain child of Jigme (c) Female political empowerment
Singye Wangchuck. (d) Female per capita income
Which of the statement(s) given above is/are correct? 11. Economic growth in country X will necessarily have to
(a) Only 1 occur if [UPSC 2013]
(b) Only 2 (a) there is technical progress in the world economy
(c) Both 1 and 2 (b) there is population growth in X
(d) None of the above (c) there is capital formation in X
(d) the volume of trade grows in the world economy

1. (c) 2. (c) 3. (c) 4. (c) 5. (c) 6. (c) 7. (c) 8. (b) 9. (c) 10. (c)
11. (c)
Chapter four
Economic Planning
in India
Meaning of Strategies of
Economic Planning Planning
Planning involves — Economic planning refers to the path Harrod Domar Strategy
of actions in terms of policy measures — The first Five Year Plan (1951-56) was
acceptance of a clearly to be followed in future, in pursuance based on this strategy. This strategy
defined set of objectives in of pre-determined objectives. emphasised the role of capital
terms of which to frame — Planning Commission (now NITI accumulation’s dual character, which on
overall policies, formulation Aayog) defines economic planning as the one hand, increases the national
the utilisation of country’s resources income (demand side role) and on the
of a strategy for promoting
for developmental activities in other hand, increases the production
the realisation of the ends accordance with national priorities. It is capacity (supply side role).
defined and working out a a consciously and judiciously carried — According to this growth model, the rate
rational solution to out process for optimum utilisation of of economic growth in an economy is
problems—an attempt to existing resources in order to fulfil dependent on the level of savings and
some well defined objectives. capital output ratio.
coordinate means
and ends.
Objectives of Nehru-Mahalanobis
Planning Strategy
The broad objectives of Five Year Plans in — This strategy was a two sector model,
that is, consumer good sector and capital
India are as follows:
good sector. The strategy emphasised
— A high rate of growth with a view to investment in heavy industry to achieve
improvement in standard of living. industrialisation for rapid economic
— Modernisation of economy in terms of development. It was based on the
adoption of new technologies and Russian experience.
social outlook. — The objective was to become self-reliant
— Economic self-reliance meaning and overcome capital constraint. This
avoiding import which can be strategy was adopted in the Second Five
produced in India. Year Plan (1956-61) and with minor
— Equity implying equitable distribution modifications, up to the Fifth Plan. It was
of wealth with social justice. a long-term strategy.
— Economic stability, which means — It is also suffered to as planning by
controlling inflation and inducement as against imperative
unemployment. planning.
18 Magbook ~ Indian Economy

Gandhian Strategy History of Planning in India


— It was enunciated by Acharya Shriman Narayan Agarwal — First attempt to initiate economic planning in India was
in his ‘Gandhian Plan’ in 1944. The basic objective of the made by M Visvesvaraya, a noted engineer and politician,
Gandhian Model is to raise the material as well as cultural in 1934, through his book, Planned Economy for India.
level of the masses so as to provide a basic standard of
— In 1938, National Planning commission was set-up under
life.
the Chairmanship of Jawaharlal Nehru by the Indian
— It laid emphasis on scientific development of agriculture National Congress.
and rapid growth of cottage and village industries.
— Its recommendations could not be implemented because
Moreover, Gandhian strategy emphasised on employment
of the beginning of the World War II and changes in the
oriented planning rather than production oriented
Indian political situation.
planning of Nehru.
— It stated that the objective of planning was to ensure an
LPG Strategy adequate standard of living for the masses. It emphasised
heavy industry and land reforms.
— Liberalisation, Privatisation and Globalisation (LPG) strategy
— In 1944, Bombay plan was presented by 8 leading
of planning was introduced by the Finance Minister,
industrialists of Bombay including JRD Tata, GD Birla and
Dr Manmohan Singh under Narasimha Rao government.
others.
— The strategy ended the ‘license-permit-raj’ and opened
— It saw future progress based on textile and consumer
the hitherto areas reserved for the public sector to
industries and saw an important role for the state in post
private sector. It allowed for foreign direct investment and
independent India.
followed an export promotion policy to boost economic
growth. In all, it changed the nature of planning from
— In 1945, People’s Plan was given by MN Roy.
centralised to ‘indicative’, wherein planning was to play a — In 1950, Sarvodaya plan was given by Jai Prakash
facilitating role. Narayan. A few points of this plan were accepted by the
— It is also referred to as planning by inducement as against government.
imperative planning.

Types of Economic Planning


Classification Meaning / Characteristic
Role of Government
Imperative Planning ˜ Also called directive, totalitarian or authoritative planning or planning by direction or command.
˜ One central authority, which decides all aspects of planning.
˜ Generally, practiced in socialist economies.
Planning ˜ Also called indicative or market incentives planning.
Inducement ˜ State regulates the private sector to achieve the set targets.
˜ Persuasion after than compulsion.

Extent of People’s Participation


Centralised Planning ˜ All economic decisions taken by the central authority.
˜ Government formulates economic plans, determines objectives, sets targets and priorities.
Decentralised ˜ Connected with capitalistic economies.
Planning ˜ Implemented through market mechanism.
˜ Empowers people as planning operates from bottom to top.
On the Basis of Time-Period
Short-Term Plan ˜ Also known as Controlling plan.
˜ Encompasses the period of one year, therefore, also known as Annual plan.
˜ Main objective-raise revenue, attain short-term economic target, bring price stability and remove BoP deficit.
Magbook ~ Economic Planning in India 19

Classification Meaning / Characteristic


Medium-Term Plan ˜ Lasts for 3 to 7 years, but normally, it is made for 5 years.
˜ Objective-raise per-capita income, raise level of employment, create self-sufficiency, reduce dependence over
foreign aid and raise revenues through domestic sources and to remove regional and intra- regional disparity.
Long-Term Plan ˜ Lasts for 10 to 30 years.
˜ Also called Perspective plan.
˜ Basic philosophy is to bring structural changes in the economy.
On the Basis of Resource Allocation
Physical Planning ˜ Concerned with physical allocation of resources with the productive yield.
˜ Aim is to bring physical balance between investment and output.
˜ Concerned with real resources of the economy.
Financial Planning ˜ Aims at establishing equilibrium between demand and supply to avoid inflation and bring economic stability.
˜ It depicts the size of economy in monetary terms.
On the Basis of Flexibility/Rigidity
Rolling Plan ˜ Concept given by Gunnar Myrdal.
˜ Refers to the rolling of a plan at intervals of usually one year, so that it continues to be a plan of certain
number of years.
˜ It is usually medium-term plan.
Fixed Plan ˜ The contents of plan are fixed in relation to a fixed time period.
˜ Contents–targets, priorities, strategies—changed only under severe unforeseen events.
Structural Planning ˜ Aims at bringing changes in socio-economic set-up of a country.
˜ Attributed to USSR (1929), which abolished Landlord system, introduction of collective farming etc.
Functional Planning ˜ No changes brought in the existing socio-economic set-up and planning made in the context of existing
institutions.
˜ Followed in France, Germany, UK etc.

Note Indicative planning is attributed to France. India adopted it in 1991. Rolling plan was introduced in India by Janata Party Government in 1978.

National Institution for Transforming India (NITI) Aayog


— National Institution for Transforming India (NITI) Aayog is a policy ‘think-tank’ of government that replaces Planning commission
and aims to involve states in economic policy-making. It will be providing strategic and technical advice to the Central and the
State Governments. Prime Minister heads the Aayog as its Chairman.
— The body occupying the building, the Planning commission, had been restructured and renamed the National Institution
for Transforming India (NITI) Aayog, even as the employees of the erstwhile Planning commission face teething troubles,
with no clearly defined function or role, Business Standard takes a look at five key counts on which the new body differs
from the Planning Commission of India.

NITI Aayog-Composition
— NITI Aayog will be headed by the Prime Minister and will have a governing council, comprising Chief Ministers of states
and Heads of all Union Territories. The Governing council replaces the earlier National Development council.
— In addition, there will also be a regional council comprising of Chief Ministers and Lieutenant Governors of Union
Territories, which will be mandated to develop plans that are region specific.
— The Aayog will have 7-8 full time members and two well-known and accomplished part-time members, drawn from
leading research organisations and major universities. Four union ministers, nominated by the Prime Minister, will also be
included in ex-officio capacity.
— On the PM’s invitation, specialists across domains, will be invited to share knowledge and add value to the planning
process, making extensive use of technology in developing sustainable plans and programme implementation. The Prime
Minister shall appoint a full time Chief Executive Officer with a fixed tenure and may sanction a dedicated secretariat, if
deemed necessary.
20 Magbook ~ Indian Economy

Planning Commission National Development


— After independence in 1950, the Planning commission was set-up Council (NDC)
under the Chairmanship of Pt Jawaharlal Nehru. It was to
— The National Development Council (NDC) is
formulate plans for the economic development of the country on
neither a constitutional body nor a statutory body.
the basis of the available physical, capital and human resources.
Union Cabinet set-up NDC in 1952, through an
— On 1st January, 2015, Government established NITI Aayog executive order.
(National Institution for Transforming India) to replace Planning
— National Development Council (NDC) is mainly
commission.
concerned with approval of Five Year Plans. The
— The Planning commission was essentially a non-political NDC is headed by the Prime Minister and consists
and non-constitutional advisory body, which makes of the Central Ministers, Chief Ministers of the State
recommendations to the government. It was set-up through an and Lt Governors, Administrators of Union
executive order of the Union Government on 15th March, 1950. Territories and Members of the Planning
Differences between NITI commission.
Aayog and Planning Commission — The Secretary of the Planning commission acts as
the Secretary of the Council. From a strictly legal
Parameter NITI Aayog Planning Commission point of view, NDC is essentially an advisory body.
Financial To be an advisory body or a Enjoyed the powers to allocate
Clout think-tank. The powers to funds to ministries and State Functions of NDC
allocate funds might be Governments.
vested in the Finance — To review the working of plan from time-to-time
ministry. and to recommend such measures as are
Full-time The number of full-time The last commission had eight necessary for achieving the aims and targets
Members members could be fewer full-time members. articulated in the National Plans.
than Planning commission. — To review the social and economic policies
States Role State Government are States role was limited to the affecting the development of the nation.
expected to play a more National Development council — To ensure maximum cooperation of people in the
significant role than they did and annual interaction during planning and improvement of administrative
in the Planning commission. plan meetings.
capacity.
Member To be known at the CEO and Secretaries or Member — To suggest programmes and schemes for the
Secretary to be appointed by the Secretaries were appointed
Prime Minister. through the usual process. development of less developed and backward
classes and regions.
Part-time To have a number of Full Planning commission had no
Members part-time members, provision for part-time members. — To assess resources required for implementing
depending on the need from plans and to suggest ways and means for raising
time- to- time. national resources.
— To prescribe guidelines for the formulation of
15 Years Vision Document in National plans.
— To consider National plans as formulated by
Place of Five Year Plan Planning commission and to approve the same.
— Narendra Modi Government has decided to discontinue the Five
Year Plan and replace it with 15 Year Vision Document. The first Five Year Plans in India
15 Year Vision Document will come into effect from 2017-18 after — After independence, India launched a programme
the end of the 12th Five Year Plan. It will be formulated with of Five Year Plans to make the optimum use of
central objective of eradication of poverty. It will come alongwith a country’s available resources and to achieve rapid
7 year National Development Agenda which will lay down the economic development.
programmes, schemes and strategies to achieve a long term
— In India, development plans were formulated and
vision. The long vision document (Perspective plan) will comprise
carried out within the framework of the mixed
three year mass economic framework.
economy.
— 2017-18 to 2032-33 Vision Document
— In India, Economic planning was adopted in the
— 2017-18 to 2024-25 National Development Agenda form of Five Year Plans and was seen as a
— 2017-18 to 2019-20 Review of Development Agenda (to be development tool on account of various reasons.
repeated after every three year)
Magbook ~ Economic Planning in India 21

These are:
— Limitations of market mechanism in view of the existing economic backwardness of India at the time of independence.
— The need for social justice as experience of the past five and-a-half decades suggests that in a free enterprise economy, economic gains
do not necessarily trickle down.
— Judicious mobilisation and allocation of resources in the context of overall development programme in the light of the resource constraint
in India.
— So far, Twelfth Five Year plans have been formulated since, the year 1951. Twelfth Five Year Plan (2012-2017), came into force
once it was approved by the NDC on 27th December, 2012.

Formulation of Five Year Plan


— The preparation of a Five Year Plan starts with the formulation of an Approach Paper, outlining the macroeconomic
dimensions, strategies and objectives of the plan.
— The Approach Paper is prepared by the Planning Commission (NITI Aayog) after intensive consultations with individuals.
— The Approach paper then presented to the National Development Council (NDC).
— Thus, based on the parameters postulated in the NDC approved Approach Paper, the Central Ministries and the states
prepare their respective plans, with the help of a large number of Steering committees or Working groups.
— Based on the reports of these Steering committees and Working groups, the States and the Central Ministries were come
with their proposals of detailed plans and programmes.

Implementation of Five Year Plan


— The Five Year Plan is implemented through Annual plans, which is a detailed description of the allocation of resources
between centre and states and for different sectoral activities in the government.
— In particular, it involves allocation of budgetary resources and detailed consideration of public sector projects, programmes
or schemes.
— The sanction of government expenditure is affected through Annual Budget, which is passed by the Parliament every year.

Brief Description of Five Year Plan


Plans Objectives Facts Assessments
First Plan ˜ Highest priority accorded to agriculture in view of ˜ Agriculture production increased dramatically.
(1951-56) large-import of foodgrain and inflation. ˜ National income went up by 18% and Per- Capita income
(Harrod ˜ Increasing the rate of investment from 5% to 7%. by 11%.
Domar Model) ˜ 31% of total plan outlay on agriculture followed by ˜ Targeted growth rate was 2.1% and First Plan achieved
transport and communication, social services, power 3.6%.
and industry. ˜ Price level was stable.
˜ Economist KN Raj was the architect.
Second Plan ˜ Rapid industrialisation with particular emphasis on ˜ Moderately successful, targeted growth rate was 4.5% but
(1956-61) the development of basic and heavy industry, also achieved 4.1%.
called Nehru Mahalanobis plan. ˜ Durgapur (UK), Bhillai (USSR) and Rourkela.
˜ To promote a socialistic pattern of society as (W Germany) Steel plants set-up with foreign help.
envisaged at Avadi Summit of Indian National ˜ Atomic Energy commission came into being and TIFR was
Congress in 1955. set–up.
˜ To increase National income by 25%, expansion of ˜ Inflation and low agricultural production and Suez crisis.
employment and reduction of inequality.
˜ To increase the rate of investment from 7% to 11%
of GDP.
˜ There was a thrust towards substitution of basic and
capital good industries in this plan.
22 Magbook ~ Indian Economy

Plans Objectives Facts Assessments


Third Plan ˜ Indian economy entered take off stage (WW Rostow). ˜ A failure because of worst famine (1965-66), in 100
(1961-66) ˜ Self-reliant and self-generating economy was the goal. years.
(Gadgil ˜ Priority to agriculture and development of basic ˜ Indo-China (1962) and Indo-Pakistan (1965), conflict
Yojana) industries. Tried to balance industry and agriculture. diverted the resources from development to defence.
˜ To increase the National Income by 30% and Per ˜ Postponement of fourth Plan by 3 years.
Capita Income by 17%. ˜ Targeted growth 5.6% achieved growth 2.8%.
˜ The situation created by Indo-Pakistan Conflict (1965),
two successive years of severe drought, devaluation of
currency by 57% general rise in prices and erosion of
resources for plan delayed.
˜ Fourth Plan delayed because between 1966 to 1969 three
Annual Plans were formulated.
Annual Plan ˜ Due to the unfortunate failure of the Third Plan, the production in various sectors of the economy became stagnant.
(1966-69) In 1966, the Government of India declared the devaluation of rupee, with a view to increase the exports of the
country. So, the Fourth Plan was postponed and 3 Annual Plans were implemented. Some of the economists called
this period, i.e. from 1966 to 1969, Plan Holiday.
Fourth Plan ˜ Objective was growth with stability and progressive ˜ First 2 years of the plan were successful with record
(1969-74) achievement of self-reliance. foodgrain production on account of Green Revolution.
˜ Laid special emphasis on improving the condition of ˜ Adoption of import-substitution policy and
under privileged and weaker sections. export-promotion policy widened the industrial base.
˜ Food security in gold was also one of its main goal. ˜ Targeted growth 5.7% however, achieved growth 3.3%.
˜ The objective is of correcting the earlier trend of ˜ The plan was failure on account of runaway inflation (due
increased concentration of wealth and economic to 1972 oil crisis or supply shock); huge influx of refugees
power. from Bangladesh post 1972 Indo-Pak War.

Fifth Plan ˜ Original approach to plan prepared by ˜ Targeted growth 4.4% and achieved growth 4.8%.
(1974-79) C Subramaniam, who proposed economic growth ˜ Fifth Plan cost calculations based on 1971-72, prices
alongwith direct attack on poverty. proved to be wrong.
˜ However, final draft prepared by DP Dhar with ˜ Fifth Plan terminated 1 year before the plan period in
objectives of removal of poverty (Garibi Hatao) and March, 1978.
attainment of self-reliance. ˜ Brought to the fore problem associated with coalition
˜ To step-up domestic rate of saving. government making a mockery of formulation of Five Year
˜ Introduction of minimum needs programme. Plan.

Rolling Plan Rolling plan (Gunnar Myrdal) was brought out by Janata Party Government under Morarji Desai in 1978. The focus of
(1978-80) the plan was enlargement of the employment potential in agriculture and allied activities to raise the income of the
lowest income classes through minimum needs programme. Annual Plan period was 1979-80.

Sixth Plan ˜ Removal of poverty through strengthening of ˜ Indian economy made an all round progress and most of
(1980-85) infrastructure for both agriculture and industry. the targets fixed by the plan was achieved.
˜ The emphasis was laid on greater management, ˜ Targeted growth 5.2%.
efficiency and monitoring of various schemes. ˜ Achieved growth 5.4%.
˜ Involvement of people in formulating schemes of
development at local level.
Seventh Plan ˜ To accelerate foodgrains production. ˜ Foodgrain production grew by 3.23% as compared to a
(1985-90) ˜ To increase employment opportunities. long-term growth rate of 2.68% between 1967-68 and
˜ To raise productivity. 1988-89.
˜ Outward looking strategy with gradual liberalisation
˜ The Indian economy finally crossed the barrier of the
over of economy. Hindu rate of growth of 3% given by Professor Raj
Krishna.
˜ Average annual growth rate was 6.0% as against the
targeted 5.0% and average of 3.5 % in the previous plans.
˜ It saw the beginning of liberalisation of Indian economy.
Annual Plan The Eighth Plan could not take off due to fast changing political situations at the centre. Therefore, from 1990-92,
(1990-92) Annual plans were formulated.
Magbook ~ Economic Planning in India 23

Plans Objectives Facts Assessments


Eighth Plan ˜ Process of fiscal reforms and economic reforms ˜ Higher economic growth rate of 6.8% achieved as against
(1992-97) initiated by Narasimha Rao Government to prevent the targeted 5.6%.
another major economic crisis. ˜ Improvement in trade and current account deficit.
˜ To increase the average industrial growth rate to ˜ Significant reduction in fiscal deficit.
7.5%. ˜ Agriculture growth and industrial growth increased.
˜ To provide a new dynamism of the economy and ˜ Unshackled private sector and foreign investment control
improve the quality of life of the common man. was the prime reason for high growth.
˜ Also called as Rao-Manmohan Singh model. ˜ Overall socio-economic development indicators low.
˜ First indicative plan. ˜ The growth became jobless and fruitless.

Ninth Plan ˜ Growth with social justice and equality. ˜ Global economic slowdown and other factors led to
Emphasis on Seven Basic Minimum Services (BMSs), revision of targeted growth rate from 7% to 6.5%, which
(1997-02) ˜

which included safe drinking water universalisation of too was not achieved.
primary education, streamlining PDS among others. ˜ The economy grew at 5.4% only.
˜ Pursued the policy of fiscal consolidation. ˜ Agriculture grew by 2.1% as against the target of 4.2%
˜ Decentralisation of planning with greater reliance on per annum.
states.
˜ Ensuring food and nutritional security to all.
˜ Empowerment of women, SC/STs/OBCs.

Tenth Plan ˜ The Tenth Plan aimed at achieving 8.1% GDP growth ˜ Increase in GDP growth to 7.6% compared to 5.5%
assuming that ICOR (Incremental Capital Output compared to 5.5% in the Ninth Plan. The lower than
(2002-07)
Ratio) will decline from 4.53% to 3.58%. targeted growth rate of 8% was due to low growth of 3%
˜ It aimed at increasing domestic saving rate from in the first year of Tenth Plan.
23.52% to 29.4% of GDP and gross capital ˜ Increase in gross domestic saving and investment.
formation to 32.2% from 24.4% of GDP. ˜ Reduction in ICOR to 4.2% though higher than targeted,
˜ To improve the overall framework of governance. but less than Ninth Plan’s ICOR of 4.53%.
˜ Agriculture was the core element. ˜ Increase in foreign exchange reserves to US $ 287 billion.
˜ However, Tenth Plan fared worst on socio-economic
indicators and the agricultural growth rate was meagre
2.1%.
Eleventh Plan ˜ Average GDP growth of 8.1% per year. ˜ The growth rate during the Eleventh Plan period was
(2007-12) ˜ Agricultural GDP growth of 4% per year. Generation about 7.9%, which is higher than the 7.8% growth rate
of 58 million employment opportunities. achieved in the Tenth Plan.
˜ Sex ratio for age group 0-6 years to be raised to 935 ˜ As against the target of 4% growth in the agriculture
by 2011-12 and to 950 by 2016-17. sector, the plan could register a growth of only 3% during
2007-12 period.
˜ The services sector continued to register a growth rate of
more than 10%. However, the industrial growth rate
showed at 7.9%.

Twelfth Five Year Plan (2012-17)


— The Finance Ministry has extended the time period for 12th plan schemes ending on 31st March by six months (i.e.,
September, 2017)
— The Approach Paper of the Twelfth Five Year Plan is concerned with the faster, sustainable and more inclusive growth. In
it, the challenge of urbanisation has been identified as one of the key focus area.
— In the opinion of Planning commission, the economy has gained in strength in many dimensions and is therefore, well
placed to achieve faster, sustainable and more inclusive growth. Having achieved 7.9% growth during the Eleventh Plan, it
is reasonable to aim at 8.0% growth for the Twelfth Plan.
— There are several imponderables, including considerable short-term uncertainties in the global economy and also
formidable supply constraints in energy and some other sectors on the domestic front. While there are possible downsides
to this scenario, we should aim at an average 8.2% GDP growth for the Twelfth Five Year Plan at this stage.
24 Magbook ~ Indian Economy

— In the Eleventh Plan, the achieved growth rate was 7.9%. — Connect all villages with all weather roads by the end of
As compared to the initial target of 9% and the revised Twelfth Five Year Plan.
target of 8.0%. The total plan size of Twelfth Plan is ` 37.7 — Upgrade national and state highways to the minimum
lakh crore, 13.7% more than the Eleventh Plan. two-lane standard by the end of Twelfth Five Year Plan.
— At the energy front, Planning commission acknowledged — Complete Eastern and Western dedicated freight corridors
that energy strategy for the Twelfth Plan requires a large by the end of 12th Five Year Plan.
number of actions by different ministries in the Central — Increase rural density to 70% by the end of Twelfth Five
Government plus action by State Governments in several Year Plan.
areas. — Ensure 50% of rural population has access to 55 LPCD
piped drinking water supply and 50% of Gram
Targets under Twelfth Plan Panchayats, achieve the Nirmal Gram status by the end
Economic Growth of Twelfth Five Year Plan.
— Real GDP Growth rate of 8.0%. (according to April, 2013) Environment and Sustainability
— Agriculture Growth rate of 4.0%. — Increase green cover (as measured by satellite imagery)
— Manufacturing Growth rate of 10.0%. by 1 million hectare every year during the Twelfth Five
— Every state must have a higher average growth rate in the Year Plan.
Twelfth Plan than that achieved in the Eleventh Plan.
— Add 30000 MW of renewable energy capacity in the Twelfth
Poverty and Employment Plan.
— Head-count ratio of consumption poverty to be reduced by — Reduce emission intensity of GDP in line with the target of
10 percentage points over the preceding estimates by the 20% to 25% reduction by 2020 over 2005 levels.
end of Twelfth Five Year Plan.
— Generate 50 million new work opportunities in the Service Delivery
non-farm sector and provide skill certification to equivalent — Provide access to banking services to 90% Indian
numbers during the Twelfth Five Year Plan. households by the end of Twelfth Five Year Plan.
— Major subsidies and welfare related beneficiary payments
Education
to be shifted to a direct cash transfer by the end of the
— Mean years of schooling to increase to 7 years by the end
Twelfth Plan, using the Aadhar platform with linked bank
of Twelfth Five Year Plan.
accounts.
— Enhance access to higher education by creating two
million additional seats for each age cohort aligned to the Alternative Scenarios During Twelfth Plan
skill needs of the economy. — To illustrate the consequences of inaction on key growth
— Eliminate gender and social gap in school enrolment (i. e. promoting policies, the Planning commission has
between girls and boys and between SCs, STs, Muslims and undertaken a systematic process of scenario planning
the rest of the population) by the end of Twelfth Five Year based on diverse views and disciplines to understand the
Plan. interplay of the principal forces, internal and external,
shaping India’s progress.
Health
— This analysis suggests three alternative scenarios of how
— Reduce IMR to 25 and MMR to 1 per 1000 livebirths and
India’s economy might develop titled, Strong Inclusive
improve Child Sex ratio (0-6 years), to 950 by the end of
Growth, Insufficient Action’ and ‘Policy Logjam.
the Twelfth Five Year Plan.
— Reduce total fertility rate to 2.1 by the end of Twelfth Five
Year Plan. Hindu Growth Rate
— Reduce under-nutrition among children aged 0-3 years to The concept of Hindu Growth rate was given by Professor
half of the NFHS-3 levels by the end of Twelfth Five Year Raj Krishna to refer to the stagnant growth rate of India till
Plan. 1970s. The word ‘Hindu’ in the term was used by some
early economists to imply that the Hindu outlook of
Infrastructure including Rural
fatalism and contentedness was responsible for the slow
Infrastructure growth. The argument was shattered in the 1990s, when
— Increase investment in infrastructure as a percentage of India achieved a growth rate of 6%+, in the context of the
GDP to 9% by the end of Twelfth Five Year Plan. economic reforms brought by Dr Manmohan Singh (then
— Increase the gross lrrigated area from 90 million hectare to Finance Minister). Thus, this new growth trajectory was
103 million hectare by the end of Twelfth Five Year Plan. termed as the Sardar Growth rate.
— Provide electricity to all villages and reduce AT and C
losses to 20% by the end of Twelfth Five Year Plan.
Magbook ~ Economic Planning in India 25
— In 2001, the Economic Advisory Council of the Prime Minister
Economic Reform advised on SGR like labour law flexibility, pension reform
based on employee contribution and pension funds being
First Generation Reform deployed in stock market, value added tax and GST, liberalised
— Since July, 1991, India has been taking up economic FDI including FDI in retail. SGR are difficult as they are directly
reform to achieve higher rate of economic growth so involved with the daily lives of the people like:
that socio-economic problems like unemployment, — user charges need to be rationalised.
poverty, regional imbalances can be successfully — manpower rationalisation in banks and PSUs through VRS.
solved. — interest rate cut e.g. on small savings will mean less return.
— It was realised that by closing economy to global
influences the country was missing on technology
development and also the gains from global trade.
India Vision 2020
India needs exports, FDI, FII for stability of balance of — Planning commission has released India Vision-2020 on 23rd
payment. Worldwide, countries were embracing market January, 2003, which presents pre-assessment of the progress
model of growth e.g. China, with proven result. So, of Indian economy for the next two decades. This document is
India could make the historical shift from centralised prepared by Mr Shyam Prasad Gupta, a member of Planning
planning to market based model of growth. commission. The salient points of the document are as follows:
— These reforms are relatively easier to implement as — The expected annual growth rate by 2020 to be 9%.
they do not involve legislative measures and can be — Elimination of unemployment, illiteracy and poverty by 2020.
implemented through executive action. Examples — Per Capita Income to get doubled by 2020.
include freeing up of exchange rate, allowing FDI and — Cent per cent registration of children (age group 6-14 years) in
FII, delicensing of certain sectors, disinvestment etc. schools.
These reforms focus on macro variables and seek to — 1.35 billion population of the country to have better living standard
achieve macroeconomic stability. by 2020.
— Environment situation to remain as unbalanced as present.
Second Generation Reforms — With 2% annual employment generation rate, 20 crores new
employment opportunities to be created by 2020.
— This term was introduced by Finance Minister, — Employment share in agriculture to come down from present 56%
Yashwant Sinha. Having begun with the reform in all to 40% by 2020.
sectors and seen the economy benefit from them. — Unorganised sector to create more additional employment
— The Second Generation reforms were initiated by the opportunities.
end of 1990’s. The reason of calling the later set of — Urban population percentage to get increased from existing 25% to
reform SGR is that they followed the initial reform, 40%.
which laid the foundation for the reform process to — Water problem in metropolitan cities to continue as such.
deepen.
Self Check
Build Your Confidence

1. Consider the following statements 7. Consider the following statements


1. NITI Aayog is structured to promote cooperative 1. NITI Aayog or National Institution for transforming India
federalism. Aayog is policy think-tank of government, that replaces
2. NITI Aayog is to centralise the planning process in Planning commission.
response to scarcity of resources. 2. NITI Aayog will provide strategic and technical advice to
Which of the statement(s) given above is/are correct? the State Government.
(a) Only 1 (b) Only 2 Which of the statement(s) given above is/are correct?
(c) Both 1 and 2 (d) Neither 1 nor 2 (a) Only 1 (b) Only 2
(c) Both 1 and 2 (d) Neither 1 nor 2
2. The main objective of the Twelfth Five Year Plan is
(a) inclusive growth and poverty reduction [UPSC 2014] 8. The term ‘Hindu rate of growth’ refers to the 3.70% per
(b) inclusive and sustainable growth annum growth rate achieved by the Indian economy over
(c) sustainable and inclusive growth to reduce unemployment the First Six Five Year Plans’. The term was coined by
(d) faster, sustainable and more inclusive growth (a) JN Bhagwati (b) KN Raj
(c) Raj Krishna (d) Sukhamoy Chakravarty
3. Consider the following statements
1. First Five Year Plan was based on Harrod Domar strategy. 9. Which one among the following Five Year Plan of India
2. Second Five Year Plan was based on economy diversified the planning shifting focus from
Nehru-Mahalanobis strategy. heary and basic industries to new sunrise industries like
Which of the statement(s) given above is/are correct?
telecommunication, computers, electronic?
(a) Only 1 (b) Only 2 (a) Second Five Year Plan (b) Fourth Five Year Plan
(c) Both 1 and 2 (d) Neither 1 nor 2 (c) Seventh Five Year Plan (d) Ninth Five Year Plan

4. In recent plans, certain words/phrases were used in the 10. Match the following
title of the plan along with ‘Growth’. They are
List I List II
1. Inclusive, 2. Faster, (Plan Model) (Proposer)
3. More Inclusive, 4. Sustainable, A. Bombay plan 1. Jai Prakash Narayan
5. More sustainable.
B. Gandhian plan 2. MN Roy
Which combination is true of the Twelfth Five Year plan? C. People’s plan 3. Sriman Narayana
(a) 1, 2 and 3 (b) 1, 4 and 5
D. Sarvodaya plan 4. Birla and Tata Group
(c) 2, 3 and 4 (d) 1, 2 and 4
Codes
5. Inclusive growth as enunciated in the Eleventh Five Year
A B C D A B C D
Plan does not include which one of the following? (a) 4 3 2 1 (b) 1 2 3 4
(a) Reduction of poverty (c) 3 2 4 1 (d) 1 4 2 3
(b) Extension of employment opportunities
(c) Reduction of gender inequality 11. The Rolling Plan concept in Nation planning was
(d) Strengthening of capital market introduced by [BPSC 2008]
(a) Indira Gandhi government
6. Consider the following statements (b) The National government
1. In the Eleventh Five Year Plan, the agriculture sector (c) The Janata Party government
contributed more than 25% in the overall GDP of the (d) Rajiv Gandhi government
India.
2. In the Twelfth Five Year Plan the growth rate of the
12. The Government of India has established NITI Aayog to
replace the [UPSC 2015]
agriculture sector was above 4%.
Which of the statement(s) given above is/are correct? (a) Human Rights Commission
(b) Finance Commission
(a) Only 1
(c) Law Commission
(b) Only 2
(d) Planning Commission
(c) Both 1 and 2
(d) Neither 1 nor 2
1. (a) 2. (d) 3. (c) 4. (c) 5. (a) 6. (d) 7. (a) 8. (c) 9. (c) 10. (a)
11. (c) 12. (d)
Chapter five
Money and Banking
M1 = Currency with the public +
Money —
Demand deposits with the Banking
Financial market — Fiat money, derives its value by being declared System + Other deposits with the
provides channels for by a government to be legal tender; i.e. it must RBI.
be accepted as a form of payment within the — M2 = M1 + Saving deposits of Post
allocation of saving to boundaries of the country, for all debts, public Office Savings Banks.
investment. The financial and private. — M3 = M1 + Time deposits with the
market thus, contribute — The money supply of a country consists Banking System.
to economic currency (bank notes and coins) and bank — M4 = M3 + Office Savings of Banks.
development to the money (the balance hold in checking accounts — The decreasing order of liquidity of
and savings accounts). these monetary aggregates is
extent that latter
— Bank money, which consists only of records M0 > M1 > M2 > M3 . The decline in
depends on the rates of (mostly computerised in modern banking), liquidity indicates the shifting of
saving investment. forms by far the largest part of the money ‘medium of exchange ‘to’ store of
supply in developed nations. value’.
— Demand deposits are those deposits
Measures of Money Supply payable by the bank on demand by
in India a customer like current and savings
account.
— Money supply is the stock of liquid assets held
by the public which can be freely exchanged
for goods and services. Indian Currency
— RBI calculates four concepts of money supply. Symbol ( `)
These are known as measures of monetary ◆
The symbol of Indian rupee ` came
aggregates or money stock measures. into use on 15th July, 2010. India is
— The working group under the Chairmanship of the fifth economy (after America,
Dr YB Reddy, then Deputy Governor of RBI Britain, Japan and European Union to
(now Former Governor of RBI) has suggested accept a unique currency symbol.
four new monetary measures. ◆
The new symbol designed by D
(M0 , M1, M2 , M3 ) and three liquidity measures Udaya Kumar, a post graduate of
(L, L 2 , L 3 ). Besides, the group also IIT Mumbai was finally selected by
recommended the publishing of Financial the Union Cabinet on 15th July,
Sector Survey ‘A Monetary Aggregates’, every 3 2010.
months. ◆
The new symbol, is an amalgamation
— M0 = Currency in circulation + Banker’s deposit of Devanagari ‘Ra’ and the
with RBI + Other deposit with RBI. Roman ‘R’ without the stem.
28 Magbook ~ Indian Economy

Liquidity Aggregates — It is a market for short-term funds with maturity ranging


from overnight to 1 year and includes financial
— L 1 = M 3 + All Deposits with the Post Office Savings
instruments that are deemed to be close substitutes of
Banks (excluding National Savings Certificates).
money.
— L 2 = L 1 + Term Deposits with Term Lending Institutions
and Refinancing Institutions (FIs) + Term Borrowing Functions of Money Market
by FIs + Certificates of Deposit issued by FIs; and The money market performs three broad functions are as
— L 3 = L 2 + Public Deposits of Non-Banking follows
Financial Companies. (i) It provides an equilibrating mechanism for demand and
supply of short-term funds.
Financial Sector (ii) It enables borrowers and lenders of short-term funds to
fulfil their borrowing and investment requirements at an
— Financial Sector of a country is one of the important
efficient market clearing price.
determining factors of the level of economic development.
(iii) It provides an avenue for Central Bank intervention in
Sound financial system induces the level of savings and
influencing both quantum and cost of liquidity in the
investment, thus, working as a stimulant for the
financial system, thereby transmitting monetary policy
development variables.
impulses to the real economy.
— Weak financial structure certainly hinders the tempo of
development process by discouraging the developmental — Efficient functioning of the money market is important for
variables. Thus, rapid economic development requires a the effectiveness of monetary policy.
sound financial system with adequate availability of finance
Regulation of Indian Money Market
and a strong system of associated financial and investment
institutions. — Indian money market is broadly divided into two
parts-organised and unorganised.
Financial sector in India comprises of Financial Intermediaries or
Financial Institutions are as follows: — The RBI is the Apex Organisation in the Indian Money
— Financial Institutions are the institutions, which are primarily Market. It carries out regulation and development of the
engaged in the collection and mobilisation of savings and Indian money market through instruments such as call ,
convert into investment. Financial institutions include all banks notice or term money market, repo market, certificate of
and non-banking financial institutions. deposit, commercial paper and Collateralised Borrowing
— Financial Markets are the markets, which are engaged in the and Lending Obligation (CBLO).
collection of savings from the surplus units of the economy and
lending to the deficit units of the economy for the investment Organised Money Market
and other purposes. Financial market consists of money market
and capital market. Call Money Market
— Financial Assets include unit shares, debentures, certificate of — The call or notice money market forms an important
deposits, life insurance policies etc. These are the instruments segment of the Indian money market.
traded in the financial markets through financial intermediaries. — Call or notice money is an amount borrowed or lent on
demand for a very short period. If the period is greater
Financial Markets than 1 day and up to 14 days, it is called the notice
— Financial market is an important part of financial sector. money; otherwise the amount is known as call money.
Financial market is that market, where financial transactions No collateral security is needed to cover these
take place. On the basis of short-term and long-term transactions.
transactions, such markets are classified as into money — The call market enables the banks and institutions to even
market and capital market. out their day-to-day deficits and surpluses of money.
Cooperative Banks, Commercial Banks and primary
Money Market dealers are allowed to borrow and lend in this market for
— The cluster of financial institutions that deal in short-term adjusting their cash reserve requirements.
securities and loans, gold and foreign exchange are termed as — This is a completely inter-bank market. Interest rates are
money market. Money has a time value and therefore, the use market determined. In view of the short tenure of these
transactions, both borrowers and lenders are required to
of it, is bought and sold against payment of interest.
have current accounts with Reserve Bank of India.
Short-term money is bought and sold on the money market
and long-term money on the capital market. Banker’s Acceptance Market
— Neither the money market nor the capital market exists in — A Banker’s Acceptance (BA) is a short-term credit

one physical location. The money market is a key investment created by a non-financial firm and
component of the financial system, as it is the function of guaranteed by a bank to make a payment. Acceptances
monetary operations conducted by the Central Bank in its are traded at discounts from face value in the secondary
pursuit of monetary policy objectives. market.
Magbook ~ Money and Banking 29

— One advantage of a banker’s acceptance is that it does — These bills are called trade bills. These trade bills are called
not need to be held until maturity and can be sold off commercial bills, when they are accepted by Commercial
in the secondary markets, where investors and Banks. If the bill is payable at a future date and the seller needs
institutions constantly trade BAs. money during the currency of the bill, the seller may approach
the bank for discounting the bill.
Collateral Loan Market
— The banks discount this bill by keeping a certain margin and
— In this market, loan is often secured against collateral
credits the proceeds. Banks, when in need of money, can also
security. Security may be in any form viz pledge,
get such bills rediscounted by financial institutions such as LIC,
mortgages etc. Thus, the market for loans secured by
UTI, GIC, ICICI and IRBI.
collateral security is called the collateral loan market.
The maturity period of the bills varies from 30 days, 60 days or
Treasury Bill Market 90 days, depending on the credit extended in the industry.
— Treasury bills are money market instruments to finance Indian Money Market
the short-term requirements of the Government of
India. These are discounted securities and thus, are
issued at a discount to face value. The return of the Organised Banking Unorganised Banking Sub-Markets
investor is the difference between the maturity value Sector Sector
and issue price.
Call Money Bill Market 364 day T-Bills Certificates Commercial
— The market that deals with treasury bills is called of Deposits (CoDs) Papers
Market
treasury bill market. These are the lowest risk category
instruments for the short-term. RBI issues treasury bills
[T-bills] at a prefixed day and for a fixed amount. Commercial Bills Treasury Bills (90-days)
— There are four types of Treasury Bills: Organisation of Indian Money Market
(i) 14 Days T-Bill It was introduced in 1997, by the RBI. Certificates of Deposits Market
Maturity is in 14 days, it is auctioned on every Friday — After treasury bills, the next lowest risk category investment
of every week and the notified amount for auction is option is Certificate of Deposit (CD) issued by banks and
` 100 crore. Financial Institution (FI).
(ii) 91Days T-Bill Maturity is in 91 days, it is auctioned — Allowed in 1989, CDs were one of RBI’s measures to
on every Friday of every week and the notified amount deregulate the cost of funds for banks and FIs.
for auction is ` 100 crore. A (CD) is a negotiable promissory note, secure and short-term,
(iii) 182 Days T-Bill Maturity is in 182 days, it is of upto a year, in nature.
auctioned on every alternate Wednesday, which is not a Repo Market
reporting week and the notified amount for auction is
— Repo is a money market instrument which helps in
` 100 crore. It was introduced on the
collateralised short-term borrowing and lending through sale or
recommendations by Vaghul Working Group.
purchase operations in debt instruments.
(iv) 364 Days T-Bill Maturity is 364 days, it is auctioned on — Initially repos were allowed in Central Government treasury
every alternate Wednesday, which is a reporting week bills and dated securities created by converting some of the
and the notified amount for the auction is ` 500 crore. It treasury bills, RBI gradually allowed repo transactions in all
was also recommended by Vaghul Working Group. government securities and T-bills of all maturities and now
— In recent times, RBI has been issuing only 91 days State Government Securities, PSU’s bonds, private corporate
and 364 days T-Bills. securities have also been made eligible for repos to broaden
— These are bought by the Reserve Bank, Commercial the repo market.
Banks, non-banking financial intermediaries, LIC, UTI Money Market Mutual Funds (MMMFs)
and GIC. Treasury bills are most liquid, because — The scheme was introduced by RBI in April, 1992 with the
Reserve Bank is always ready to buy and discount objective of providing an additional short-term avenue to the
them. individual investors. They have now been brought under the
purview of SEBI since March, 2000.
Commercial Bill Market
— It is the market that deals in commercial bills. Commercial Paper Market
Commercial bills of exchange are negotiable — Commercial Papers (CPs) are negotiable short-term unsecured
instruments drawn by the seller or drawer of the goods promissory notes with fixed maturities, issued by well-rated
on the buyer or drawer of the good for the value of the organisations. These are generally sold on discount basis.
goods delivered. — Organisations can issue CPs either directly or through banks
or merchant banks (called as dealers).
30 Magbook ~ Indian Economy

Unorganised Money Market Besides helping diversify funding sources, the cost of
borrowing could also turn out to be lower than domestic
— The sector consists of unregulated non-bank financial
markets. In 2013, the first masala bonds were issued
intermediaries such as money lenders Chit funds, Nidhis etc.
by the International Finance Corporation (IFC), an arm
— Chit funds are savings institutions. They are of various types
of the World Bank. IFC then named them Masala bonds
and don’t have any standardised form. Chit funds have
to give a local flavour by calling to mind Indian culture
regular members, who make periodic contributions.
and cuisine.
— At periodic intervals funds are given to a member based on — Masala bond will help the Indian corporates to reduce
a pre determined criterion, usually on the basis of bids or
its interest cost burden on the debt amount on its
draw of lots. All members are assured of their turn before
balance sheet. The more of foreign funds can be used
the round ends.
for infrastructural development in the country. Overall,
— Chit funds are prevalent in almost all states, but Kerala and the development of a Masala bond market would be
Tamil Nadu account for the major part. They exist in both positive for Indian firms, opening up potentially
organised and unorganised form. significant new sources of funding over External
— Organised Chit funds are regulated by registrar of Chit funds Commercial Borrowings (ECBs).
and the relevant legislation in this regard is the Chit Funds
Act, 1982. There is however, regulatory confusion since Capital Market
Collective Investment Schemes (CIS) are to be registered — It is one of the most important segments of the Indian
and regulated by SEBI. Many Chit funds take advantage of financial system. It is the market available to the
the regulatory loopholes. companies for meeting their requirements of the
— Nidhis are a kind of mutual benefit funds. Their dealings are long-term funds. These are markets for buying and
restricted to members only and they operate in the selling equity and debt instruments.
unregulated credit market. — The market consists of a number of individuals and
— Deposits mobilised by them are not much. Their principal institutions (including the government) that channelise
source of funds is from the members and they provide loans the supply and demand for long -term capital and
to members at relatively reasonable rates and are secured. claims on it.
— Money lenders and loan companies are present all across — The demand for long-term capital comes predominantly
the country. They generally give loans to wholesale traders, from private sector manufacturing industries,
artisans and other self-employed persons. They charge high agriculture sector, trade and the government agencies,
rates of interest from 26% to 48% and 50 people who while the supply of funds for the capital market comes
approach them are generally unable to get loans from largely from individual and corporate savings, banks,
Commercial Banks. insurance companies, specialised financing agencies
Promissory Note and the surplus of governments. The Indian capital
market is broadly divided into the Industrial Securities
— It is a legal document between a lender and a borrower,
Market and Gilt-edged Market.
whereby the latter agrees to certain conditions for the
repayment of the sum of money borrowed.
(i) Industrial Securities Market
— Promissory note is signed when one borrows from a
— The industrial securities market refers to the market,
Commercial Bank.
which deals in equities and debentures of the
— Particular forms of promissory notes, known as commercial corporates. It is further divided into primary market and
paper, can be bought and sold. secondary market.
Dated Government Securities Primary Market
— These are securities issued by the Government of India and — Primary market (new issue market) deals with new
State Governments. The date of maturity is specified in the securities, i.e. securities, which were not previously
securities, therefore, they are known as dated securities. available and are offered to the investing public for the
Masala Bond first time. It is the market for raising fresh capital in the
form of shares and debentures.
— Masala bonds are rupee denominated overseas bonds.
Masala bonds will help to internationalise the Indian rupee — It provides the issuing company with additional funds
and also deepen the Indian financial system( Public and for starting a new enterprise or for either expansion or
Private Sector). By issuing bonds in rupees, an Indian diversification of an existing one and thus, its
company is shielded against the risk of currency fluctuation, contribution to company financing is direct. The new
typically associated with borrowing in foreign currency. offerings by the companies are made either as an Initial
Public Offering (IPO) or rights issue.
Magbook ~ Money and Banking 31

Secondary Market/Stock Market — In particular, it is responsible for


— Secondary Market or Stock Market (old issues market or — institutional reforms in the securities markets.
stock exchange) is the market for buying and selling — building regulatory and market institutions.
securities of the existing companies. Under this, securities — strengthening investor protection mechanism and
are traded after being initially offered to the public in the — providing efficient legislative framework for securities markets,
such as Securities and Exchange Board of India Act, 1992
primary market and listed on the stock exchange.
(SEBI Act, 1992); Securities Contracts (Regulation) Act, 1956
— The stock exchanges are the exclusive centres for trading and the Depositories Act, 1996.
of securities. It is a sensitive barometer and reflects the
trends in the economy through fluctuations in the prices Securities and Exchange Board of
of various securities. India (SEBI)
(ii) Gilt-Edged Market — It is the regulatory authority established under the
SEBI Act, 1992, in order to protect the interests of the
— The gilt-edged market refers to the market for
investors in securities as well as promote the development
government and semi-government securities, backed by
of the capital market.
the Reserve Bank of India (RBI). Government securities
— It involves regulating the business in stock exchanges
are tradeable debt instruments issued by the government
supervising the working of stock brokers, share transfer
for meeting its financial requirements.
agents, merchant bankers, underwriters etc as well
— The term gilt-edged means ‘of the best quality’. This is as prohibiting unfair trade practices in the securities market.
because the government securities do not suffer from risk
— The main functions of SEBI are as follows:
of default and are highly liquid (as they can be easily sold
— To regulate the business of the stock market and other
in the market at their current price). The open market
securities market.
operations of the RBI are also conducted in such — To promote and regulate the self-regulatory organisations.
securities. — To prohibit fraudulent and unfair trade practices in securities
market.
Other Instruments of Capital Market — To promote awareness among investors and training of
intermediaries about safety of market.

Derivatives the term derivative indicate that it has no
— To prohibit insider trading in securities market.
independent value i.e. its value is entirely derived from an
— To regulate huge acquisition of shares and takeover of
underlying asset. The underlying asset can be securities,
companies.
commodities, currency etc. Derivative is a forward future
option or any other hybrid contract of fixed duration, linked
for the purpose of contact fulfillment to the value of an asset. Reforms in Capital Market

Futures Contract means a legally binding agreement to buy or of India
sell the underlying security on a future date. — The capital market has witnessed major reforms in the

Options Contract is a type of derivatives contract which gives 1990s and thereafter. It is on the average of growth. Thus,
the buyer or holder of the contract the right (but not the the Government of India and SEBI have taken a number of
obligation) to buy or sell the underlying asset at a measures in order to improve the working of the Indian
pre-determined price within or at the end of a specified Stock Exchanges and to make it more progressive and
period ‘call’ and ‘put’ are the two types of options contract. vibrant.

Forwards Contract is also an agreement to buy or sell an asset The major reforms undertaken include are as follows:
on a future date. However, unlike a futures contract it is — Credit Rating Agencies Three credit rating agencies viz the
between two private parties and there is no guarantee of Credit Rating Information Services of India Limited
fulfillment of the contract. (CRISIL-1988), the Investment Information and Credit Rating
Agency of India Limited (ICRA - 1991) and Credit Analysis and
Research Limited (CARL) were set-up in order to assess the
Regulatory Framework financial health of different financial institutions and agencies
related to the stock market activities.
— In India, the capital market is regulated by the Capital
Markets Division of the Department of Economic Affairs, — Merchant Banking Activities Many Indian and foreign
Commercial Banks have set-up their merchant banking
Ministry of Finance.
divisions in the last few years. It has proved as a helping hand
— The division is responsible for formulating the policies for the factors related to the capital market.
related to the orderly growth and development of the — Growth of Electronic Transactions Due to technological
securities markets (i.e. share, debts and derivatives) as development in the last few years, the physical transaction
well as protecting the interests of the investors. with more paper work is reduced. It saves money, time and
energy of investors.
32 Magbook ~ Indian Economy

— Growing Mutual Fund Industry The growing of mutual funds in India Secondary Market
has certainly helped the capital market to grow. A big diversification in
— Rajiv Gandhi Equity Savings Scheme On 23rd
terms of schemes, maturity etc has taken place in mutual funds in
India. It has given a wide choice for the common investors to enter the November, 2012, the government notified a new
capital market. tax saving scheme called the Rajiv Gandhi Equity
— Growing Stock Exchanges Initially, the BSE was the main exchange, but Savings Scheme (RGESS) exclusively for first-time
now after the setting up of the NSE and the OTCEI, stock exchanges retail investors in the securities market.
have spread across the country. Recently, a new Inter-connected Stock — The scheme provides a 50% deduction of
Exchange of India has joined the existing stock exchanges. amount invested during the year, upto a
— Investor’s Protection Under the purview of the SEBI, the Central maximum investment of ` 50,000 per financial
Government of India has set-up the Investors Education and Protection year, from his/her taxable income for that year, for
Fund (IEPF) in 2001. It works in educating and guiding investors and to three consecutive assesment years.
protect the interest of the small investors from frauds and also
malpractices in the capital market.
— Growth of Derivative Transactions Since, June 2000, the NSE has Qualified Foreign
introduced the derivatives trading in the equities. These
innovative products have given various options for investment leading to
Investor (QFI)
the expansion of the capital market. — QFIs shall mean a person who fulfills the following
criteria are as follows:
New Law of SEBI Resident in a country that is a member of the
— In August 2014, the Securities Laws (Amendment) Act, 2014, gave Financial Action Task Force (FATF) or a country
SEBI additional powers, including to order the arrest of violators that is a member of a group which is a member
and seek call data records of individuals under investigation. of FATF.
— The new law gave SEBI the powers to search and obtain — Resident in a country that is a signatory to
information, including call records, about any suspected entity IOSCOs MoU (Appendix a Signatories) or a
from within or outside the firm. signatory of a bilateral MoU with Securities and
Exchange Board of India (SEBI). A QFI should
Forward Markets Commission with SEBI
neither be a person resident in India nor should
— The government notified on 1st September, 2015 the merger of be registered with the SEBI as a Foreign
commodities market regulator Forward Markets Commission (FMC) Institutional Investor sub-account or Foreign
with SEBI with effect 28th September, 2015. Venture Capital Investor. A QFI should be set-up
— As a result, of this notification Foreign Contribution Regulation Act, with a SEBI registered Qualified Depository
1952 will get repealed and regulation of commodity derivatives Participant (QDP) to commence activities. The
market will shift to the Securities and Exchange Board of India QDP shall provide inter alia custody services.
(SEBI) under Securities Contracts Regulation Act (SCRA) 1956
Recent Initiatives for Further Development of
with effect from 28th September, 2015..
Corporate Bond Markets
— The commission allows commodity trading in 22 exchange in
— To permit banks to take limited membership in
India, of which 6 are national. Currently, there are three national
SEBI-approved stock exchanges for the purpose
and six regional bourses for commodity futures in the country.
of undertaking proprietary transactions in the
corporate bond markets.
New Policy Initiatives — To enhance liquidity in the corporate bond
— In the overall context of the evolving macro-economic situation in markets the Insurance Regulatory and
the country and global financial developments, the government in Development Authority of India (IRDAI) has
close collaboration with the RBI and SEBI has recently taken a permitted insurance companies to participate in
number of initiatives to meet the growing capital needs of the Indian the repo market. The IRDAI has also permitted
economy. insurance companies to become users of Credit
Some of the initiatives are as follows: Default Swap (CDS).
Primary Market — Mutual funds have been permitted to participate
in CDS in corporate debt securities, as users.
— SEBI (Alternative Investment Funds) Regulations, 2012 With a
view to extending the reach of regulation to unregulated funds, — Revised guidelines on CDS for corporate bonds
ensuring systemic stability, increasing market efficiency, by the RBI provide that in addition to listed
encouraging new capital formation and providing investor corporate bonds, CDS shall also be permitted on
protection, SEBI has notified new regulations covering Alternate unlisted, but rated corporate bonds even for
Investment Funds (AIFs). issues other than infrastructure companies.
Magbook ~ Money and Banking 33

— CDS shall be permitted on securities with original maturity up ◆


Guwahati Stock Exchange, Guwahati
to one year like CPs, certificates of deposit and ◆
Hyderabad Stock Exchange, Hyderabad
non-convertible debentures with original maturity less than 1 ◆
Jaipur Stock Exchange, Jaipur
year as reference or deliverable obligations. ◆
Ludhiana Stock Exchange, Ludhiana
Stock Exchanges in India ◆
Chennai Stock Exchange, Chennai
— Bombay Stock Exchange (BSE), the oldest stock exchange in

MP Stock Exchange, Indore
Asia, was established in 1875. It is synonymous with Dalal ◆
Pune Stock Exchange, Pune
Street. ◆
Interconnected State Exchange of India Limited
— BSE was corporatised and renamed BSE Limited in 2005. In Commodity Exchanges
1894, the Ahmedabad Stock Exchange was started to
facilitate dealing in the shares of textile mills.

Multi Commodity Exchange of India Limited (MCX)
— In 1908, Calcutta Stock Exchange was started to facilitate

National Commodity and Derivatives Exchange Limited
market for shares of plantations and jute mills. At present,
(NCDEX)
there are 22 stock exchanges in the country. Two types of

Indian National Multi-Commodity Exchange (NMCE)
transaction take place in stock exchanges. These are as ◆
Indian Commodity Exchange Limited (ICEX)
follows :
— Investment Transaction Sale or purchase of securities National Stock Exchange (NSE)
undertaken with the long-term prospect relating to their yield and — NSE was promoted by leading financial institutions at
price. the behest of the Government of India and was
— Speculative Transaction Sale or purchase of securities incorporated in November, 1992, as a tax-paying
undertaken with short-term gain from differences in yield and
company unlike other stock exchanges in the country.
price. In this, delivery of securities or the payment of full price is
rare. — On the basis of the recommendations of high powered
Pherwani Committee, the National Stock Exchange was
— Speculative transaction of different types are as follows:
incorporated in November, 1992. In April 1993, it was
— Spot Transaction involves delivery of and payment for securities
recognised as a stock exchange and commenced
on the same day.
operations in 1994. In October 1995, NSE became
— Cash Transaction are ready delivery transaction, wherein delivery
largest stock exchange in the country.
of and payment for securities is completed within a period of one
to 7 days. — Trading at NSE can be classified under two broad
— Forward Transaction involves delivery of and payment for categories:
securities will be made on certain fixed settlement days, coming (i) Wholesale debt categories
once in 15 or 30 days.
(ii) Capital market
— On the recommendation of the Narasimham Committee, SEBI
was given the power to control and regulate the new issues — Wholesale debt market operations are similar to money
market as well as stock exchange through Amendment of the market operations, where institutions and corporate
Capital Issues Control Act, 1947. bodies enter into high value transactions in financial
Approved Stock Exchanges in India instruments such as government securities, treasury
bills, public sector unit bonds, commercial paper,

UP Stock Exchange, Kanpur
certificate of deposit etc.

Vadodara Stock Exchange, Vadodara
— NSE has several advantages over the traditional trading

Coimbatore Stock Exchange, Coimbatore exchanges.

United Stock Exchange of India Limited They are as follows:

Bombay Stock Exchange, Mumbai — NSE brings an integrated stock market trading network

Over the Counter Exchange of India, Mumbai across the nation.

National Stock Exchange, Mumbai — Investors can trade at the same price from anywhere in the
country since inter-market operations are streamlined

Ahmedabad Stock Exchange, Ahmedabad coupled with the countrywide access to the securities.

Bangalore Stock Exchange, Bengaluru — Delays in communication, late payments and the

Bhubaneswar Stock Exchange, Bhubaneswar malpractices prevailing in the traditional trading
mechanism can be done away with greater operational

Calcutta Stock Exchange, Kolkata efficiency and informational transparency in the stock

Cochin Stock Exchange, Cochin market operations with the support of total computerised

Delhi Stock Exchange, Delhi network.
34 Magbook ~ Indian Economy

Bombay Stock Exchange (BSE) Stock Market Indices


— Established in 1875, BSE Limited (formerly known as A Stock Market Index is created by selecting a group of stock that are
Bombay Stock Exchange Limited), is Asia’s first stock representative of the whole market of a specified sector or segment of
exchange and one of India’s leading exchange the market. An index is calculated with reference to a base period and
groups. Over the past 137 years, BSE has facilitated a base index value. An index is used to give information about the
the growth of the Indian corporate sector by providing price movements of products in the financial commodities or any other
it an efficient capital raising platform. markets. Financial indexes are constructed to measure price
— Some indices of BSE are given below: movements of stocks, bonds, T-bills and other forms of investments.
— SENSEX — MIDCAP Stock market indexes are meant to capture the overall behaviour of
— SMLCAP — BSE-100 equity markets. Stock market indices are useful for a variety of
— BSE-200 — BSE-500 reasons. Some of them are
— Around 5000 companies are listed on BSE making it They provide a historical comparison of returns on money invested in
world’s number one exchange in terms of listed the stock market against other forms of investments such as gold or
members. BSE limited is world’s 5th most active debt.
exchange in terms of number of transactions handled ◆
They can be used as a standard, against which to compare the
through its electronic trading system. It is also one of performance of an equity fund.
the world’s leading exchange (5th largest in May, ◆
It is a lead indicator to the performance of the overall economy or a
2012) for Index options trading (Source-World sector of the economy.
Federation of Exchanges). ◆
Stock indexes reflect highly up to date information.
— BSE is the first exchange in India and second in the ◆
Modern financial applications such as Index funds, index futures,
world to obtain an ISO 9001:2000 certification.
index options play an important role in financial investment and risk
— BSE’s popular equity index the SENSEX is India’s most management.
widely tracked stock market benchmark index. It is
traded internationally on the EUREX as well as leading SENSEX
exchanges of the BRCS nations (Brazil, Russia, China
and South Africa).
(The Barometer of Indian Capital Market)
— The Bombay Stock Exchange launched BSE
— BSE sensitive index also referred to as BSE-30 is a free float
market capitalisation-weighted stock market index of 30 well
Carbonex, the first carbon based thematic index in
established and financially sound companies listed in Bombay
the country. Which takes a strategic view of
Stock Exchange.
organisational commitment to climate change
mitigation. This index has been launched with the
— The free float market capitalisation of a company is determined
aim of creating a benchmark and increasing by multiplying, the price of its stocks by the number of shares
issued by a company, which is readily available for trading on
awareness about the risks posed by climate change.
the stock exchange.
— It will enable investors to track performance of the
constituent companies of BSE-100 index regarding their
— The base year or period of SENSEX was 1978 − 79 = 100. The
commitment to greenhouse gases emission reduction. calculation of SENSEX involves dividing the free float market
capitalisation of 30 companies in the index by a number called
MCX-SX the index divisor.

(MCX Stock Exchange Limited) Over The Counter Exchange of India


— It is a private stock exchange headquartered in (OTCEI)
Mumbai, which was founded in 2008. It offers
— Traditionally, trading in stock exchanges in India followed a
currency futures contracts for US Dollar-Rupee,
conventional style, where people used to gather at the exchange
Euro-Rupee, British Pound-Rupee, Japanese
offices and bids and offers were made by open outcry. This
Yen-Rupee.
old-age trading mechanism in the Indian Stock Markets used to
— It offers electronic trading platform in currency create many functional inefficiencies.
futures contracts. The exchange received — Lack of liquidity and transparency, long settlement periods and
permissions to deal in interest rate derivatives, equity, benami transactions are a few examples that adversely affected
futures and options on equity and wholesale debt
investors. In order to overcome these inefficiencies, OTCEI was
segment, video SEBI’s letter dated 10th July, 2012.
incorporated in 1990, under the Companies Act, 1956. OTCEI
is the first screen based Nationwide Stock Exchange in India.
Magbook ~ Money and Banking 35

The exchange is ranked behind the Multi-Commodity


Reference Rates-MIID, MIBOR Exchange (MCX) and the National Commodity & Derivative

A reference rate is an accurate measure of the market price. Exchange (NCDEX). ICEX Limited is a screen based on-line
In the fixed income market, it is an interest rate that the derivatives exchange for commodities and has been
market respects and closely watches. It plays a useful role in established on August 28, 2015.
a variety of situations.

NSE had developed MIBID (Mumbai Inter Bank Bid Rate) Other Investment Agents
and MIBOR (Mumbai Inter Bank Offer Rate) for the overnight — Venture Capital Fund Its a type of fund, which provides
market. It was launched in 1998. They are the reference rates. early stage, high risk, high potential growth start-up
Then, NSE launched the 14 days MIBID or MIBOR and then the companies, their funding requirements. The funding is
1 month and the 3 months MIBOR and MIBID. Thus, all the based on the hope that a successful venture will provide
four categories of MIBOR and MIBID are now available. super-normal returns. The funding is usually made after the

It is the simple average of the quotes by the various ‘seed funding’ round and in the ‘growth funding round’, that
participants in the market-banks, PDs, institutions polled on a is at the stage, in which companies just starts to grow, but
daily basis. after it has created a certain base.

LIBOR (London Inter-Bank Offered Rate) It is the average of — Angel Investor It is an individual with large financial
interest rates provided by leading banks in London that they resources, who provides capital to a business start-up in
would be charged if borrowing from other banks. It is used as exchange of equity or convertible debt. They provide the
a global benchmark interest rate by many banks around the initial seed money or ongoing support to the company to
world. carry it through difficult times. They are more interested in
helping the business succeed than making super-profits
Commodity Futures Market and in this sense, they are different from venture capital.
Usually, Angel Investors belong to the entrepreneur family
— Commodities traded on the commodity futures market or friends.
during 2009, included a variety of agricultural — Hedge Fund It is an aggressively managed portfolio of
commodities, bullion, crude oil, energy and metal investments that uses advanced investment strategies
products. Agricultural commodities, bullion and energy such as long, short, leveraged and derivative positions in
products accounted for a large share of the commodities both domestic and international markets to generate high
traded in the commodities future market. returns. Hedge funds are typically not open for investment
— The Central Government has announced a decision to to the general public and only accept investments from a
merge the commodities market regulator, Forward Markets small group of people, who can provide very large initial
Commission (FMC), with the capital markets regulator, minimum investment.
Securities and Exchange Board of India (SEBI). — Bank Sathi It enables the beneficiaries in rural areas to
make deposits and withdrawals. This facility is available in
National Multi-Commodity branchless areas of Bank.
Exchange — Participatory Notes are financial instruments used by
National Multi Commodity Exchange (NMCE) will merge with wealthy investo to diversify their investments. These are
Indian Commodity Exchange (ICEX) creating the country’s third not regulated and usually forms part of Hedge Funds.
biggest commodity exchange. The ICEX will hold a 62.8% stake — Sovereign Wealth Fund (SWF) is a pool of money derived
in the merged entity, while NMCE shareholders will own the rest from a country’s financial resources. For the purpose of
(37.2%). investing in areas, which will benefit the country’s
The merger has been approved by the boards of both economy and people in the future. e.g. India could start a
exchanges and is expected to be completed by December, SWF to invest in energy resources abroad, so as to
subject to regulatory approvals. This is the first merger deal in safeguard our energy security for the future.
the commodity exchange space in India.
— Mutual Funds It is an investment, vehicle, which is made
The NMCE was launched on November 26, 2002 as India’s up of a pool of funds collected from many investors for
first online, demutualised commodity exchange by a group of
the purpose of investing in securities such as stocks,
Indian commodity-based corporations and public agencies.
bonds, money market instruments and similar assets.
The NMCE is India’s third-largest commodity exchange
Mutual funds are managed by professional fund
offering contracts on oils and oil seeds, coffee, rubber and
managers.
spices.
36 Magbook ~ Indian Economy

— Main functions of RBI are as follows:


Portfolio Investment — Maintaining monetary stability so that business and economic
In investment terminology, portfolio refers to a collection of life can deliver welfare gains of a properly functioning mixed
financial assets such as stocks, bonds, cash etc with any individual economy.
investor, hedge fund, bank etc. Portfolios are designed to serve the — Maintaining financial stability and ensuring sound financial
objectives of investors such as relating to risk, rate of return on institutions so that monetary policy can be safely pursued
investment etc. Based on these objectives, the portfolio can have and economic units can conduct their business with
different ratios of cash, stock, bond etc. confidence.
— Maintaining a stable payments system so that financial
transaction can be safely and efficiently executed.

Banking in India — Promoting the development of financial infrastructure in


terms of markets and systems and to enable it to operate
— History of Indian banking goes back to 19th century it efficiently, i.e. playing a leading role in developing a sound
failed. First successful bank in India was Bank of Bengal financial system so that it can discharge its regulatory
set-up in 1806. First Commercial Bank in country was function efficiently.
Awadh Commercial Bank established in 1881. — Ensuring that credit allocation by the financial system
broadly reflect the national economic priorities and societal
— In 1921, Imperial Bank, of limited liability of India was concerns.
set-up. There were two important steps in the banking — Regulating the overall volume of money and credit in the
sector after independence in 1949. Nationalisation of economy with a view to ensuring a reasonable degree of
Reserve Bank of India and the Banking Regulation Act, price stability.
which empowered RBI to regulate banking sector in
country. Role of the RBI
— The Punjab National Bank, established in Lahore in 1895, RBI plays the following roles in the Indian banking and
has survived to the present and is now one of the largest financial system are as follows:
banks in India. Note Issuing Authority
— The largest bank-Imperial Bank of India was nationalised in — RBI has had the sole authoring to issue currency notes
1955 and renamed as State Bank of India followed by other than ` 1 notes or coins and coins of smaller
formation of its 7 associates in 1959. denominations since, its inception. ` 1 notes or coins and
— The step toward social banking was taken with the coins of smaller denomination are issued by the Central
nationalisation of 14 Commercial Banks on 19th July, Government, but are put into circulation through the
1969. Six more Commercial Banks were nationalised on RBI.
15th August, 1980. — RBI can issue notes against the securing of coins or
— Banking crisis during 1913-1917 and failure of 588 banks in bullion, foreign securities, rupee coins, Government of
various parts of the country during the decade ended 1949 India securities as such bills of exchange (promissory
underlined the need for regulating and controlling notes as are eligible for purchase by it. The Reserve
Commercial Banks. The Banking Companies Act was passed Bank has adopted Minimum Reserve System for the note
in February, 1949, which was subsequently amended to read issue. Since 1957, it maintains gold and foreign
as Banking Regulation Act, 1949. This Act provided the legal exchange reserves of ` 200 crore of which atleast ` 115
framework for regulation of the banking system in India. crore should by in gold.
— Now, the Indian Banks have overseas presence in the form
of physical branches, representative offices, joint ventures
Printing of Securities and Minting in India
and subsidiaries. Security Press Station Related by
Currency Notes Press (1928) Nasik Bank notes from `
Reserve Bank of India (RBI) Security Paper (Established Hoshangabad
1 to 100
Banks and
— The RBI was set-up on the basis of Hilton Young 1967-68) currency notes
Commission recommendation in April, 1935, with the paper
enactment of RBI Act, 1934. Bank Notes Press (1974) Dewas Bank notes of `
— The RBI continued to serve as the Central Bank to Burma 20, 50, 200, 100,
(Myanmar), until Japanese occupation of Myanmar in 500 and 2000
April, 1947. Security Notes Printing Hyderabad Union excise duty
— The RBI also continued to serve as Central Bank to Press (Established 1982) stamps
Pakistan, until June, 1948. India Security Press(1992) Nasik Postal material
postal stamps etc
— The RBI was nationalised in 1949 and its First Indian
Governor was CD Deshmukh.
Magbook ~ Money and Banking 37

Security Press Station Related by Credit Control


Modernised Currency Notes Mysore (Karnataka) — It is an important tool used by RBI, a major weapon of
Press (1995) Sarbani (West Bengal) the monetary policy used to control the demand and
Coins are minted at four places viz, Mumbai, Kolkata, supply of money (liquidity) in the economy. Central Bank
Hyderabad and Noida. administers control over the credit that the Commercial
` 1 note released after 20 years Banks grant. Such a method is used by RBI to bring
economic development with stability.
— In November 1994, printing of ` 1 note was stopped mainly
due to highest cost and for freeing capacity to print Need for Credit Control
currency notes of higher denomination. — To encourage the overall growth of the priority sector.
— Printing of ` 2 and 5 notes were discontinued in 1995. — To keep a check over the channelisation of credit.
— Notes of ` 1 to be issued would be legal tender as provided — To achieve the objective of controlling inflation as well as
in the Coinage Act, 2011. deflation.
— To boost the economy by facilitating the flow of adequate
Banker to the Government
volume of bank credit to different sectors.
— RBI has the obligation to transact the banking business of the
— Stability in exchange rate and money market of the
Union and State Governments. In this capacity, it accepts ,
country.
money on account of these governments makes payments on
their behalf and carries out their exchange and remittance
operations. Banker to Banks Monetary Policy Committee
— RBI has a special relationship with the banks. It controls the Government recantly approves a six member Monetary
amount of their reserves (SLR and CRR) and holds all or part Policy Committee, that will set policy interest rates. Out of
of their reserves. Banks borrow from the RBI in times of need the six members, three members are from RBI including the
and RBI is in effect the lender of last resort. RBI is the governors, who would have a casting vote.
ultimate source of money and credit in India. The other three external members in the committee would
Regulator and Supervisor be appointed by the government. Besides, the six members,
a finance ministry nominee would also take part in the
— In this role, RBI provides the broad parameters within which deliberations of the committee to convey the government’s
the banking and financial system of India functions. Its view on policy, but he won’t have a voting right.
regulatory powers are provided by the RBI Act and the
Banking Regulation Act. RBI also regulates many types of
Non-Banking Financial Companies (NBFCs). Some of the
regulatory powers of RBI are as follow finance:
Methods of Credit Control
— Issuing licenses for new banks. There are two types of methods of credit control are as follows:
— Prescribing minimum requirements related to paid-up capital,
reserves etc. Quantitative/Credit Control
— Inspecting the working of banks with regard to organisational — Quantitative or credit control is used to control the volume
set-up, branch expansion etc.
of credit and indirectly to control the inflationary and
— Conducting investigations into complaints of fraud, irregularities
deflationary pressures caused by expansion and
etc in respect of banks.
contraction of credit.
— Approving or forcing amalgamations, reconstruction or
liquidation of banks. — The quantitative or credit control consists of:
— Controlling appointments or termination of Chairman and Chief — Bank Rate It is also called the rediscount rate. It is the rate,
Executive Officers of private sector banks. at which the RBI allows finance to Commercial Banks. It is
currently at 9%.
Custodian of Foreign Reserves — Cash Reserve Requirement (CRR) Since, 1962, the RBI
— As the custodian of foreign reserves, RBI is responsible for has been empowered to vary the CRR requirement between
managing the investment and utilisation of the country’s 3% and 15% of the total demand and time deposits. The
RBI (Amendment) Bill, 2006, empowers RBI to prescribe
foreign reserves in the best possible manner. With the
CRR cash that banks deposit with the RBI without any floor
introduction of floating exchange rate system and rate or ceiling rate.
convertibility of the rupee, RBI also has act to stabilise the — Statutory Liquidity Ratio (SLR) It is the ratio of liquid asset,
foreign exchange market. which all commercial banks have to keep in the form of cash,
— RBI’s function in this role is to develop and regulate the gold and unencumbered approved securities equal to not
foreign exchange market and to facilitate external trade and more than 40% of their total demand and time deposits
liabilities.
payment.
38 Magbook ~ Indian Economy

— Open Market Operations (OMOs) It role as a credit control


instrument emerged after economic reforms of 1991, when
Quantitative Easing
Indian economy was flushed with excessive inflow of foreign — It refers to an extreme form of monetary easing through
funds. Under OMOs, when the RBI sells G-secs in the which the Central Bank floods the financial system with
market, it withdraws money or liquidity from the market and
liquidity.
thus, reduces volume of credit leading to control of
inflation. — It is done to induce bank lending to productive sectors of
— Repo Rate It was introduced in December, 1992, by RBI. It the economy and thus, promote growth during times when
is the rate, at which RBI lends short-term money to the banks are overcautious to lend the money due to prevailing
banks against securities. When the repo rate increases situation of recession or depression.
(Dearer Money Policy) borrowing from the RBI becomes
more expensive and when the repo rate decreases, (Cheaper
Money Policy) borrowing becomes cheaper. Repo rate injects
Liquidity Adjustment Facility (LAF)
liquidity in the market. — Repo rate and reverse Repo rate are the parts of Liquidity
— Reverse Repo Rate It was introduced in November, 1996. It Adjustment Facility (LAF) of RBI.
is the rate, at which banks park short-term excess liquidity — LAF allows the RBI to manage market liquiding on a daily
with the RBI. An increase in the reverse repo rate means basis and to send interest rate signals to the market.
that the RBI is ready to borrow money from the banks at
higher rate of interest . As a result, banks would prefer to — LAF operates through repo and reverse repo auctions.
keep more and more surplus funds with the RBI. Reverse It has now becomes the principal operating instrument of
repo rate withdraws liquidity from the market. monetary policy.
— Other banking operation activities are Marginal Standing
Facility Rate (MSFR), Net Demand and Time Liabilities etc. Market Stabilisation Scheme
— It is used by the RBI in times of volatility in exchange rate.
Marginal Standing Facility (MSF) — Here, RBI right release or buy foreign exchange in the

MSF scheme came into effect in 2011. It is a very short-term market to stabilise the exchange rate.
borrowing scheme for Scheduled Commercial Banks. MSF — Under MSS, RBI issues bonds on behalf of the government.
rate is the rate at which these banks can borrow funds
overnight from RBI against government securities. Qualitative Credit Control

Banks can use MSF during severe cash shortage or acute — Qualitative credit control is used by RBI for the selective
shortage of liquidity. MSF reduces volatility in the overnight purposes, some of which are as follows:
lending rates in the inter-bank market and enables smooth
— Margin Requirements This refers to difference between the
transmission of monetary policy. Under MSF, Banks can securities offered and amount borrowed by the banks.
borrow upto 2% of the Net Demand and Time Liabilities — Consumer Credit Regulations This refers to issuing rules
(NDTL). regarding down payments and maximum maturities of
instalment credit for purchase of goods.
— RBI Guidelines RBI issues oral or written statements, appeals,
MCLR guidelines, warnings etc to the banks.
— The Reserve Bank of India has brought a new — Rationing of Credit The RBI controls the credit granted or
methodology of setting lending rate by commercial banks allocated by Commercial Banks.
under the name Marginal Cost of Funds based Lending — Moral suasion an application of pressure, but not force to get
Rate (MCLR). It has modified the existing base rate members to adhere to a policy RBI gives advices and
system from April 2016 onwards. suggestions to the bankers to follow the instructions given
— As per the new guidelines by the RBI, banks have to by it.
prepare Marginal Cost of Funds based Lending Rate
RBI Controls, Inflation and Growth
(MCLR) which will be the internal benchmark lending
rates. — RBI can influence inflation and growth in the economy to a
large extent through its instruments of control. If RBI
— Based upon this MCLR, interest rate for different types of
squeeses out liquidity from the economy by selling
customers should be fixed in accordance with their
securities, increasing repo rates, increasing CRR etc, then
riskiness. The base rate will be now determined on the
the demand in the economy is reduced and inflation is
basis of the MCLR calculation. The MCLR should be
brought under control.
revised monthly by considering some new factors
including the repo rate and other borrowing rates. — However, in case inflation is due to supply side shortages,
RBI controls have less influence. Similarly, increasing
— Specifically the repo rate and other borrowing rates that
liquidity in economy means that households have more
were not explicitly considered under the base rate
money to consume, industries have more money to invest
system.
Magbook ~ Money and Banking 39
in plant and machinery etc, all of which lead to
increase in economic activity. Thus, it can be seen Nationalised Banks in India
that measures taken by RBI, which control inflation 1. Allahabad Bank 11. Indian Overseas Bank
can hurt growth and measures which boost growth, 2. Andhra Bank 12. Oriental Bank of Commerce
can cause inflation. 3. Bank of Baroda 13. Punjab and Sind Bank
RBI Guideline for Small Banking 4. Bank of India 14. Punjab National Bank
— The RBI issued draft guidelines for those sealing a license to 5. Bank of Maharashtra 15. Syndicate Bank
set-up a payments banks or small banks, as a part of its 6. Canara Bank 16. UCO Bank
efforts to expand banking services to more businesses and 7. Central Bank of India 17. Union Bank of India
poor household. 8. Corporation Bank 18. United Bank of India
— The minimum paid capital required for both categories 9. Dena Bank 19. Vijaya Bank
of bank licenses would have to contributes atleast
10. Indian Bank 20. IDBI Bank Limited
40% initially.

Scheduled Commercial — On 15th April, 1980, six more banks having demand and time
liabilities of not less than ` 200 crores were nationalised. The
Banks undertakings of these banks are taken over and vest in six
— All banks which are mentioned in the Second Schedule corresponding new banks under the banking companies
of RBI Act, 1934 are known as Scheduled Banks. (Acquisition and Transfer of Undertakings) Act, 1980.
— These banks comprise Scheduled Commercial Banks — Later on, in the year 1993, the government merged New Bank
and Scheduled Cooperative Banks. Advances, of India with Punjab National Bank. It was the only merger
deposits, money at call, short notice etc. are included between Nationalised Banks and resulted in the reduction of
in the assets of commercial Bank of India. Scheduled the number of Nationalised Banks from 20 to 19.
Commercial Banks in India are categorised into five
— In the group wise classification, since 31st December, 2007
different groups according to their ownership or nature
IDBI Bank Limited has been included in Nationalised Banks.
of operation.
— These bank groups are as follows:
— State Bank of India and its associates State Bank of India
— Nationalised Banks — State Bank of India (SBI) was previously called Imperial Bank
— Private Sector Banks of India in 1921, which was created by amalgamation of 3
— Foreign Banks and Presidency Banks viz, Bank of Bengal, Bank of Bombay and
— Regional Rural Banks Bank of Madras. It was nationalised in 1955.

Public Sector Banks Bharatiya Mahila Bank


— After 1969 Commercial Banks are broadly classified
— Former Prime Minister Dr Manmohan Singh and UPA
into Nationalised or Public Sector Banks and Private Chairperson, Sonia Gandhi jointly inaugurated India’s first all
Sector Banks. The State Bank of India and its five women bank, Bharatiya Mahila Bank in Mumbai on 19th
Associate Banks alongwith Nationalised Banks are the November, 2013, on the birth anniversary of former Prime
Public Sector Banks. Minister Indira Gandhi.

Nationalised Banks Merging of SBI and Mahila Bank


— From 1st February, 1969, the government imposed — SBI, merged its associate bank and BMB with itself on 1
social control on banks by introducing certain April, 2017. With this merger, SBI becomes one of top 50
provisions in the Banking Regulation Act, 1949. It global banks.
imposed severe restrictions on the composition of the — The Union Cabinet, on 15th June, 2016, approved the
Board of Directors and internal management and merger of 5 associate banks as well as BMB with State Bank
administration of banking companies. of India.
— It also introduced restrictions on advances by banking
Merging of Bank of Baroda, Vijaya Bank and
companies. These were intended to ensure that the
Dena Bank
bank advances were not confined to large-scale
industries and big business houses, but were also — Vijaya Bank and Dena Bank merged with Bank of Baroda
directed, in due proportion to other important sectors (BoB) on 1st April, 2019. This merge has created BoB as the
like agriculture, small-scale industries and exports. 3rd largest public sector in India.
40 Magbook ~ Indian Economy

information about its shareholders, financial position and the


Private Banks dealings with Indian parties.
— All those banks where creator parts of stake or equity — The request is examined keeping in view the financial
are held by the private shareholders are called as private soundness of the bank, international and home country
sector banks. In India, private sector banks are known ranking, international presence, economic and trade
with two names; old private sector banks and new relations between the two countries and supervisory
private sector banks. standards in the home country etc.

Old Private Sector Bank


Regional Rural Banks
— The banks which were not nationalised at the time
of nationalisation of banks that took place during 1969
— In 1976, the Parliament enacted the Regional Rural Banks
Act, 1976 to provide for the incorporation, regulation and
and 1980 are known as the old private sector banks.
winding up of Regional Rural Banks. The Act has been
These were not nationalised, because of their small size
made effective from the 26th September, 1975.
and regional focus.
— The equity of the RRBs is contributed by the Central
New Private Sector Bank Government, concerned State Government and the sponsor
bank in the proportion of 50:15:35.
— The banks, which came in operation after 1991, with the
introduction of economic reforms and financial sector
— The objective of the RRBs is to develop the rural economy by
reforms are known as new private sector banks. providing; for the purpose of development of agriculture,
Banking Regulation Act was then amended in 1993, trade, commerce, industry and other productive activities in
which permitted the entry of new private sector banks in the rural areas, credit and other facilities, particularly to the
the Indian banking sector. small and marginal farmers, agricultural labourers, artisans
and small entrepreneurs and for matters connected there
Licence to New Private Banks with and incidental there to.
IDFC Bank — Besides, the Reserve Bank which is the regulatory authority
— Prime Minister Narendra Modi on October 19, 2015 for the RRBs in accordance with the provisions of the
inaugurated the IDFC Bank in New Delhi. IDFC Bank Banking Regulations Act, 1949, the Banking
Limited, which started its operations on October 1, Regulations Act empowers NABARD (National Bank for
2015, is an Indian Banking company with headquarters Agriculture and Rural Development) to undertake the
in Mumbai that forms a part of IDFC, an integrated inspection of RRBs.
infrastructure finance company. — A Regional Rural Bank seeking permission of the Reserve
— IDFC Bank was granted a universal banking license in Bank for opening branches has to obtain the
July, 2015 by the Reserve Bank of India (RBI). It was recommendation of NABARD.
selected along with micro-lender Bandhan in the last — RRB (Amendment) Bill, 2014 this amendment to raise the
round of award of licenses. authorised capital of the RRBs from ` 5 crore to ` 2000
crore. The bill also provides that the authorised capital of
Bandhan Bank any RRB shall not be reduced below ` 1 crore.
— Bandhan Bank Limited appointed its Chairman and
Board of Directors on July 9, 2015. The bank started its Lead Bank Scheme
operations in India from August 23, 2015. It is the first — Lead Bank Scheme based on area approach was launched
bank established in Eastern India post Independence. in 1969, on the recommendation of Dr Gadgil Committee
— Former Chief Economic Adviser to the Indian government and Narasimham Committee. Under the LBS, all the 14
Ashok Kumar Lahiri has been appointed as the Nationalised Banks and a few Private Sector Banks were
Chairman. Chandra Shekhar Ghosh, founder of Bandhan allotted specific districts and were asked to play the lead role
Financial Services Limited, was appointed as the in coordinating credit deployment. The services area
Managing Director and Chief Executive Officer (MD & approach was implemented under the purview of lead Bank
CEO) of the bank. They both will be in the board of scheme.
directors as well.
Scheduled Co-operative
Foreign Banks Banks
— Foreign Banks are allowed to operate in India through — Co-operative Banks have also played a limited, but important
branches and representative offices. A new Foreign role in the banking system of the country. Scheduled
Bank desirous of opening a branch in India is required Co-operative Banks consist of Scheduled State Co-operative
to apply Reserve Bank of India giving relevant Banks and Scheduled Urban Co-operative Banks.
Magbook ~ Money and Banking 41

Names of selected applicants


State Co-operative Banks (SCBs) —

— Au Financiers (India) Limited (Jaipur)


— State Co-operative Bank means the Principal co-operative — Capital Local Area Bank Limited (Jalandhar)
society in a state, the primary object of which is the financing — Disha Microfin Private Limited (Ahmedabad)
of other co-operative societies in the state. — Equitas Holdings Private Limited (Chennai)
— The Banking Ombudsman Scheme, 1995 notified by RBI on — ESAF Microfinance and Investments Private Limited
14th June, 1995 was in terms of powers conferred on the (Chennai)
bank by Section 35A of the Banking Regulation Act, 1949 (10 — Janalakshmi Financial Services Private Limited
of 1949) to provide for a system of redressal of grievances (Bengaluru)
against banks. — RGVN (North-East) Microfinance Limited (Guwahati)
— Suryoday Micro Finance Private Limited (Navi Mumbai)
Urban Co-operative Banks (UCBs) — Ujjivan Financial Services Private Limited (Bengaluru)
— The UCBs are registered under the Co-operative Societies Acts — Utkarsh Micro Finance Private Limited (Varanasi)
of the respective State Governments. UCBs having a
multi-state presence are registered under the Multi-state
Bimal Jalan Committee/
Co-operative Societies Act and regulated by the Central New Bank Licenses, 2014
Government. — A committee, under the Chairmanship of former RBI
— Besides, the Reserve Bank also has regulatory and supervisory Governor Bimal Jalan, was constituted to scrutinise
authority for bank related operations under certain provisions the application for new banks in India. The committee
of the Banking Regulation Act, 1949 (as applicable to submitted its report in February 2014. Recommending
Co-operative Societies). for the ‘in-principal approval’ of banking licenses to
Bandhan Microfinance and IDFC (Infrastructure
Payment Banks Development and Finance Corporation).
— The Reserve Bank of India (RBI) granted ‘in-principle’ approval — The RBI had issued guidelines for new banks in
to 11 entities, including Reliance Industries, Aditya Birla Nuvo, February, 2013 and invited applications from the
Vodafone and Airtel, to set-up payments banks and proposed various stakeholders. The Bimal Jalan Committee
such licenses ‘on tap’ in future on August 18, 2015. recommended the name of the two entities, from
— The other entities which have been given ‘in-principle’ approval among the list of several entities, like Indian post,
are—Department of Posts, Cholamandalam Distribution Anil Ambani Group, Aditya Birla Group, Bajaj
Services, Tech Mahindra, National Securities Depository Finance, Muthoot Finance, Religare Enterprise etc.
Limited (NSDL), Fino PayTech, Sun Pharma’s Dilip Shantilal — However, the committee has put forth certain
Sanghvi and PayTM’s Vijay Shekhar Sharma. conditions before the entities, in order to get the
— The ‘in-principle’ approval granted will be valid for a period of banking license. Within a period of 18 months these two
18 months, during which time the applicants have to comply entities are required to
with the requirements under the guidelines and fulfil the other — get a net worth of ` 1000 crore or more;
conditions as may be stipulated by the RBI. — open at least 25% branches in unbanked rural areas.

Small Finance Banks Types of Banking System


— The Reserve Bank of India (RBI) has granted ‘in-principle’
There are three types of banking are as follows:
approval to set-up small finance banks under the “Guidelines
for Licensing of Small Finance Banks in the Private Sector” Core Banking
(Guidelines) on September 16, 2015.
— Core banking is often associated with retail banking
— The ‘in-principle’ approval granted will be valid for 18 months and many banks treat the retail customers as their
to enable the applicants to comply with the requirements core banking customers. Business is usually
under the Guidelines and fulfil other conditions as may be managed via the Corporate banking division of the
stipulated by the RBI.
institution. Core banking covers basic depositing and
— On being satisfied that the applicants have complied with the lending of money.
requisite conditions laid down by it as part of ‘in-principle’
— Normal core banking functions will include
approval, the RBI would consider granting them a licence for
transaction accounts, loans, mortgages and
commencement of banking business under Section-22(1) of the
Banking Regulation Act, 1949. Until a regular licence is issued, payments. Banks make these services available
the applicants cannot undertake any banking business. across multiple channels like ATMs.
— It is also known as Core Banking solution.
42 Magbook ~ Indian Economy

Retail Banking Narasimham-II


— When a bank executes transactions directly with consumers, — The Narasimham-II Committee was tasked with the
rather than corporations or other banks, then it is called Retail progress review of the implementation of the
Banking. banking reforms since, 1992 with the aim of further
strengthening the financial institutions of India.
Narrow Banking — It focussed on issues like size of banks and capital
— It restricts banks to hold liquid and safe government bonds. adequacy ratio among other things. M Narasimham,
— It is also called as safe bank. Chairman, submitted the report of the committee in
April, 1998.
Inter-Bank Transfer Damodaran Committee
— It is a special service that allows you to transfer funds
electronically to accounts in other banks in India through. — The committee, headed by former SEBI Chairman M
— NEFT The acronym ‘NEFT’ stands for National Electronic Funds Damodaran, was set-up by the Central Bank to look
into the issues of customer services and evaluate
Transfer. Funds are transferred to the credit account with the
the existing system of grievance redressal
other participating bank using RBI’s NEFT service. RBI Acts as
mechanism prevalent in banks, its structure and
the service provider and transfers the credit to the other bank’s
efficacy and recommend measures for expeditious
account.
resolution of complaints.
— RTGS The acronym ‘RTGS’ stands for Real Time Gross
Settlement. The RTGS system facilitates transfer of funds from Recommendations
accounts in one bank to another on a ‘real time’ and on ‘gross — Bank should offer no-frill savings accounts with
settlement’ basis. The RTGS system is the fastest possible inter certain basic facilities such as cheque book and
bank money transfer facility available through secure banking ATM card without prescribing any minimum
channels in India. balance.
— Minimum or Maximum account for RTGS or NEFT. Transactions — All fixed deposit receipts should prominently indicate
under Retail and Corporate Internet Banking. the annualised interest rate to help customers take
more informed decisions. The Indian Banks Association
Types Retails Corporate should standardise the account opening form for all
Minimum Maximum Minimum Maximum banks, similar to the one used for loans. Title deeds of
RTGS ` 2 Lakhs ` 5 Lakhs ` 2 Lakhs ` 50 Lakhs for property should be returned to customers within 15
Vyapaar E ` 500 days of the full settlement of home loans.
crore for vistar.
NEFT No Minimum ` 5 Lakhs No Minimum ` 50 Lakhs for Swabhiman
Vyapaar E ` 500
crore for vistar.

A major financial inclusion initiative was formally
launched as ‘Swabhiman’ on 10th February, 2011,
which aims at providing branchless banking through
Banking Sector Reforms the use of technology. Banks will provide basic services
like deposits, withdrawal and remittances using the
Narasimham Committee services of Business Correspondents.
Recommendation ◆
The initiative enables government subsidies and social
— Deregulation of interest rate. security benefits to be directly credited to the accounts
— Reduction in reserve requirement. of the beneficiaries, enabling them to draw the money
from the business correspondents in their village itself.
— Prudential norms.
— Supervision of Commercial Banks.
— Measures to improve the competitive efficiency in banking Khandelwal Committee Report
sector. — Government constituted a Committee on Human
Resources issues of Public Sector Banks (PSBs)
Narasimham-I under the Chairmanship of Dr AK Khandelwal, who
— The purpose of the Narasimham-I Committee was to study all has submitted its report.
aspects relating to the structure, organisation, functions and — The committee made 105 recommendations on
procedures of the financial systems and to recommend matters related to Manpower and Recruitment
improvements in their efficiency and productivity. Planning, Training, Career Planning, Performance
— The committee submitted its report to the Finance Minister in Management, Reward Management, Succession
November, 1991. Planning and Leadership Development, Motivation,
Magbook ~ Money and Banking 43

Professionalisation of HR, Wages, Service Conditions and — A Payment Bank can become Banking Correspondent
Welfare etc. (BC) of another bank and offer all products / services
— As 49 recommendations required further deliberations, the which a BC can offer.
remaining 56 recommendations were forwarded to PSBs — Payment Banks can distribute non-risk sharing financial
with the request that an HR Plan for each bank be products like mutual fund units and insurance
prepared and got approved by the respective Board of products, etc.
Directors. — The revenue of these banks would come mainly from the
transaction fees.
Nachiket Mor Committee
— The RBI appointed committee on comprehensive financial Banks Board Bureau
services for small business and low income under the — Prime Minister Narendra Modi has approved the guidelines
Chairmanship of Sri Nachiket Mor.
of Banks Board Bureau (BBB) on 28th February, 2016.
Recommendation of committee are as follows: BBB starts its work from 1st April, 2016.
— Every adult (above 18 years) of over country should have a — BBB will work as Search Committe or appointments
bank account by 1st January, 2016. This account will be
known as Universal Electronic Bank Account (UEBA).
board, appointment of Chairman of PSBs (Public Sector
Banks) and will consist of professionals with two
— Every resident should be issued an account at a time of
receiving Aadhaar number (UIDAI) by a bank itself. government representative viz Secretary of Financial
— It recommends abolition of interest subsidies and loan waivers. Services and Public Enterprises.
— It recommends raising priority sector lending cap for bank to — Vinod Rai appointed as the First Chairman of the Banks
50% from the current 40%. Board Bureau. He was the former Comptroller and
— It also proposed for creation of a payment bank to provide Auditor General (CAG) of India.
payment services including credit, insurance and risk — The government has maintained the BBB as a holding
management products.
company for state run banks, an idea first mooted at the
maiden banking conclave Gyan Sangam in January, 2015.
Payment Bank
On 23rd September, 2013, Committee on Comprehensive
Financial Services for Small Business and Low Income Indradhanush to Revamp PSU Banks
Households headed by Nachiket Mor, was formed by the RBI. ◆
To revive the fortunes of public sector banks, the government
On 7th January, 2014, the Nachiket Mor Committee unveiled a seven-point plan ‘Indradhanush’ encompassing
submitted its final report. Among its various ` 20000 crore immediate fund infusion, creation of a single
recommendations, it recommended the formation of a new holding company and minimising political interference on
category of bank called Payment Bank. On 17th July, 2014, August 14, 2015. The seven Key reforms of Indradhanush
the RBI released the draft guidelines for Payment Banks, mission include appointments, de-stressing capitalisation,
seeking comments for interested entities and the general empowerment, frame work of accountability and governance
public. On 27th November, RBI released the final guidelines reforms.
for Payment Banks. ◆
The strategy, Indradhanush (Rainbow), focuses on systemic
The key aspirants to payment banking business include changes in state-run lenders, including a fresh look at hiring,
telecom firms, prepaid payment instruments / payment a comprehensive plan to de-stress bloated lenders, capital
solution providers retail chains, large business correspondents infusion, accountability incentives with higher rewards
and business conglomerates. Out of them, telecom firms have including Stock Options and cleaning up governance.
an advantage over others mainly because they already have a
distribution network in rural areas. Most of the prepaid
payment instruments / payment solution providers are tech Basel Norms
sauuy and already working in the field of mobile payments.
— It was in 1988 that the central banking bodies of the
Scope of activities of Payment Banks developed economies agreed upon the provision of Capital
— Payment Banks can accept demand deposits. This implies Adequacy Ratio (CAR), also known as the Basel Accord.
that customers can open savings accounts as well as The accord was agreed upon at Basel, Switzerland, at a
current accounts; but NO time deposits (such as FDs). meeting of the Bank of International Settlements (BIS).
— An account balance cannot exceed ` 1 lakh for an This accord provides recommendations on banking,
individual customer. Payment Banks can issue ATM/ debit regulations with regard to capital risk, market risk and
cards but not credit cards. operational risk. It’s objective was to ensure that financial
— Payment Banks can not give loans. Payments and institutions have enough capital to meet obligations and
remittance services through various channels. absorb unexpected losses.
44 Magbook ~ Indian Economy

Basel I Banking Ombudsman


— The Basel Committee on Bank Supervision (BCBS)
— Banking Ombudsman Scheme was introduced by the RBI
published a set of minimal capital requirements for
in 1995 under the Banking Regulation Act, 1949. It is a
banks, to maintain a certain amount of free capital (ratio) senior official appointed by the RBI to redress customer
to their assets, as a cushion against probable losses in complaints against deficiency in certain banking services.
investment and loans. Basel I primarily, focus on credit
— Decision of Banking Ombudsman can be appealed against
risk. In 1988, this ratio capital was decided to be 8%. The
to the appellate authority (vested in a Deputy Governor of
CAR is the percentage of the total capital to the total RBI).
weighted assets.
— Banking Ombudsman can award compensation to the
— CAR = (Total of Tier-I and Tier-II capital)/ Risk weighted complainant. In this, it takes into account the loss of the
assets. complainant’s time, expenses incurred and the
— Thus, CAR is also known as Capital to Risk-Weighted harassment and mental anguish suffered.
Assets Ratio (CRAR). It is used to protect the depositors — It has jurisdiction over all Commercial Banks, RRBs,
and promote the stability and efficiency of the financial Scheduled primary co-operative banks, NBFCs etc. It
system. deals with matters less than or equal to ` 10 lakh.

Basel II Development Financial


— It attempts to integrate Basel I capital standards with
national regulations, by setting the minimum capital Institutions
requirement of financial institutions with the goal of — Financial institutions are an important part of the Indian
ensuring institution liquidity. It aims securing at
financial system as they provide medium to long-term
international convergence on regulations governing the
finance to different sectors of the economy.
CAR.
— The institutions have been set-up to meet the growing
— Minimum capital requirements
demands of particular segments, such as export, rural
— Supervisory review
housing and small industries.
— Market discipline
— These institutions have been playing a crucial role in
Basel III channelising credit to the needy sectors and addressing
the challenges or issues faced by them.
— It seek to strengthen the existing capital requirements and
— The four financial institutions - EXIM Bank, National Bank
introduce a global liquidity standard to enable the banks to
for Agriculture and Rural Development (NABARD), National
weather financial storms. It mandates the banks to increase
Housing Bank (NHB) and Small Industries Development
the loss absorbing capital from 2% to 4.5% by January,
Bank of India (SIDBI) are under full-fledged regulation and
2015. Also, banks will be required to hold a capital
supervision of the Reserve Bank.
conservation buffer of 2.5% of withstand future period of
stress. — As in the case of Commercial Banks, prudential norms
relating to income recognition, asset classification and
Basel III Guidelines, 2015 provisioning and capital adequacy ratio are applicable to
these financial institutions as well. These institutions also
— The Reserve Bank of India (RBI) released on 28th May, are subject to on-site inspection as well as off-site
2015, the draft guidelines on the Net Stable Funding surveillance.
Ratio (NSFR) under Basel III framework on liquidity
— Since, all the banks are directly or indirectly contributing
standards of banks.
to the development works in the country, thus, all are
— The NFSR is defined as the amount of available stable development financial institutions.
funding relative to the amount of required stable funding.
— In draft guidelines released, the Central Bank said that Financial Stability and
banks will have to maintain Net Stable Funding Ratio
(NSFR) from January 2018.
Development Council (FSDC)
— Basel III norms define the capital of the banks in different — Financial Stability and Development Council is an
ways. It consider common equity and retained earnings apex-level body constituted by the Government of India.
as the predominant component of capital, but restrict the The idea to create such a super regulatory body was first
inclusion of items such as-deferred tax assets, mortgage mooted by the Raghuram Rajan Committee in 2008.
service rights and investments in financial institutions to Finally in 2010, the then Finance Minister of India, Pranab
no more than 15% of the common equity component. Mukherjee, decided to set-up such an autonomous body
Magbook ~ Money and Banking 45

dealing with macroprudential and financial regularities in


the entire financial sector of India. The council is headed
Industrial Finance Corporation of
by the Finance Minister and has the Reserve Bank of India Limited (IFCI)
India (RBI) Governor and chairpersons of the Securities — It was the first development finance institution set-up in
and Exchange Board of India, Insurance Regulatory and 1948 under the IFCI Act in order to provide long-term
Development Authority and Pension Fund Regulatory and institutional credit to medium and large industries. It aims
Development Authority as other members alongwith to provide financial assistance to industry by way of rupee
finance ministry officials. and foreign currency loans, underwrites or subscribes the
issue of stocks, shares, bonds and debentures of
FSDC will perform following roles
industrial concerns etc.
— To engage in macroprudential supervision of the economy,
including the functioning of large financial conglomerates
— It has also diversified its activities in the field of merchant
and address inter-regulatory coordination issues. banking, syndication of loans, formulation of rehabilitation
programmes, assignments relating to amalgamations and
— To focus on financial literacy and financial inclusion. mergers etc.
— To periodically look into issue relating to financial
development. Industrial Development Bank of
National Housing Bank (NHB) India (IDBI)
— It was established in July, 1964, as an apex financial
— National Housing Bank was set-up on 9th, July, 1988
institution for industrial development in the country. It
under the National Housing Bank Act, 1987 as a
caters to the diversified needs of medium and large-scale
wholly-owned subsidiary of the Reserve Bank to act as an
industries in the form of financial assistance, both direct
apex level institution for housing.
and indirect. Direct assistance is provided by way of
— NHB has been established to achieve, among other project loans, underwriting of and direct subscription to
things, the following objectives: industrial securities, soft loans, technical refund loans,
— To provide a sound, healthy, viable and cost effective housing etc. While, indirect assistance is in the form of refinance
finance system to all segments of the population and to facilities to industrial concerns.
integrate the housing finance system with the overall financial
system. Small Industries Development
— To promote a network of dedicated housing finance
institutions to adequately serve various regions and different
Bank of India (SIDBI)
income groups. — It was set-up by the Government of India in April, 1990,
— To augment resources for the sector and channelise them for as a wholly owned subsidiary of IDBI. It is the principal
housing. financial institution for promotion, financing and
— To make housing credit more affordable. development of small-scale industries in the economy. It
— To regulate the activities of housing finance companies based aims to empower the Micro, Small and Medium
on regulatory and supervisory authority derived under the act. Enterprises (MSME) sector with a view to contributing to
— To encourage augmentation of supply of buildable land and the process of economic growth, employment generation
also building materials for housing and to upgrade the housing and balanced regional development.
stock in the country.
— To encourage public agencies to emerge as facilitators and
suppliers of serviced land for housing.
MUDRA Bank
— Micro Units Development and Refinance Agency Bank (or
National Bank for Agriculture and MUDRA Bank) was launched by Prime Minister Narendra
Modi on 8th April, 2015 with a corpus of ` 20,000 crore
Rural Development (NABARD) and a credit guarantee corpus of ` 3,000 crore.
— National Bank of Agriculture and Rural Development — It is a public sector financial institution in India. It
(NABARD) is one of the subsidiaries where the majority provides loans at low rates to small entrepreneurs.
stake is held by the Reserve Bank. NABARD is an Apex MUDRA Bank will be set-up through a statutory
Development Bank with a mandate for facilitating credit flow enactment.
for promotion and development of agriculture, small-scale
— It will be responsible for developing and refinancing all
industries, cottage and village industries, handicrafts and
Micro-Finance Institutions (MFIs) which are in the
other rural crafts.
business of lending to micro and small business entities
— It also has the mandate to support all other allied economic engaged in manufacturing, trading and service activities.
activities in rural areas, promote integrated and sustainable The formation of the agency was initially announced in the
rural development and secure prosperity of rural areas. 2015 Union Budget of India in February 2015.
46 Magbook ~ Indian Economy

Formation of MUDRA Bank Industrial Investment Bank of


— The MUDRA banks will be set-up under the Pradhan India Limited (IIBI)
Mantri MUDRA Yojana Scheme. The bank will intially
function as a non-banking financial company and a — It was set-up in 1985 under the Industrial Reconstruction
subsidiary of the Small Industries Development Bank of Bank of India Act, 1984, as the principal credit and
India. Later, it will be made into a separate company. reconstruction agency for sick industrial units. It was
converted into IIBI on 17th March, 1997, as a
— It will also serve as a regulator for other Micro-Finance
full-fledged development financial institution. It assists
Institutions (MFIs) and provide them refinancing servies. It
industry mainly in medium and large sector through wide
will provide guidelines for MFIs and give them ratings.
ranging products and services. Besides project finance,
— The bank would partner with state level/regional level IIBI also provides short duration non-project
co-ordinators to provide finance to Last Mile Financer of asset-backed financing in the form of underwriting or
small/micro business enterprises. direct subscription, deferred payment guarantees and
Objectives of MUDRA Bank working capital or other short-term loans to companies to
meet their fund requirements.
The objectives of the MUDRA Bank are as follows:
— Regulate the lender and the borrower of microfinance and
brings stability to the microfinance system through Export-Import (EXIM) Bank
regulation and inclusive participation. ◆
Recognising the important role of exports in maintaining the
— Extend finance and credit support to Microfinance viability of external sector and in generating employment, the
Institutions (MFI) and agencies that lend money to small Reserve Bank had sought to ensure adequate availability of
businesses, retailers, self-help groups and individuals. Concessional Bank credit to exporters. It took the lead role in
— Register all MFIs and introduce a system of performance setting up the Export Import Bank of India (EXIM Bank) in
rating and accreditation for the first time. January, 1982.
— This will help last mile borrowers of finance to evaluate and

In recent years, with the liberalisation of real and financial
approach the MFI that meets their requirement best and
sectors of the economy, interest rates on export credit have
been rationalised within the overall monetary and credit
whose past record is most satisfactory. This will also
policy framework.
introduce an element of competitiveness among the MFIs.
The ultimate beneficiary will be the borrower.

In order to provide adequate credit to exporters on a priority
basis, the Reserve Bank has also prescribed a minimum
Differences Between SIDBI and proportion of banks’ adjusted net bank credit to be lent to
MUDRA Bank exporters by Foreign Banks.
It is very important to understand certain things/context/facts
and differences between SIDBI and MUDRA.
Difference between SIDBI and MUDRA bank are as follows:
New Gold Investment Schemes
— SIDBI is an apex small units development bank where
The Government had launched Sovereign Gold Bonds and
Gold Monetisation Schemes on 5th November, 2015. The
MUDRA will initially be started as a department of SIDBI.
main objective of the schemes is to reduce the demand for
— The role of SIDBI remains to promote and finance the physical gold and shift a part of the gold imported every year
small scale sector, implement government plans and for investment purposes into financial savings.
co-ordinate with other organisations while the role of
MUDRA has been conceived more in the context of Sovereign Gold Bonds
present state of microfinance sector. — These are issued by the RBI on behalf of the Government
— Over a period of time, MUDRA, replacing RBI, may emerge of India in rupees and denominated in grams of gold and
as a regulatory body for Microfinance sector housed by restricted for sale to the resident Indian entities only both
emerging NBFC-MFIs. (This is vehemently protested by in demat and paper form.
MFIs who wish to function under regulatory powers of — The minimum and maximum investment limits are 2
RBI.) grams and 500 grams of gold per person per fiscal year
— MUDRA may refinance other MFIs to finance SHGs (Self respectively. The rate of interest for the year 2015-16 is
Helf Groups) to promote micro entrepreneurship. 2.75% per annum, payable on a half yearly basis.
— So, MUDRA can be seen as an initiative to reach the last — The tenure of the Bond is for a period of 8 years with exit
mile for underfinanced small scale units that could not option from fifth year onwards. KYC norms are the same
benefit from insitutional sources of finance. as that for gold. Exemption from capital gains tax is also
available.
Magbook ~ Money and Banking 47

— Redemption is made in the rupee value equivalent to the — The reason for the establishment was to safeguard the
price of gold at the time of maturity. In the first two interest of the policy holders and for the upgradation of the
tranches of SGB, total subscription of 3788 kg of gold entire insurance sector.
amounting to ` 993 crore were received from about 3.90 — The Insurance Regulatory and Development Authority of
lakh applications. India has been authorised to register the new insurance
companies in India.
Gold Monetisation Scheme
— Bureau of Indian Standards (BIS) certified Collection,
Purity Testing Centres (CPTC) to collect the gold from Insurance Companies
the customer on behalf of the banks. — Insurance industry includes two sectors; Life Insurance and
— The minimum quantity of gold (bullion or jewellery) General Insurance. Life Insurance in India was introduced
which can be deposited is 30 grams and there is no limit by Britishers. A British firm in 1818 established the Oriental
for maximum deposit. Life Insurance Company at Calcutta (now Kolkata).
— Gold Saving Account can be opened with any of the — Since, the opening up, the number of participants in the
designated bank and denomination in grams of gold for insurance industry has gone up from 7 insurers (including
short-term period of 1-3 years, a medium-term period of LIC, four public sector general insurers, one specialised
5-7 years and a long-term period of 12-15 years . insurer and the GIC as the national re-insurer) in 2000 to
— The CPTCs transfer the gold to the refiners. The banks 49 insurers as on 30th September, 2011.
will have a tripartite / bipartite Legal Agreement with — Insurance Companies in India The insurance companies
refiners and CPTCs. offer protection against losses. They deal in life
— For the year 2015-16, interest rates has been fixed as insurance, marine insurance, vehicle insurance and
2.25% and 2.5% for the medium and long term so on.
respectively. Redemption is made in cash/gold for — The insurance companies collect the little savings of the
short-term and in cash for medium and long-term investors and then reinvest those savings in the market.
deposits. As of 2nd February, 2016, a total of 1030.2 kg The indigenous insurance companies are collaborating with
of gold have been mobilised through the scheme. different foreign insurance companies after the liberalisation
process. This step has been incorporated to expand the
Non-Banking Financial Indian insurance market and make it competitive.
Companies (NBFCs) Types of Non-Banking Financial Institutions
— NBFCs are essentially banks, since, they perform the
Institution Principle Business
basic twin functions of attracting deposit from the public
and making loans. However, unlike Commercial Banks, NBFCs Receiving deposits under any scheme
they are not incorporated as a bank and are not governed and lending.
by the provision of the Banking Regulation Act, 1949. Equipment Leasing Equipment leasing and financing.
— With the Enactment of RBI (Amendment) Act, 1997, the RBI Company
now control the functioning of NBFCs.
Hire Purchase Finance Hire purchase transaction.
— NBFCs as a whole account for 11.2% of assets of the total
Company
financial system.
— Two broad categories of NBFCs are as follows : Invest Company Buying or selling of securities.
(i) Deposit taking NBFCs (NBFCs-D).
Loan Company Making loan or advances for an
(ii) Non-deposit taking NBFCs (NBFCs -ND).
activity other than its own.
— Capital to Risk-weighted Assets Ratio (CRAR) norms were
made applicable to NBFCs-D in 1998. The CRAR norm for Residuary Non-Banking Business same as NBFCs, but does
NBFC-D is 12% (15% in case of unrated NBFCs-D). Companies (RNBCs) not belong to any of the categories
under NBFCs.
Insurance Regulatory and Mutual Benefit Financial Any company notified by the Central
Development Authority of India Company (MBFC) Government as a Nidhi Company
under Companies Act, 1956.
(IRDAI)
Miscellaneous Called Asset Finance Companies,
— The Insurance Regulatory and Development Authority of Non-Banking Companies which manages, conducts and
India was established in the year 1999 by the (MNBCs) supervises Chit Funds.
Government of India.
48 Magbook ~ Indian Economy

Self Check
Build Your Confidence
1. Consider the following statements Which of the statement(s) given above is/are
1. Indian depository receipt is an instrument denominated in Indian correct?
Rupees in the form of a depository receipt created by the (a) Only 1 (b) Only 2
custodian of securities registered with the Securities and (c) Both 1 and 2 (d) Neither 1 nor 2
Exchange Board of India against the underlying equity of issuing 7. What is ‘NIKKEI’? [BPSC 2000]
company.
(a) Share Price Index of Tokyo Share Market
2. Standard Chartered PLC became the first global company to file
(b) Name of Japanese Central Bank
for an issue of Indian depository receipts in India.
(c) Japanese Name of Country’s Planning
Which of the statement(s) given above is/are correct? Commission
(a) Only 1 (b) Only 2 (d) Foreign Exchange Market of Japan
(c) Both 1 and 2 (d) Neither 1 nor 2
8. The minimum and maximum investment limits
2. ‘Basel III Accord’ after seen in the news, seeks to [UPSC 2015] under sovereign gold bonds are  gm and 
(a) Develop national strategies for the conservation of biological gm of gold per person per fiscal year
diversity respectively.
(b) Improve banking Sector’s ability to deal with financial risk (a) 5 and 200 (b) 2 and 500
management (c) 15 and 700 (d) 50 and 800
(c) reduce the green house gas emisssions
(d) Transfer technology from developed countries to poor countries 9. Consider the following statements [IAS 2004]
1. The National Housing Bank (NHB), the apex
3. With reference to Indian Capital market, consider the following
institution of housing finance in India, was
statements
set-up as a wholly-owned subsidiary of the RBI.
1. CRISIL was set-up in the 8th Five Year Plan. 2. The Small Industrial Development Bank of
2. CRISIL rates the debt instruments of the public sectors. India (SIDBI) was established as a
Which of the statement(s) given above is/are correct? wholly-owned subsidiary of the Industrial
(a) Only 1 (b) Only 2 Development Bank of India (IDBI).
(c) Both 1 and 2 (d) Neither 1 nor 2 Which of the statement(s) given above is/are
correct?
4. Which one of the following Companies is eligible for the financial
assistance and loans from the Industrial Finance Corporation of (a) Only 1 (b) Only 2
India (IFCI)? (c) Both 1 and 2 (d) Neither 1 nor 2
(a) Limited Public Companies (b) Public Co-operatives 10. A rise in SENSEX means [IAS 2000]
(c) Private Limited Companies (d) All of these above (a) a rise in prices of shares of all companies
5. Consider the following statements registered with Bombay Stock Exchange
(b) a rise in prices of shares of all companies
1. A committee, under the chairmanship of former RBI Governor
registered with National Stock Exchange
Bimal Jalan, was constituted to scrutinise the application for
(c) a rise in prices of shares of all companies
new Banks in India.
belonging to group of companies registered
2. The committee recommended to give banking licenses to with Bombay Stock Exchange
Bandhan Micro Finance and Infrastructure Development and (d) None of the above
Finance Corporation (IDFC).
Which of the statement(s) given above is/are correct? 11. Consider the following with policy reference to
(a) Only 1 (b) Only 2 Indian Economy [UPSC 2015]
(c) Both 1 and 2 (d) Neither 1 nor 2 1. Policy rate
2. Open market operations
6. Consider the following statements about the Indian Capital
Market 3. Public debt
1. The Security Exchange Board of India (SEBI) was set-up in the 4. Public reverse
7th Five Year Plan. Which of the policies given above is/are
2. The Capital Issue (Control) Act, 1947 was repealed and replaced components of monetary policy are correct?
by the SEBI. (a) Only 1 (b) 2, 3 and 4
(c) 1 and 2 (d) 1, 3 and 4
1. (c) 2. (b) 3. (b) 4. (d) 5. (c) 6. (c) 7. (a) 8. (b) 9. (c) 10. (c)
11. (c)
Chapter six
Inflation
Germany after World War I
Inflation —
experienced such inflation; while
India has restored — Inflation refers to the persistent rise in Bolivia experienced it in mid-1985.
general price level in the country over a
macroeconomic and
period of time. Inflation could be monetary or Bottleneck Inflation
financial stability, but price inflation. During periods of inflation, — It takes place when the supply falls
structural impediments there is an increase of the money supply. drastically and the demand remains
to growth and — When you have inflation more money is being at the same level.
persistently high circulated, which causes the currency to lose
its purchasing power, which leads to an
inflation remain key
increase in the price of goods and services.
Causes of
concerns policies meant Over the course of many years, economic Inflation
to reduce inflation often cycles go through periods of inflation, — Inflation is caused due to a mismatch
lead to lower growth deflation and stagflation. between demand and supply, i.e.
and thus, reduction in — Each one of these, has a specific effect on when demand exceeds supply. Thus,
the overall economy as a whole and inflation can occur due to changes in
employment while
sometimes can lead to long periods of the demand side or the supply side
policies meant to recessions or depressions in the economy. or both.
increase growth and
employment are Types of Inflation Demand Side
accompanied by high Inflation
inflation.
Low Inflation/Creeping — It is also known as demand pull
Inflation inflation. Increase in demand can
occur due to many reasons, such as:
— It is an inflation that is slow and on
— Increase in public expenditure,
predictable lines.
especially by the government
— This inflation takes place in a longer run and operating large fiscal deficits.
the range is generally in a single digit. — Loose Monetary Policy of the Central
Bank, which leads to low interest
Galloping Inflation rates and thus, higher consumption.
— Rapid GDP growth, which leads to
— It is a very high inflation running in the range
more employment, higher wages.
of double digits or triple digits.
— Increase in population.
— It is also known as hoping inflation, jumping — Depreciation of exchange rate, which
inflation or running inflation. reduces imports, increases exports
and thus, pulls up demand.
Hyper Inflation — Reduction in direct taxes, which
puts more money in the hands of
— It is an inflation, which is large and
households.
accelerating, which might have annual rates
— Speculation in commodities market
in millions.
etc.
50 Magbook ~ Indian Economy

Supply Side Inflation On Farmers


— It is also known as cost push inflation. — Farmers usually gain during inflation, because they can get
Factors influencing inflation from the supply side can also better prices for their harvest during inflation.
be many, such as:
— Backward agricultural sector, which is not able to produce On Investors
enough food. — Those, who invest in debentures and fixed-interest bearing
— Inefficient storage, transportation and marketing securities, bonds etc lose during inflation. However,
infrastructure, which leads to wastage and reduction in
investors in equities benefit because more dividend is
supplies.
yielded on account of high profit made by joint-stock
— Hoarding by traders of essential items, artificially reduces
supply and causes inflation. companies during inflation.
— Rise in the prices of crude oil, fertilizers etc. — Inflation will lead to deterioration of gross domestic savings
— Rise in labour costs. and less capital formation in the economy and less
— Higher costs of imported materials. long-term economic growth rate of the economy.
— Higher costs of capital due to squeezing of credit by the
Central Bank.
— Cartelisation by a few big suppliers to fix prices arbitrarily to Other Inflation Related Concepts
make undue profits. Deflation A general decline in prices, often caused by a
— Monopoly of a single supplier in the market, enabling him to reduction in the supply of money or credit. Deflation can be
set arbitrary prices. also caused by a decrease in government, personal or
— Pushing up of profits by the management of a company by investment spending. The opposite of inflation, deflation has
increasing the prices also leads to inflation.
the side effect of increased unemployment since, there is a
— It has to be understood that it is not always easy to lower level of demand in the economy, which can lead to an
differentiate between demand and supply side inflation and
economic depression.
an example from the demand side can also be explained
from the supply side and vice-versa. Stagflation When you have a slow economy with high inflation
rates and unemployment, stagflation is usually the result.
When the economy does not grow and prices continue to rise
Effects of Inflation you have a stagflation cycle in the economy.
— The effect of inflation is different on different Disinflation This is a reduction in the rate of inflation over time,
communities. When price rises or the value of money even though inflation itself may be positive.
falls, some groups of the society gain, some lose and Reflation It is an attempt to bring back inflation in an economy,
some stand in between. Let us discuss the effects of which is in deflation so as to induce growth.
inflation on distribution of income and wealth, production
on the society as a whole

On Business Community Measures of Inflation


— Inflation is welcomed by entrepreneurs and businessmen — Inflation refers to the changes in general price level in the
because they stand to profit by rising prices. country over a period of time. There are three standard
— They find that the value of their inventories and stock of measures of inflation, viz
goods is rising in money terms. They also find that prices (i) Wholesale Price Index (WPI)
are rising faster than the costs of production, so that their (ii) Consumer Price Index (CPI)
profit is greatly enhanced. (iii) GDP deflator.
— In India, to measure the price level, the Wholesale Price
On Fixed Income Groups Index (WPI) and the Consumer Price Index (CPI) are used.
— Inflation hits wage-earners and salaried people very hard.
Although wage-earners, by the grace of trade unions, can Wholesale Price Index (WPI)
chase galloping prices, they seldom win the race. — It measures the change in wholesale prices on weekly
— Since, wages do not rise at the same rate and at the basis. On the basis of weekly indices, average annual WPI
same time as the general price level, the cost of living is worked out. Average annual wholesale prices of the
index rises and the real income of the wage earner current year are related to average annual wholesale prices
decreases. of the base year (assumed as 100).
Magbook ~ Inflation 51

New Series WPI New Consumer Price Indices


The new series of the WPI has 697 items as compared to 676 — At the retail level, CPI is meant to reflect the cost of living
items in old series. New vegetables and fruits such as radish, conditions and is computed on the basis of the changes
carrot, cucumber, bitter gourd, mosambi (sweet lime), in the level of retail prices of selected goods and services,
pomegranate, jackfruit and pear have been added to the list of on which consumers spend the major part of their
primary articles. In the mineral group, items like copper income.
concentrate, lead concentrate and garnet have dolomite and — Therefore, a broad-based CPI for the country as a whole,
magnesite have been removed. including both services and manufacturing products, has
Natural gas has been added as a new item. In the greater relevance for Monetary Policy formulation.
manufacturing items list, around 173 new items including — In India, however, data on CPI relates to different
conveyer belt, rubber tread, steel cables, tissues paper and segments of the population rather than the entire
wooden splint have been added, while 135 items like
population. With a view to addressing this issue, the
khandsari (unrefind raw white sugar), poppadom and video
Reserve Bank has taken the initiative and prepared an
CD players have been removed.
approach paper on CPI (Urban) and CPI (Rural).
— Subsequently, the Central Statistical Organisation (CSO)
Producer Price Index has taken up the work for generating data on CPI (Urban)
and CPI (Rural). The new CPIs once compiled will go a
Even, when we have a revised WPI, we would still not have a long way in filling a major data gap in price statistics.
Producer Price Index (PPI). The PPI covers price changes
— The Central Statisticals Office (CSO), Ministry of Statistics
faced by the producers on primary, intermediate and finished
and Programme Implementation, has revised the base
goods and services ready for the market.
year of the CPI from 2010 to 2012.
The primary difference between the WPI and the PPI is, in
addition to the coverage, that the WPI reflects changes in the April, 2017 (Provisional)
average cost of production including mark-ups and taxes,
while the PPI measures price changes of transacted goods at Indices Rural Urban Combined
the gate excluding taxes. CPI 3.75 3.96 3.89
The purpose of the PPI is to provide a measure of prices CFPI 1.89 2.42 2.01
received by producers of commodities. The PPI usually covers
the industrial (manufacturing) sector as well as public utilities Inflation Based on the WPI VS CPI
(electricity, gas and communications). Some countries also
— Inflation in all major indices largely followed each other.
include agriculture, mining, transportation and business
The gap between WPI and CPIs had widened in 2009-10,
services.
due to higher food inflation as food items have a much
larger weight in CPI vis-a-vis the WPI.
— Food items contribute a weight of 46.20% in CPI-IW and
Consumer Price Index (CPI) 69.15% in CPI-AL as against 24.31% in WPI.
— It measures the change in retail prices on monthly basis.
On the basis of monthly indices, average annual CPI is Introduction of CPI (Urban) and CPI (Rural)
worked out. Average annual retail prices for the current — The Central Statistical Organisation (CSO) has taken up a
year are related to the average annual retail prices of the new initiative of compilation of CPI (Urban) and CPI
base year (assumed as 100). Like Wholesale Price Index, (Rural) and CPI (rural+urban) for all States or UTs and all
different goods are accorded weights depending on their India by considering all sections of the urban and rural
relative significance. population.
— It needs to be emphasised that while WPI includes goods — In urban areas, all cities or towns having population (2001
only, CPI includes both goods as well as services. Another Population Census) more than 9 lakh and all State or
important feature of CPI is that it focuses on a Union Territory capitals not covered therein were selected
homogeneous group of consumers. purposively. In all 310 towns have been selected either on
— CPI reflects cost of living of the concerned category of purposive or random basis, from which 1114 quotations
consumers. CPI for the industrial workers is a widely used (price scheduled) are canvassed every month.
index in India. It is also used to determine the dearness — In rural areas, with a view to having a manageable
allowances of government employees. workload and considering that the CPI (Rural) would
— Besides, CPI of agricultural labourers, CPI for urban provide the price changes for the entire rural population of
non-manual employees are also calculated by the the country, a total of 1183 villages have been selected at
economists for some specific purposes. all India level.
52 Magbook ~ Indian Economy

The CSO has also decided to bring out a national CPI by


—
merging CPI (Urban) and CPI (Rural) with appropriate Measures to Check Inflation
weights, as derived from NSSO 61st round of Consumer — The handling of India’s inflation challenge consists of a
Expenditure Survey (2004-05) data. careful combination of effort on the part of the RBI and
government, including the Ministry of Finance and
GDP Deflator several other ministries, alongwith advisory support by
— It refers to the ratio between GDP at current prices and GDP the Inter-Ministerial Group (IMG) on inflation.
at constant prices. If GDP at Current Prices = GDP at — The IMG on inflation recommended several steps for
Constant Prices, GDP deflator = 1, implying no change in improving supply chains from the farmer to the
price level. If GDP deflator is found to be 2, it implies rise in consumer. It recommended amending the Agricultural
price level by a factor of 2 and if GDP deflator is found to be Produce Marketing Committee (APMC) Acts in order to
4 , it implies a rise in price level by a factor of 4. cut down on the large middleman price margin. It also
— GDP deflator is acclaimed as a better measure of price recommended that one way to improve the supply chain
behaviour because it covers all goods and services and benefit farmers is to allow FDI in multi-brand retail.
produced in the country. — The IMG argued that if this was permitted within a
carefully crafted regulatory framework, there could be
Core Inflation large gains for farmers and also for ordinary consumers.
— Another way to analyse inflation data is by looking at core — The fiscal measure, the administrative measure and the
inflation, which is generally a chosen measure of inflation monetary measure are the three different ways to
that excludes the more volatile categories like food and contain inflation.
energy prices. The main argument, here is that the Central
Bank should effectively be responding to the movements in Fiscal Measures
permanent component of the price level rather than — Fiscal policy can be effectively employed to check
temporary deviations. It is, therefore, a preferred tool for inflation. Manipulation of public expenditure, taxation
framing long-term policy. and public debt can be used for this purpose.
— Government can spend more money in the
Change in Reporting of developmental sphere to increase supply by improving
Inflation productivity. Tax incentives can also be used to improve
the supply situation.
— At present, the WPI for all commodities including
manufactured products is released only on a monthly basis. Administrative Measures
However, until recently WPI for primary articles and the fuel — The authority to take decisions on this front is with the
group was also being released on a weekly basis. But, it was executive branch of the government. Under this
observed over a period of time that, there was a tendency measure, they:
for upward revisions in the indices reported once the final
— remove levy obligations in respect of imported materials.
numbers were later released.
— ban export of constrained materials.
— The higher frequency weekly reporting was thus, prone to — maintain the central issue price, particularly for rice and
more statistical noise and sometimes provided a misleading wheat.
picture, so the trade-off was between the more frequent — suspend future trading.
and less reliable data and less frequent, but more reliable — allot rice and wheat under Open Market Sale Scheme
data. International practice for reporting CPI inflation is also (OMSS) and many more.
on a monthly basis.
— In view of this, the Cabinet Committee on Economic Affairs Monetary Measures
(CCEA) in its meeting held on 24th January, 2011, agreed to — Monetary measures come under the purview of Reserve
discontinue the weekly release of WPI for the commodities. Bank of India (RBI). Through, the Monetary Policy
Items under the groups primary articles and fuel and review, RBI tries to control price rise and maintain
power with immediate effect. WPI shall, henceforth, be economic growth and financial stability.
released on a monthly basis only.
Self Check
Build Your Confidence

1. Consider the following statements 7. In India, inflation measured by the [ RPSC 2013]
1. Headline inflation is a measure of the total inflation (a) Wholesale Price index number
within an economy. (b) Consumer Price index for urban non-manual worker
2. Headline inflation is affected by areas of the market (c) Consumer Price index for agriculture labours
which may experience sudden inflationary spikes (d) national income defaultor
such as food, vegetables or energy.
8. Which one of the following is likely to be the most
Which of the statement (s) given above is/are correct? inflationary, in its effect? [IAS 2013]
(a) Only 1 (b) Only 2
(a) Repayment of public debt
(c) Both 1 and 2 (d) Neither 1 nor 2
(b) Borrowing from the public to finance a budget deficit
2. Economic growth is usually coupled with [IAS 2011] (c) Borrowing from banks to finance a budget deficit
(a) deflation (d) Creating new money to finance a budget deficit
(b) inflation 9. Which one of the following measures is generally used by
(c) stagflation the government to contain the recession of the economy?
(d) hyper inflation [UPPCS 2009]
3. The rate of inflation in India is measured in respect (a) Increasing money supply
(a) Consumer Price index (b) Increasing government spending
(b) Wholesale Price index (c) Decreasing taxation
(c) money supply (d) All of the above
(d) cost of living index for industrial worker 10. Consider the following statements regarding the
4. Which of the following is likely to be the most determination of inflation in India.
inflationary in its effect? 1. Food Price index consists of two sub-components, namely
primary food articles and manufactured food products.
(a) Repayment of public debt
(b) Borrowing from the public to finance a budget deficit 2. The weight of the primary food articles is less than the
(c) Borrowing from banks to finance a budget deficit manufactured food products.
(d) Creating new money to finance a budget deficit Which of the statement (s) given above is/are correct?
(a) Only 1 (b) Only 2
5. A rise in general level of prices may be caused by (c) Both 1 and 2 (d) Neither 1 nor 2
1. an increase in the money supply.
2. a decrease in the aggregate level of output. 11. Consider the following statements
3. an increase in the effective demand. 1. CPI measures price change in both goods and services.
Which of the statement(s) given above is/are correct? 2. WPI does not measure price change in services.
(a) Only 1 (b) 1 and 2 Which of the statement(s) given above is/are correct?
(c) 2 and 3 (d) All of these (a) Only 1 (b) Only 2
(c) Both 1 and 2 (d) Neither 1 nor 2
6. Consider the following statements
1. Inflation benefits the debtors. 12. With reference to inflation in India, which of the following
statements is correct? [UPSC 2015]
2. Inflation benefits the bond holders.
(a) Controlling the inflation in India is the responsibility of the
Which of the statement (s) given above is/are correct?
Government of India only
(a) Only 1
(b) The Reserve Bank of India has no role in controlling the
(b) Only 2
inflation
(c) Both 1 and 2
(c) Decreased money circulation helps in controlling the inflation
(d) Neither 1 nor 2
(d) Increased money circulation helps in controlling the inflation

1. (c) 2. (b) 3. (b) 4. (d) 5. (d) 6. (a) 7. (a) 8. (d) 9. (d) 10. (a)
11. (c) 12. (c)
Chapter seven
Public Finance
Public Finance (ii) Unearned Revenue is that revenue,
which is mobilised by the government
Public finance is the — The study of government’s revenue and without any contractual obligations to
expenditure is called as public finance. The the payee. It includes taxes, fines
study of the financial
boundary of public finance in modern time and forfeitures, special assessment
health of State is not limited to ways and means of levies, escheats, gifts and grants etc.
Government and local government income and expenditure only, Sources of public revenue can also
bodies in India. Public but it also studies public debt, financial be classified into Tax and Non-Tax
administration and Fiscal policy of the revenue.
finance assesses the
economy. So, we can say that public
government revenue finance studies revenue and expenditure of Tax Revenue
and government government of an economy and all the
— Tax is a compulsory payment by the
expenditure of the activities related to it. Public finance can be
citizens to the government to meet the
divided into five sections which are as
public authorities and public expenditure. It is legally imposed
follows :
by the government on the taxpayer and
one or the other to (i) Public revenue in no case taxpayer can deny to pay
achieve desirable effects (ii) Public expenditure taxes to the government.
and avoid undesirable (iii) Public debt
ones. (iv) Fiscal policy and Types of Tax
(v) Financial administration — Tax can be direct or indirect : income
tax, wealth tax, gift tax etc are the
Public Revenue examples of direct taxes and sales
taxes, excise duty, customs duty etc
— Public revenue, an indispensable organ of
are the examples of indirect taxes.
public finance operation, includes all
income and receipts of the government
through various sources. Different means of
Direct Tax
revenue to the government are called — A direct tax is that, which is borne by
sources of public revenue. Sources of the person on whom it is levied. A
public revenue can be broadly divided into direct tax cannot be shifted to other
two categories such as : person.
(i) Earned Revenue is the kind of revenue, — Direct as well as indirect money burden
which is received from certain assured of the direct tax is on the person on
sources kept under the complete whom the tax is imposed. Impact of the
disposal of governmental ownership. It tax as well as incidence of the tax is on
includes public domain like rent, the same person.
royalties, sales of forest products etc and — As a proportion of gross tax revenue,
commercial revenues like profits of direct taxes have been accounting for
public sector enterprises, public utility over a half of total tax revenue since
services etc. 2007-08.
Magbook ~ Public Finance 55

Some of the direct taxes are as follows : Value Added Tax (VAT)
— Personal Income Tax It is the tax levied directly on the — VAT is a multi point sales tax with set-off for tax paid on
income of individuals by the Central Government. purchases of inputs. There is no cascading (tax on tax) effect
Income sources is added for taxation. as there is credit mechanism for tax paid on inputs. The tax is
— Corporate Tax It is levied on the profit of the levied on the value of the product and consumption only. Total
companies or corporations. Now, the corporate tax burden of the tax is borne by the consumer only.
rate is 30%. To prevent companies from avoiding — VAT is simply a new name for the sales tax of states, in which a
taxes a Minimum Alternate Tax (MAT) at 15% of book number of other indirect taxes have been merged. Haryana was
profit is levied. It is the largest source of revenue of the first state to introduce VAT from 1st April, 2003. Now, most
the Central Government, covering about 18% of the states have introduced VAT.
total revenue. — Value added = Total sales − Cost of intermediate consumption.
— Wealth Tax This tax is levied on the net wealth of the
individuals, Hindu undivided family and joint stock Advantages of VAT
companies. To assess net wealth, net obligations are — No cascading effect.
deducted from its market value. It is a minor source — Increase in tax base.
of revenue of the government, primarily imposed to
— Reduced evasion due to transparency.
reduce concentration of wealth in the society.
— Multiple taxes are merged and tax system is simplified.
Gift Tax Central Value Added Tax (CENVAT)
— This tax is imposed by the Central Government on all
— The basic purpose of CENVAT is to eliminate the cascading
donations and gifts over and above the prescribed
effects of the taxes by Tax Credit system.
limits to the family members. However, donation
given by the charitable institutions and companies is — Under the CENVAT scheme, a manufactures of final product
not covered under gift tax. This tax is basically or provider of taxable service shall be allowed to take credit of
imposed to check the evasion of estate duty and duty of excise as well as service tax paid on input received.
wealth tax. Custom Duties
— Interest Tax This tax is imposed on the interest — These duties are imposed on commodities, which are to be
income of the commercial banks on their gross loans imported or exported from India. In other words, when a goods
and advances. Now, it is not in force in India. cross the political boundary of a country or come from other
— Recent Initiatives for Direct Taxes Alternate Minimum countries, custom duties are imposed. Like excise duties,
Tax (AMT) has been extended to non-company customs duties also contribute largely to the government
assets. This move has been taken to widen the tax revenue.
base. In order to bring about greater certainty and to
Service Tax
reduce litigation in matters related to transfer, pricing
and international taxation, the Advance Pricing — Comparatively a new concept in India, service tax is a tax
Agreement (APA) scheme has been notified. imposed on the person, who avails any specified service. Its
importance as a source of revenue has been increasing in
Indirect Tax recent years.
— The government is receiving more and more revenue from
— Indirect taxes are those taxes, which have their service tax. Because of this after year, more and more services
primary burden or impact on one person, but that are being covered under the service tax net.
person succeeds in shifting his burden on to others.
— This tax was introduced in India in 1994-95. With economic
— Consequently, the final or the real burden of the taxes
growth and expansion of service sector in the economy,
or the incidence has to be borne by a third person. In revenue from service tax has been increasing over the years.
India, sales tax, excise duty, custom duty etc are the From Budget 2014-15, the negative list concept in service tax
examples of indirect taxes. has been reformed and a number of services have been
Some of the indirect taxes are as follows : brought under the ambit of the service tax. However, some of
Central Excise Duties the components under the negative list have been kept intact.
— Corporate tax is largest source of revenue generation for the
— Central excise duties are imposed by the Central
Indian Government.
Government on the goods produced within the
country except certain goods on which State — Surplus budget is the revenue of the financial year are greater
Governments are empowered to impose tax. These than anticipated expenditures.
goods include liquor, drugs etc.
56 Magbook ~ Indian Economy

duty (known under CVD), Special Duty of Custom Duty


Ways and Means Advances (WMA) (SAD), Service tax and gratuity surcharge related to the

Ways and means advances are provided by the RBI to the supply of goods or services were separate taxes. All these
states, banking with it, to help them to tide over temporary included in CGST.
mismatches in the cash flow of their receipts and payments. — State GST (SGST) Taxes imposed and collected by the State

Such advances are under the RBI Act, 1934, repayable in government on goods and services are levied under State
each case not later than 3 months from the date of making GST system. Earlier, State governments pay VAT under State
that advance. taxes, purchase tax, entry tax, entertainment tax,

These are two types of WMA is normal and special. advertisement tax including State excise and surcharge
related to State sub-tax and imposition, lottery taxes, tax on
speculation and gambling. All these taxes now included in
Goods and Services Tax (GST) SGST.
— Government of India implemented GST from 1st July, — Integrated GST (IGST, State Indemnification) The proposed
2017. It converts the country into unified market, Goods and Services tax provides an integrated GST, (IGST)
replacing most indirect taxes with one tax. It is levied levied on international commodities and services. It is
both on goods (manufacturing) and services. imposed and recovered by the Central Government.
— It is an integrated scheme of taxation that does not The amount of taxes received under this tax is distributed to
discriminate between goods and services and is a part the state for the loss of revenue generated to the States.
of the proposed tax reforms that centre on evolving an — Union Territory GST (UTGST) Arrangement or provision
efficient and harmonised consumption tax system in the under the UTGST and Tax system is for the Union Territory
country. Key features of the GST are as follows : where they do not have their own Legislative Assemblies,
(i) Two components one levied by the centre (referred to such as Andaman and Nicobar Islands, Daman and Diu and
as Central GST) and the other levied by the states Dadar and Nagar Haveli etc. These Union Territories have
(referred to as State GST), rates for which would be the provision to and collect taxes by the Central
prescribed appropriately. Government.
(ii) The Central GST and the State GST would be Taxes Out of GST
applicable to all transactions of goods and services
except the exempted goods and services. ◆
Taxes that are not included in any of the provision of the GST,
(iii) The Empowered committee has decided to adopt a they contain alcohol, real estate, crude oil, petrol, natural gas and
two-rate structure a lower rate for necessary items and the fuel for turbine. All these items will be out of GST provision
goods of basic importance and a standard rate for and the current taxation system will be applicable on them.
goods in general. There will also be a special rate for
precious metals and a list of exempted items. Direct Tax Code (DTC)
(iv) The GST will be levied on import of goods and services — DTC was proposed by the United Progressive Alliance (UPA)
into the country. Government to consolidate the law relating to the direct
(v) The administration of the Central GST to the Centre taxes. The bill seek to replace the Income Tax Act, 1961
and for State GST to the states would be given. and Wealth Tax Act, 1957. The bill, in its original form,
widened the tax slabs and lowers corporate tax rates. It also
(vi) Central Taxes replaced by GST Bill Central Excise
removed a number of exemptions and grant for some other.
Duty, Additional Duties of Excise and Customs,
DTC provision introduced in the Budget 2012-13 are as
Special Additional Duty of Customs (SAD), Service Tax
follows :
and Cesses and Surcharges on supply of goods and
— General Anti- Avoidance Rule (GAAR).
services.
— Advance Pricing Agreement (APA).
(vii) State Taxes Subsumed in the GST Bill VAT, Central
— Income tax exemption limit rose to ` 2 lakh.
Sales Tax, Purchase Tax, Luxury Tax, Entry Tax,
Entertainment Tax, Taxes on advertisements, lotteries, — Upper limit of 20% tax slab rose to ` 10 lakh.
betting, gambling and State Cesses and Surcharges. — 20% cut in Securities Transaction Tax (STT).
Types of GST
GAAR
Under the GST form, four types of GST are
— Central GST (CGST) Under the CGST, there is a
— The General Anti-Avoidance Rule (GAAR) was proposed in
provision to impose tax on the supply of goods and mid-March as a hart of budget for fiscal year 2013.
services by the Central government. Earlier, Central — GAAR was scheduled to come into effect from 1st April,
Excise, Excise (Drugs and Toilet construction), Excise 2013. In the Budget 2013-14, it was announced that a
duties on the taxes imposed by the Central Government, modified version of GAAR provisions will come into effect
(Goods of special importance), additional duty of custom from April, 2016.
Magbook ~ Public Finance 57

— GAAR aims to target tax evaders partly by stopping Indian — Duties of excise on tobacco and certain goods
companies and investors from routing investment to Mauritius manufactured or produced in India.
or other tax heavens for sole purpose of avoiding tax. — Estate duty in respect of property other than agricultural
— GAAR was scheduled to come into effect from 1st April, land.
2013. In the Budget 2013-14, it was announced that a — Fees in respect of any matters in the Union list, but not
modified version of General Anti-Avoidance Rule (GAAR) including any fees taken in any court.
provisions will come into effect from April, 2016. A number — Foreign loans.
of representation were received against GAAR provisions
— Lotteries organised by the Government of India or the
introduced in the last budget.
Government’s State.
Non-Tax Revenue — Post office savings bank.
— Post and telegraphs, telephones, wireless, broadcasting
— Non-tax revenue are those receipts, which are received from
and other like forms of communication.
sources other than taxes like fees, fines etc. Some of the
non-tax revenue are as follows : — Property of the union.
— Fee, License and Permit One of the main sources of non-tax — Public debt of the union.
revenue of the government are fee, license and permit. — Railways.
— Fee A fee is a payment to the government for the services that it — Rates of stamps duty in respect of bills to exchanges,
renders to the people. Examples of fees are: land registration fee, cheques, promissory notes etc.
birth and death registration fee, passport fee, court fee etc. — Reserve Bank of India.
— License and Permit The amount that the government charges for — Taxes on income other than agricultural income.
allowing the people to perform a given job, is called license or — Taxes on the capital value of the assets, exclusive of
permit fee. “License fees are charged to give permission for
agricultural land of individuals and companies.
something by the government.” e.g. driving license, import
license. There is a difference between fees and license fees. — Taxes other than stamp duties on transactions in stock
License fees are paid, when a person is permitted to do some exchanges and future markets.
specific job by the government. No service is provided to the — Taxes on the sale or purchase of newspapers and on
license holder. advertisements published therein.
— Escheat Escheat refers to that income of the state, which arises — Terminal taxes on goods or passengers, carried by
out of the property that comes to it for want to a legal heir. Such a railways, sea or air.
property has no claimant. State alone has the legal right over it.
— Special Assessment It is yet another source of non-tax revenue State Sources
of the government. Special assessment is that payment, which is
made by the owners of those properties, whose value has
— Capitation tax.
appreciated due to developmental activities of the government. — Duties in respect of succession to agricultural land.
— Fines and Penalties Fines and penalties are those payments, — Duties of excise on certain goods produced or
which are made by the law breakers to the government by manufactured in the state, such as alcoholic liquids
way of economic punishment. The aim is not to earn opium etc.
revenue. Its actual aim is to force the people to be law — Estate duty in respect of agricultural land.
abiding (follow the rules and regulations). — Fees in respect of any matters in the State list, but not
— It is determined by the government in an arbitrary manner including fees taken in any court.
and not on the basis of administrative cost. — Land revenue.
— Rates of stamps duty in respect of documents other
— Income from Public Enterprises Several public enterprises
than those specified in the Union List.
are owned by the government. For instance, Indian
— Taxes on agricultural income.
Railways, Nangal Fertilizer Factory, Indian Oil, Bhilai Steel
Plant etc. — Taxes on land and buildings.
— Profit from sale proceeds of the products of these — Taxes on mineral rights, subject to limitations imposed
enterprises constitutes the income of the government. by Parliament relating to mineral development.
— Taxes on the consumption or sale of electricity.
Sources of Revenue — Taxes on the entry of goods into a local area for
consumption, use or sale therein.
— The following list will show the respective sources of revenue — Taxes on the sale and purchase of goods other than
for the Union and the States. newspapers.
Union Sources — Taxes on advertisements other than those published in
newspapers.
— Corporation tax. — Taxes on goods and passengers carried by road or on
— Currency, coinage and legal tender, foreign exchange. inland waterways.
58 Magbook ~ Indian Economy

— Taxes on vehicles. — Pigouvian Tax It is a tax, which is imposed on bodies having


— Taxes on animals and boats. negative externalities. An example of pigouvian tax is the carbon
— Taxes on professions, trades callings
tax levied in some countries for causing pollution.
and employments. — Tobin Tax It is a tax levied on foreign exchange transactions
— Taxes on luxuries, including taxes on
both when foreign capital enters a country and when it leaves. It
entertainments, amusements, betting and gambling. is meant to check speculative flows.
— Tolls.
— Transfer Pricing It is the price at which divisions of a company
transact with each other. Transactions may include trade of
Duties Levied by the Union, but Collected and
supplies or labour. It is used when individual entities of a larger
Appropriated by the States (Article 268)
firm are treated as separately run entities.
— Stamp duties and duties of excise on medicinal and
— Specific Duty Tax is levied based on weight or quantity.
toilet preparations (those mentioned in the Union
list) shall be levied by the Government of India but
— Ad Valorem Tax is levied based on value and not an weight or
shall be collected. quantity.
— In the case, where such duties are leviable within any — Withholding Tax It means withholding tax of certain payments
Union Territory, by the Government of India. such as salary to employees, payments to contractors, interest
— In other cases, by the States within which such duties etc. It is the same as Tax Deducted at Source (TDS).
are respectively leviable. — Capital Gains Tax It is the tax on gains made from buying and
Taxes Levied and Collected by the Union, but Assigned selling assets such as land, shares etc. Gain made on assets
to the States (Article 269) held for over three years (one year for shares) is called
— Duties in respect of succession to property other
long-term capital gain.
than agricultural land. — Base Erosion and Profit Shifting (BEPS) It is of major
— Estate duty in respect of property other than
significance for developing countries due to their heavy reliance
agricultural land. on corporate income tax, particularly from multinational
enterprises. BEPS refers to tax planning strategies that exploit
— Taxes on railway fares and freights.
gaps and mismatches in tax rules to artificially shift profits to low
— Taxes other than stamps duties on transactions in
or no-tax locations where there is little or no economic activity.
stock exchanges and future markets.
— Taxes on the sale or purchase of newspapers and Tax Related Primary Concepts
on advertisements published therein.
— Tax Evasion It is the illegal evasion of taxes by individuals,
— Terminal taxes on goods or passengers carried by
corporations and trusts. Tax evasion often entails taxpayers
railways, sea or air. deliberately misrepresenting the true state of their affairs to the
— Taxes on the sale or purchase of goods other than tax authorities to reduce their tax liability and includes dishonest
newspapers where such sale or purchase takes tax reporting, such as declaring less income, profits or gains
place in the course of inter-state trade or than the amounts actually earned or overstating deductions.
commerce. — Tax Shifting Transferring some or all of a tax burden of an entity
Taxes which are Levied and Collected by the Union, (such as a subsidiary) to another (such as the parent firm) is tax
but which may be distributed between the Union and shifting. Tax shift or tax swap is a change in taxation that
the States (Articles 270 and 272) eliminates or reduces one or several taxes and establishes or
— Taxes on income other than agricultural income. increases others while keeping the overall revenue unchange.
— Union duties of excise other than duties and taxes — Tax Avoidance It is the legal usage of tax regime to one’s own
referred to in Articles 268, 268A and 269. advantage to reduce the amount of tax that is payable by means
— Taxes on income does not include corporation tax. that are within the law. Tax shattering is very similar term and
The distribution of income tax proceeds between tax havens are jurisdictions which facilitate reduced taxes. The
the union and the states is made on the basis of the term tax mitigation is sometimes used, its original use was by
recommendations of the Finance commission. tax advisers as an alternative to the pejorative term tax
avoidance.
Important Terms Related to — Fiscal Space It is a relatively new term that refers to the
Taxation flexibility of a government in its spending choices and more
generally, to the financial well-being of a government. Peter
— Tax Haven It is a country or territory where certain
Haller (2005) defined it ‘‘as room in a government’s budget that
taxes are levied at a low rate or not at all. Tax haven
allows it to provide resources for a desired purpose without
lead to loss of revenue for governments, money
jeopardising the sustainability of its financial position or the
laundering etc. Cayman islands, Gibraltar,
stability of the economy.’’
Liechtenstein etc are some of the tax havens.
Magbook ~ Public Finance 59
* Higher fiscal deficits usually lead to rising public debt. Tax Expenditure/Revenue
India’s Central Government liabilities GDP ratio has
infact come down since 2002-03. Foregone
* The government appointed a committee headed by — There is significant divergence between the statutory
Dr Vijay Kelkar to check out a roadmap for fiscal rates of taxes as notified in the various schedules and
consolidation. the actual or effective rate of taxation, which is
essentially a simple ratio of tax revenue collected to
Public Expenditure the tax base. The arises on account of the exemptions
to the tax rate specified in the schedule.
— Before independence, the activities of the Government in India — In the case of corporate taxpayers, deduction on
were influenced by classical thinking. Classical economists account of accelerated depreciation, deduction for
opposed the idea of entrusting too many functions to the export profits of export-oriented units located in
government and thus, public expenditure remained limited. Special Economic Zones (SEZs) and profits of
— After independence, the government decided to interference businesses in the power and telecom sectors were
directly in the economy of the country. It was realised that some of the major incentives.
economic growth and welfare of the people cannot be increased — The exemption under Section 80C of the Income Tax
without government playing a major role. act lead to a significant revenue loss.
— Centralised planning and the public sector were to play a — As far as indirect taxes are concerned, revenue
major role in this. Thus, the scope of public expenditure foregone is defined as the difference between duty
increased rapidly after independence. that would have been payable, but for the issue of
exemption notification and actual duty paid in terms
Sources of Government Expenditure of the relevant notification.
There are some sources of Government expenditure given belows : Advantages of Foregoing Revenue
— Subsidies.
— Generates more investment than would have been
— Purchase and provision of goods and services.
possible if the tax was collected and spent by the
— Investments and money transfer. government.
— Creation and maintenance of the military police emergency — Induces economic growth which leads to higher tax
and five fighting organisation. revenues in the future.
Disadvantages of Foregoing Revenue
Debt Management Strategy — Distorts resource allocation.
— In a more aimed at a focused targeting of subsidies for the — Leads to multiplicity of rates, legal complexities,
country’s debt management. The Nilekani Task force was litigation etc.
set-up to come up with an IT improve intro strategy for — Government needs more money to spend on social
kerosene. The main recommendations are as follows : and physical infrastructure of the country which is
— Set-up a strong, robust IT infrastructure backbone to reform the instead going to the private sector.
functioning of PDS, which would be an incentive based system for — Amount of revenue foregone has become too large in
all stakeholders.
recent years (more than ` 5 lakh crore in a year),
— End to end computerisation of PDS across the country. which is even more than the fiscal deficit.
— PDS Network (PDSN), as the National Information Utility (NIU) to
be the common software platform for all states, to implement and
operate the IT infrastructure for PDS. Public Debt
— Aadhar (UIN) to be used in PDS to simplify ration card registration, — Public debt in the Indian context refers to the
cleaning up the beneficiary database.
borrowings of the Central and State Governments.
— Provision of maximum choice to the beneficiaries in terms of
Public debt of Central Government consists of internal
location, mix of commodities, convenient quantities and in any
number of installments and choice to purchase commodities or and external debt and other liabilities.
receive a direct transfer of subsidy.
— Set-up a Core Subsidy Management System (CSMS) for direct Internal Debt
transfer of subsidies on kerosene, LPG and fertilizer through — Includes market borrowings, money raised by issuing
Aadhar Enabled Bank Account (AEBA) to facilitate robust
bonds treasury bills, special securities issued to the
identification of the beneficiary.
RBI etc.
— Fertilizers movement and sale at market price with reimbursement
of subsidy directly to the beneficiary.
60 Magbook ~ Indian Economy

External Debt any other Constitution. The Finance commission was appointed
2 years after the implementation of the Constitution and every
— Includes borrowings from foreign countries and 5 years thereafter.
international financial institutions. — The President has the power to appoint a new Finance commission
— Non-government external debt includes NRI even before the expiry of 5 years, if he deems it necessary. In
deposits, trade credit, external commercial other words, the appointment of the Finance commission is a
borrowings etc. continuous process according to our Constitution.
— India’s external debt is held in multiple currencies, — Finance commission, generally, presents its recommendations
the largest of which is the United States dollar
regarding the allocation of revenue between the centre and the
(48.2%).
state, so that, there is no discord between the centre and the
states.
Necessity of Public Debt — Main objective of the Finance commission is to assess the
— Public debt is considered necessary for the following financial requirements of the states to determine amount of
reasons : grants- in-aid to be given to various states and to strengthen the
— Smoothening out tax rates. Borrowing means that tax federal financial system.
rates need not be increased or decreased sharply.
— In times of recession economy can be pump-primed by Objectives of Finance Commission
spending money through borrowing.
— To determine the basis for the allocation of funds collected from
— Financing war or other emergency expenditure.
the taxes, which are divisible between the centre and the states.
— To finance expenditure on social sector for human
— To formulate the principles regarding the grants to the states
capital formation.
from the centre.
— Towards capital formation to boost economic activities
— To continue the agreements made between the Government of
in the country.
India and the states or to recommend changes in them.
— Despite these benefits public debt is also criticised — To consider any other financial matter in the interest of the
when it reaches a high level. Reasons for its criticism
country, on being notified by the President to do so.
are as follows :
— On the basis of this arrangement, 15th Finance Commissions
— It creates a burden of interest which has to be paid out
of the current revenue.
have been set-up so far :
— It often does not lead to direct or indirect returns which Finance Finance
makes its repayment difficult. Chairman Chairman
Commission Commission
— It reduces availability of funds to the private sector.
1st (1951) Mr KC Niyogi 9th (1987) Mr NKP Salve
— It can increase inflationary tendencies in the economy
by increasing the money supply. 2nd (1956) Mr K Santhanam 10th (1992) Shri KC Pant
3rd (1961) Mr AK Chanda 11th (1998) Prof AM Khusro
Debt to GDP Ratio 4th (1966) Mr RV Rajamannar 12th (2004) Dr C Rangarajan
— It is the ratio of a country’s national debt to its Gross 5th (1968) Mr Mahaveer Tyagi 13th (2008) Dr Vijay L Kelkar
Domestic Product (GDP). By comparing what a 6th (1972) Mr Brahmananda Reddy 14th (2012) YV Reddy
country owes to what it produces the debt-to-GDP
7th (1977) Mr JM Shelat 15th (2017) N K Singh
ratio indicates the country’s ability to pay back it’s
debt. 8th (1982) Mr YB Chavan

* Raja J Chelliah committee on 29th August,


1991 to make the recommendations for
15th Finance Commission
making the tax system more elastic, broad — Former Planning Commission member N K Singh was
based and also to suggest measures required appointed as the chairman of the 15th Finance Commission,
simplifying the existing laws and regulations. which has been asked to look into the impact of the Goods and
* The higher the debt to GDP ratio, the less Services Tax (GST) on finances of both the centre and the
likely country will be to pay its debt back states.
and higher the risk of its default. — The other members of the commission, which is required to
submit its report by October 2019, are former economic affairs
Finance Commission secretary Shaktikanta Das and former chief economic adviser
Ashok Lahiri, NITI Aayog member Ramesh Chand and
— According to Article 280 of Constitution, the Georgetown University professor Anoop Singh.
President appoints a Finance commission after — The commission will review the current status of the finance,
every 5 years. This provision is also a Fundamental deficit, debt levels, cash balances and fiscal discipline efforts of
feature of Indian Constitution, which is not found in the union and the states.
Magbook ~ Public Finance 61

14th Finance Commission Fiscal Policy


— The 14th Finance commission, headed by former RBI — Fiscal policy is that part of government policy, which is
Governor YV Reddy, has endorsed the compensation concerned with raising revenue through taxation and with
road map for the goods and services tax finalised by deciding on the amount and purpose of government
the Centre, but has called for an autonomous and spending. The idea of using fiscal policy to combat recessions
independent GST compensation fund. was introduced by John Maynard Keynes in the 1930s.
— Members of the 14th Finance Commission YV Reddy, Objectives of Fiscal Policy
Abhijit Sen, M Govinda Rao, Sushama Nath and
Sudipto Mundle. The Government of India on 24th
— Fiscal policy in India has two major objectives :
February, 2015 accepted recommendations of the (i) Improving the growth performance of the economy.
14th Finance commission for increasing share of (ii) Ensuring social justice to the people.
states in central taxes to 42%. — Fiscal policy influences the growth performance in the
— The commission recommended increase in the share following manner :
of states in the centre’s tax revenue from the current — Influencing Resource Mobilisation India has done well in this area
32% to 42%, the single largest increase ever as reflected in the tax GDP ratio which increased from 6.3% in
recommended. 1950-51 to 16.2% in 2011-12.
— Influencing Efficiency of Resource Allocation This is done by an
Recommendations of 14th efficient and rational allocation of resources to maximise the rate
Finance Commissions of growth.
— Fiscal policy also tries to achieve social justice through
— The 14th Finance commission is of the view that tax
various measures :
devolution should be the primary route for transfer of
— Relying more on direct taxes than indirect taxes.
resources to the states. In understanding the states
— Increasing taxes on the rich and expenditure on luxury goods
needs, it has ignored the plan and non-plan
while lowering them on the poor and goods of common use.
distinctions.
— Spending on welfare and development projects.
— In recommending an horizontal distribution, it has
used broad parameters population (1971), changes in Fiscal Imbalance and Deficit
population since then, income distance, forest cover
and area, among others. Financing
— It has recommended distribution of grants to states for — Fiscal imbalance is a situation, in which there is a gap
local bodies using 2011 population data with weight of between revenue and expenditure. Deficit financing is a
90% and area with weight of 10%. method of filling this gap which in India means borrowing
— Grants to states are divided into two. One, grant to from the RBI against the issue of Treasury Bills and running
duly constituted Gram Panchayats. Two grant to duly down of accumulated cash balances. This amounts to
constituted municipal bodies. creation of money.
— The need for deficit financing arises when the government
— It has divided grants into two parts. A basic grant and
fails to mobilise enough resources to fund its plans. Since,
a performance one for Gram panchayats and
cutting down plan expenditure would hurt growth, the
Municipal, bodies.
government instead resorts to deficit financing.
— The ratio of basic to performance grant is 90 : 10 for
panchayats and 80 : 20 for municipalities. Consequences of Deficit Financing
— During a recession deficit financing can have positive effects.
7th Pay Commission It leads to an increase in Aggregate demand, which puts the
— The 900-page report of the 7th Pay Commission hitherto idle machinery and capital equipment into operation
headed by Justice AK Mathur was presented to thus, leading to an increase in production and the general
Finance Minister Arun Jaitley on November 19, 2015 economic activity.
with a recommendation that the new scales be — There is a time lag between the increase in demand due to
implemented from January 1, 2016. deficit financing and the increase in supply of goods and
services. This leads to high inflation during this period.
— The panel recommended a 14.27% increase in basic
pay, the lowest in 70 years. The previous 6th Pay — The level of inflation depends on how much of the resources
Commission had recommended a 20% hike which the are spent on producing consumer goods and how much on
government doubled while implementing it in 2008. capital equipment. Production of consumer goods leads to
increased supply and thus, lower inflation.
62 Magbook ~ Indian Economy

— Its members included RBI Governor Urjit


Kelkar Committee Report on Patel, Chief Economic Advisor Arvind
Fiscal Consolidation Subramanian, former Finance Secretary Sumit
The Vijay Kelkar committee, which was given the task of preparing a fiscal Bose, and National Institute of Public Finance
consolidation plan has suggested number of steps, which are given below: and Policy Director Rathin Roy.

Reduce subsidy to 2% of GDP in 2013-14 and 1.8% in 2014-15, from a — Review Committee Report has recommended a
projected 2.6% in 2012-13. debt to GDP ratio of 60% for the general

Immediate increase in the price of diesel by ` 4 litre that of kerosene by (combined) government by 2023, comprising
` 2 litre and cooking gas by ` 50 per cylinder. 40% for the Central Government and 20% for
the State Governments. As per the Constitution

Instead of subsidies provision for diesel, kerosene, gas move to system of
of India, it is mandatory for a State to take the
market based price by March, 2014.
Central Government’s consent for raising any

Implement of goods and service tax.
loan if the former owes any outstanding

Reduce excise and ST rates to 8%. liabilities to the latter.

A 360° profile of all tax paying individual and institution should be created
to help decrease tax evasion and tax fraud.

The proposed Food Security Bill should be appropriately phased taking into
Financial
account the present difficult fiscal challenge. Administration
— All financial activities involving issues of
Fiscal Responsibility and Budget financial administration including public
budget, its passing, auditing and similar other
Management Act, (FRBM) 2003 matters. Without a extensive study of relevant
— FRBM Act, 2003 was passed by the Union Government to provide a dimensions of financial administration the
legislative control over the fiscal situation of the country, which had subject of public finance remains incomplete.
deteriorated earlier. It was meant to bring fiscal discipline, increase
plan expenditure, leave the RBI with autonomy as far as money Union Budget
creation was concerned, meet the consumption expenditure of the
government from its own resources etc. — The budget is an extensive account of the
— The Fiscal Responsibility and Budget Management Act, 2003 (FRBM government’s finances, in which revenues from
Act) has been amended as part of the Finance Bill, 2012. It has all sources and expenses of all activities
introduced two concepts to reform the expenditure aspect of the undertaken are aggregated.
fiscal policy. — Union budget is an expression of the Fiscal
These are : Policy of the Government.
— Effective Revenue Deficit It excludes from the conventional revenue deficit, — The Finance Minister presents the Union
grants for the creation of capital assets. This is an important development budget every year in the Parliament that
for the reason that while the revenue deficit of the consolidated general contains the Government of India’s revenue
government fully reflects total capital expenditure incurred, in the accounts and expenditure for 1 fiscal year, which runs
of the centre, these transfers are shown as reserve expenditure. from 1st April to 31st March.
Therefore, the mandate of eliminating the conventional revenue deficit of
the centre becomes problematic. With this amendment, the endeavour of Historical Preview
the government under the FRBM Act, 2003 would be to eliminate the
effective revenue deficit.
— The term budget is actually derived from a
French word Bougette, which means a sack or
— Medium-term Expenditure Framework Statement It will set forth a
pouch. It was first used in France in 1803. In
three-year rolling target for expenditure indicators. It would help in
undertaking a de-novo exercise for allocating resources for prioritised
the Constitution of India, the term budget is
schemes and weeding out others that have outlived their utility. nowhere used.
— It is rather mentioned as Annual Financial
NK Singh Committe statement under Article 112 comprising the
Revenue budget, Capital budget and also the
— The FRBM Review Committee headed by former Revenue Secretary,
estimates for the next fiscal year called
NK Singh was appointed by the government to review the
budgeted estimates.
implementation of FRBM. In its report submitted in January 2017,
titled, ‘The Committee suggested that a rule based fiscal policy by
— As per the British legacy, the Union Budget of
limiting government debt, fiscal deficit and revenue deficits to certain India used to be presented on the evening of
targets is good for fiscal consolidation in India. last working day of the month of February to
follow the British budget.
Magbook ~ Public Finance 63

During the NDA regime, then Finance Minister Yashwant


—
Sinha was the first to present the budget on 28th
Types of Budgeting
February, 2001 at 11 am. (The budget has to be passed Zero-Based Budgeting
by the Lok Sabha before it can come into effect on 1st
— It is a method of budgeting, in which all budgetary
April.)
allocations are set to nil at the beginning of a financial year.
Preparation of Budget Gender Budgeting
— The budget is prepared by the budget division of — It came into being in 2004-05. To contribute towards the
Department of Economic affairs in the Ministry of Finance women empowerment and removal of inequality based on
(MoF), after consulting with other ministries and the gender, role of budgeting has been accepted through this
Planning Commission. step.
— The process majorly includes following steps which may
Capital Budgeting
be sequential or overlapping too.
— Capital budgeting or investment appraisal is the planning
Overall Budget process used to determine whether an organisation‘s
— Overall budget are available for more than 1 fiscal year long-term investments such as new machinery,
but are not distributed to individual fiscal year. It is a replacement of machinery, new plants, new products, and
hierarchical and structure containing budget structure research development projects are worth the funding of
elements (budget hierarchy). cash through the firm‘s capitalisation structure (debt, equity
or retained earnings).
Passing of Finance Bill
— Capital budget includes capital receipts and payments of
(Under Rule 219 of the Lok Sabha)
the government. Loans from public, foreign governments
Classification of the Budget and RBl form a major part of the government’s capital
— Budget of the Union Government is classified into revenue receipts. Capital expenditure is the expenditure on
account and capital account. development of machinery, equipment, building, health
facilities, education, etc.
Revenue Account
— Consists of all those receipts or expenditure or that do not Outcome Budget
entail sale or creation of assets or increase or decrease of — An Outcome budget measures the development outcomes
liabilities. of all government programmes. For instance, it will tell a
— Capital account consists of receipts or expenditure from citizen if money has been allocated for building a primary
liquidation or creation of assets or increase or decrease of health centre has it indeed come up. In other words, it is
liabilities. Expenditure is also divided into two expenditure: a mean to develop a linkage between the money spent by
(i) Plan Expenditure Consists of money going to annual a government and the results which follow.
plans of the Union and State Governments. — Outcome budgeting in India was introduced by the
(ii) Non-Plan Expenditure It is the expenditure not falling Finance Minister P Chidambaram from Budget 2005-06. It
under the annual plans. It has a small capital is based on the idea that financial outlays in the budget do
component whose largest chunk is on defense. Both not necessarily lead to outcomes, while the people of the
plan and non-plan expenditure are divided into revenue country are concerned with the outcomes.
and capital account as usual. * The first such mini-budget was presented by TT
Krishnamachari on 30th November, 1956, in form of
Stages in Budget Enactment fresh taxation proposals through Finance Bills,
The budget goes through the following six stages in the demanded by the prevailing domestic and
Parliament : International Economic Situation.
(i) Presentation of the budget on the floor of the House * John Mathai proposed the first Budget of Republic
before the Lok Sabha. of India in 1950 and also the creation of Planning
commission.
(ii) General discussion on the budget.
* Finance Minister Morarji Desai has given Budget
(iii) Vote of account. for the maximum number of times (10), followed by
(iv) Scrutiny by departmentally related Standing P Chidambaram, who has given 9 Budgets.
committees.
* CD Deshmukh was the first Indian Governor of
(v) Voting on demands for grants. RBI to have presented the Interim budget for the
(vi) Passing of Appropriation Bill (Article 114 of the year 1951-52.
Constitution of India).
64 Magbook ~ Indian Economy

* Ms Indira Gandhi is the only woman to hold the post of the — Primary deficit, the pure deficit derived after
Finance Minister and to have presented the budget in her deducting the interest payments and structural
capacity as the Prime Minister of India in 1978. and cyclical deficit part of the public sector
* Plan expenditure was for the first time presented separately deficit.
in the budget for 1959-60. — Income deficit (the difference between family
income and the poverty threshold).
Deficit — Trade deficit ( when the value of imports exceed
the value of exports).
A deficit is the amount by which a sum falls short of some reference
amount. The meaning of deficit differs from that of debt which is an — Introduction of new schemes would entail more
accumulation of yearly deficits. spending and it goes just opposite to what we
are trying to do, i.e. reduce deficit. Import duty is
a tax collected on imports and some exports by
Types of Deficits the customs authorities of a country. It is usually
— Revenue Deficit It is the difference between the revenue receipt on based on the value of the goods that are
tax and non-tax side and the revenue expenditure. Revenue imported. There are two distinct goals to import
expenditure is synonymous with consumption and duties : to raise income for Local Government,
non-development. and to give a market advantage to locally grown
— Fiscal Deficit It is the difference between what the government or produced goods that are not subject to import
earns and its total expenditure. duties.

Fiscal Deficit = Difference between country’s expenditures and earnings.

Fiscal Deficit = Revenue Receipts (Net tax revenue + Non-tax revenue) + Financial Stability and
Capital Receipts (only recoveries of loans and other receipts) – Total Development Council
expenditure (Plan and Non-plan)
It is an apex-level body constituted by Government
— Budget Deficit It considers only the difference between the total of India, which was first mooted by Raghuram Rajan
budgeted receipt and the expenditure. It was abolished in 1997. committee in 2008. It envisages to strengthen and
— Monetised Deficit It is the borrowing made from the RBI, through institutionalise the mechanism of maintaining
printing fresh currency. It is resorted to, when government cannot financial stability, financial sector development,
borrow from market. inter-regulatory coordination alongwith monitoring
— Gross Fiscal Deficit The Gross Fiscal Deficit (GFD) of government is macro-prudential regulation of economy.
the excess of its total expenditure, current and capital, including
loans net of recovery, over revenue receipts (including external
Financial Sector Legislative
grants) and non-debt capital receipts. Reform Commission
— Gross Fiscal Deficit = Total Expenditure – (Revenue Receipts + Non-debt It is a body set-up of Ministry of Finance by
Creating Capital Receipts). Government of India to review and rewrite the
— Net Fiscal Deficit The Net Fiscal Deficit (NFD) is the gross fiscal legal-institutional architecture of the Indian
deficit reduced by net lending by government. financial sector, which is chaired by a former judge
— Primary Deficit Amount by which a Government’s total expenditure of the Supreme Court of India and have an electric
exceeds, its total revenue, excluding interest payments on its debt. mixer of expert members drawn from the fields of
— Primary deficit = Fiscal deficit – Interest payments. finance, economics, public administration, law etc.
— Gross Primary Deficit The Gross Primary (GFD) Deficit (GPD) is the
Gross Fiscal Deficit less interest payment while the primary revenue
deficit is the revenue deficit less interest payments. Railway Budget Merged with
Action of the Government to Reduce Union Budget
The separate Railway Budget, a 92 year old
the Deficit —
tradition, has been scrapped by the Modi
A deficit is the amount by which a sum falls short of some reference Government. With the scrapping of the Rail
amount. In economics, a deficit is an excess of expenditures over Budget, Railway also not have to pay a special
revenue in a given time period. In more specific cases, it may refer to dividend to the government for getting gross
— Balance of Payments (BOP) deficit, when the Balance of Payments budgetary support. The Central Government also
is negative. approved removal of plan and non-plan
— Government budget deficit that is deficit spending.
expenditure.
Self Check
Build Your Confidence
1. Consider the following taxes (b) net increase in Union Government’s borrowings from the
1. Sales Tax 2. VAT 3. Property Tax Reserve Bank of India
Which of the tax given above are Ad-valorem taxes? (c) the sum of budgetary deficit and net increase in internal
(a) 1 and 2 (b) 2 and 3 and external borrowing
(c) 1 and 3 (d) All of these (d) the sum of monetised deficit and budgetary deficit

2. Consider the following statements 8. Consider the following statements


1. Gift tax for the first time was imposed in 1958. 1. 14th Finance commission has recommended to
increase in the shave of states in the centre’s tax
2. Gift tax was imposed as a complement of the estate
revenue from the current 32% to 42%, the single largest
duty.
increase even recommended.
Which of the statement(s) given above is/are correct?
2. The 14th Finance commission, headed by former RBI
(a) Only 1 (b) Only 2
Governor YV Reddy did not endorse the compensation
(c) Both 1 and 2 (d) Neither 1 nor 2
road map for the goods and services tax finalised by the
3. Consider the following statements centre and also denied for an autonomous and
1. Personal income tax independent GST Compensation Fund.
2. Import duty Which of the statement(s) given above is/are correct?
3. Service tax (a) Only 1 (b) Only 2
Which of the taxes given above is/are indirect tax? (c) Both 1 and 2 (d) Neither 1 nor 2
(a) 1 and 2 (b) 2 and 3 9. Which committee recommended abolition of tax rebates
(c) 1 and 3 (d) All of these under Section 88? [UPPCS 2000]
4. Consider the following statements [UPPCS 2009] (a) Chelliah committee (b) Kelkar committee
(c) Shome committee (d) None of these
1. Service tax was introduced in 1994-95.
2. There are 119 services in 2013-14 on which service tax 10. Consider the following taxes [IAS 2001]
is imposed. 1. Corporation tax 2. Customs duty
Which of the statement(s) given above is/are correct? 3. Wealth tax 4. Excise duty
(a) Only 1 (b) Only 2 Which of the statement(s) given above is/are indirect
(c) Both 1 and 2 (d) Neither 1 nor 2 taxes?
5. Value added tax is [UPPCS 2009] (a) Only 1 (b) 2 and 4
(a) an ad-valorem tax on domestic final consumption (c) 1 and 3 (d) 2 and 3
collected at all stages between production and point of 11. These has been a persistent deficit budget after year.
final sale Which of the following actions can be taken by the
(b) an ad-valorem tax on final consumption collected at the government to reduce the deficit? [UPSC 2015]
manufacturing level 1. Reducing revenue expenditure
(c) tax on final consumption collected at the consumption
2. Introducing new welfare schemes
rate
3. Rationalising subsidies
(d) special tax levied by the states on products from other
states 4. Expanding industries
Select the correct answer using the codes given below
6. Fiscal deficit implies (a) 1 and 3 (b) Only 1
(a) total expenditure-(revenue receipt+recovery of loan+ (c) 2 and 3 (d) All of these
receipts from disinvestment)
(b) total expenditure-total receipt 12. A decrease in tax to GDP ratio of a country indicates,
(c) total expenditure-(Revenue receipt+receipt from which of the following statements?
disinvestment) 1. Slowing economic growth rate income.
(d) total expenditure-Disinvestment receipt 2. Less equitable distribution of National income.
Select the correct answer using the codes given below
7. Fiscal deficit in the Union budget means [IAS 1994]
(a) Only 1 (b) Only 2
(a) difference between current expenditure and current
(c) Both 1 and 2 (d) Neither 1 nor 2
revenue

1. (d) 2. (c) 3. (b) 4. (a) 5. (a) 6. (a) 7. (c) 8. (a) 9. (b) 10. (b)
11. (a) 12. (b)
Chapter eight
India’s Balance of
Payments
Components of Capital Account
India’s balance of Balance of There are the principle forms of capital account
payments has been Payments (BoP) transactions :
under increasing stress — When the difference in the value of — Foreign Investment It has two sub-components
which are as follows:
recently. Experts have imports and exports of all the three
items i.e. visible, invisible and (i) Foreign Direct Investment (FDI) referring
declined while imports to the purchase of assets in the rest of the
capital transfers, is taken into
have not fallen account, it is called Balance of world, which allows control over that
significantly, resulting assets.
Payments (BoP).
e.g. purchase of a firm by TATA in the rest
in increasing trade and — Thus, an overall record of all of the world.
current account economic transactions of a country in
(ii) Portfolio Investment referring to purchase
a given period, with rest of the world.
deficits. India’s growing of an asset in the rest of the world, without
— Balance of Payments (BoP) account any control over that asset. Portfolio
external exposures can broadly comprises of the following investment into India also consists of
also be attributed to components: Foreign Institutional Investment (FII).
the increasing — Current Account of Balance of e.g. purchase of the some shares of a
integration of India’s Payments consist of all transactions company by TATA in the rest of the world.
relating to goods, services and — Loans It has two sub-components which are as
economy with the rest follows:
income. It is functionally classified
of the world. into merchandise or visible and (i) Commercial Borrowings referring to
invisibles. Current account deficit is borrowing by a country (including
the situation where payments on the government and the private sector) from
current account out of the country the international money market. This
are more than the payments into involves market rate of interest without
the country. In current account considerations of any concession.
surplus, there is a net inward (ii) Borrowings as External Assistance
payment into the country on the referring to borrowing by a country with
current account. considerations of assistance. It involves
lower rate of interest compared to that
— Capital account is that account
prevailing in the open market.
which records all such transactions
— Banking Capital Transactions referring to
between residents of a country and
transactions of external financial assets and
rest of the world, which causes a liabilities of Commercial Banks and Cooperative
change in the asset or liability Banks operating as authorised dealers in foreign
status of the residents of a country exchange. These transactions include NRI deposits.
or its government. Investments (FDI — Reserve Account The official reserve account
and FII) and Borrowings External records the change in stock of reserve assets (also
Commercial Borrowing (ECB) are known as foreign exchange reserves) at the
part of the capital account. country’s monetary authority.
Magbook ~ India’s Balance of Payments 67

— Net Errors and Omissions This is the last component of the — Devaluation and Depreciation of the Rupee The
Balance of Payments and principally exists to correct any devaluation and depreciation of the rupee have led to an
possible errors made in accounting for the three other increase in the price of imports. Exports have become
accounts. They are often referred to as balancing items. cheaper, the low price and income elasticities of demand
for exports have resulted in slow increase in exports.
BoP Trends — Slow Rise in Export Earnings Export earnings rose,
India had faced pressure on Balance of Payments (BoP), however, they were not sufficient enough to meet the
since, planning period due to either internal or external factors. rising imports. Thus, rise in exports has neither been
Period I (1956-57 to 1975-76) substantial nor continuous. The growth in exports has not
The period comprising the 2nd, 3rd and 4th Plans and first 2 been sufficient enough to finance the rising imports.
years of 5th Plan saw heavy deficit in Balance of Payments — Debt Service The Balance of Payments (BoP) problem
(BoP) and extremely tight payment position. This period has also aggravated due to the rising obligation of
witnessed three wars, several droughts and the first oil shock amortising payments in 2011-12, debt service ratio was
in 1973, though the government resorted to serve import 6% with the ever increasing imports and slow pace of
controls and foreign exchange regulation etc. exports, the most effective solution for India’s Balance of
Period II (1976-77 to 1979-80) Payments (BoP) problem is cost reduction and
competitiveness in global market.
It was relatively short period and was a golden period as far as
BoP is concerned. In this period, India had a small current — Appreciation The recent appreciation of the rupee has
account surplus of 0.6% of the GDP and also possessed foreign made exports costlier and imports cheaper. It may also
exchange reserves equivalent to about seven months imports. add to the Balance of Payments (BoP) problem.

Period III (1980-81 to 1990-91)


This period broadly corresponds to the period of 6th and 7th
Foreign Capital
Plans and was marked by severe BoP difficulties. The reasons — Foreign capital inflow to the country can be either in the
for severe difficulties are as follows: Widening trade deficits, form of concessional assistance or non-concessional flow.
Gradual decline in net receipts from invisibles, Reductions — Non-concessional flows include mainly External
inflows of concessional assistance to India principally from Commercial Borrowings (ECBs), loans on market terms,
World Bank group, The third oil shock during 1990-91, During NRI deposits and foreign investment.
1990s, domestic political developments affected confidence
— Foreign investment can be categorised into Foreign Direct
abroad in Indian economy etc, International rating agencies
downgraded India, Substantial outflow of deposits held by Investment (FDI) and Foreign Institutional
Non-Resident Indians (NRIs), Reserves declined to a low of $ Investment (FII).
0.9 billion in January 1991.
Foreign Direct Investment (FDI)
Period IV (1991-92 Onwards) — It refers to direct investment in the productive capacities
The reforms of 1990’s have facilitated India to move away of a country by someone from outside the country. Such
from closed economy framework towards a more open and an investment can be in the form of setting up a new
liberal economy. Foreign exchange reserves were built to very plant or through purchase of shares of a company, where
comfortable positions and the difficulties of BoP came under
the shareholding gives the foreign entity control over the
control and the reasons for the same are as follows:
business of the company.
— Trade balance has always been in deficit since, imports have
always exceeded exports. — A foreign company can set-up its business in India from
— When current account deficits are larger than capital account two ways, by setting up a company under the Companies
surpluses, foreign exchange reserves are also used to cover Act or by setting-up an unincorporated entity like Liaison
these deficits. office, project office or branch office.
The important reasons for deficit in India’s BoP position can — The government of India has amended FDI inflow in
be cited as follows: 2014, the government increased foreign investment limit
— Irreversible Trade Deficit Our imperative imports of oil and from 26% to 49% in insurance sector. It also launched
coal and India’s passion for gold. Make in India intiative in September 2014 under which
— Rise in Imports The reasons for rapid rise in imports are FDI policy for 25 sectors was liberalised further.
building industrial base (in the early stages), increase in — As per April 2015, FDI inflow in India increased by 48%
export related imports (gems, jewellery, capital goods) since the launch of Make in India initiative. India was
increase in imports of industrial raw materials, rise in the ranking 15th in the world in 2013 in terms of FDI inflow.
price and imports of Petroleum, Oil and Lubricants (POL) It rose up to 9th position in 2014. While in 2015 india
products etc. become top destination for foreign direct investmation.
68 Magbook ~ Indian Economy

Sectors which are Open to FDI FIPB is Abolished


— Most sectors are at least partially open to FDI, subject to a The Union Cabinet has approved the abolition of 25 year old
FIPB. Henceforth, concerned ministries will be responsible for
cap and specific conditions. There are two entry routes for
direct approval of foreign investment proposals. The decision
FDI in India. In sectors where FDI is allowed up to 100%,
fails in line with Finance Minister Arun Jaitley’s proposal to
FDI enters under the automatic route, subject to sectoral
scrap FIPB 2016-2017 Union Budget. FIPB was constituted
regulations and other conditions.
in the mid-nineties under the Prime Minister’s office following
— In this instance, no approval is required from the Reserve economic liberalisation. Rationale behind this move is that
Bank of India (RBI) or the government however, the over 90% of the FDI inflows in value terms enters through
investment must be notified to the RBI’s regional office automatic route. The government expects that scrapping of
within 30 days. FIPB would help in ease of doing business. At present, only
— In sectors, where FDI is allowed under the government 11 sectors, including defence and retail trading need
route, prior approval from the Foreign Investment government approval for Foreign Direct Investment (FDI).
Promotion Board (FIPB) is required. FIPB
recommendations must be cleared by the Ministry of Foreign Institutional
Finance for FDI proposals below or equivalent to ` 50 Investment (FII)
billion and by the Cabinet Committee of Economic Affairs — A foreign institutional investor is one which is registered in
for FDI proposals of more than ` 50 billion. a country outside of the one, in which it is investing. In
Key Changes in FDI Limits India, it is used to refer to companies which invest in the
country’s financial markets.
Sector/Activity % of Entry Route — In 2013, India accepted the internationally, laid down
FDI/Equity definition of FII to remove the ambiguity between FII and
Defense Sector 100% (FIPB route-49% at FDI. Now, when an investor has a stake of less than 10%
present) in a company, it will be treated as FII and where and
Telecom Services 100% Automatic upto 49% investor has a stake of more than 10%, it will be treated
government route beyond as FDI.
49% and upto 100%
— FIIs may invest in securities in both the primary and
Tea Plantation 100% Automatic upto 49% secondary market in shares, debentures and warrants of
government route beyond
listed or unlisted companies.
49% and upto 100%
Asset Reconstruction 100% Automatic upto 49%
— Some of the entities eligible to be treated as FII in India
Company government route beyond are pension funds, mutual funds, banks, investment
49% and upto 100% trusts, sovereign wealth funds, Foreign Central Banks etc.
Petroleum and Natural 49% Automatic route — FIIs can invest in securities in the primary and secondary
Gas markets dated government securities, commercial paper,
Commodity Exchange 49% Automatic route derivatives, units of schemes floated by domestic mutual
Power Exchanges 49% Automatic route funds (including UTI), Indian depository receipts and
Stock Exchanges/Clearing 49% Automatic route security receipts.
Corporations
Means of FII Investment
Credit Information 74% Automatic route
Companies — FIIs can invest through a registered broker on recognised
Courier Services 100% Automatic route
Indian stock exchanges. They can purchase shares or
convertible debentures either through private placement
Single Brand Product 100% Automatic upto 49%
Retail Trading government route beyond
or through offer for sale.
49% and upto 100% — FIIs can also invest through a sub-account (a person
Insurance Sector 49% FIPB route (The CCEA outside India on whose behalf investments are proposed
on 24th December, 2014, to be made).
49% FDI m insurance) — FIIs can also issue Offshore Derivative Instruments (ODIs)
Pharmaceuticals 100% Automatic route to persons who are regulated by an appropriate foreign
Animal Husbandry 100% Automatic route regulatory authority and after compliance with KYC norms.
Food item 100% Automatic route — Large amount of FII investment into India comes through
e-Commerce 100% Automatic route participatory notes.
Magbook ~ India’s Balance of Payments 69

FDI vs FII — Spot exchange rates, at which different currencies are


trades for immediate exchange.
FDI inflows are preferred over FII inflows for the following reasons:
— The rise in the value of one currency relative to another
— FDI is considered to be long-term and stable investment
currency is called appreciation.
whereas FII is considered as hot money i.e. it can move out
quickly during adverse circumstances leading to instability
and volatility in the exchange rate and the stock market. Foreign Exchange
— Since, FDI represents ownership, it leads to the inflow of — Foreign exchange reserves are an important component
better technology management practices etc while FII is of the BoP and an essential element in the analysis of an
generally only interested in short-term gains. economy’s external position.
— While the above arguments are true to a certain extent, — India’s foreign exchange reserves comprise Foreign
FII’s have also been influencing the incorporation of better Currency Assets (FCAs), gold, Special Drawing Rights
technology and management in the companies where they (SDRs) and Reserve Tranche Position (RTP) in the
hold shares. International Monetary Fund (IMF).

Participatory Notes (P-Notes) Exchange Rate


— These are financial instruments used by investors or hedge — Exchange rate is the rate, at which Indian rupee will be
funds, that are not registered with the Securities and exchanged vis-a-vis other international currencies, say US
Exchange Board of India to invest in Indian securities. dollar, in the foreign exchange market.
— Indian based brokerages buy India-based securities and — The rupee was historically linked to the British pound
then issue participatory notes to foreign investors. Any sterling till 1946. After independence, India had to fix and
dividends or capital gains collected from the underlying maintain the external value of the rupee in terms of gold
securities go back to the investors. or the US dollar as required under IMF rules. Therefore,
— P. Notes are overseas Derivative Instruments that have India fixed the value of rupee at ` 3.30 per US dollar. This
Indian stocks as their underlying assets. They allow foreign was the official rate of exchange and RBI would buy and
investors to buy stocks listed on Indian exchanges through sell foreign currencies at this rate.
FIIs without being registered. — The 1994-95, budget announced full convertibility of the
rupee on current account i.e. freedom to buy or sell
Qualified Foreign Investor (QFI) foreign exchange in connection with the current account
— QFI is a person, who fulfills the following criteria: transactions under Article VIII of the IMF.
— Resident in a country or group which is a member of Financial
Action Task Force (FATF). NEER and REER
— Resident in a country that is signatory to 1OSCO’s MoU or a
signatory of a bilateral MoU with SEBI. — The Nominal Effective Exchange Rate (NEER) and Real
— Such person should not be resident in India or registered with Effective Exchange Rate (REER) indices are used as
SEBI as an FII or a sub-account of FII. indicators of external competitiveness of the country over
— QFIs are distinct from FIIs and non-resident Indians. They are a period of time.
allowed to invest directly into mutual funds and stocks of
— NEER is the weighted average of bilateral nominal
Indian companies. It was felt that foreign investors had been
kept at bay owing to concerns relating to money laundering exchange rates of the home currency in terms of foreign
and due diligence by the government and regulators. currencies, while REER is defined as a weighted average
— For this reason, the new scheme of QFI was started to ensure of nominal exchange rates, adjusted for home and foreign
more foreign capital inflows, reduce market volatility and country relative price differentials.
deepen the markets. — Nominal rupee depreciation, while having some adverse
— Global Depository Receipts (GDRs) These are equity effects such as greater imported inflation, is also useful
instruments issued in international markets like London, over time in offsetting higher domestic inflation and
Luxemburg etc. Indian companies use GDRs to raise capital ensuring Indian exports remain competitive.
from abroad. GDRs are designated in dollars, euros etc. — REER captures movements in cross currency exchange
— American Depository Receipts (ADRs) These are the rates as well as inflation differentials between India and
equity instruments issued to American retail and its major trading partner and reflects the degree of
institutional investors. They are listed in New York, either external competitiveness of Indian products.
on Nasdaq or New York Stock Exchange. — The RBI has been constructing 6 currency (US dollar,
— Indian Depository Receipts (IDRs) These are similar to euro for euro zone, pound sterling, Japanese Yen,
ADR or GDR. They are used by non-Indian companies in Chinese renminbi and Hong Kong dollar) and 36 currency
the Indian stock markets for issuing equity to Indian indices of NEER and REER.
investors.
70 Magbook ~ Indian Economy

The foreign Exchange Regulation Act (FERA) was


Present Exchange Rate Policy —
legislation passed by the Indian Parliament in 1973 and
— India has transited from fixed exchange rate policy to a come into force on 1st January, 1974.
market determined exchange rate. It is broadly a floating — FERA imposed stringent regulation on certain kinds of
rate regime with the central intervening only for reducing
payments. It deals with foreign exchange and scousitles
excess volatility, preventing the emergence of destabilising
and the transactions which had an impact on the foreign
speculative activities, maintaining adequate reserves and
exchange and import and export of currency.
developing on orderly foreign exchange market.
— The Foreign Exchange Management Act (FEMA) was an
— At present, the exchange rate policy is guided by the
act passed in the winter session of parliament in 1999
broad principles of careful monitoring and management
which replaced foreign exchange Regulation Act.
of exchange rates with flexibility, while allowing the
— FEMA has brought a new management regime of foreign
underlying demand and supply conditions to determine
exchange consistent with the emerging frame work of the
exchange rate movements over a period in an orderly
World Trade Organisation (WTO).
manner.

Policy Options with RBI to Manage Main Features of FEMA


Exchange Rate — Activities such as payments made to any person outside
India or receipts from them, alongwith the deals in foreign
— Using Policy Rates RBI can use policy rates (repo, CRR,
security is restricted. It is FEMA that gives the Central
SLR etc) to manage the exchange rate. By lowering
Government the power to impose the restrictions.
interest rates, supply of rupee’s increased in the market
leading to depreciation of the currency. Increasing — Restrictions are imposed on people living in India, who
interest rates on the other hand, takes out rupee from the carry out transactions in foreign exchange, foreign security
system leading to shortage of rupee supply and thus, or who own or hold immovable properties abroad.
appreciation of the rupee. — Without general or specific permission of the Reserve
— Using Forex Reserves RBI can sell forex reserves and Bank of India, FEMA restricts the transactions involving
buy Indian rupees leading to demand for rupee. Based on foreign exchange or foreign security and payments from
weekly forex reserves data, RBI seems to be selling forex outside the country to India the transactions should be by
reserves selectively to support rupee. Its intervention has an authorised person.
been limited as liquidity in money markets has remained — Deals in foreign exchange under the current account by an
tight and further intervention only tightens liquidity authorised person can be restricted by the Central
further. Government, based on public interest. Although, selling or
drawing of foreign exchange is done through an authorised
— Easing Capital Controls Capital controls could be eased to
person, the RBI is empowered by this act to subject the
allow more capital inflows. “Resisting currency
capital transactions to a number of restrictions.
depreciation is best done by increasing the supply of
foreign currency by expanding market participation.” This — People living in India will be permitted to carry out
in essence, has been RBI’s response to depreciating transactions in foreign exchange, foreign security or to own
rupee. or hold immovable property abroad, if the currency,
security or property was owned or acquired, when he or
FERA and FEMA she was living outside India or when it was inherited to
— FERA, (Foreign Exchange Regulation Act) in place since, him or her by someone living outside India.
1974 did not succeed in restricting activities such as the — Exporters are needed to furnish their export details to RBI.
expansion of Trans National Corporations (TNCs). After To ensure that the transactions are carried out properly,
the Amendment of FERA in 1993, it was decided that the RBI may ask the exporters to comply to its necessary
act would become the FEMA (Foreign Exchange requirements.
Management Act).
— This was done in order to relax the controls on foreign Capital Account
exchange in India, as a result of economic liberalisation.
FEMA served to make transactions for external trade Convertibility in India
(exports and imports) easier. Involving current account for — Capital Account Convertibility (CAC) for Indian Economy
external trade no longer required RBI’s permission. refers to the abolition of all limitations with respect to the
— The deals in foreign exchange were to be ‘managed’ movement of capital from India to different countries across
instead of ‘regulated’. The switch to FEMA shows the the globe. According to the Tarapore Committee, Capital
change on the part of the government in terms of foreign Account Convertibility refers to the freedom to convert local
capital. financial assets into foreign financial assets and vice-versa
at market determined rates of exchange.
Magbook ~ India’s Balance of Payments 71

— It is associated with changes of ownership in foreign or — During good times, domestic borrowers could enjoy triple
domestic financial assets and liabilities and embodies the benefits of
creation and liquidation of claims on or by the rest of the — Lower interest rates,
world. — Longer maturity and
— RBI appointed the Second Tarapore Committee to set out — Capital gains due to domestic currency appreciation. This
the framework for fuller capital account convertibility. would happen, when the local currency is appreciating due
to surge in capital flows and the debt service liability is
falling in domestic currency terms. The opposite would
India’s External Debt happen, when the domestic currency is depreciating due to
reversal of capital flows during crisis situations, as happened
— India’s external debt has increased over time and India is during the 2008, global crisis.
one of the highest indebted country of the world in terms of
— A sharp depreciation in local currency would mean
total debt outstanding.
corresponding increase in debt service liability, as more
— Gross external debt is defined, at a point of time, as ‘‘The domestic currency would be required to buy the same
outstanding amount of those actual current and not amount of foreign exchange for debt service payments.
contingent liabilities that require payments of principal and
— This would lead to erosion in profit margin and have
interest by the debtor at same points in the future and that
market-to-market implications for the corporate.
are owed to non-residents by residents of an economy.’’
There would also be debt overhang problem, as the
— India’s external debt stock at end March, 2015 stood at volume of debt would rise in local currency terms.
US $ 475.8 billion recording an increase of 6.6% over end
— Together, these could create corporate distress,
March, 2014. The rise in external debt is largely due to
especially because the rupee tends to depreciate
higher NRI deposits.
precisely, when the Indian economy is also under stress
— The maturity profile of India’s external debt indicates the and corporate revenues and margins are under pressure.
dominance of long-term borrowings. Long-term external
debt accounted for 76.9% of the total external debt, while
the remaining 23.1% was short-term debt. NRI Deposits
— Non-Resident Indian (NRI), deposits are of three types:
Concepts of External Debt (i) Non-Resident (External) Rupee Account (NRE
Sovereign (Government) and Non-Sovereign Account) Deposits were introduced in 1970. Any NRI
(Non-Government) Debt can open an NRE account with funds remitted to India
through a bank abroad. The amount held in these
— Sovereign debt includes:
deposits together with the interest accrued can be
— External debt outstanding on account of loans received by
repatriated.
Government of India, under the external assistance programme
and civilian component of rupee debt. (ii) Foreign Currency Non-Resident (Banks) [Deposits
— Other government debt comprising borrowings from IMF, (FCNR-B)] were introduced with effect from 15th May,
defence debt component of rupee debt as well as foreign 1993. These are term deposits maintained only in
currency defence debt. pound sterling, US dollar, Japanese yen, euro,
— FII in Government Securities. Non-sovereign includes the Canadian dollar and Australian dollar. The minimum
remaining components of external debt. All other debt is maturity period of these deposits was raised from 6
non-sovereign debt. months to 1 year effective October, 1999. From
26th July, 2005, banks have been allowed to accept
External Commercial Borrowings (ECBs) FCNR (B) deposits up to a maximum maturity period of
— The definition of commercial borrowing includes loans from 5 years against the earlier maximum limit of 3 years.
Commercial Banks, other commercial financial institutions, (iii) Non-Resident Ordinary Rupee (NRO) Accounts Any
money raised through issue of securitised instruments like person, resident outside India may open and maintain
bonds including India Development Bonds (IDBs) and NRO account with an authorised dealer or in authorised
Resurgent India Bonds (RIBs), Floating Rate Notes (FRN) bank for the purpose of putting through bonafide
etc. transactions denominated in Indian rupee. NRO
— It also includes borrowings through buyers’ credit and Accounts may be opened or maintained in the form of
supplier credit mechanism of the concerned countries current, saving, recurring or fixed deposits. NRI or
International Finance Corporation, Washington [IFC(W)], Persons of Indian Origin (PIO), may remit an amount
Nordic Investment Bank and private sector borrowings from not exceeding USD 1 million per financial year, out of
Asian Development Bank (ADB). the balances held in NRO accounts.
Self Check
Build Your Confidence
1. In the context of International Trade Dumping refers to 5. Consider the following items
(a) selling a commodity cheaper in foreign market and costly in 1. Gems and jewellery
domestic market 2. Chemicals and related products
(b) sending cheap goods to developing countries by developed 3. Engineering goods 4. Textiles
countries Which of the items given above are the top four items in
(c) free distribution of used products by developed countries in India’s manufactured exports?
developing countries (a) 1, 2 and 3 (b) 1, 2 and 4
(d) All of the above (c) 2, 3 and 4 (d) All of these
2. Consider the following statements 6. Areas which are engines for economic growth
1. General Agreement on Trade in services is a treaty of the supported by quality infrastructure and
World Trade Organisation (WTO) that entered into force in complemented by an attractive fiscal package are
January 1995. known as
2. All members of the WTO are signatories to the General (a) Export Processing Zones (b) Duty Free Tariff Zones
Agreement on Trade in Services. (c) Special Economic Zones (d) Technology Parks
Which of the statement(s) given above is/ are correct?
7. The SEZ Act 2005 which came inot effect in February
(a) Only 1 (b) Only 2 2006 has certain objectives? In this context consider
(c) Both 1 and 2 (d) Neither 1 nor 2 the following :
3. Consider the following statements 1. Development of infrastructure facilities
1. The shares of exports of textiles and gems and jewellery to 2. Promotion of investment from foreign sources
us fell while those of chemicals and related items and 3. Promotion of export of services only
engineering goods increased. Which of the above is/are objective of this Act?
2. In the case of China, the shares of textiles and engineering (a) 1 and 2 (b) 2 only
goods increased and that of chemicals and related products (c) 2 and 3 (d) 1, 2 and 3
decreased.
8. Asia’s first Export Processing Zone (EPZ) was set-up
Which of the above statement(s) about India’s export
in
compositions between 2000-01 and 2013-14 is/are correct?
(a) Kandla (b) Goa
(a) Only 1 (b) Only 2
(c) Kochi (d) Tuticorin
(c) Both 1 and 2 (d) Neither 1 nor 2
9. Five different earlier schemes have been merged into
4. Consider the following statements
a single scheme, namely
1. Among the major items of import, the value of Petroleum, Oil
(a) Service Exports from India Scheme
and Lubricants (POL), constitutes 36.7% of total imports in
(b) Merchandise Exports from India Scheme
2013-14.
(c) Pradhan Mantri Swasthya Suraksha Yojana
2. The import of gold declined from 1037 tonnes in 2012-13 to
(d) National Urban Health Mission
664 tonnes in 2013-14.
Which of the above statement(s) about India’s import 10. Which one of the following items has gained the
composition is/are correct? highest growth rate in the import compostion of the
(a) Only 1 Indian Economy in the last decade?
(b) Only 2 (a) Electronic goods
(c) Both 1 and 2 (b) Gold
(d) Neither 1 nor 2 (c) Project goods
(d) Pulp and Water paper
Chapter nine
India’s Foreign Trade
— Foreign trade plays a significant role with special emphasis on improving case of
The patterns of India’s in the economy of each country. doing business.
foreign trade have Foreign trade helps a country to — The government is pitching india as a friendly
changed considerably utilise its natural resources and to destination for manufacturing and exporting
since the early 1990s. export its surplus production. It can goods some key features of the new foreign
import technical know-how. tradepolicy are
From the financial
— A country can industrialise itself by — India to be made a significant participant in world
year 1990-91 to the importing necessary capital, trade by 2020.
one of 2017-18 the machines and raw materials from — Merchandise exports from India Scheme (MEIS) to
total value of goods more advanced and industrialised promote specific services for specific markets
nations. By proper control of foreign foreign trade policy.
exports increased more — FTP would reduce export obligation by 25% and
trade, employment, output, prices,
than 16 times (from industrialisation and economic
given boost to domestic manufacturing.
$18 billion to over — Industrial products to be supported in major
development of the country can be
markets rates ranging from 2% to 3%.
$300 billion). During influenced favourably.
— Branding companies planned to promote exports in
the same time span, — According to the Economic Survey sectors where India has traditional strength.
2017-18, among India’s trading
goods imports
partners, the top 5 countries with New Schemes in FTP 2015-20
increased almost which India has negative bilateral — PFTP 2015-20 introduces two new schemes,
20 times. trade balance are China, namely ‘‘Merchandise Exports from India
Switzerland, Saudi Arabia, Iraq and Scheme (MEIS’’ for export of specified goods to
South Korea while the top 5 specified markets and Services Exports from
countries with which it has surplus India Scheme (SEIS) for increasing exports of
trade balance are USA, UAE, notified services, in place of a plethora of
Bangladesh, Nepal and UK. India schemes earlier, with different conditions for
has the highest trade deficit with eligibiltiy and usage.
China. Its share in India total trade
deficit is 47.1% in 2016-17. Merchadise Exports from India
Scheme (MEIS)
Foreign Trade Policy — Now 5 different earlier schemes (Focus Product
(FTP), 2015-20 Scheme, Market Linked Focus Product Scheme,
Focus Market Scheme, Agri Infrastructure
— The new five years foreign Trade
Incentive Scheme and VKGUY) have been
policy 2015-20 provides a
merged into a single scheme, namely
framework for increasing exports of
Merchandise Export from India Scheme (MEIS).
goods and services as well as
Incentives under this scheme are available on
generation of employment and
exports of notified goods to notified markets at
increasing value addition in the
notified rates. In order to avail this benefit, all
country in keeping with the ‘Make in
exporters are advised to declare on all shipping
India’ vision. The focus of the new
bills from 1st June, 2015 onwards mandatorily
policy is to support both the
as ‘‘we intent to claim rewards under MEIS’’.
manufacturing and services sectors,
74 Magbook ~ Indian Economy

Service Exports from India Scheme (SEIS) Top Export Destinations (2018-19 % share)
— Served from India Scheme (SFIS) has been replaced USA 16 %
with Service Exports from India Scheme (SEIS). Now, all UAE 9%
service providers located in India and earning foreign China 5.1 %
exchange, regardless of the Constitution or profile of the Hong Kong 4.1 %
service provider, who is exporting notified services, Singapore 3%
would be eligible for the benefits at the rate of 3% or UK 3.2 %
5% of net foreign exchange earnings. Germany 2.8 %
Bangladesh 2.7 %
— The reward issued as Duty Credit Scrip under this
Netharlands 2.7 %
scheme and goods imported by using this scrip will be Nepal 2.3 %
freely transferable and usable for all types of
goods/services for payment of custom duty, excise duty Indian Imports
and service tax. Incentives (MEIS and SEIS) now are — An import is any good or service brought into one country
available to units located in SEZs also. from another country in a legitimate fashion, typically for use
in trade. Import goods or services are provided to domestic
India’s Foreign Trade Position
consumers by foreign producers.
India’s participation in foreign trade was continuously
— An import in the receiving country is an export to the
declining till 1980. Since 2001, it has continually improved.
sending country. Import of goods normally requires
As per the current ranking, India is the 17th largest
involvement of the custom authorities in both the country of
exporter and 11th largest importer of foreign trade.
import and the country of export and is often subject to
According to Economic Survey 2017-18, among India’s
import quotas, tariffs and trade agreements.
trading partners, the top 5 countries with which India has
negative bilateral trade balance are China, Switzertand,
— Imports in India are reported by the Directorate General of
Saudi Arabia, Iraq and South Korea while the top 5 Commerce. Among the major items of import, the value of
countries with which it has surplus trade balance are USA, Petroleum, Oil and Lubricants (POL) grew in the financial
UAE, Bangladesh, Nepal and UK. India has the highest year of 2017-18. The other major items of import are gold,
trade deficit with China. pearl/stones, petroleum products, coal/coke, telecom
instruments, iron and steel in the financial year of 2017-18.
India’s Trading Capital goods is the other major import category.
— However, the quantum of capital goods imports has actually
India’s overall exports (Merchandise and Services
Combined) in April, 2018 to March, 2019 are estimated to increased in 2012-13 as indicated earlier. India’s main
be USD 535.45 billion, exhibiting a positive growth of import partners are China (13.5% of total imports), Saudi
7.97% over the same period last year. Overall imports in Arabia, United Arab Emirates, Switzerland, United States,
April, 2018-March 2019 are estimated to be USD 631.29 Indonesia, Qatar in the financial year 2015.
billion, exhibiting a positive growth of 8.48% over the same Top Import Sources (FY 2018-19 % Share)
period.
China 14.5
Indian Exports USA 6.4
UAE 5.3
Exports measure the amount of goods or services that
Saudi Arabia 5.6
domestic producers provide to foreign consumers. It is
Switzerland 3.6
good that is sent to another country for sale. In the past, S. Korea 3.2
export of commercial quantities of goods normally required Indonesia 3.2
involvement of the custom authorities in both the country of Hong Kong 3.1
export and the country of import. Iran 3.0
More recently, with the advent of small trades over the Singapore 2.7
internet such as through Amazon and E-bay, exports have
largely by passed the involvement of customs in many Import/Export Controls
countries due to the low individual values of these trades.
Nonetheless, these small exports are still subject to legal
Imports
restrictions applied by the country of export. Around 5% tariff lines are under import controls. 11,600 tariff
Indian exports averaged 246.76 INR billion from 1978 until lines are free for import. Restrictions removed over the next 10
2013. India is also one of Asia’s largest refined product years, removing almost all the quantitative restrictions. Presently,
exporters with petroleum accounting for around 18% of prohibited items–53, restricted items–485 and state trading
total exports. items–33.
Magbook ~ India’s Foreign Trade 75

Exports Overall Trade Balance


Controls primarily on account of security, public health, morals, — Taking merchandise and services together, overall
exhaustible resources and environment grounds. Prohibited trade deficit for April-March 2017-18 is estimated
items–59, restricted items–155 and state trading items–12. at US $ 87.17 Billion (estimated) as compared to US $
Special provision for these items under Weapons of Mass 47.70 Billion during April-March 2016-17. Taking
Destruction Act, 2005. Export Facilitation Committee looks into merchandise and services together, overall trade deficit
applications for licence for these items–special chemical, for April-March 2018-19 is estimated at USD 95.85
organisms, materials, equipments and tech. Billion as compared to USD 86.05 Billion (Actual) in
April-March 2017-18.
Trade Composition
Import Cover
Export Composition — The stock of foreign exchange reserves in terms of
— Noticeable compositional changes have taken place in India’s months of retrained imports of goods as at end of year. It
export basket between 2000-01 and 2013-14 with the share measures the number of months of money available in
of petroleum, crude and products increasing by nearly five the national bank to cover the cost of imports. It is an
times to 20.1%, catapulted by its 33.5% growth Compound operational definition of import cover. An import coverage
Annual Growth Rate (CAGR). ratio is the share (or percentage) of a country’s own
— The commodity composition of India’s trade has undergone imports that is subject to a particular non-tariff barrier or
many changes since liberalisation and has been driven by anyone of a specified group of non-tariff barriers.
trade policy, movements in international prices, and the — They are calculated by attaching actual values to
changing pattern of domestic demand. bilateral trade flows between various exporters and the
importing country. Import cover is an important indicator
Trade Composition Exports of the stability of the currency. During the currency
— India shipped US $261 billion worth of goods around the crisis of 2013, when foreign exchange reserves fell to
globe in 2016. India’s top 10 exports accounted for 59.4% of around $275 billion, import cover dipped to around
the overall value of its global shipments. seven months. According to currency explits, eight to
— Based on statistics fromt the International Monetary Fund’s ten months of import cover is essential for the stability
World Economic Outlook Database, India’s total Gross of the currency.
Domestic Product amounted to $ 8.721 trillon as of October
2016. Therefore, exports accounted for about 3% of total Export Promotion
Indian economic output. — The main thrust of India’s Foreign Trade Policy has
India’s Top 10 Exports been to promote exportable goods and produce
importable goods in the country to meet the domestic
List of top 10 export product categories according to Department
demand for foreign goods. A brief account of these
of Commerce, Government of India : Natural or cultured pearls,
major policy thrust are as follows:
precious and semi-precious stones, Mineral fuels, Vehicle other
— Export promotion refers to policies and measures of the
than railways, Nuclear reactors, boilers, Organic chemicals,
government designed to encourage exports with a view to
Pharmaceutical products, lron and steel, Apparel and clothing
improving forex reserves and correcting the BoP deficit.
accessories, Electrical machinery, Cereals. The significance of export promotion (as a strategy to
Imports combat BoP deficit), has come into greater focus after
imports have been liberalised in accordance with the
— Indian imported US $ 356.7 billion worth of goods from emerging trend towards globalisation. The government is
around the globe in 2016. Down by 8.7% from 2015 to 2016. trying to encourage exports through various kinds of cash
India’s top 10 imports accounted for almost three-quarters incentives, subsidies as well as concessions.
(74.3%) of the overall value of its product purchases from
other countries. Indian imports represent 2.2% of total global Measures for Export Promotion
imports which totaled $ 16.473 trillion one year earlier in 2015. — Several committees have been appointed to offer
suggestions for the promotion of exports, viz Gorewala
India’s Top Imports Products
Committee 1950, De Souza Committee 1957, Import
The following product groups represent the highest dollar value and Export Policy Committee 1962, Alexander
in India’s import purchases during 2018-19. Also shown is the
Committee 1979, Tondon Committee 1980, Abid Hussain
percentage share each product category represents in terms of
Committee 1984 and Rangarajan Committee 1991.
overall imports into India: Pulses, Project goods, Fertilisers, crude
and manufactured,Pulp and water paper, Gold, Newsprint, — It is the recommendation of these committees that
Machine tools, Iron and steel, Medical and pharmacautical several steps have been initiated for the promotion of
products, Fruits and vegetables. exports.
76 Magbook ~ Indian Economy

The central government has sanctianed 60 AEZs


Export Processing Zone (EPZ) —
comprising about 40 agricultural cammodities. AEZs is
— A Free Trade Zone (FTZ) or Export Processing Zone (EPZ), spread across 20 states in the country.
also called foreign-trade zone, formerly free port, is an — His objective of setting up AEZs is to coverage the efforts
area within which goods may be landed, handled, made hitherto by various central and state government
manufactured or reconfigured and re-exported without the departments for increasing exports of agricultural
intervention of the customs authorities. commodities from India.
— Only when the goods are moved to consumers within the
country, in which the zone is located do they become
subject to the prevailing customs duties.
General Agreement on
— Free-trade zones are organised around major seaports, Tariffs and Trade
international airports and national frontiers-areas with — The General Agreement of Tariffs and Trade (GATT) was a
many geographic advantages for trade. It is a region multilateral agreement regulations international trade.
where a group of countries has agreed to reduce or According to its preamble, its purpose was the
eliminate trade barriers. Free trade zones can be defined “Substantial reduction of tariffs and other trade barriers
as labour intensive manufacturing centres that involve the and the elimination of preferences, on a reciprocal and
import of raw materials or components and the export of mutually advantageous basis.”
factory products. — It was negotiated during the United Nations Conference
on Trade and Employment and was the outcome of the
Special Economic Zone (SEZ) failure of negotiating governments to create the
— Asia’s first Export Processing Zone (EPZ), was set-up in International Trade Organisation (ITO).
Kandla, India in 1965. The first SEZ Policy was — GATT was signed in 1947 and lasted until 1994, when it
announced in April 2000, which inter-alia provided for to was replaced by the World Trade Organisation in 1995.
make SEZ an engine of growth supported by quality The original GATT text (GATT 1947) is still in effect under
infrastructure backed up by attractive fiscal package. the WTO framework, subject to the modifications of GATT
— To import stability to the SEZ regime, SEZ Act, 2005 1994.
was enacted and which came into effect from 10th
February, 2006. As per the provisions of the SEZ Act, GATT and the World Trade
2005 100% FDI is allowed in SEZs through the automatic Organisation (Uruguay Round)
route. Incentives The act offers a highly attractive fiscal
incentive package, which ensures
— In 1993, the GATT was updated (GATT 1994) to include
new obligations upon its signatories. One of the most
— Exemption from custom duties, central excise duties, service
tax, central sales taxes and securities transaction tax to both significant changes was the creation of the World Trade
the developers and the units. Organisation (WTO).
— Tax holidays for 15 years (currently the units enjoy a 7 years — The 75 existing GATT members and the European
tax holiday), i.e. 100% tax exemption for 5 years, 50% for the Communities became the founding members of the WTO
next 5 years and 50% for the ploughed back export profits for
on 1st January, 1995.
the next 5 years.
— 100% income tax exemption for 10 years in a block period of — The other GATT members rejoined the WTO in the
15 years for SEZ developers. following 2 years (the last being Congo in 1997). Since,
— Infrastructure Provisions have been made for the founding of the WTO, 21 new non-GATT members
— The establishment of free trade and warehousing zones to have joined and 29 are currently negotiating membership.
create world class trade-related infrastructure to facilitate — There are total of 164 member countries in the WTO.
import and export of goods aimed at making India a global Afghanistan became the new member as of 29 July, 2016.
trading hub. The setting up of offshore banking units and Russia and Vanuatu became members in 2012 but Syria
units in an International Financial Service Centre in SEZs.
and SFR Yugoslavia have not rejoined the WTO.
— The public private participation in infrastructure development.
— The setting up of a ‘SEZ authority’ in each Central Black Money
Government SEZ for developing new infrastructure and — Black money is unaccounted for and untaxed cash
strengthening the existing one. generated by dealings in black economy. Black money
Agri Export Zones (AEZ) proceeds are usually received in cash from underground
economic activity and, as such, is not taxed.
— With the prime objective of boosting agricultural exports
from India in March 2001, Government of India
— Recipients of black money hide it, spend it only in the
announced a policy of setting up the Agri Export Zones underground economy, or attempt to give it the
(AEZs) across the country. appearance of legitimacy through money laundering.
Self Check
Build Your Confidence
1. Consider the following statements 7. Which of the following constitute capital account?
1. Applying a tax on foreign inflows into a country is known [UPSC 2013]
as ‘Tobin Tax’. 1. Foreign Loans
2. Brazil has applied ‘Tobin Tax’ many times. 2. Foreign Direct Investment
Which of the statement(s) given above is/are correct? 3. Private Remittances
(a) Only 1 (b) Only 2 4. Portfolio Investment
(c) Both 1 and 2 (d) Neither 1 nor 2 Codes
(a) 1, 2 and 3 (b) 1, 2 and 4
2. Consider the following statements (c) 2, 3 and 4 (d) 1, 3 and 4
1. The term Hot Money is associated with Foreign Portfolio
Investment. 8. The problem of international liquidity is related to the
2. Transfer pricing practices relate to Foreign Direct
non-availability of [UPSC 2015]
Investment. (a) goods and series
Which of the statement(s) given above is/are correct? (b) gold and silver
(c) dollers and other hard currencies
(a) Only 1 (b) Only 2
(d) exportable surplus
(c) Both 1 and 2 (d) Neither 1 nor 2
3. The Balance of Payments (BoP) of a country is a 9. A great deal of Foreign Direct Investment (FDI) to India
systematic record of [UPCS 2013]
comes from Mauritius comparing with many major and
mature economics like UK and France. Why?
(a) all import and export transactions of a country during a
(a) India has preference for certain countries as regards
given period of time, normally a year
receiving FDI.
(b) goods exported from a country during a year
(b) India has double taxation avoidance agreement with
(c) economic transaction between the government of one
Mauritius.
country to another
(c) Most citizens of Mauritius have ethnic identity with India
(d) capital movements from one country to another
and so they feel secure to invest in India.
4. The term ‘Paper Gold’ means [UPPCS 2004] (d) Impending dangers of global climatic change prompt
(a) Special Drawing Right (SDRs) of the IMF Mauritius to make huge investments in India.
(b) special accommodation facility of the World Bank
10. Which one of the following measures is not likely to aid
(c) currencies still on gold standard
in improving India’s Balance of Payment position?
(d) deficit financing
(a) Promotion of Import substitution policy.
5. Consider the following statements (b) Levying of higher duties on export.
1. Foreign Exchange Management Act (FEMA) was passed (c) Imposition of higher tariff on imports
in 1991 to replace Foreign Exchange Regulation Act. (d) Devalution of Rupee
2. FEMA serves to make transactions for external trade 11. Who maintains the foreign exchange reserve in India?
(exports and imports) easier.
(a) State Bank of India
Which of the statement(s) given above is/are correct?
(b) Reserve Bank of India
(a) Only 1 (b) Only 2 (c) Ministry of Finance government of India
(c) Both 1 and 2 (d) Neither 1 nor 2 (d) Export-Import Bank of India
6. Which one of the following factors is tercet in account to 12. The important reasons for deficit in India’s BoP position
calculate the Balance of Payments (BoP) of a country? is/are
(a) Current account (a) Devaluation and Depreciation of the Rupee
(b) Changes in the foreign exchange reserves (b) Debt Service
(c) Errors and omissions (c) Both (a) and (b)
(d) All of the above (d) Neither (a) nor (b)
Chapter ten
Demographic Profile of India
These four stages have been described below:
Demography
India comes next only to — Demography is a statistical study of First Stage
China as regards to the human population. It studies a variety of — Stage of high birth rate and high death
size of its population, variables related to population like size, rate.
growth, distribution, density, composition Birth and death rates are both high,
but is seventh in the —
and their spatial and temporal variations. population growth is slow and fluctuating.
world as regards to the — A broader study of demography also
area. Thus, on 2.4% of includes the relationship between the Second Stage
world’s area and with aforementioned variables and the (Early Expanding)
1.8% of world’s income, economic, social, cultural and other
— Stage of high birth rate and low death rate:
factors.
India is maintaining 18% — Birth rate remains high death rate falls.
— Demographics are the quantifiable Population begins to rise rapidly.
of world's population. It statistics of the given population based
clearly indicates that on the study of demography. Third Stage (Late Expanding)
there is excessive — Stage of declining birth rate and low death
burden of population Theory of Demographic rate:
in India. Transition — This stage is characterised by decline in birth
— The Demographic Transition theory is a rate, low death rate and low population
generalised description of the changing growth (growth rate of population declines).
pattern of mortality fertility and growth — Birth rate starts to fall; death rate continues
to fall. Population continues to rise.
rates as societies move from one
demographic regime to another.
— Theory of Demographic Transition is
Fourth Stage (Low Fluctuating)
— Stage of low birth rate and low death rate:
credited to Frank W Notestein, who
— In the fourth stage of demographic transition,
gave his theory in 1945. This theory
a low birth rate and low death rate lead to a
was based on the data from Western stationary or declining population.
countries, which experienced a — It is called a stage of stationary population.
transition in demography from the stage — Birth and death rates both are low.
of high birth rates and high mortality to Population is steady or declining as in many
a stage of low birth rates and low Western European nations at present.
mortality with a consequent declining
population.
— According to this theory, all
Optimum Theory of
countries pass through stages of Population
demographic transition, which is — This theory states that in every country,
accompanied by industrialisation and there is an optimum level of population.
economic development. While Notestein “The optimum population is that, which
gave three stages of demographic gives the maximum income per head.” If
transition, sex expanded them into four the population exceeds the optimum level,
stages. there is the problem of over population.
Magbook ~ Demographic Profile of India 79

— 1961-1971 This period witnessed an increase in population


Population Theory of Malthus by 10 crore 90 lakhs. Growth rate was 24.8%.
— Thomas Robert Malthus was the first economist to propose — 1971-1981 During this period, population in India rose to
a systematic theory of population. He gave his ideas 68 crore 33 lakhs. Thus, 13 crore 51 lakhs persons were
regarding population in his book Essay on the Principle of added to the total size of India’s population.
Population. He argued that while food production could
only increase in arithmetic progression, human populations (iv) Period of High Growth with Definite
grow exponentially. Thus, he predicted a future when Signs of Slowing Down (1981 to Present)
humans would have no resources to survive on. — 1981-1991 In this period, population went up to 84 crore
— He supported ‘preventive’ and ‘positive’ checks on 63 lakhs making addition of 16 crore in 10 years.
population growth such as late marriage. He was however, — 1991-2001 In 2001, the population of India went up to
against birth control after marriage. 102.90 crore. Thus, between the period 1991-2001, the
— Most modern economists disagree with Malthus since, he population of India increased 18.07 crore.
neglects the possibilities from technology and the fact that — 2001-2011 In 2011, the population of India, was 121.08
a larger population increases the chances of someone crores. This represents an increase of 18.14 crore in the
achieving breakthroughs in technology. previous decade. It was the first time since, census began
that the decadal population growth was lower than the
Size and Growth of India’s previous decade.
Population
— India’s population increased rapidly in the post
independence period. Between 1951-61, it increased by
Birth and Death Rates
more than 7.82 crore or by nearly 21.6%, which — Birth and death rates in India, are high compared to most
exceeded its growth rate of the previous 40 years. This countries in the world. Birth rate refers to number of
excessive rise in population is called population explosion. children born per thousand persons in a year. Death rate
refers to number of people dying per thousand persons
— Since 1951, population has been increasing constantly.
in a year. When it is said that birth rate in India is 23, it
Between 1971-81, growth rate of population was 24.8%
means every year 23 children are born per thousand
and between 1981-91, it was 23.8%.
persons on an average.
— India’s population growth rate, has decelerated to 17.64%
— It is clear that birth rate in India, is still very high in
in the decade 2001-2011, the slowest rate to growth in
comparison to birth rate of developed nations like Japan,
this past century. Study of the growth of India’s population
can be divided into four periods of time: Germany, Canada and UK.
— If, we compare the birth rates of India and China, we find
(i) Period of Stable Population (1891 to 1921) that birth rate in India is double the birth rate in China.
— Between 1891 and 1921, rate of growth of population in However, death rate in India is almost same in
India, was low. In these 30 years, population increased by comparison to the death rates of other countries.
1.26 crore.
It was so because in these years, calamities and
—
epidemics, like famines, plague, malaria etc took a heavy
Density of Population
toll of human lives. The epidemic in 1918, took a toll of — Density of population refers to average number of people
140 lakh human lives. living per square kilometre. Density of population in a
country is measured by dividing its total population by
(ii) Period of Growth of Population
total land area.
(1921 to 1951)
— In 2011, India’s total population was 121 crore that used
— Since 1921, population has been increasing at a rapid
to live over an area of 32.80 lakh sq km. According to
rate. The trend of growth of population in India, since
Census 2011, density of population in India was 382 per
1921, has been consistently on the rise. That is why,
sq km.
Census Commissioner has referred the year 1921 as Year
of Great Divide. This increase was higher than that of the Real Population Density
previous 30 years. — Instead of measuring population density for the entire
(iii) Period of Population Explosion area of the country, some parts of which might not be
(1951 to 1981) liveable, a more accurate measure is considered to be
— In this period, population increased at a very fast rate. (real population density) or physiological density, which
This period is called period of population explosion. measures the population per sq km of arable land
— 1951-1961 In this period, growth rate was recorded to be available in a country. This measure signifies the feeding
21.6%, which was highest for any decade before that. potential of the country to its citizens.
80 Magbook ~ Indian Economy

— India’s physiological density is much lower compared to many Migration Effect


other countries due to the availability of large arable land. — Rural life in India, suffers from many difficulties, such
as less opportunities of employment, low level of
Census 2011 income, lack of educational and training facilities, lack
of health and medical facilities etc. In order to get rid

The Census 2011 is the 15th National Census survey
of these difficulties, rural people migrate to urban
conducted by the Census Organisation of India.
areas.

Mr C Chandramouli is the Commissioner and Registrar
General of the Indian 2011 Census.
Age Structure/Composition

The 2011, Indian National census has been conducted in
— Age structure of the population of a country indicates
2 phases – house listing and population.
the extent, to which the population of that country is

The National Census survey covered all the 28 States of productive from the economic point of view.
the country and 7 Union Territories including 640
— Population in the age group of 15-60 years is known
districts, 497 cities, 5767 Tehsils and over 6 lakh
as working population. Population in the age group of
villages.
0-14 years and above 60 years is known as
non-working or dependent population. Higher
Rural-Urban Population proportion of working population is beneficial for the
— Ratio of urban population to the total population of a country economic development of the country.
is an index of the level of industrialisation of that country. As — In India, percentage of population in the age group of
industries gather momentum in a country, ratio of urban 0-14 years is still high. In India, percentage of
population goes on rising. population above 60 years is increasing. That indicates
— India is an agricultural country, so ratio of urban population is higher life expectancy and reduction in death rate.
less than the rural population.
Demographic Dividend
Important Facts Related to Rural/Urban — It refers to an opportunity before a country with a high
Population (2011 Census) share of population between the ages of 15 and 64, to
— In the decade from 2001 to 2011, rural population increased boost economic growth. This stage is reached when
by 90.47 million, while urban population increased by 91 the country experiences lower fertility rates, which
million. For the first time, increase in urban population has means that the dependent population below the age of
outpaced the growth in rural population. In percentage terms, 15 years is low.
the increase in rural and urban population was 12.18% and — The population above 64 years of age is also low due
31.80% respectively. to the lower life expectancies of the older generations.
— Himachal Pradesh (89.96%) has the largest proportion of On the other hand, the population between 15-64 is
rural population, while Delhi (97.5%) has the highest high, due to the higher birth rates in the previous
proportion of urban population. generation. This reduces the dependency ratio (share
— Sex ratio in the country improved by 7 points, from 933 in of population not engaged in productive employment
2001 to 940 in 2011. The improvement in rural areas was of and dependent on others) and thus, boosts the
one point from 946 to 947, while in urban areas, it improved economic growth.
by 26 points from 900 to 926. Kerala has the highest sex ratio — A larger working population also means that the
in total (1084), rural (1077) and urban (1091) population. domestic savings rate is high (since, the dependent
— Child sex ratio (0-6 age group) has dropped in the country by population does not increase savings, but reduces
13 points between 2001 and 2011 (927 to 914). In rural them) and thus, investment and economic growth is
areas, the fell was higher at 15 points (934-919), while in higher. Many East Asian countries were able to
urban areas, it fell by 4 points (906-902). Andaman and achieve high economic growth rates by utilising their
Nicobar Islands has the highest child sex ratio in rural areas demographic dividend.
(975), while Nagaland has the highest in urban areas (979). — Demographic dividend can only be useful, if it is
— Literacy rate for the country as a whole was 74.04% and was accompanied by supportive national policies, which
68.91% in rural areas and 84.98% in urban areas. The improve literacy, provide employment, health care etc.
increase in literacy rates was 9.21 points (total), 10.17 points High share of young population in a country can also
(rural) and 5.06 points (urban). Two Main causes of rise in have negative consequences like social unrest, crime
urban population in India are as follows: and high divorce rates etc.
Magbook ~ Demographic Profile of India 81

Demographic Dividend: India Male rate of literacy was 82.14% and female rate of
literacy was 65.46%.
Population
1961 1971 1981 1991 2001 2011 — The highest literacy rate was in Kerala. It was 93.91%.
Group/Year
— In Bihar, the literacy rate was just 63.82%.
0-14 41 41.2 39.5 37.7 34.3 30.2
— The female literacy rate of Kerala was 91.98%, which is
15-64 56 55.5 57 58.4 61.4 64.8
also the highest in India.
65 and above 3 3.3 3.5 3.9 4.3 5
— The lowest female literacy rate was in Rajasthan, it was
Number shows the share of population as a percentage of total only 52.66%. In Punjab, the female literacy rate was
population. 71.34%. In Haryana, it was 66.77% and in Himachal
Source World Bank Pradesh, it was 76.6%. In Jharkhand, it was only 56.21%.
— It may be noted that all the States and Union Territories
Population Pyramid have shown increase in literacy rate during 2001-2011.
— Population pyramid is a graphical illustration of the
different age groups in a population along with the male National Population Policy
and female population. The horizontal axis represents the — Population policy refers to all those legal, administrative
absolute numbers of population, with one side programmes and other government efforts, which aim at
representing the male population and the other side reducing birth rate and improving the quality of life.
representing the female population.
— After independence, the Government of India adopted a
— The vertical axis is divided into equal divisions national policy on population with the objective to check
representing different age groups such that it the increase in birth rate and improve the standard of
encompasses the entire population of the country or living of people. This policy has been revised from time to
region. time and its scope has been widened. It has been very
effective in initiating measures for population control.
Life Expectancy
— Expectation of life refers to the average life of the New National Population Policy (2000)
people of a country. In India, expectation of life of — The Government of India announced its New
the people is very short. It has improved as a result of National Population Policy on 15th February, 2000. The
planned efforts. National Population Policy (NPP) affirms the commitment
Average life expectancy is shown in the table below : of government towards voluntary consent of citizens, while
availing of reproductive healthcare services.
Expectation of Life (in years) — The New National Population Policy (NPP) provides a
Years Life Expectancy Years Life Expectancy policy framework to meet the reproductive and child
health needs of the people of India for the next 10 years.
1921 19.4 1971 52.0
1931 26.9 1981 54.0 Objective of NPP, 2000
1941 32.0 1991 59.0 — The immediate objective of the NPP 2000 is to address
the unmet needs for centra caption, healthcare,
1951 33.0 2001 64.0
infrastructure and health personnel and to provide
1961 41.0 2011 69.89 integrated service delivery for basic reproductive and child
— In other countries of the world, expectation of life is much healthcare.
longer than ours. For instance, in Australia it is 79 years,
in Japan 82 years, in England 77 years, in America 78 Twelfth Plan and Family Welfare
years, in Sweden and in Canada it is 80 years. Programmes
— The main targets of twelfth Plan regarding family welfare
Sex Ratio are given below:
— All over the world, males out number the females. Sex ratio in
the world is 986 females to 1000 males. According to 2011 Particular Target (2016-17)
Census, sex ratio in India was 940 females to 1000 males. Infant Mortality Rate (IMR) 25 per 1000

Literacy Maternal Mortality Rate (MMR) 100 per 100000


— Any person above the age of 7 years, who can read and Total Fertility Rate 2.1
write in any language is treated as literate. According to
Sex Ratio for 0-6 years age group from 914 to 950
the Census 2011, the rate of literacy in India was 74.04%.
82 Magbook ~ Indian Economy

Socio-Economic and Caste Census (SECC) Percentage of Elderly Persons to


— The first post-independence, socio-economic and caste Total Population (India)
census was carried out by the Government in 2011. It will Total Population % of Elderly
be carried out by the State Governments with the financial Years
(in million) Persons
and technical support of the Government of India.
2001 70.69 6.90
— The census will identify poor households based on certain
2006 83.58 7.50
automatic inclusion criterias, automatic exclusion criterias
2011 98.47 8.30
and deprivation indicators.
2016 118.10 9.30
Census, 2011 Old Age People 2021 143.24 10.70
— India has the second largest population of older (60+) 2026 84.62 12.40
persons in the world. The number of older persons in the
population is expected to increase from 71 million in 2001 Religion Based Census Report
to 173 million in 2026. — The Census 2011 data on Population by Religious
— A majority (80%) of the elderly population in India is in the Communities, released by the Registrar General of India
rural areas, thus, making service delivery to them a on August 25, 2015. It showed that the percentage of
challenge and there are a large number of 80+ persons in Hindus dipped by 0.7 Percentage Points (PP) in the
the country. decade 2001 to 2011, pulling it for the first time
below 80%.
National Population Register (NPR)
— The distribution is total population by six major religious
— The Government of India has decided to create a National
communities namely, Hindu, Muslim, Christian, Sikh,
Population Register (NPR) for the country.
Buddhist, Jain besides ‘Other Religions and Persuasions’
— NPR will be a register of the usual residents of the and ‘Religion not stated’.
country. It is compulsory for every citizen of the country to
register in a National Register of Indian Citizens (NRIC). Population by Religious Communities
NRIC will be prepared from NPR after verification of based on Census 2011
citizenship.
Religion Population
— The data collected from NPR will be sent to UIDAI, for
Hindus 96.63 crore (79.8%)
de-duplication and issue of Aadhar number. A person,
who has already registered with UIDAI will have to enrol Muslims 17.22 crore (14.2%)
again in the NPR. Christians 2.78 crore (2.3%)
Sikhs 2.08 crore (1.7%)
Aadhar Buddhists 0.84 crore (0.7%)
— Aadhar is a 12 digit unique number, which the Unique
Identification Authority of India (UIDAI) will issue to Jains 0.45 crore (0.4%)
residents of India on a voluntary basis. Other Religions & Persuasions 0.79 crore (0.7%)
— The number will be generated randomly and will not be Religion Not Stated 0.29 crore (0.2%)
based on any classification. The numbers will be linked to
the basic biometric information of the person, including Population by Religious Communities in India
photograph, iris and fingerprints.
Religious Community Person Male Female
UIDAI All Religious
1028610328 532156772 496453556
(Unique Identification Authority of India) Communities
— UIDAI is an attached office under the Planning Commission Hindus 827578868 428678554 398900314
of India, which was set-up in January, 2009, to issue unique Muslims 138188240 71374134 66814106
identify numbers to people in the country and own and
Christians 24080016 11984663 12095353
operate the Aadhar number database on an on-going
basis.The agency will issue only the number and not the Sikhs 19215730 10152298 9063432
smart cards. Buddhists 7955207 4074155 3881052
— The agency is headed by a chairman, who holds a cabinet Jains 4225053 2177398 2047655
rank. The first Chairman of UIDAI is Nandan Nilekani, Other Communities 6639626 3332551 3307075
former co-chairman of Infosys technologies.
Magbook ~ Demographic Profile of India 83
State-Wise Sex Ratio in 2001 and 2011
Sex Ratio Population Density Decadal Rate Literacy Rate
State/Union Territory Total Population
(per 1000 females) (per km) (growth rate) (in %)
Total India 1210569573 943 382 17.7 74.04
Jammu and Kashmir 1,25,41,302 889 124 23.6 67.2
Himachal Pradesh 68,64,602 972 123 12.9 82.8
Punjab 2,77,43,338 895 551 13.9 75.8
Uttarakhand 1,00,86,292 963 189 18.8 78.8
Haryana 2,53,51,462 879 573 19.9 75.6
Rajasthan 6,85,48,437 928 200 21.9 66.1
Uttar Pradesh 19,98,12,341 912 829 20.2 67.7
Bihar 10,40,99,452 918 1106 25.4 61.8
Sikkim 6,10,577 890 86 12.9 81.4
Arunachal Pradesh 13,83,727 938 17 26.0 65.4
Nagaland 19,78,502 931 119 − 0.6 79.6
Manipur 25,70,340 985 128 24.5 79.2
Mizoram 10,97,206 976 52 23.5 91.3
Tripura 36,73,917 960 350 14.8 87.2
Meghalaya 29,66,889 989 132 27.9 74.4
Assam 3,12,05,576 958 398 17.1 72.2
West Bengal 9,12,76,115 950 1028 13.8 76.3
Jharkhand 3,29,88,134 949 414 22.4 66.4
Odisha 4,19,74,218 979 270 14.0 72.9
Chhattisgarh 2,55,45,198 991 189 22.6 70.3
Madhya Pradesh 7,26,26,809 931 236 20.3 69.3
Gujarat 6,04,39,629 919 308 19.3 78.0
Maharashtra 11,23,74,333 929 365 16.0 82.3
Uttar Pradesh 8,45,80,777 993 308 11.0 67.0
Telangana 3,52,86,757 988 307 17.87 66.4
Karnataka 6,10,95,297 973 317 15.67 75.4
Goa 14,58,545 973 394 8.2 88.7
Kerala 3,34,06,061 1084 860 4.9 94.0
Tamil Nadu 7,21,47,030 996 555 15.6 80.1
Delhi 1,67,87,941 868 11320 21.2 86.2
Chandigarh 11,55,450 818 9258 17.2 86.0
Dadra and Nagar Haveli 2,43,247 618 2191 53.8 87.1
Daman and Diu 3,43,709 774 700 55.9 76.2
Lakshadweep 64,473 947 2149 6.3 91.8
Puducherry 12,47,953 1037 2547 28.1 85.8
Andaman and Nicobar 3,80,581 876 46 6.9 86.6
84 Magbook ~ Indian Economy

Self Check
Build Your Confidence
1. Consider the following statements 6. Which arrangement of following would show the correct
1. This is constant increase in population density of India sequence of demographic transition as typically
since 1901. associated with economic development? [UPSC 2013]
2. There is constant decline in sex ratio since 1901. 1. High birth rate with high death rate.
Which of the statement(s) given above is/are correct? 2. Low birth rate with low death rate.
(a) Only 1 (b) Only 2 3. High birth rate with low death rate.
(c) Both 1 and 2 (d) Neither 1 nor 2 Codes
(a) 1, 2, 3 (b) 1, 3, 2
2. Which of the following statements is correct?
(c) 3, 1, 2 (d) 2, 1, 3
(a) No modifications are done in census even though the
results obtained form post- enumeration surveys might 7. Consider the following statements
not match with those obtained from actual census 1. Assam has the highest percentage of poverty according
(b) Census is a mandatory exercise mentioned in the to the Suresh Tendulkar Committee in India.
Constitution to be executed by all states as well as union 2. In India, the infant mortality rate per thousand of live
(c) Census has not been mentioned in Constitution, but is birth of girls is more than for boys.
carried out as a policy parameter Which of the statement(s) given above is/are correct?
(d) National Population Policy, 2000 had an immediate (a) Only 1
objective to bring fertility rate to replacement levels (b) Only 2
3. Consider the following statements regarding census of (c) Both 1 and 2
India. (d) Neither 1 nor 2
1. Census is held all places in India simultaneously. 8. Which of the following statement deals with the benefit
2. Census, 2011 shows decline in population for the first of National Population Register?
time in history of India. (a) Strengthen security of the country and improve planning
Which of the statement(s) given above is/are correct? and prevent indentity theft
(a) Only 1 (b) Only 2 (b) Gives the basic biometric information
(c) Both 1 and 2 (d) None of these (c) Estimated the community based population
(d) Provision for population education in educational
4. Which of the following causes are major hurdle in
institution
tapping demographic dividend in India?
1. US appropriate labour policy. 9. What percentage of the total population of the world
2. Less female practice pation in workforce. resides in India, as estimated in 2011? [UPPCS 2012]
3. Migration. (a)15 (b) 17.5
4. Low inclusiveness and less employment generation. (c) 20 (d) 22.5
Which of the following statements is/are correct? 10. During which decade did the population record a
(a) 1 and 2 (b) 2 and 3 negative growth rate in India? [UPPCS 2012]
(c) 1, 2 and 4 (d) All of these
(a) 1921-1931 (b) 1911-1921
5. Consider the following statements (c) 1941-1951 (d) 1941
1. In India, the unemployment rate on the Current Daily 11. What will be true if India have more number of people in
Basis (CDS) by NSSO data in always more than 25%. the productive age groups?
2. In last decade, the employment generated by the public 1. More incomes
sector is always more than of the private sector. 2. More demand of products
Which of the statement(s) given above is/are correct? 3. High GDP
(a) Only 1 (b) Only 2
Select the correct answer using the codes given below
(c) Both 1 and 2 (d) Neither 1 nor 2
(a) Only 2 (b) 2 and 3
(c) 1 and 3 (d) All of these
Chapter eleven
Agriculture
Development of Tertiary Sector Tertiary sector
Agricultural Sector —
provides helpful services to the industries and
The Indian economy — Agriculture is the primary industry in India. agriculture like banking, warehousing etc.
The agriculture sector of India has — Internal Trade is mostly done in agricultural
continues to be occupied almost 43% of India’s produce. e.g. various means of transport get
predominantly an geographical area and over 58% of the bulk of their business by the movement of
Agricultural rural households depend on agriculture as agricultural goods.
Economy in terms of their principal means of livelihood. — Contribution in Foreign Trade Agriculture plays
— As per the NSSO, in 2011-12, the share of an important role in the international trade.
employment of
agriculture in employment was 48.9%. In Jute, tea, coffee and spices are the country’s
labour force and as 2015-16, agriculture contributed 17.4% to well known conventional exports.
a source of India’s GDP, as compared to 18.3% in — Fertilizers, harvesters and thrashers are the
subsistence for the 2013-14. notable import items meant exclusively for
millions in the — Agriculture and allied sector contributes agriculture sector of the economy.
15.2% in the financial year 2016-17. For International Importance India is the largest
countryside. It is an the year 2018-19, growth in the agriculture
—
producer of coconuts, mangoes, bananas, milk
undisputed fact that and allied activities was estimated at 2.7%, and dairy products, cashew nuts, pulses,
the dream of down from 5% in 2017-18. ginger, turmeric and black pepper. It is also
‘Inclusive growth’ — This is the biggest unorganised sector of the the 2nd largest producer of rice, wheat, sugar,
economy accounting for more than 90% cotton, fruits and vegetables.
will remain a far-cry, share in total unorganised labour force.
if the growth plans
fail to account for Agriculture Export Policy, 2018 to double
Five Year Plan and
the significance of agricultural exports from present US $ 30 Agriculture
agriculture. To billion to 60 billion by 2022 and reach 100
billion in the next few years. — 1st Plan (1951-56) The 1st Plan aimed at
quote Professor solving food crisis, hence, highest priority to
Gunnar Myrdal, “It is agriculture with allocation of more than 14% of
Features of Indian the total plan outlay.The growth in agriculture
in the agriculture
sector that the
Agriculture Sector remained 2.71%.
— Largest Employment Providing Sector — 2nd Plan (1956-61) This plan saw significant
battle for long-term Agriculture in India, is the most important reduction in agricultural outlay. It was 11.7%
economic source of employment. of the total plan outlay.This plan witnessed a
growth of 3.15% in agricultural sector.
development of — Basis for Industrial Development Agriculture
offers raw materials including (cotton, — 3rd Plan (1961-66) 2nd Plan experience and
India will be won or
sugarcane and oilseeds) for industries like recognition that agricultural production is the
lost.” limiting factor, the 3rd Plan fixed ambitious
textiles, sugar and oil-processing etc.
Besides, it also offers market for the targets of production for all agricultural crops.
expanding industrial sector of the economy. This plan also saw the introduction of Intensive
Agricultural District Programme (IADP),
— Industries producing capital goods (like
followed by High Yielding Variety Programme
tractors, thrashers and harvesters) are
(HYVP). Agricultural growth fell to a low of
directly dependent upon agricultural sector.
0.73%.
86 Magbook ~ Indian Economy

— 4th Plan (1969-74) This plan aimed at systematic — Some areas, where the plan will be focussed are as follows:
application of science and technology to improve — Public-Private Partnership for better extension and marketing
agricultural practices. The allocation to agriculture sector services. This includes modifying Agricultural Produce Market
was 15% of the total plan outlay. Agricultural growth Committee (APMC) acts to encourage setting up of private
reached a level of 4.16% during this plan. markets and contract farming.
— Development of soil testing and product quality testing facilities.
— 5th Plan (1974-79) The 5th Plan was the only period,
— Distribution of more institutional credit equitably, greater focus
when the actual foodgrain production exceeded the
on small and marginal farmers, improving productivity in
targeted production.The agricultural growth under this
rain-fed areas, retaining youth in agriculture and funding for
plan was 3.28%. research and innovations.
— 6th Plan (1980-85) Agriculture growth rate in this plan
was 4.3% as against the targeted 3.8% . The year
1983-84, of the plan is hailed as the Second Green
Green Revolution in India
Revolution. Agricultural sector during this plan made a — The introduction of high-yielding varieties of seeds after
growth of 2.52%. It was the result of expansion in 1965 and the increased use of fertilizers and irrigation are
supplies of inputs and services to farmers, agricultural known collectively as the Green Revolution, which provided
extension and better management. the increase in production needed to make India
— 7th Plan (1985-90) Total plan outlay on agriculture was self-sufficient in foodgrains.
6% and except cotton, none of the targets fixed for — The term ‘Green Revolution’ is a general one that is applied
various sectors was achieved. During this plan to successful agricultural experiments in many third world
agricultural growth was 3.47%. countries. It is not specific to India, but it was most
— 8th Plan (1992-97) Agriculture growth rate in this plan successful in India.
was 2.44% on account of weather and climate — There were three basic elements in the method of the Green
conditions being favourable.The agricultural sector Revolution:
registered an impressive growth rate of 4.68%. (i) Continued expansion of farming areas.
— 9th Plan (1997-2002) Agriculture growth rate in this (ii) Double-cropping existing farmland.
plan was 2.44%. All the set targets were not achieved (iii) Using seeds with improved yields.
and hence, 9th Plan was a failure on agriculture front.
In this plan, the agricultural growth fell to 2.02%. Drawbacks of First Green
— 10th Plan (2002-07) This plan adopted the prescription Revolution
of the National Agricultural Policy (NAP), 2000 and
— While the first Green Revolution achieved many successes,
therefore, envisaged better management of resources,
there were also many flaws in its strategy, which were not
soil and water, so as to promote sustainable and
envisaged at that time. These flaws include, negative impact
inclusive agricultural growth. The agricultural sector
on environment and health due to excessive use of fertilizers
grow at 2.3%.
and pesticides; depletion of soil nutrients; depletion of water
— 11th Plan (2007–12) This Plan witnessed an average resources including ground water; higher costs of input etc.
annual growth of 3.6% in the Gross Domestic Product
(GDP) from agricultural and allied sector against a target
of 4.0%. While it may appear that the performance of
Bringing Green Revolution in
the agriculture and allied sector has fallen short of the Eastern India Programme (BGREI)
target, production has improved remarkably, growing The BGREI was launched in 2010-11, as a part of the Rashtriya
twice as fast as population. Krishi Vikas Yojana. It was implemented in the Eastern region of
the country. It focused on resource allocation and utilisation. It
Agriculture in the 12th Plan (2012-17) has resulted in a robust increase in foodgrain production,
— As against the 12th Five Year Plan (2012-17) target 4% growth rate being estimated at 11.9% during 2011-12 as
growth for the agriculture and allied sectors, the growth against the overall growth rate of 2.2% of the country as a
registered was 4.2% in 2013-14, -0.2% in 2014-15 whole. The scheme is being continued, with an allocation of
1.2% in 2015-16, and 4.9% in 2016-17. ` 1000 crore in 2013-14.
— Some of the challenges that agriculture might face
during the 12th Plan include a shrinking land base, — Certain other conditions have also emerged after the first
dwindling water resources, adverse impact of climate Green Revolution, which are having a negative impact on
change, shortage of farm labour and increasing costs agriculture like, land constraints due to diversion of land to
and uncertainties associated with volatility in other economic areas; climate change; diversion of crops to
international markets. The 12th Plan targets an average bio-diesel; fragmentation of land holdings making farming
annual growth of 4% in agriculture. unviable.
Magbook ~ Agriculture 87

— For these reasons and to ensure the food security of the Rainbow Revolution
country, there is a need for a Second Green Revolution in — In July 2000, the Central Government of India had
the country, which would address all the problems.
announced the first-ever National Agriculture Policy. The
policy aimed at achieving a growth rate of over 4% per
Second Green Revolution annum by introducing Rainbow Revolution in the next two
in India decades so that the total GDP growth can be sustained at
6.5%.
— Second Green Revolution will consist of a number of
different programmes working towards the same goals. Tricolour Revolution
Some of the initiatives, which will help in this direction are — The reference to a Tricolour Revolution was made by
as follows: Prime Minister Narendra Modi. This phrase has three
— Increasing crop yields in Eastern states. components these are as follows:
— Organic farming and contract farming.
(i) Saffron Energy Revolution for promotion and better
— Amending the Agricultural Produce Marketing Committee utilisation of solar energy.
(APMC) Acts.
(ii) White Revolution to ensure cattle welfare and further the
— Investing in research to drought proof crops as well as to
goals of White Revolution.
tackle climate change.
(iii) Blue Revolution for fishermen’s welfare, cleansing rivers
— Investing in supply chain and cold chains.
and sea and conserving water.
— Encouraging private investments through tax law
amendments. Operation Green
— Use of plant breeding and biotechnology. — The Finance Minister Arun Jaitley has announced
— Rain water harvesting and watershed development. Operation Green on the lines of Operation Flood for
— Improving credit availability. enhancing the production of tomato, onion and potatoes in
— Refocusing on land reforms. budget 2018-19. A sum of ` 500 crore has been allocated
— Improving soil quality and reclaiming degraded land. for this new measure. It may help in doubling the income
of farmers by the end of 2022.

Major Agricultural Revolutions National Commission on Farmers


Green Revolution : Cereals, wheat and — It was appointed in 2004, under the chairmanship of Dr
leguminous plants. MS Swaminathan.
White Revolution : Milk and dairy products — The commission suggested an Agricultural Renewal Plan,

Silver Revolution : Egg and poultry which has five components


Yellow Revolution : Edible oil (i) Soil health enhancement with special focus on dry
farming.
Blue Revolution : Fishery
(ii) Irrigation water supply augmentation and demand
Pink Revolution : Prawns/Meat processing management.
Golden Revolution : Honey (iii) Credit and insurance facilities like creation of agriculture
Golden Fibre Revolution : Jute risk fund.
Silver Fibre Revolution : Cotton (iv) Technological reforms in the form of proper integration
of production and post-harvest technologies,
development of a cadre of rural farm science managers
and lab-to-land demonstrations.
Evergreen Revolution
(v) Assured and remunerative marketing.
— The concept was given by renowned agricultural scientist
— The commission also suggested a Risk Stabilisation Fund
Dr MS Swaminathan. Evergreen Revolution emphasises
and a farmer centric Minimum Support Price and Market
on organic agriculture and green agriculture with the help
Intervention Scheme (MIS) and creation of Pani
of integrated pest management, integrated nutrient supply
Panchayats.
and integrated natural resource management. The core of
the Evergreen Revolution is sustainability.
— Bringing Green Revolution to Eastern India, initiated in
Agricultural Inputs
2010-11, intends to address the constraints limiting the — It play a crucial role in determining yield levels and in turn
productivity of ‘rice based cropping systems’ in Eastern augmentation of level of production in the long run.
India comprising seven states, viz, Assam, Bihar, Improvement in yield depends on application of
Chhattisgarh, Jharkhand, Odisha, Eastern Uttar Pradesh technology, use of quality seeds, fertilizers, pesticides,
and West Bengal. micro-nutrients and irrigation.
88 Magbook ~ Indian Economy

Seeds (National Seeds Policy, 2002) — Since 2010, government is implementing a Nutrient Based
Subsidy Scheme (NBS) in which a fixed subsidy is
— Seeds are a critical input for long-term sustained growth
announced on per kg of nutrient annually. Additional subsidy
of agriculture. In India, more than four-fifths of farmers
is given to micro-nutrients. The prices of urea however,
rely on farm saved seeds leading to a low seed
remain under statutory price control.
replacement rate. Hence, the Central Government has
been addressing this issue through various
programmes/schemes.
Irrigation
— Indian Seed Programme involving the participation of — It is one of the most important inputs for enhancing
Central and State Governments, the Indian Council of productivity and is required at different critical stages of
Agricultural Research (ICAR), state agricultural universities, plant growth of various crops. The Government of India has
co-operatives and the private sector and farmers and plant taken up irrigation potential creation through public funding
breeders. and is assisting farmers to create potential on their own
— The Protection of Plant Varieties and Farmers’ Rights (PPV farms.
and FRs) authority established in November 2005, at New
— The total irrigation potential in the country has increased
Delhi, has been mandated to implement provisions of the
PPV and FR Act, 2001.
from 81.1 million hectare in 1991-92 to about 140 million
hectare in 2014-15.
— PPV and FR Act has been passed within the context of Sui
Generis System of the WTO, so as to effectively block the Initiation of the Accelerated Irrigation
efforts of MNCs to capture the seed market by getting
Benefit Programme (AIBP)
patents in their favour and gradually buying out small seed
growers in the country. — From 1996-97, to extend assistance for the completion of
incomplete irrigation schemes. Under this programme,
Sui Generis System projects approved by the Planning Commission are eligible
— TRIPS Agreement offers three options for plant varieties for assistance. In the budget 2012-13, a government owned
and their protection, viz, Patent System, Sui Generis water resource finance company is being operationalised to
Systems and combination of two. mobilise large resources to fund irrigation.
— Under Sui Generis System, farmer has the right to save, National Mission on Micro Irrigation (NMMI)
use, exchange share or sell the farm produce including
— Irrigation consumes more than 80% of the water resources
seeds. However, farmer cannot sell the branded seeds.
of the country. Availability of adequate quantity and quality of
Seed Bank water is the key to achieve higher productivity levels.
— A scheme for the establishment and maintenance of a — This mission will result in 2.85 million hectare to be brought
seed bank has been in operation since, 1999-2000. under micro irrigation; savings in use of irrigation water,
— The basic objective of the scheme is to make available fertilizer and electricity; increase in production and
seeds for meeting contingent requirements and also productivity of crops; convergence with other on going
develop infrastructure for production and distribution of schemes of Department of Agriculture and Cooperation (DAC)
seeds. and other ministries on creation of water harvesting structures
and linking the same with Micro Irrigation System for higher
— The scheme is being implemented through National
water use efficiency and enhanced return to the farmers.
Seed Corporation of India and 12 State Seeds
Corporations of various states. Pradhan Mantri Krishi Sinchai Yojana
(PMKSY)
Fertilizers — With an eye on improving farm productivity, the government
— India is meeting 85% of its urea requirement through has decided to spend ` 50,000 crore over the next 5 years
indigenous production, but depends heavily on imports under the Pradhan Mantri Krishi Sinchai Yojana (PMKSY).
for its phosphatic and potash (P and K) fertilizer — The major objective of the PMKSY is to achieve convergence
requirements. of investments in irrigation at the field level expand cultivable
Fertilizer Subsidy area under assured irrigation improve on farm water use
— Fertilizer subsidy is borne by the Union Government. efficiency to reduce wastage of water, enhance adoption of
The two objectives of providing fertilizer subsidy are as precision irrigation and other water saving technologies.
follows: Neeranchal Watershed Yojana
(i) Making fertilizers available to the farmers at affordable — The project is implemented by the Union Ministry of Rural
prices so as to encourage intensive cultivation. Development over a six-year period (2016-21) to achieve
(ii) Attracting more investment to the domestic fertilizer objectives of PMKSY on 7th October, 2015. It will support
industry. the Pradhan Mantri Krishi Sinchayee Yojana (PMKSY) in
Magbook ~ Agriculture 89

hydrology and water management, agricultural Power and Irrigation Subsidies


production systems, capacity building and — Since water and electricity fall within the state domain, power
monitoring, and evaluation. It seeks to ensure access and irrigation subsidies are provided by the State Governments.
to irrigation to every farmland (Har Khet Ko Pani) and — Irrigation subsidies are incurred on account of the pricing of
for efficient use of water (Per Drop More Crop).
irrigation water provided to the farmers by the Sate
Governments.
Sprinkler Irrigation — Consequences of power and irrigation subsidies.
Under sprinkler irrigation, water is sprinkled under — Increased fiscal burden.
pressure on to the crop through a set of nozzles attached — Less revenue available for investment in irrigation and other large
to a network of pipes in the form of rainfall. This system scale projects.
is suitable for high density horticultural crops. The — Over exploitation of ground water resources by farmers.
sprinkler system sets, unlike drip system are moveable. — Inefficient use of irrigation water leading to water logging and
Hence, one sprinkler set could cover more than one salinity.
hectare by shifting from one place to another. * One of the emphasis of 12th Five Year Plan is to
encourage public private partnership in agriculture so
Drip Irrigation as to bridge the gap in dryland areas and rapidly
It is also known as trickle irrigation or micro irrigation, it diversity agriculture.
is an irrigation method that saves water and fertilizer by * The main purpose of the Rainbow Revolution was to
allowing water to drip slowly to the roots of plants, either achieve the growth rate of over 4% per annum.
on to the soil surface or directly on to the root zone,
through a network of valves, pipes, tubing and emitters. It
is done through emitters fitted on a network of pipes
Food Security
(mains, sub-mains and laterals). The emitting devices — The World Food Summit of 1996 defined food security as
could be drippers, micro sprinklers, mini sprinklers, micro existing when “All people, at all times, have physical, social
jets, misters, fan jets, micro sprayers and forgers. and economic access to sufficient, safe and nutritious food to
meet their dietary needs and food preferences for an active
and healthy life” according to FAO (Food and Agriculture
Rainfed Area Development Programme Organisation).
— Given the importance of rainfed agriculture in India, — India’s food security programme tries to tackle some of these
the Rainfed Area Development Programme (RADP) problems through various intervention. The main interventions
was launched by the government as a pilot scheme can be said to be the public distribution system and the
under the RKVY focusing on small and marginal National Food Security Act, 2013.
farmers and farming systems.
— It adopted a holistic ‘end–to–end approach’ covering Public Distribution System (PDS)
integrated farming, on farm water management, — Presently, PDS is operated under the joint responsibility of the
storage marketing and value addition of farm produce
Central and the State Governments. The Central Government,
in order to enhance farmers’ income in rainfed areas.
through FCI, has assumed the responsibility for procurement,
National Rainfed Area Authority (NRAA) storage, transportation and bulk allocation of foodgrains to the
State Governments.
— The government has set-up National Rainfed Area
Authority (NRAA), an expert body to provide the — The operational responsibility including allocation within states,
much needed knowledge inputs regarding systematic identification of families below the poverty line, issue of ration
upgradation and management of country’s dry land cards and supervision of the functioning of fair price shops,
and rainfed agriculture. An order for setting up the rest with the State Governments.
authority was issued on 3rd November, 2006. — Under the PDS, presently, the commodities namely wheat, rice,
— The NRAA has a two tier structure. The 1st tier is the sugar and kerosene are being allocated to the states/UTs for
governing board that provides necessary leadership distribution. Some states/UTs also distribute additional/items of
and appropriate coordination in implementation of mass consumption oils, iodised salt, spices etc.
programmes. — As of date, there are about 4.99 lakh Fair Price Shops (FPS)
— The 2nd tier is the Executive Committee consisting of across India.
technical experts and representatives from — The Targeted Public Distribution System (TPDS) was
stakeholder ministries. introduced with effect from June 1997.
90 Magbook ~ Indian Economy

Revamped Public Distribution Price Fixation


System (RPDS) — Another method of intervention in the market mechanism has
been the announcement of different administered prices viz
— The Revamped Public Distribution System (RPDS)
minimum support prices statutory minimum prices
was launched in June, 1992, with a view to
procurement prices issue prices.
strengthen and streamline the PDS as well as to
improve its reach in the far-flung, hilly, remote and — These prices are announced for different agricultural crops by
inaccessible areas, where a substantial section of the the Government of India on the recommendations of
poor live. Commission for Agricultural Costs and Prices (CACP).
— Foodgrains for distribution in RPDS areas were issued — Minimum Supports Prices (MSP) These are in the nature of a
to the states at 50 paise below the Central Issue guarantee to the producers in that prices paid to the farmers
Price. The scale of issue was upto 20 kg per card. cannot be lower than the MSP.
— The RPDS included area approach for ensuring — Procurement Prices These are higher than the MSP and are
effective reach of the PDS commodities, their delivery the prices at which government buys from farmers. In recent
by State Governments at the doorstep of FPSs in the years, government has been announcing endless procurement
identified areas, additional ration cards to the left out so, that farmers have been selling to the government at
families, infrastructure requirements like additional procurement prices.
Fair Price Shops, storage capacity etc and additional — Central Issues Prices (CIP) It indicate the prices at which
commodities such as tea, salt, pulses, soap etc for government supplies produce to the fair price shops and ration
distribution through PDS outlets. depots. Wheat and rice are issued to the State
Governments/UTs at CIP for distribution through the PDS.
Targeted Public Distribution States may choose to provide additional subsidy to the
beneficiaries by reducing prices below CIP.
System (TPDS) — Price support through MSP and procurement prices is
— The TPDS as it operated earlier had been widely extended only for specific crops. This has led to a change in
criticised for its failure to serve the population below cropping pattern in the country towards certain specific crops
the poverty line. Therefore, on the basis of the such as rice and wheat whose MSP has increased a lot. It has
recommendations of the Chief Ministers Conference also benefitted farmers in those states where such crops are
held in July, 1996, an effort was made to streamline produced in a larger number.
the PDS, through the introduction of the Targeted
Public Distribution System (TPDS) in June, 1997. Decentralised Procurement Scheme
— This system follows a 2 tier subsidised pricing — In view of a this, a decentralised procurement scheme was
structure for families, Below Poverty Line (BPL) and started in 1997, under which State Governments themselves
for those Above Poverty Line (APL). procure and distribute foodgrains. The difference between the
— The identification of poor under the scheme is done economic cost fixed for the State and the Central Issue Price
by the states as per the state-wise poverty estimates (CIP) is passed on to the states as subsidy.
of Planning Commission. — The objectives of this scheme are to cover more farmers under
— In order to make the TPDS more focused and MSP operations, improving efficiency of PDS, providing
targeted towards the poor, the Antyodaya Anna foodgrains suited to local tastes and reducing transportation
Yojana was launched in December 2000. costs.
The scheme contemplates identification of 10 million
—
poor families and providing them with 25 kg of
National Food Security Act 2013,
foodgrains per family per month at a low price of ` 2 (NFSA)
per kg for wheat and ` 3 per kg for rice. — NFSA is the biggest intervention of its kind in the world in the
realm of food security. If implemented property this law can
Agricultural Prices and improve the lives of millions in this country.
Procurement — Some of the highlights of this act are as follows:
— The Government of India undertakes an agricultural — It extends to the whole of India.
pricing policy and procurement programme to provide — Priority households are entitled to 5 kgs of food grains per person
reasonable returns to the farmers and instil certainty per month and Antyodaya households to 35 kgs per household per
month.
and confidence in them.
— Combined coverage of priority and Antyodaya households will
— The procurement programme is also essential to the extend to 75% of the rural population and 50% of the urban
functioning of the Public Distribution System (PDS). population.
Magbook ~ Agriculture 91

— PDS issue prices will be 3/2/11 per kg for rice/wheat/millets. These — The Indian food processing industry stands at
may be revised after 3 years. $135 billion and is estimated to grow with a CAGR of
— For children in the age group 6 months to 6 years, an 10% to reach $ 200 billion by 2015.
age-appropriate meal will be provided through the local
— The industry is segmented into sectors namely, milk
Anganwadi.
and allied products (dairy), meat and poultry, seafood,
— For children aged 6-14 years, one free mid-day meal in all
bakery and confectionery, fruit and vegetables, grain,
government and government aided schools up to class VII.
pulses and oilseeds (staple) products, alcoholic and
— For children below 6 months ‘exclusive breast feeding will be
promoted’. non-alcoholic products (beverages) and packaged
foods. The classification is not distinct as many
— Every pregnant and lactating mother is entitled to a free meal at
the local Anganwadi (during pregnancy and 6 months after) and processed products overlap different segments.
maternity benefits of ` 6000 t o b e p a i d i n i n s t a l l m e n t s .
— The act does not specify criteria for identification of eligible
Food Corporation of India (FCI)
households. Central Government will determine state-wise coverage FCI was set-up in 1965 with the primary duty to undertake
and states will then identify the beneficiaries. the purchase, storage, movement, transport, distribution and
— State food commissions will be created to monitor implementation sale of foodgrains and other foodstuffs.
of the act.
It has also been entrusted with maintaining buffer stocks of
— Grievance Redressal System consists of the district. foodgrains on behalf of the government. It is the sole
Grievance Redressal officer and the State Food Commission.
repository of foodgrains meant for the PDS.
— Transparency provisions include placing PDS records in the public
domain, conducting periodic social audits, use of information and
communication technology and setting up of vigilance committees.
— The act also states that the Central and State Governments will
Land Reforms
endeavour to undertake PDS reforms. — With the twin objectives of achieving social equity and
ensuring economic growth the land reforms
International Fund for programme was built around three major issues as
Agricultural Development follows:
(i) Abolition of intermediaries.
IFAD is an International Financial Institution and a
specialised UN agency based in Rome – the UN’s food and (ii) Settlement and regulation of tenancy.
agricultural hub. It is a unique partnership of 165 members (iii) Regulation of size of holdings.
from the Organisation of the Petroleum Exporting Countries — After independence, the government has undertaken
(OPEC), other developing countries and the Organisation for many land reform measures e.g.
Economic Cooperation and Development (OECD). — Zamindari System has been abolished. The actual
FAD provides a strong global platform for discussing rural cultivator has been given either the ownership right or
policy issues – and for increasing awareness about why the right of occupancy tenant.
investment in agriculture and rural development is critical to — Tenancy System has been reformed by enacting various
reducing poverty and improving food security. legislative measures in different states.
— Ceiling on landholdings has been fixed.
— By 2004, about 1633 lakh hectare of holdings have been
Storage Capacity and Constraints consolidated.
— There are three agencies in the public sector, which are as — Co-operative Farming has also been developed.
follow engaged in building large scale storage/warehousing — In order to improve the conditions of landless farmers,
Acharya Vinoba Bhave launched Bhoodan Movement in the
capacity namely; Food Corporation of India (FCI), Central
country.
Warehousing Corporation (CWC) and 17 States Warehousing
Corporations (SWCs).
— While the capacity available with FCI is used mainly for storage
National Land Records
of foodgrains that with CWC and SWCs is used for storage of Modernisation Programme
foodgrains as well as certain other items. (NLRMP)
— The Government of India decided to implement the
Food Processing Industry centrally-sponsored scheme in the shape of the
— India is the 3rd largest producer of food in the world, after National Land Records Modernisation Programme
China and the US. (NLRMP) by merging two existing centrally-Sponsored
Schemes of Computerisation of Land Records (CLR)
— Food processing industry is the 5th largest industry in India, in
and Strengthening of Revenue Administration and
terms of production, consumption, exports and expected
Updating of Land Records (SRA and ULR).
growth.
92 Magbook ~ Indian Economy

— The integrated programme seeks to achieve the following - (iii) Mix of federal and unitary types.
modernise management of land records, minimise scope (iv) No separate banks exist and long-term credit is provided
of land/property disputes enhance transparency in the by the long-term section of State Co-operative Banks
land records maintenance system and facilitate moving (SCBs) co-operatives accounted for 17.2% of
eventually, towards guaranteed conclusive titles to institutional agricultural credit in 2011-12.
immovable properties in the country.
— A single window to handle land records. Commercial Banks and Rural
— The mirror principle, which refers to the fact that cadastral Credit
records mirror the ground reality. Share of commercial banks in rural credit was meagre just after
— The curtain principle, which indicates that the record of title is independence.
a true depiction of the ownership status.
— Title insurance, which guarantees the title for its correctness
Regional Rural Banks (RRBs)
and identifies the title holder against loss arising on account — RRBs were set-up to supplement the efforts of
of any defect therein. co-operatives and commercial banks.
— In 1976, the Parliament enacted the Regional Rural Banks
Agricultural Finance and Act, 1976 to provide for the incorporation, regulation and
winding up of Regional Rural Banks. The Act has been
Credit Facilities made effective from the 26th September, 1975.
— Agricultural credit is disbursed through a multi-agency — The equity of the RRBs is contributed by the Central
network comprising of Commercial Banks (CBs), Regional Government, concerned State Government and the
Rural Banks (RRBs) and co-operatives. With their vast sponsor bank in the proportion of 50:15:35.
network (covering almost all villages in the country), wide — Besides the Reserve Bank, which is the regulatory
coverage and outreach extending to the remotest parts of authority for the RRBs in accordance with the provisions
the country, the Co-operative Credit Institutions, both in of the Banking Regulations Act, 1949, the Banking
short and long-term structure, are the main institutional Regulations Act empowers NABARD (National Bank for
agencies for the dispensation of agricultural credit. Agriculture and Rural Development) to undertake the
— After nationalisation, Commercial Banks have also started inspection of RRBs.
giving loans for farming operations. Regional Rural Banks — Area of RRBs is limited to a specified region comprising one
and farmer service societies also strengthen the rural or more districts of a state. They grant direct loans and
credit programmes. advances only to small and marginal farmers, rural artisans,
— National Bank for Agriculture and Rural Development agricultural labourers and others of small means for
(NABARD) has been established as an apex agricultural productive purposes. Lending rates of RRBs cannot be
finance institution. higher than those of co-operative societies in any particular
state.
Co-operative Credit Societies
RRB (Amendment) Act, 2015
— Rural co-operative credit institutions in India have been
organised into short-term and long-term structures. — The Regional Rural Banks (Amendment) Act seeks to
— The short-term co-operative credit structure consists of amend the existing Act so as to increase the authorised
three-tiers–Primary Agricultural Credit Societies (PACS) at capital of each Regional Rural Bank (RRB) from ` 5 crore
the village level, District Central Co-operative Banks to ` 2000 crore divided into ` 200 crore of fully paid
(DCCB) at the district level and State Co-operative Banks shares of ` 10 each.
(SCB) organised at the state level. — The bill also provides that the authorised capital of any
— For long-term credit requirements of the farmers Regional Rural Bank shall not be reduced below ` 1crore
long-term credit co-operatives have been set-up. These and shares in all cases to be fully paid up shares of ` 10
are organised at two levels and differ from state to state. each. It also provides that the issued capital of each rural
Generally they are of four types which are as follows: bank shall not be less than ` 1crore.
(i) Unitary structure in which State Co-operative — At present, there are 56 RRBs and they are doing well.
Agricultural and Rural Development Banks (SCARDBs) The amendment to raise the authorised capital of the
operate at state level through their branches and have RRBs from ` 5 crore to ` 2000 crore, will strengthen these
direct membership of individuals. institutions and further deepen financial inclusion.
(ii) Federal structure in which Primary Co-operative Agricultural Debt Waiver and Debt Relief Scheme,
Agricultural and Rural Development Banks (PCARDBs) 2008
operate as independent units at the primary level and — The Finance Minister, in his budget speech for 2008-2009,
federate themselves into SCARDS at the state level. announced a Debt Waiver and Debt Relief Scheme for
Magbook ~ Agriculture 93
farmers. The scheme covered direct agricultural loans TRIFED
extended to ‘marginal and small farmers’ and ‘other farmers’ — Tribal Co-operative Marketing Development Federation of
by Scheduled Commercial Banks, Regional Rural Banks,
India Limited (TRIFED) came into existence in 1987 and
Co-operative Credit Institutions (including Urban
got registered under the Multi-State Co-operative
Co-operative Banks) and Local Area Banks.
Societies Act, 1984. Now, the Multi -State Co-operative
— In the case of a small or marginal farmer, the entire eligible Societies Act, 2002.
amount was waived. In the case of ‘other farmers’, there
was a One Time Settlement (OTS) Scheme, under which Recommendations of Task Force on
the farmer was given a rebate of 25% of the ‘eligible Credit Related Issues of Farmers
amount’ subject to the condition that the farmer pays the (Chairman Umesh Chandra Sarangi)
balance of 75% of the ‘eligible amount’. — The task force looked into the issue of large number of
— A National Level Monitoring Committee to monitor the farmers, who had taken loans from private
implementation of the scheme was also constituted. moneylenders, not being covered under the loan waiver
Kisan Credit Card (KCC) Scheme scheme.
— Financial literacy and counselling campaigns be
— Kisan Credit Cards were started by the Government of
undertaken to increase awareness among farmers on the
India, Reserve Bank of India (RBI) and National Bank for
KCC.
Agricultural and Rural Development (NABARD) in August
1998, to help the farmers access timely and adequate — Banks be encouraged to educate their rural branch staff
credit. Since 1998, about 10.78 crore KCCs had been about the KCC.
issued up to October 2011. — Banks use farmers co-operatives and SHG federations as
— The scheme includes reasonable components of banking correspondents to increase out reach.
consumption credit and investment credit within the overall — The KCC be technology-enabled, including the
credit limit sanctioned to the borrowers, to provide conversion to a smart card with withdrawals and
adequate and timely credit support to the farmers for their remittances enabled at Automated Teller Machines
cultivation needs. Budget 2012-13, has expanded the (ATMs), Points of Sale (PoS) and through hand held
scope of KCCs as now they can be used as smart cards machines; banks need to have Core Banking Solutions
and ATMs. The card is valid for 3 years and subject to (CBSs) in place at the earliest, to enable technology to
annual renewals. benefit the farmer.
— The KCC limit be fixed for five years, based on the
NABARD : An Overview banker’s assessment of total credit needs of the farmer
— NABARD was set-up by the Government of India as a for a full year and that the limit be operated by the
development bank with the mandate of facilitating credit borrower as and when needed, with no sublimits for
flow for promotion and development of agriculture and kharif and rabi or for different stages of cultivation.
integrated rural development. — There should be automatic renewal and annual increase
— The mandate also covers supporting all other allied in credit limit linked to inflation rate.
economic activities in rural areas, promoting sustainable
rural development and ushering in prosperity in the rural Agricultural Marketing and
areas.
— It is an apex institution handling matters concerning policy,
Extension Services
planning and operations in the field of credit for agriculture — Organised marketing of agricultural commodities is being
and for other economic and developmental activities in promoted in the country through a network of regulated
rural areas. markets. Most of the states and Union Territories have
— Essentially, it is a refinancing agency for financial enacted legislations (the Agriculture Produce Marketing
institutions offering production and investment credits for Committee [APMC] Act) to provide for regulation of
promoting agricultural and developmental activities in rural agricultural produce markets.
areas. — Seventeen states or UTs have amended their APMC Act
and the remaining are in the process of doing so.
NAFED
— As per the Economic Survey 2014-15, India has 2,477
— National Agricultural Co-operative Marketing Federation of principal regulated primary agricultural markets in the
India Limited (NAFED) is the Apex Co-operative country. These markets are governed by APMC Acts and
Organisation at the national level. administered by a separate Agricultural Produce
— It deals in procurement distribution export and import of Marketing Committee (APMC) which has its own
selected agricultural commodities. It was established in marketing regulations. This set up hinders the free flow
1958. of agricultural commodities from one market to another.
94 Magbook ~ Indian Economy

National Agriculture Market Modified NAIS (MNAIS)


(NAM) — With the aim of further improving crop insurance schemes, the
— On 14th April, 2016, Prime Minister launched MNAIS is under implementation on pilot basis in 50 districts in the
National Agriculture Market (NAM) as a pan-India country from rabi 2010-11 season.
electronic trading portal for farm produce which Agriculture Insurance Company of India Limited
creates a unified national market for agricultural
— AICIL was incorporated under the Companies Act, 1956 in 2002 as
commodities by integrating the existing Agriculture
a specialised participation from General Insurance Company four
Produce Market Committee (APMC) markets.
public sector GICs and NABARD.
— This portal provides a single window service for
all APMC related services and information, such Pilot Weather Based Crop Insurance Scheme
as commodity arrivals and prices, provision for (PWBCIS)
responding to the trade offers, buy and sell trade
— Similarly, the WBCIS is also being implemented as a central-sector
offers, among other services.
scheme from kharif 2007 season. The scheme is intended to
provide insurance protection to farmers against adverse weather
National Agricultural Market incidence such as deficit and excess rainfall, high or low
Yojana temperature and humidity that are deemed to adversely impact
— The purpose behind NAM is the creation of a crop production.
common national market for agricultural — The WBCIS is based on actuarial rates of premium, but to make the
commodities through an e-platform network. scheme attractive, premium actually charged from farmers has
— As a part of National Agricultural Market (NAM), been restricted to be on a par with the NAIS from kharif 2007-08 to
Prime Minister Narendra Modi launched the kharif 2010-11.
national e-agriculture market on 21st April, 2016. * Agriculture, including allied activities, accounted for only
These e-mandis (markets) will integrate the 17.4% of the GDP in 2014-15 (at constant price 2011-12). Its
various vegetable markets across country. role in the country’s economy is much bigger with its share in
total employment as high as 48.9% according to the NSSO
— The Kisan Call Centre Scheme was launched in
2011-12.
2004, to provide agricultural information to the * The National Land Records Modernisation Programme
farming community through toll-free telephone (NLRMP) which started in 2008 aims at updating and
lines. A country-wide common 11 digit number digitising land records by the end of the 12th Plan.
—1800-180-1551has been allocated for KCCs. * Besides an increase in the Minimum Support Price (MSP) for
— The Agri-clinic and Agri-business Centres wheat and rice, inadequate open market availability relative
Scheme was launched in 2002, to provide to demand, particularly for wheat, has also resulted in a build
extension services to farmers on payment basis -up of price pressure and hardening of inflation for cereals.
through setting up of economically viable self
employment ventures. NABARD monitors the Commodity Futures Market
credit support to Agri-clinics through Commercial — The commodity futures market facilitates the price discovery
banks. process and provides a platform for price risk management in
commodities. Currently, 113 commodities are notified for futures
Agriculture Insurance trading of which 51 are actively traded in 5 national and 16 regional
commodity specific exchanges.
— There are various major crop insurance schemes
Forward Markets Commission
under implementation in the country:
— The Forward Markets Commission (FMC) is the regulator for
National Agricultural Insurance commodity futures trading, which it regulates under the provisions
Scheme (NAIS) of the Forward Contracts (Regulation) Act, 1952. Its chief
responsibilities are as follows:
— The NAIS is a government sponsored central — To ensure the participation of physical market stakeholders, especially
sector crop insurance scheme being implemented farmers, as hedgers in the commodity futures market by increasing the
in the country since 1999-2000, season with the level of awareness of physical market participants and policy makers
objective of providing financial support to farmers about the economic role of this market.
in the event of failure of crops as a result of — It also ensures the dissemination of spot and future prices of agriculture
natural calamities, pests and diseases. The commodities at Agricultural Produce Market Committees (APMCs).
Agriculture Insurance Company of India Limited is — It also grants recognition to the new commodity exchanges under the
the implementing agency for the scheme. Forward Contracts (Regulation) Act, 1952.
Magbook ~ Agriculture 95

expenditure of ` 33,269 crore. Schemes which are part of


Major Schemes of Green Revolution-Krishonnati Yojana are
Agriculture — Mission for Intergrated Development of Horticulture (MIDH)
— National Food Security Mission (NFSM)
— There are various major scheme related to agriculture, — National Mission for Sustainable Agriculture (NMSA)
which is given below: — Submission on Agriculture Extension (SMAE)
PM Kisan Samman Nidhi Yojana — Submission on Seeds and Planting Material (SMSP)
— Submission on Agricultural Mechanisation (SMAM)
— The Government with a view to augment the income of — Submission on Plant Protection and Plan Quarantine (SMPPQ)
the farm families is implementing a Central Sector — Integrated Scheme on Agriculture Census, Economics and
Scheme, namely Pradhan Mantri Kisan Samman Nidhi Statistics (ISACES)
Yojana. PM Kisan Nidhi Yojana is for small and marginal — Integrated Scheme on Agriculture Cooperation (ISAC)
farmers for helping in financial condition for purchase of — Integrated Scheme on Agriculture Marketing (ISAM)
seed, fertilizers etc. — National e-Governance Plan (NeGP-A)
— This scheme was started on 24 February, 2019 from Aajeevika Grameen Express Yojana
Gorakhpur (UP) under total of ` 6000 amount is to be
farmer in 3 equal installments in every four months. The — The Government of India has launched Aajeevika Grameen
scheme is expected to cover around 14.5 crore farmers Express Yojana (AGEY) in August 2017, to provide an
for year 2019-20. Under the scheme, financial benefit alternative source of livelihood to members of Self Help
has been provided to all small and marginal landlord Groups (SHGs). This scheme has been launched under
farmers with total cultivable holding upto 2 hectares. Deen Dayal Antyodaya Yojana - National Rural Livelihood
Mission (DAY-NRLM). The main objective of AGEY is to
KUSUM Scheme provide an alternative source of livelihood to members of
— The Union Government has announced ` 1.4 lakh crore SHGs under DAY-NRLM by facilitating them to operate
Kisan Urja Suraksha evam Utthaan Mahaabhiyan public transport services in backward rural areas.
(KUSUM) scheme for promoting solar farming i.e. — This will provide safe, affordable and community- monitored
decentralised solar power production of upto 28,250 MW rural transport services like e-rickshaws, three and four
to help farmers. The Union Budget 2018-19 has allocated wheeler motorised transport vehicles to connect remote
` 48,000 crore for the scheme for the ten-year period. villages with key services and amenities including access
— Government will provide 60% subsidy on solar pumps to to markets, education and health for the overall economic
farmers. It will be shared between Centre and States development of the area. The sub-scheme will be
while 30% will be provided through bank loans. The implemented in 250 blocks in the country on a pilot basis
balance cost will be borne by farmers. for a period of three years from 2017-18 to 2019-20.

Gobardhan Yojana Pradhan Mantri Fasal Bima Yojana


— Union Minister for Drinking Water and Sanitation, Uma — This is the new crop damage insurance scheme that has
Bharti, launched the Gobardhan scheme on 30th April, been approved by the Union Cabinet in January 2016. It
2018 from Haryana’s Karnal district with an aim to will replace the existing 2 insurance schemes NAIS and
positively contribute to cleanliness and generate energy Modified NAIS.
from solid waste. — The theme of the Scheme is One Nation–One Scheme. In
— The government aims to cover nearly 700 districts under this, all shortcomings and weaknesses of all previous
the scheme in this financial year (2018-19). The scheme schemes were removed and incorporated with the best
focuses on managing and converting cattle dung and features of all schemes.
solid waste from farms and fields to useful compost, Highlights of the Scheme
biogas and bio-CNG. It will also help in keeping villages — There will be a uniform premium of only 2% to be paid by
clean and generate energy. It will increase income of farmers for all Kharif crops and 1.5% for all Rabi crops.
farmers and cattle herders. In case of annual commercial and horticultural crops, the
Green Revolution-Krishonnati Yojana premium to be paid by farmers will be only 5%.
— The premium rates to be paid by farmers are very low and
— Cabinet Committee on Economic Affairs (CCEA) has
balance premium will be paid by the government to
approved continuation of Umbrella Scheme, Green
provide full insured amount to the farmers against crop
Revolution—Krishonnati Yojana in agriculture sector
loss on account of natural calamities.
beyond 12th Five Year Plan from 2017-18 to 2019-20.
The Umbrella scheme comprises of 11 schemes and
— There is no upper limit on government subsidy. Even if
missions. These schemes will continue for three financial balance premium is 90%, it will be borne by the
years, i.e., 2017-18, 2018-19 and 2019-20 with an government.
96 Magbook ~ Indian Economy

— Earlier, there was a provision of capping the premium National Mission on Food
rate which resulted in low claims being paid to farmers.
This capping was done to limit government outgo on Processing (NMFP)
the premium subsidy. This capping has now been — NMFP has been launched under the 12th Plan for a
removed and farmers will get claim against full sum decentralised implementation of various schemes under the
insured without any reduction. ministry of food processing with the help of State
— The use of technology will be encouraged to a great Governments.
extent. Smart phones will be used to capture and — It consists of the following main schemes technology up
upload data of crop cutting to reduce the delays in gradation of food processing industries, cold chain facilities
claim payment to farmers. Remote sensing will be used for non-horticultural produces, modernisation of abattoirs,
to reduce the number of crop cutting experiments. primary processing centres/collection centres in rural areas,
upgradation of quality of street food etc.
‘Soil Health Card’ Scheme
— Narendra Modi, Prime Minister of India launched the National Food Processing
Union government’s nationwide ‘Soil Health Card’ Development Council (NFPDC)
Scheme (SHCS) from Suratgarh, Rajasthan on February
NFPDC has been set-up to provide guidance to all schemes of the
19, 2015 where he described agriculture as the key to
ministry of food processing including NMFP. It will comprise the
poverty eradication.
Agriculture Minister as Chairman, representatives of State
— The SHCS is an attempt to check deteriorating soil Government, industry associations and related government officials.
quality and spruce up farm productivity and will provide
all 145 million farm owners in the country with a SHC
in the next three years. In this way, the Ministry has National Food Security Mission
been sanctioned ` 568 crore for the next three years. (NFSM)
— This ambitious scheme was announced when finance — It was launched in rabi 2007-08. The mission aims to
minister Arun Jaitley presented it in the first budget in
increase production through area expansion and productivity;
July, 2014. The budget allotted ` 100 crore for issuing
create employment opportunities and enhance the farm level
cards and an additional ` 56 crore to set-up 100
economy to restore confidence of farmers. The NFSM is
mobile soil testing laboratories across the country.
being implemented in 476 Districts of 17 States.
Mega Food Park Scheme (MFPS) — To intensify the pulses production programme, since
2010-11, two additional programmes have been adopted
— The 10th Plan scheme of Food Parks was renamed as
under NFSM. These are as follows:
the Mega Food Park Scheme (MFPS) in 2008. The
— Merging of the pulse component of the Integrated Scheme of
scheme has been launched with the objective of
Oilseeds, Pulses, Oil Palm and Maize (ISOPOM) with the NFSM.
implementing the express objectives of the Vision 2015, Jharkhand and Assam have also been included under the
document through creation of excellent infrastructure. programme.
— At present, 13 mega food parks are at various stages of — Accelerated Pulse Production Programme (APPP) was initiated to
implementation. boost the production of pulses by active promotion of
— By February, 2013, the government had approved a technologies in 1000 clusters of 100 hectare each.
total of 30 mega food parks. — The National Food Security Mission (NFSM) during the 12th
— 11th Plan (2007-12), aimed at setting up 30 mega food Five Year plan will have 5 components
parks. In line with this, Budget 2011-12, announced (i) NFSM-Rice (ii) NFSM-Wheat (iii) NFSM-Pules
setting up another 15 mega food parks in addition to (iv) NFSM-Coarse cereals and (v) NFSM-Commercial crops.
the already existing. 12th Plan (2012-17), has targeted — The mission is being continued during 12th Five Year Plan
to set-up 50 mega food parks during the plan period. with new targets of additional production of food grains of 25
— Objectives of the Mega Food Park Scheme are as million tons of food grains comprising of 10 million tons rice,
follows: 8 million tons of wheat and 4 million tons of pulses and 3
— To provide state of the art infrastructure for food processing million tons of corse cereal by the end of 12th Five Year Plan
in the country on a pre-identified cluster basis. (2012-17).
— To ensure value addition of agricultural commodities. National Horticulture Mission (NHM)
— To establish a sustainable raw material supply chain for
each cluster. To facilitate induction of latest technology.
— It was launched in 2005-06 for the holistic development of
the horticulture sector through ensuring forward and
— Quality assurance through better process control and
capacity building. backward linkages and with the active participation of all the
stakeholders.
Magbook ~ Agriculture 97

— 18 States and the 3 Union territories of Andaman and National Mission for Sustainable
Nicobar Islands Lakshadweep and Puducherry are covered
under the mission and the mission covers 372 districts. Agriculture (NMSA)
— NMSA is one of the 8 plans, under the National Action
National Bamboo Mission (NBM) Plan on Climate Change (NAPCC) and will be
— The NBM is a Centrally Sponsored Scheme which was implemented during the 12th Five Year Plan. It seeks to
launched in 2006 -07 for harnessing the potential of the transform agriculture into an ecologically sustainable
bamboo crop in the country. At present, it is being climate resilient production system, while at the same
implemented in 27 States with a total outlay of ` 568.23 time, exploiting its fullest potential and thereby ensuring
crore. ` 1200 crore have been allotted for this mission in food security, equitable access to food resources,
the budget of 2018-19. enhancing livelihood opportunities and contributing to
economic stability at the national level.
Horticulture Mission for North-East and
Objectives of NMSA
Himalayan States (HMNEH)
— The objectives of NMSA are as follows
— The technology mission for North-Eastern states including
— To devise strategic plans at the agro-climatic zone level.
Sikkim, aimed at the holistic development of all the
— To enhance agricultural productivity through customised
horticulture crops, has now been renamed as HMNEH.
interventions such as use of biotechnology.
— The main objective of the mission is to set-up nurseries for — To facilitate access to information and institutional support by
production and distribution of quality planting materials. expanding automatic weather station networks to the
Panchayat level and linking them to existing insurance
Rashtriya Krishi Vikas Yojana (RKVY) mechanisms.
— The RKVY was launched in 2007-08, with an outlay of — To promote ‘laboratory to land’ research by creating model
` 25000 crore in the 11th Plan for incentivising states to villages and model farm units in rainfed and dryland areas.
enhance public investment to achieve a 9% growth rate in — To strategise long-term interventions for emission reduction
from energy and non-energy uses by way of introduction of
agriculture and allied sectors.
suitable crop varieties and farm practices, livestock and
— The RKVY permits taking up national priorities as manure management.
sub-schemes allowing flexibility in project selection and — To realise the enormous potential of growth in dryland farming.
implementation.
— The sub-scheme under RKVY are as follows: Organic Farming
— Bringing Green Revolution to Eastern region - Uttar Pradesh, — It is a form of agriculture that relies on techniques such
Jharkhand, Bihar, West Bengal, Assam, Odisha and as crop rotation, green manure compost and biological
Chhattisgarh introduced in 2010-11. pest control.
— Integrated development of 60000 pulses and oilseeds villages — The main objectives of National Project on Organic
in rainfed areas introduced in 2010-11. Farming include:
— Promotion of Oil Palm. — Capacity building through service providers.
— Financial and technical support for setting up of organic
— Initiative on village clusters.
input production unit such as fruits and vegetable market,
— Nutri-cereals. waste compost, bio-fertilizers and bio-pesticides and
— National Mission for protein supplements. vermiculture hatcheries.
— Human resource development through training and
— Accelerated Fodder Development Programme.
demonstrations.
— Rainfed Area Development Programme and — Awareness creation and market development.
— National Mission on saffron-economic revival of Jammu and — Quality control of organic inputs.
Kashmir saffron introduced in 2010-11. — Biological assessment of soil health.

Precision Farming Indian Council of Agricultural


Also known as satellite farming, it uses satellite technology, Research (ICAR)
information technology and GIS Systems to improve Crop — ICAR is an autonomous organisation, under the
management. It is based on observing and responding to Department of Agricultural Research and Education,
intra-fields variations. It helps in matching farming practices with Ministry of Agriculture, Government of India. The council
crop needs, reducing ecological footprint and boosting is the apex body for coordinating, guiding and managing
competitiveness through more efficient practices like improved research and education in agriculture including
management of fertilizer usage etc. horticulture, fisheries and animal sciences in the entire
country. It was established in 16th July, 1929.
98 Magbook ~ Indian Economy

The Poultry Development Scheme comprising three


Animal Husbandry, —
components, namely Assistance to State Poultry Farms,
Dairying and Fisheries Rural Backyard Poultry Development and Poultry Estates
is being implemented.
— The 11th Five Year Plan envisaged overall growth of
4.8% per annum, for the sector. In 2011-12, this sector
contributed 127 million tonnes of milk, 66.45 billion eggs,
Fisheries
44.73 million kg wool and 5.51 million tonnes of meat. The — Fish is an important source of and also an important
18th Livestock Census (2007) has placed total livestock source of livelihood, production of fish, both marines
population at 529.7 million and total of poultry birds at and inland , has gone up from 5.6 million tonnes in
648.8 million. 2000-01 to 10.8 million tonnes in 2014-15, an increase
of 6.4% over 2013-14. The exports of marine products
Dairy Sector have increased significantly in the year 2011-12.
— India ranks first in the world in milk production. The per India’s Position in World Agriculture Production
capita availability of milk has also increased from 176 g per
day in 1990-91 to 322 g in 2014-15. Produce Global Rank
— The Ministry of Agriculture is implementing important Wheat 2nd
schemes, namely the Intensive Dairy Development
Programme, Strengthening infrastructure for quality and Rice/Paddy 2nd
clean milk production and Assistance to Cooperative and Total Pulses 1st
Dairy Entrepreneurship Development Scheme, in the dairy
Groundnut (in shell) 2nd
sector. A major programme for genetic improvement called
the National Project for Cattle and Buffalo Breeding Vegetables (with melons) 2nd
(NPCBB) was also launched in 2000. Fruits (excluding melons) 2nd
National Dairy Plan Sugarcane 2nd
— It is a strategic plan prepared by NDDB for achievement of Tea 3rd
180 million tonnes of milk production by 2021-22. Under Jute and Jute like fibres 1st
the plan, districts are separated into 324 high potential
districts for intensive development and 282 low potential Cotton (lint) 2nd
districts for further expansion. The plan envisages breed Total Milk 1st
improvement through artificial Insemination and through
Total Eggs 3rd
natural service, setting up plants to augment cattle feed,
by-pass protein and mineral mixture and expanding or Total Meat 5th
strengthening milk processing infrastructure.

Livestock Insurance MOBILE APPS FOR FARMERS


— A centrally sponsored scheme for livestock insurance is — Union Agriculture and Farmers Wealth Minister, Shri
being implemented in all the states with the twin objectives Radha Mohan Singh launched two mobile apps for the
of providing protection mechanism to farmers and cattle farmers on 23rd December, 2015.
bearers against any eventual loss of their animals due to
death and to demonstrate the benefit of the insurance of Mobile app ‘Crop Insurance’
livestock and popularise it with the ultimate goal of attaining — It will help the farmers not only to find out complete
qualitative improvement in livestock and its products. details about insurance cover available in their area, but
— The scheme benefits farmers and cattle bearer with also to calculate the insurance premium for notified
indigenous/cross-breed milch cattle and buffaloes in 300 crops, coverage amount and loan amount in case of a
selected districts. The benefit of subsidy is to be restricted loaned farmer.
to two animals per beneficiary per household.
Mobile app ‘Agrimarket Mobile’
Poultry — This app automatically captures the location of person
— Four regional Central Poultry Development Organisations using mobile GPS and fetches the market prices of crops
located at Chandigarh, Bhubaneshwar, Mumbai and in those markets which fall within the range of 50 km.
Hessarghatta are focusing on production of stocks suitable There is another option to get price of any market and
for backyard rearing, training to the farmers to upgrade any crop in case person does not want to use GPS
their technical skills. location.
Self Check
Build Your Confidence
1. The Government of India fixes the Minimum Support 7. Consider the following statements
Prices after taking into account the recommendations of 1. Agriculture and allied sectors contribute more than 15%
which among the following bodies? of Gross Domestic Product of India.
(a) Ministry of Consumer Affairs, Food and Public Distribution 2. Share in total employment by agriculture as high as
(b) Cabinet Committee of Economic Affairs 48.9%.
(c) Planning Commission Which of the statement(s) given above is/are correct?
(d) Commission for Agricultural Costs and Prices (a) Only 1 (b) Only 2
2. Consider the following statements. (c) Both 1 and 2 (d) Neither 1 nor 2
1. The concept Evergreen Revolution was given by 8. Consider the following statements.
renowned agricultural scientist Norman Borlaug. 1. The accelerated Irrigation Benefits Programme was
2. Evergreen Revolution emphasises on organic launched during 1996-97 to provide loan assistance to
agriculture and green agriculture with the help of poor farmers.
integrated post management, integrated nutrient supply 2. The Command Area Development Programmes was
and integrated natural resource management. launched in 1974-75 for the development of water use
Which of the statement(s) given above is/are correct? efficiency.
(a) Only 1 Which of the statement (s) given above is /are correct?
(b) Only 2 (a) Only 1 (b) Only 2
(c) Both 1 and 2 (c) Both 1 and 2 (d) Neither 1 nor 2
(d) Neither 1 nor 2
9. National Horticulture Mission was launched in which of
3. Government of India established TRIFED in August 1987, the following Five Year Plans?
with an objective to save tribals from exploitation by
(a) Ninth Five Year Plan (b) Tenth Five Year Plan
private traders. Which among the following is the
(c) Eleventh Five Year Plan (d) None of these
precise full form of TRIFED?
(a) Tribal Federation of India Limited
10. The National Food Security Mission (NFSM) aims to
(b) Tribal Marketing Development Federation of India Limited
enhance the production of
(c) Tribal Co-operative Marketing Development Federation of 1. Rice 2. Wheat
India Limited 3. Pulses 4. Coarse Cereals
(d) Tribal Co-operative Federation of India Limited 5. vegetables.
4. The importance of agriculture in Indian economy is Select the correct answer using the codes given below
indicated by its contribution to which of the following? (a) 1, 2 and 3 (b) 1, 2 and 5
(c) 2, 3 and 4 (d) 1, 2, 3 and 4
(a) National income and employment [UPPCS 2006]
(b) Industrial development and international trade 11. Bringing Green Revolution to Eastern India in a sub
(c) Supply of foodgrains scheme of
(d) All of the above (a) National Mission on Agriculture Extension and Technology
5. The Green Revolution in India has contributed to (b) National Mission for Sustainable Agriculture
(a) inter-regional inequality [WBPCS 2007]
(c) Rashtriya Krishi Vikas Yojana
(b) inter-class inequality
(d) It is not a sub-scheme.
(c) inter-crop inequality 12. Consider the following statements about the National
(d) All of the above Agricultural Insurance Scheme (NAIS).
6. The price at which the government purchases foodgrains 1. The Scheme has been implemented from Rabi 1999-2000
for maintaining the Public Distribution System and for season.
building up buffer Stocks are known as [IAS 2001] 2. The scheme is available non-loanee farmers only.
(a) Minimum Support Prices Which of the statement(s) given above is/are correct?
(b) Procurement Prices (a) Only 1 (b) Only 2
(c) Issue Prices (c) Both 1 and 2 (d) Neither 1 nor 2
(d) Ceiling Prices

1. (d) 2. (b) 3. (c) 4. (d) 5. (d) 6. (b) 7. (b) 8. (c) 9. (b) 10. (d)
11. (d) 12. (c)
Chapter twelve
Indian Industry
— Meeting High Income Demand Beyond
Industries in Indian certain limits, the demands of the people

Before independenc
Economy are usually for industrial products alone.
— Absorbing Surplus Labour (Employment
— Industry refers to an economic activity Generation) underdeveloped countries like
e, Industrial Policy concerned with the processing of raw materials India are characterised by surplus labour
of the Government and manufacture of goods in factories. and rapidly growing population. Industries
was characterised Industries are often classified based on their are expected to create employment
principle product e.g. steel industry, opportunities and facilitate economic
by Laissez-Faire i.e.
automobile industry, textile industry etc. development.
non-interference — The Indian Government has been trying to — Bringing Technological Progress Research
policy in the affairs promote rapid industrial growth since, and Development. It is associated with the
process of industrialisation, which further
of industries. independence. As a result of the various
leeds to development.
Industrial efforts, the industrial sector in India has grown
in multifarious dimensions.
development was Industrial Policy
left to the exclusive Role of Industry — The Department of Industrial Policy
care of private Promotion (DIPP) has been renamed as
— Rapid growth of national income is possible
sector. However, the Department for Promotion of Industry
only through industrialisation as growth in
and Internal Trade (DPIIT) under the
in the agriculture is limited by factors including
Ministry of Commerce and Industry. It is
natural factors. Industrial growth in 8th Five
post-independence responsible for the entire industrial policy.
Year Plan was 7.29%, in 9th Plan 4.29%, in
era, government 10th Plan (2002-07), 9.17% and in 11th Plan
— An industrial policy provides guidelines for
has been taking an the effective co-ordination of the activities
(approx) 10%. Some of the broad goals and
of various sectors of the economy. The
active interest in objectives of industrial development in India
evolution of industrial policy in India may
are as follows:
the development of be studied in the background to see, how
— Industries can provide better quality and more
industries in India. far it has worked as a potent tool to realise
employment than the agriculture sector. The
the goal of planned development.
So far, government share of industry in total employment increased
from 16.2% in 1999-2000 to 21.9% in 2009-10. — When India achieved independence in
has formulated five
— Value addition in the industrial sector can earn 1947, the national consensus was in
industrial policies more foreign exchange than simply exporting raw favour of rapid industrialisation of the
i.e. Industrial Policy materials. economy, which was seen not only as the
— The industrial sector provides goods for the key to economic development, but also to
1948, 1956, 1977, development of basic infrastructure of the country economic sovereignty.
1980 and 1991 like power, telecom etc which then provides the — In the subsequent years, India’s Industrial
respectively. basis for the growth of other sectors like
Policy evolved through successive
agriculture and services.
Industrial Policy Resolutions and Industrial
— National security requires that products for
Policy Statements. Specific priorities for
defence and other strategic sectors be produced
within the country itself so as to guard against industrial development were also laid down
adverse eventualities like sanctions, wars etc. in the successive Five Year Plans.
Magbook ~ Indian Industry 101
The main aims of the industrial licensing policy were the
Industrial Policy Resolution, —
development and control of industrial investment and
1948 (IPR) production as per national priorities, checking the concentration
— The 1st Industrial Policy was announced in April 1948, by of industries and ensuring balanced regional development.
then Industrial Minister, Late SP Mukherjee. Its historic — However, from time-to-time, many deficiencies in the
importance lies in the fact that it ushered in the system of licensing system came to light. The government set-up
‘Mixed Economy’ in the country i.e. it entrusted the task of several committees for the study of the licensing system and
industrial development on both private and public sectors. giving suggestions for its improvement.
Salient Features of IPR, 1948 — Such committees included RK Hazari Committee, 1964 and
Dr Subimal Dutt Committee, 1967. On the basis of Subimal
— Development of mixed economy.
Dutt Committee recommendation, government enacted the
— State programmes for the development of industries. Monopolies and Restrictive Trade Practises (MRTP) Act, 1969.
— Promotion of small-scale and cottage industries.
— Foreign investment was allowed, but effective control Monopolies and Restrictive Trade
should be with Indians. Practises Act, 1969 (MRTP)
— Industries classified into four categories: ◆
MRTP Act was enacted in 1969 and MRTP Commission was
(i) Public Sector
constituted in 1970, to prevent the concentration of economic
(ii) Mixed Sector power and to prohibit restrictive or unfair trade practices.
(iii) Controlled Private Sector ◆
Under the act, companies having assets beyond the threshold limit
(iv) Private and Co-operative Sector
(i.e. ` 20 crores in 1985) were placed under the purview of the act.
Certain restrictions are imposed on such companies like prior
Industrial Policy Resolution,

approval of the MRTP. Commission for establishment of new


1956 undertakings, expansion of undertakings, mergers and
— IPR, 1956 was the most comprehensive industrial policy, acquisitions.
which was formulated in the backdrop of the adoption of
the Constitution and the socio-economic goals. Industrial Policy Statement, 1977
— The policy may be described as the ‘Economic — The thrust of the policy was on decentralisation of the
Constitution’ of India, as it is not only outlined the basic
industries and the promotion of small-scale and cottage
framework of the future industrial policies (especially upto
industries. It introduced the concept of tiny sector within the
1991), but also of the general economic policies.
small-scale sector.
— Its main objectives were to accelerate the rate of
economic growth and to speed up industrialisation for Industrial Policy Statement, 1980
achieving a socialistic pattern of society.
— The policy emphasised the optimum utilisation of installed
Salient Features (IPR 1956) capacity, technological upgradation and modernisation.
— The policy industries divided into three categories: — The policy selectively liberalised the industrial sector i.e.
(i) Schedule A (Public Sector) Seventeen industries MRTP Act was liberalised, scope of licensing was reduced,
were exclusively reserved for the public sector. simplified the procedure for regularisation of unauthorised
(ii) Schedule B (Mixed Sector) Twelve industries were excess capacity etc.
placed in the mixed sector of public and private
enterprise. These were to be progressively An Evaluation of Industrial
state-owned and in which state would generally
set-up new units. Policy Before 1991
(iii) Schedule C (Private Sector) All the remaining — Several official committees as well as other experts bring to
industries and their future development would, in light the major deficiencies of the industrial policies before
general be left to the initiative and enterprise of the 1991. Some of the important criticism are as follows:
private sector. — The objective of the licensing policy was to ensure capacity
creation according to the priorities decided in the Five Year Plan.
Industrial Licensing But due to inherent demerits of the licensing system, in some
— The Industries (Development and Regulation) Act, 1951, areas excess capacity was created and there was under
empowered the government to issue licenses for the utilisation of capacity while at same time, despite availability of
setting-up of new industries, expansion of existing ones licenses, private players deliberately under produced goods,
and for diversification of products. which helped develop monopoly and monopolistic conditions in
the economy.
102 Magbook ~ Indian Economy

— Targets were not fixed for private sectors and they invested — In case of cities with population of one million or above,
only assured profit and avoided risk. excepting non-pollutant industries, all other units will be
— The objective of licensing policy was to check monopoly in the set-up at a distance of 25 km from the city limits.
economy, but it infact led to development of monopoly in the — MRTP Limit Scrapped The threshold limit of ` 100 crore
economy. Big industrial houses succeeded in getting the worth of assets for classification of a company as MRTP
licenses, while others failed.
company was removed, such companies were to be
— Due to licensing policy, developed states gained, more units recognised on case-by-case evaluation basis.
were set-up in already developed states, while poor and under
— Mandatory Convertibility Clause was Abolished It is the
developed states lagged.
condition imposed by the financial institutions on private
— Rigidity and complex nature of licensing caused delay in
companies that a part of their lending would be converted
permission and there was problem of delay in projects, which
slowed down the growth rate.
into equity at some future date.
— New Small Enterprise Policy A separate policy was
New Industrial Policy, 1991 (NIP) announced by the government in August 1991, for the
promotion of small-scale industries.
— The Government of India announced the New Industrial
Policy on 24th July, 1991. The main objective of this — Like the private enterprises, sick PSUs were also placed
policy is to unshackle the Indian industrial economy from under the purview of the Board for Industrial and
administrative and legal controls. Financial Reconstruction (BIFR).
— Its main aim is to raise industrial efficiency to the — Disinvestment of the shares of PSUs was initiated.
international level through substantial deregulation of the
industrial sector of the country. Competition Act, 2002
— The Competition Act was enacted by the government in
2002, on the recommendation of the SVS Raghavan
Industries Requiring Compulsory Committee. It repealed the MRTP Act and the MRTP
Licensing (Presently) Commission was replaced by the Competition Commission

Distillation and brewing of alcoholic drinks. of India (CCI).

Cigars and cigarettes of tobacco and manufactured tobacco — The objectives of the act are to encourage competition,
substitutes. prevent abuse of dominance (rather than dominance as

Electronic aerospace and defence equipment all types. such) and to ensure a level playing field for all the

Industrial explosives including match boxes. enterprises in the Indian economy.

Specific hazardous chemicals viz Hydrocyanic acid, Phosgene,
Isocyanates and diisocyanates of hydrocarbon. Companies Act, 2013
— This act introduces significant changes in the provisions
Main Features of NIP, 1991 related to governance, e-management, compliance and
— Delicensing The industrial licensing was abolished enforcement, disclosure norms, auditors and mergers and
irrespective of the level of investment, except for acquisitions. Also, new concepts such as one-person
18 specified industries like defence, atomic energy etc. company, small companies, dormant company, class
Since then, most of these industries were delicensed and action suits, registered valuers and corporate social
now only three industries fall under the purview of responsibility have been included. The changes in the Act
industrial licensing. 2013 has far reaching implications that are set to
— Foreign Investment Foreign capital investment limit was significantly changes the manner, in which corporates
raised from 40% to 51% in high technology and high operate in India. The Act 2013 has introduced several
investment priority industries. new concepts and has also tried to streamine many of the
— Foreign Technology Automatic approval was granted for requirements by introducing new definitions.
foreign technology agreements upto the limit of ` 200 crore — One Person Company The Act 2013 introduces a new type
subject to 5% royalty on domestic sales and 8% on exports. of entity to the existing list i.e. apart from forming a public
— Foreign Investment Promotion Board (FIPB) It was or private limited company, the Act 2013 enables the
established to expeditiously clear foreign investment formation of a new entity a ‘One Person Company’ (OPC).
proposals. It serves as a single window clearing agency for — Small Company A small company has been defined as a
the FDI proposals. company, other than a public company.
— Industrial Location Policy Excepting the big cities with — Paid-up share capital of which does not exceed 50 lakh INR.
population of one million, in other cities, industrial — Turnover of which as per its last profit and loss account does
licensing will not be required, but for those industries, not exceed 2 crore. INR or such higher amount as may be
where licensing is compulsory. prescribed which shall not be more than 20 crore INR.
Magbook ~ Indian Industry 103
— Dormant Company The Act 2013 states that a company
can be classified as dormant when it is formed and
Objectives of the Public Sector
registered under this Act 2013 for a future project or to ◆
To capture commanding heights of the economy i.e. to take up
hold an asset or intellectual property and has no significant strategic role in the industrialisation of the country.
accounting transaction. ◆
To accelerate the rate of economic growth through creation of
— National Financial Reporting Authority (NFRA) The Act basic infrastructure.
2013 requires the Constitution of NFRA, which has been ◆
To generate employment.
bestowed with significant powers not only in issuing the ◆
To promote balanced regional development.
authoritative pronouncements, but also in regulating the ◆
To generate surplus resources for development.
audit profession. ◆
To promote exports and to develop import substitution industries.
— Serious Fraud Investigation Office (SFIO) The Act 2013 ◆
To check concentration of economic power.
has bestowed legal status to SFIO.
Corporate Social Responsibility The Act 2013 makes an
—
effort to introduce the culture of Corporate Social
Expansion of Public Sector
Responsibility (CSR). In Indian corporates by requiring — There were only five Central Public Sector Enterprises
companies to formulate a corporate social responsibility (CPSEs) in 1951, with investment of ` 29 crore. The
policy and atleast incur a given minimum expenditure on number of CPSEs (excluding financial institutions) has
social activities. increased to 331 by 31st March, 2018 (out of which 257
were in operation).
The Companies (Amed.) Act, 2015 — There were over 800 state level public enterprises with
total investment in public sector in the entire country (i.e.
Important point related to Companies Act, 2015 are as follows: Centre + States) stood at over ` 6 lakh crore.
— Omitting requirement for minimum paid-up share capital
and consequential changes. The minimum paid-up share Contribution of Public Sector
capital requirement of INR 1,00,000 (in case of a private — The public sector was instrumental in the creation of
company) and INR 5,00,000 (in case of a public infrastructure and the development of basic industrial
company) under the Company Act, 2013 has been done structure of the country.
away with. — PSUs did a commendable job in the promotion of strategic
— Making common seal optional and consequential changes and key industries like atomic energy, armaments and
for authorisation for execution of documents. ammunition, aircrafts, heavy machinery, iron and steel,
— Doing away with the requirement for filling a declaration by coal, drugs, fertilizers etc.
a company before commencement of business or — The public sector provided employment to about 70% of
exercising its borrowing powers. the workers employed in the organised sector. Presently,
— Prohibiting public inspection of board resolutions field in public sector contributes about 24% to the GDP and
the registry. accounts for over 20% of the Gross Domestic Capital
— Empowering Audit Committee to give omnibus approvals Formation (investment).
for related party transaction on annual basis.
Problems of the Public Sector
— Replacing ‘special resolution with resolution’ for approval of
— The return on capital invested in PSUs has been deplorably
related party transactions by non-related shareholders.
low due to low profitability and losses of some PSUs.
Problems related with the Price Policy i.e. Administered
Public Sector —

Prices of the products of PSUs were deliberately kept


— At the time of independence, the country was lower than the market prices.
predominantly agrarian and lacked basic industries and — Lack of autonomy to the management of the PSUs due to
infrastructure facilities. The economy needed a big push. excessive political interference.
— The push could not come from the Indian private sector, — Low efficiency due to lack of incentives for better
which was starved of funds and lacked technical and performance.
managerial abilities. Further, it was incapable of taking risk — Excessive overheads especially in providing housing and
involved in long gestation investments. So, the other amenities to the employees e.g. townships.
development in the public sector became imperative. — Over staffing inflated the wage bills.
— The expansion of public sector in the field of industries — Inappropriate investment decisions like inappropriate
took place in a big way with the launching of the Second location, technology, product mix etc.
Plan (1956-61), which gave top priority to the industrial — Indiscriminate expansion of the public sector in almost all
growth of the country. areas.
104 Magbook ~ Indian Economy

The coveted status empowers the boards of these firms


Public Sector Reforms —
to take investment decisions up to ` 5000 crore as
— To improve the performance of the PSUs, the government against the present ` 1000 crore limit for Navratnas
has adopted the following measures, especially in the post without seeking government approval.
reform (1991 onwards) era. — The Maharatna firms would now be free to decide on
— Memorandum of Understanding the concept of investments up to 15% of their net worth in a project,
Memorandum of Understanding (MoU) was introduced in limited to an absolute ceiling of ` 5000 crore.
1987, on the recommendation of the ‘Committee to Review
the Policy for the Public Enterprises’ headed by Mr Arjun Maharatna CPSEs
Sengupta. — Oil and Natural Gas Corporation Limited (ONGC)
— MoU refers to the agreement between the concerned ministry — Indian Oil Corporation Limited (IOCL)
and the management of a PSUs in which the latter is provided — Steel Authority of India Limited (SAIL)
a reasonable degree of autonomy and simultaneously held — NTPC Limited
accountable for the performance of the PSUs. — Coal India Limited (CIL)
— New Industrial Policy, 1991 The policy contained the — Bharat Heavy Electricals Limited (BHEL)
following reformative measures for PSUs; dereservation, — Gas Authority of India Limited (GAIL)
disinvestment, professionalisation of management, — Bharat Petroleum Corporation Limited
reference of sick PSUs to the BIFR and expanding the
scope of MoUs.
Policy of Navratnas
— Voluntary Retirement Scheme (VRS) The VRS (or Golden
— Navratna was the title given originally to nine Public
Handshake Scheme) was launched in 1988, for the
Sector Enterprises (PSEs), identified by the Government
rationalisation of manpower in the central PSUs. The
of India in 1997, as its most prestigious, which allowed
scheme enabled the PSUs to shed their excess staff by
them greater autonomy to compete in the global
offering attractive compensation package to the workers,
market.
who seek voluntary retirement.
— Dismantling of Administered Price Mechanism (APM) The Criteria for Navratna Status for
government has initiated steps for dismantling of price
controls in respect of a number of products of PSUs. e.g. PSUs
it removed the price and distribution controls on iron, steel — It should have a schedule ‘A’ and Miniratna category—
and cement. The government also decontrolled the prices 1 status.
of most of the fertilizers and petro-products. — It should have atleast three ‘excellent’ or ‘very good’
Memorandum of Understanding (MoU) ratings during the
Policy of Maharatnas last 5 years.
— Maharatna Scheme was introduced for Central Public — It should have a composite score of 60 out of 100 marks
Sector Enterprises (CPSEs), with effect from 19th May, based on its performance during the last 3 years on
2010, in order to empower mega CPSEs to expand their these 6 criterias-net profit to net worth, manpower
operations and emerge as global giants. cost-to-cost of production or services, gross margin as
— The objective of the scheme is to delegate enhanced capital employed, gross profit as turnover, earnings per
powers to the boards of identified large-sized Navratna share, inter-sectoral comparison based on net profit to
CPSEs, so as to facilitate expansion of their operations, net worth.
both in domestic as well as global markets. — The company should also have four independent directors
— CPSEs fulfiling the following criteria are eligible to be on its board.
considered for grant of Maharatna status: — Navratna status empowers the PSUs to invest up to `
— Having Navratna status. 1000 crore or 15% of their net worth on a single project
— Listed on the Indian Stock Exchange, with a minimum without seeking government approval. The overall ceiling
prescribed public shareholding under SEBI regulations. on such investments in all projects put together is 30% of
— An average annual turnover of more than ` 25000 crore the net worth of the company.
during the last 3 years.
Navratna CPSEs
— An average annual net worth of more than ` 15000 crore
during the last 3 years.
— Bharat Electronics Limited
— An average annual net profit after tax of more than ` 5000 — Hindustan Aeronautics Limited
crore during the last 3 years. — Hindustan Petroleum Corporation Limited
— Significant global presence or international operations. — Mahanagar Telephone Nigam Limited
Magbook ~ Indian Industry 105

National Aluminium Company Limited


—
Objectives of Disinvestment
— NMDC Limited
— To transfer the resources from non-strategic sector to the strategic
— Oil India Limited
sector, which is much higher on social priority such as basic health,
— Power Finance Corporation Limited family welfare, primary education etc.
— Power Grid Corporation of India Limited
— To raise funds to cover up the fiscal deficit of the government.
— Container Corporation of India Limited — To improve efficiency of the public sector by inducing private
— Engineers India Limited initiative and competition.
— National Buildings Construction Corporation Limited — To enhance accountability of the PSUs by exposing them to the
— Rashtriya Ispat Nigam Limited capital market. To reduce political interference by imparting
— Rural Electrification Corporation Limited market orientation to the enterprise.
— Shipping Corporation of India Limited — Bring down government equity in all non- strategic PSUs to
— Neyveli Lignite Corporation Limited 26% or lower, if necessary.
— Restructure and revive potentially viable PSUs.
Policy of Miniratnas —

—
Close down PSUs, which can not be revived.
Fully protect the interest of workers.
— The government has also accorded the status of
Miniratna to some profit making PSEs. There are two
types of Miniratnas — Disinvestment vs Privatisation
Category I and Category II. Disinvestment refers to selling of equity of a PSU to a private
organisation or to general public. Privatisation refers to providing for
Category-I Miniratna
larger role for private capital and enterprise in the functioning of an
— These are companies, which have made a profit in economy. Privatisation is a wider term than disinvestment.
each of last 3 years and earned a profit of ` 30 crore Disinvestment is one of the means for achieving privatisation.
in atleast one of the 3 years. Privatisation may result from any of the following
— They are allowed to incur capital expenditure without ◆
Disinvestment
government approval upto ` 500 crore or equal to ◆
Denationalisation (i.e. complete sell off of a PSUs)
their net worth whichever is lower. There are 60 ◆
Transfer of management and control of a PSUs to the private sector
Miniratnas of this category at present {June 2017}. ◆
Dereservation of areas reserved for the public sector etc.
Category-II Miniratna
— These are companies, which have made profits for The Disinvestment Process
the last 3 years continuously and have a positive net — In 1992, government constituted a committee on the
worth. They can incur capital expenditure upto ` 300 disinvestment of shares in PSE’s headed by Dr C Rangarajan to
crores or 50% of their net worth whichever is lower. recommend on the policy of disinvestment. The committee
— There are presently (June 2017) 15 such category-II recommended that upto 49% equity of the PSUs under the
Miniratnas. exclusive participation of the state could be disinvested but for
rest of the industries disinvestment can be allowed upto 74%.
Disinvestment — Further, the government constituted a 5 member Disinvestment
Commission, under the chairmanship of Shri GV Ramakrishnan
— The New Industrial Policy, 1991, envisaged
in August, 1996, to draw up a comprehensive policy for the
disinvestment of a part of government shareholdings
long-term disinvestment programme.
in selected PSUs as an important element of public
sector reforms. In pursuit of this, the process of — The commission was mandated to advise the government on
disinvestment began in 1991-92, with the sale of the extent, methodology, strategy and timing of disinvestment.
minority stakes in some PSUs. — In May 2004, the government adopted National Common
— The primary aim of disinvestment in this phase was Minimum Programme, which outlined the policy of the
to raise non-inflationary finance to plug budgetary government with respect to the public sector as follows:
deficit. But the focus of disinvestment shifted to — In general, profit making PSU’s will not be privatised.
strategic sale since 1999, in which substantial chunk — In case of privatisation of profitable PSU’s, government will retain at
of government equity is sold to private sector least 51% of the equity and the management control of the
enterprises with an objective to improve the enterprise.
performance of the PSUs and to reorient public — Navratnas PSUs will be retained under the public sector.
investment. — Chronically, loss-making companies will be either sold-off or closed,
after all workers get their legitimate dues and compensation.
106 Magbook ~ Indian Economy

— All privatisations will be considered on a transparent and — In accordance with the provision of Micro, Small and
consultative case-by-case basis. Medium Enterprises Development (MSMED) Act, 2006, the
— A Board for Reconstruction of Public Sector Enterprises Micro, Small and Medium Enterprises (MSMEs) are
(BRPSEs) to be constituted. classified in two classes:
— A National Investment Fund will be established. (i) Manufacturing Enterprises The enterprises engaged in
— On 25th November, 2005, the government decided, in the manufacture or production of goods pertaining to
principle, to list large, profitable CPSEs on Domestic any industry specified in the 1st Schedule to the
Stock Exchanges and to selectively sell small portions of Industries (Development and Regulation) Act, 1951.
equity in listed, profitable CPSEs (other than Navratnas). The manufacturing enterprise are defined in terms of
investment in plant and machinery.
National Investment Fund
Enterprises Manufacturing Sector
— In pursuance of the policy laid down in NCMP, the
Central Government set-up National Investment Fund in Investment in Plant and Machineries
November 2005. The proceeds from disinvestment of
Micro Enterprises Does not exceed ` 25 lakh.
CPSUs will be channelised into NIF, which is to be
maintained outside the Consolidated Fund of India. Small Enterprises More than ` 25 lakh, but does not
exceed ` 5 crore.
— NIF will be professionally managed to provide
sustainable returns to the government, without depleting Medium Enterprises More than ` 5 crore, but does not
the corpus. Selected Public Sector Mutual Funds will be exceed ` 10 crore.
entrusted with the management of the corpus of NIF.
75% of the Annual Income of NIF will be used to finance (ii) Service Enterprises The enterprises engaged in
selected social sector schemes, which promote providing or rendering of services and are defined in
education, health and employment. terms of investment in equipment.
— The residual 25% of the annual income of the fund will Enterprises Service Sector
be used to meet the capital investment requirements of
profitable and revivable CPSUs that yield adequate Investment in Equipments
returns or in order to enlarge their capital base to Micro Enterprises Does not exceed ` 10 lakh.
finance expansion diversification.
Small Enterprises More than ` 10 lakh, but does not
— The following Public Sector Mutual Funds have been exceed ` 2 crore.
appointed initially as Fund Managers, to manage the
funds of NIF. Medium Enterprises More than ` 2 crore, but does not
exceed ` 5 crore.
— UTI Assets Management Company Limited
— SBI Funds Management Company (Private) Limited
— Life Insurance Corporation, Asset Management Company MSMEs Policy, 2012
Limited
— The policy was notified in March 2012.
— The policy envisages that every Central Ministry and PSU
The Micro, Small and shall set an annual goal for procurement for the Micro and
Medium Enterprises (MSMEs) Small Enterprises (MSEs) sector with the objective of
achieving minimum 20% of the total annual purchase of
— Over the last five decades, the Small-Scale Industries MSEs in a period of 3 years.
(SSIs) sector has acquired place of prominence in the
economy of the country. It has contributed Cottage and Village Industries
significantly to the growth of the GDP, employment
generation and exports. Cottage Industries
— The sector now includes not only SSI units but also — This type of industry is run by family members on full or
Small-Scale Service and Business Enterprises (SSSBEs) part time basis. It has negligible capital investment. There
and is thus, referred to as the small enterprises sector. is hand made production using own tools and materials.
— In India, Small-Scale Industries (SSIs) can be Village Industries
differentiated from the large-scale industries on the basis — The industries established in rural area having population
of three criteria viz volume of investment in the unit, below 10000 and having less than 15000 as fixed capital
annual turnover and number of workers employed.
investment per worker are termed as village industries.
Magbook ~ Indian Industry 107

Differences Between Cottage and — They are not able to get loans, since, they cannot offer good
Small-Scale Industries security.
— Cottage industries are mostly in rural areas, while small — Due to old methods of production, the goods are many times
industries are mostly in urban areas. of poor quality. They lack marketing support due to lack of
— Cottage industries are run by family members, while funds.
small industries have hired labourers. — Their cost of production is high due to expensive raw
— Cottage industries have very little capital investment, material.
while small industries can have upto ` 5 crore invested. — They have mostly focussed on producing artistic goods,
— Cottage industries produce mostly traditional goods, whose demand is limited and so production cannot be
while small industries produce many modern goods increased beyond a point.
also. — They have to compete with large-scale industries in many
— Cottage industries are located in the homes of workers, areas. Large industries are able to achieve low costs of
while small industries are not located in homes. production due to economies of scale.

Contribution of Small-Scale Government Measures to Promote


Industries in Indian Economy Small-Scale Industries
— The small-Scale sector accounts for over 80% of the — Government initiated several measures for the promotion of
manufacturing sector’s employment. small-scale and cottage industries immediately after
independence. The importance of these industries was
— It contributed significantly towards the economic
recognised in the very first Industrial Policy Resolution of
growth of the nation, with over 39% of the industrial
1948 and reiterated in all future industrial policy statements.
production.
— Steps taken by the government for the development of these
— The small-scale accounts for over 34% of the total
industries can be categorised as organisational, financial,
exports and about 45% of the manufacturing exports.
fiscal, technical etc.
Further, over 90% of exports of the SSls consists of
non-traditional items like sports goods, readymade Organisational Measures
garments, processed foods, chemicals etc. — Establishment of Boards Government constituted several
— SSIs are conducive for the economic development of boards at all India level to provide an organisational structure,
underdeveloped countries like India. Such industries through which the promotional efforts were to be carried out.
are relatively labour intensive, so they make These boards include Cottage Industries Board, Handloom
economical use of the scarce capital. Board, Handicraft Board, Khadi and Village Industries Board
— Small-scale industries are instrumental in reducing the etc.
inequalities in wealth. In these industries, capital is — Industrial Estates The policy of setting-up Industrial Estates
widely distributed in small quantities and the surplus of was initiated in 1955, for the construction of factory premises
these industries is distributed among large number of and to provide basic facilities like power, water, roads etc.
people. — District Industries Centre (DIC) The concept of DIC was
— Small-scale industries brings about regional dispersal introduced in the Industrial Policy Statement of 1977. This
of industries and alleviates regional imbalances. programme was initiated in 1979, to cater to all the
— Small-scale industries, make use of local resources requirements of small-scale and village industries, under one
including the capital and entrepreneurial skills, which roof.
would have remained unused for want of such
Financial Measures
industries.
— Small Industries Development Fund (SIDF) It was set-up in
— Small industry sector has performed exceedingly well
1986, to provide refinance (i.e. finance to the financial
and enabled the country to achieve a wide measure of
institutions in lieu of their-lending to SSIs) assistance for
industrial growth and diversification.
development, expansion, modernisation, rehabilitation of
SSIs.
Problems of Small-Scale — National Equity Fund (NEF) It was set-up in 1987, to
Industries provide initial capital for setting-up of new projects in
— These industries are not able to get raw material of small-scale sector in the form of equity (i.e. shares).
adequate quality. The raw material they get, is also — Single Window Scheme (SWS) It was introduced in 1988,
high in price. to provide short-term as well as long-term financial assistance
— They are not able to get regular power supply. to SSIs.
108 Magbook ~ Indian Economy

— Small Industries Development Bank of India (SIDBI) It Credit Rating Scheme for Micro and
was established in October 1989, by amalgamation of Small
Industries Development Fund (SIDF) and Natural Equity
Small Enterprises
Fund (NEF). SIDBI is the apex financial institution for small — National Small Industries Corporation (NSIC) in
enterprises sector. It provides finance to SSI, refinance consultation with credit rating agencies and Indian Bank’s
assistance and coordinates the activities of other financial Association has formulated a Performance and Credit
institutions for the provision of credit to SSIs. Rating Scheme for Micro and Small Enterprises.
— NSIC is the nodal agency for the implementation of the
Fiscal Measures scheme. NSIC empanelled the leading credit rating
— Small-scale enterprises having turnover, upto ` 1 crore are agencies like CARE, CRISIL, Dun and Bradstreet, FITCH,
fully exempted from the excise duty. ICRA, ONICRA and SME Rating Agency of India Limited
— Concessional rate of custom duties are levied on import of (SMERA) to conduct the rating of interested Micro and
certain kind of raw materials and components used by Small Enterprises under the scheme.
SSIs.
— Price and purchase preference is granted to products National Manufacturing
manufactured in the small-scale sector in government Competitiveness Programme
purchase programme.
— The National Manufacturing Competitiveness Programme
Technical Measures (NMCP) for the MSMEs aims at enhancing the
— Small-Scale Industries Development Organisation (SIDO) It competitiveness of enterprise in this sector. There are 10
was established in 1954. SIDO provides technical, components of the NMCP, which have been approved
managerial, economic and marketing assistance to SSls and are available for MSMEs. These are as follows:
through its network of extension centres and service — Lean Manufacturing Competitiveness Scheme (LMCS) for
institutes. MSMEs.
— Council for Advancement of Rural Technology (CART) It — Design Clinics Scheme for design expertise to MSMEs
manufacturing sector.
was established in 1982, to provide technical assistance to
rural industries. — Marketing Assistance and Technology Upgradation Scheme
for MSMEs.
— Technology Development and Modernisation Fund (TDMF)
— Enabling manufacturing sector to be competitive through
It was set-up for the technological upgradation and
Quality Management Standards (QMS) and Quality
modernisation of the export oriented units. Technology Tools QTT.
— Technology of Quality upgradation support for MSMEs.
Schemes/Programmes for — Promotion of Information and Communication Technology
Small Industries (ICT) in MSME sector.
— Setting-up Mini Tool Room and Training Centres under PPP
Rajiv Gandhi Udyami Mitra Yojana Mode.
— Under this scheme, the selected lead agencies i.e. Udyami — Marketing Assistance or Support to MSEs (Bar Code).
Mitras are providing guidance and assistance to the — Building Awareness on Intellectual Rights for MSMEs.
potential entrepreneurs registered with them, in preparation — Scheme for Providing Support for ‘‘Entrepreneurial and
of project report, arranging finance, selection of technology, Managerial Developments of SMEs through Incubators’’.
plant and machinery, marketing tie-ups with buyers, as well
as obtaining various approvals, clearances and NOCs etc. Procurement Policy
— Udyami Mitras are expected to assist the new entrepreneurs — Government has approved a public procurement policy
in the establishment and successful running of the for Micro and Small Enterprises , which mandates that
enterprise for the first-sixth months. Central Ministries, Departments or PSUs have to procure
minimum 20% of their total annual purchases from
— The scheme is beneficial to all potential first generation
MSEs. Out of this 20%, a share of 4% has to be
entrepreneurs, in all towns as well as rural areas, by
earmarked for the MSEs owned by SC/ST entrepreneurs.
encouraging establishment of new enterprises and thereby
creating new job opportunities locally. — Central Ministries, Departments or PSUs will prepare their
annual plan for setting of goal of 20% procurement and
— A Udyami Helpline (a Call Centre for MSMEs) with toll-free
will mention in their annual reports. The policy will help to
number 1800-180-6763 is in operation to provide
promote MSEs by improving their market access and
information, support, guidance and assistance to first
competitiveness through increased participation by MSEs
generation entrepreneurs as well as other existing
in government purchases and encouraging linkages
entrepreneurs.
between MSEs and large enterprises.
Magbook ~ Indian Industry 109

Credit Linked Capital Subsidy Industrial Sickness


Scheme for Micro and Small — The government defined the industrial sickness for the first
Enterprises time in the Sick Industrial Companies (Special Provisions)
Act, 1985. According to this act, a medium or large (i.e.
— The scheme aims at facilitating technology up-gradation non-SSI) company was defined as sick if
of Micro and Small Enterprises (MSEs) by providing 15%
— It was registered for atleast 7 years (later reduced to 5 years).
capital subsidy (limited to maximum ` 15 lakhs) for
— It incurred cash losses in the current year and the preceding year.
purchase of Plant and Machinery.
— Its entire net worth (i.e. paid-up capital and reserves) was
eroded.
Reservation of Items for SSIs
— A company is regarded, as weak or incipiently sick on the
— The policy to reserve certain items for the small-scale erosion of 50% of its peak net worth during any of
sector was introduced in 1967. It aims to promote the preceding five financial years. Industrial sickness has been
SSIs by protecting them from competition with the covered in the Companies (Second Amendment) Act, 2002
large-scale units. and Companies Act, 2012.
— In April 1967, there were only 47 items in the reserved
category, which were increased in several phases to 873 Causes of Industrial Sickness
in 1984. — The causes of industries sickness can be classified into two
categories i.e. internal and external causes.
New Small Enterprise Policy, 1991
Internal Causes
— Government announced a separate industrial policy for
the small enterprise sector on 6th August, 1991. It was
— Originate within the unit, so they can be controlled by the
titled as, “Policy Measures for Promoting and unit. These include
Strengthening Small, Tiny and Village Enterprises”. — faults at planning and construction stage
— inappropriate plant and machinery
Salient Features — management problems
— The ceiling of investment for the ‘tiny sector’ was raised — entrepreneurial problems
from ` 2 lakh to ` 5 lakh. — labour problems
— Large units including foreign firms were allowed to — financial problems etc
purchase upto 24% equity (shares) of the small-scale External Causes
industries. Scope of tiny sector was enlarged to include
— Supposed to originate outside the unit, so are not under the
all industry related service and business enterprises.
control of the unit.
— Introduction of a new legal form of business organisation,
Such factors include
limited partnership. In this form, the liability of atleast
— erratic supply of inputs
one partner is unlimited whereas other partners have
— power cuts
their liability limited to the invested capital.
— demand and credit constraints
Current Policy on SSIs — changes in government policies etc
— The report of the Task Force on micro, small and
medium enterprises, presented to the Honorable PM on Revival of Sick Units
30th January, 2010, provides a roadmap for the Government under take the following measures to revive the
development and promotion of MSMEs. sick industrial units:
— The detailed recommendations cover 6 major thematic
Industrial Investment Bank of India (IIBI)
areas, namely credit, marketing, labour, rehabilitation
— The government established the Industrial Reconstruction
and exit policy, infrastructure, technology and skill
Corporation of India (IRCI) in 1971, as a company
development and taxation as also special measures for
registered under the Companies Act. Its objective was to
the North-Eastern region and Jammu and Kashmir.
revive and rehabilitate the sick units by providing financial,
Abid Hussain Committee Report managerial and technical assistance.
— In 1985, IRCI was renamed as Industrial Reconstruction
— The Abid Hussain Committee is appointed by the
Bank of India (IRBI) and was converted into a statutory
Government of India to suggest measures to improve the
corporation. Further in 1997, the IRBI was transformed into
performance of small-scale industries. The committee
a full-fledged development financial institution and
submitted its report in January 1997.
rechristened as Industrial Investment Bank of India (IIBI).
110 Magbook ~ Indian Economy

Board for Industrial and Financial — The Sixth Economic Census had also the same coverage
Reconstruction (BIFR) as that of Fifth Economic Census. However,
— The government set-up BIFR in 1987, under the Sick establishments engaged in public administration, defence
Industrial Companies (Special Provisions) Act (SICA), and compulsory social security activities have been
1985. The BIFR is an autonomous quasi-judicial body to excluded as data pertaining to them are available with the
take final decision regarding revival and rehabilitation or government through administrative records and also due
winding up of the sick units. to the difficulties faced in collecting information from sue’s
establishments during the Fifth Economic Census.
— It is mandatory for a sick or weak unit to refer itself to the
BIFR. On receipt of such reference, the board ascertains — The Sixth Economic Census separately identified
whether the company is indeed sick. handicrafts handloom establishments for the first time.
— If sickness is confirmed, the board may allow the company — Further, Enumeration Blocks (EBs) of Population Census,
sometime to make its net worth positive on its own, 2011 have been used as the primary geographical units
prepare revival and rehabilitation package or wind-up the for collection of data.
unit.
Corporate Social Responsibility
* High interest rates and slower growth inhousehold
or retail credit resulted in slower growth in
(CSR)
consumer durables. — According to the provision for Corporate Social
* Industry is creating jobs, which have been relatively Responsibility (CSR), every company having net worth of
low-productivity jobs. As a result, per capita income ` 500 crore or more or turnover of ` 1000 crore or more or
in India has not benefited as much from a net profit of ` 5 crore or more during any financial year
inter-sectoral migration of workers out of agriculture is required to constitute a Corporate Social Responsibility
as other Asian countries have. Committee.
— The Corporate Social Responsibility Committee will
Economic Census formulate a Corporate Social Responsibility Policy.
— The Central Sector Scheme on Economic Census (EC) — Such a company is required to spend atleast 2% of the
and follow-up surveys was for the first-time formulated by average net profits of the company made during the three
the then Central Statistical Organisation (CSO) way back immediately preceding financial years, in pursuance of its
in 1976 with the following two main objectives: Corporate Social Responsibility Policy.
(i) To provide a frame (list) of all establishments in the
country, from which samples could be drawn for Index of Industrial Production (IIP)
collecting detailed information on operational and — IIP is an index for measuring the level of industrial
economic variables for any specific industry. activity in the country. All India IIP is a composite
(ii) To compile information on these variable or indicator that measures the short-term changes in the
parameters of all the establishments of the country volume of production of a basket of industrial products
including their distribution at All India, State or during a given period with respect to that in a chosen
District etc levels for comprehensive analysis of the base period.
structure of the economy. — It is complied and published monthly by the Central
— Since then, six such EC’s have been conducted so far, Statistics Office (CSO) with the time lag of 6 weeks from
during the years 1977, 1980, 1990, 1998, 2005 and the reference month.
2013. The Economic Census data, over the years, have — The Index of Industrial Production (IIP) with 2004-05 as
provided a base for under taking follow-up surveys by base is the leading indicator for industrial performance
NSSO and other governmental and non-governmental in the country. Compiled on a monthly basis, the current
agencies to study the structure and composition of the IIP series based on 399 products or product groups is
various industrial sectors and their contribution. aggregated into three broad groups of mining,
manufacturing and electricity. The IIP as an index shows
Sixth Economic Census both the level of production and growth.
— Central statistics office in the Ministry of Statistics and — The Central Statistics Office (CSO) of the Ministry of
Programme Implementation (MoSPI) conducted the Sixth Statistics and Programme Implementation (MoSPI)
Economic Census during January 2013 to April 2014 in released the new series of the IIP with 2004-05, as its
collaboration with directorates of economics and statistics new base on 10th June, 2011, replacing the IIP series
in all the States and Union Territories. with 1993-94 as base.
Magbook ~ Indian Industry 111

— Industry associations lobbying for their members’


Core Sector (often conflicting) interests are important stakeholders.

The Core Sector consists of eight core industries in the — Other stakeholder groups, who must be involved in the
economy having a weightage of 40.27% in the IIP. These consultations in a more systematic and productive way are
eight sectors are Coal, Crude Oil, Natural Gas, Petroleum unions, land owners etc.
Refinery Products, Fertilizers, Steel, Cement and Electricity. — There are many over-sight bodies and committees, perhaps
The weightage of Core Industries as follows too many. There is need to sharpen their roles and improve
co-ordination amongst them.

Natural Gas (6.88%).

Fertilizers (2.63%). National Manufacturing Plan as

Steel (17.92%). Suggested by the 12th Plan

Crude Oil (8.98%). — India’s strategic objectives for the manufacturing sector in

Coal (16.33%). the next 15 years should be to bring about a quantitative

Refinery Products (28.04%). and qualitative change through a set of policies and plans

Cement (5.37%). with the following five objectives.

Electricity (19.85%) (i) Increase manufacturing sector growth to 12-14%,
over the medium term to make it the engine of growth
for the economy.
Purchasing Managers Index (PMI) (ii) The 2-4% differential over the medium term growth
— The PMI is a composite index of 5 ‘sub-indicators’, which rate of the overall economy will enable manufacturing
are extracted though surveys of purchasing managers form to contribute at least 25% of GDP by 2025.
around the country, chosen for their geographic and (iii) Increase the rate of job creation in manufacturing
industry diversification benefits. to create 100 million additional jobs by the year
— In India, the most widely followed PMI is the one prepared 2025. Emphasis should be given to creation of
by HSBC called the India Manufacturing PMI. appropriate skill sets among the rural migrant and
— The five sub-indexes in this system and their weightage urban poor to make growth inclusive.
are as follows: (iv) Increase domestic value addition and technological
— New orders-0.3, output-0.25, employment 0.2, suppliers delivery ‘depth’ in manufacturing.
times 00.15, stock of items purchased 00.1. A PMI of more than (v) Enhance global competitiveness of Indian
50, represents expansion of the manufacturing sector, compared manufacturing through appropriate policy support.
to the previous month. A reading under 50, represents a — Ensure sustainability of growth, particularly with regard to
contraction, while a reading at 50 indicates no change. the environment.

India’s Manufacturing Sector National Manufacturing Policy,


— The 11th Plan had targeted growth in manufacturing at 2011 (NMP)
10-11%, but actual performance was only about 7.7%. It is a The major objectives of the National Manufacturing Policy are
matter of concern that the manufacturing sector has not as follows:
shared in the dynamism of the economy not just in the — to increase the sectoral share of manufacturing in GDP to
11th Plan, but even in preceding plan periods. As a result, atleast 25% by the year 2022;
the share of the manufacturing sector in GDP is only 15% in — to increase the rate of job creation, so as to create 100 million
India, compared with 34% in China and 40% in Thailand. additional jobs by the year 2022;
— The slow pace of growth in the manufacturing sector at — to enhance global competitiveness, domestic value addition,
this stage of India’s development is not an acceptable technological depth and environmental sustainability of growth;
outcome. Manufacturing must provide a large portion — to provide a productive environment to persons transitioning
of the additional employment opportunities as opposed from the primary to the secondary and tertiary sectors
to agriculture, for India’s increasing number of youth. through the creation of large integrated industrial townships
called National Investment and Manufacturing Zones
— The challenges to developing and implementing a cohesive
(NIMZs) with state-of-the-art infrastructure; land use on the
manufacturing strategy : basis of zoning; clean and energy efficient technologies;
— There is a multiplicity of ministries dealing with different aspects — to ensure compliance of labour and environmental laws, while
of industry e.g. commerce, labour, environment, science, finance introducing procedural simplifications and rationalisation, so
etc. The states have a major role in facilitating the growth of that the regulatory burden on industry is reduced;
manufacturing in terms of provision of infrastructure,
— to provide an enabling environment for tapping the potential
management of various local regulations and managing labour
of the private sector and the entrepreneurial skills of the
related laws.
younger population.
112 Magbook ~ Indian Economy

Highlights of the National Delhi-Mumbai Industrial Corridor Project


(DMIC)
Manufacturing Policy — The DMIC is proposed to be developed on side
— It will set-up National Investment and Manufacturing Zones along the alignment of the 1483 km long Western
(NIMZs). The minimum land area of each NIMZ or Dedicated Rail Freight Corridor between Dadri (Uttar
greenfield integrated industrial townships with the modern Pradesh) and Jawaharlal Nehru Port Trust ( JNPT),
infrastructure is to be 5000 hectares.
Navi Mumbai. The project seeks to create a strong
— The first phase of the NIMZ will be established along the economic base with a globally competitive
Delhi-Mumbai Industrial Corridor. environment and state-of-the-art infrastructure to
— The National Investment and Manufacturing Zone (NIMZ) activate local commerce, enhance investments and
proposed under the National Manufacturing Policy will be attain sustainable development.
managed by a Special Purpose Vehicle (SPV), headed by a — A model industrial corridor of international standards is
government official and have experts, including those on proposed to be developed with emphasis on expanding
environment. the manufacturing and services base and creating a
— The industrial townships will be self-governing and ‘Global Manufacturing and Trading Hub’. The DMIC
autonomous bodies. Single window clearance will be runs across the 6 states of Uttar Pradesh, Haryana,
provided to improve the regulatory environment. Madhya Pradesh, Rajasthan, Gujarat and Maharashtra
— The Central Government will create the enabling policy and majority of projects in DMIC are envisaged to be
framework, provide incentives for infrastructure implemented through public private partnership.
development on a private-public partnership basis — The DMIC Development Corporation (DMICDC) was
through appropriate financing instruments, while State incorporated in January 2008, for project development,
Governments will identify suitable land and be equity coordination and implementation of the numerous
holders in the NIMZs. projects in the DMIC.
— The special purpose vehicle, which will administer the NIMZ — 6 investment regions and 6 industrial areas are
will set-up skill development centres on a build own and approved to be developed in Phase I of the project.
operate basis.
— Private sector will be given standards deduction of 150% of
expenditure for skill development institutes.
Some Large-Scale
— With a view to protect the interests of labour in cases of Industries
closure of units, the policy has a mechanism of fund to
insure the workers against such loss.
Iron and Steel Industry
— The focus will also be on green manufacturing. In this — ‘Steel was a symbol of strength and a portent of the
regard, a Technology Acquisition Fund will be set-up to glory of India of the future’—Jawaharlal Nehru.
acquire global technologies and build a patent pool — The first public-owned steel plant was Rourkela
especially for equipment manufacturing that seeks to reduce Integrated Steel Plant was set-up in 1954 with the help
energy consumption. of German Collaboration.
— Environmental clearances will continue to be given as per — Steel Authority of India Limited (SAIL) was set-up in
existing rules and regulations, but the Environment Ministry 1974 and was responsible for the development of the
has agreed to give priority in processing cases from the steel industry and for major inputs for the industry.
manufacturing zones. — India is the fourth largest producer of the Crude Steel in
— The Environment Ministry will also designate officials from the world after China, Japan and the USA in 2010. In
the State Pollution Control Board to ensure speedy 2009, India was ranked 3rd. India is the largest
clearances. producer of Sponge Iron since, 2002.

NIMZs Cotton and Synthetic Textile


The NMP provides for the development of NIMZs as
—

integrated industrial townships with state-of-the-art


Industry
infrastructure and land use on the basis of zoning; — It is the largest industry in India, accounting for about
clean and energy-efficient technology; necessary social 20% of industrial output, provides employment to
infrastructure and skill development facilities to provide 20 million persons and contributes 33% to total export
a productive environment to persons transiting from the earnings.
primary sector to the secondary and tertiary sectors. — The Indian Textile Industry is predominantly cotton based
with 65%.
Magbook ~ Indian Industry 113

— The organised Textile Industry comprises of (i) spinning mills; — Dual price mechanism with partial control is applied to
(ii) coarse and medium composite mills and (iii) fine and sugar industry. Under this, the government fixes the
superfine composite mills. ratio levy and free sale sugar quota in the ratio 28 : 72.
— Average per capita consumption of cloth has increased The levy sugar is sold to consumers through fair price
steadily since eighties. It stood at 39 m in 2008-09. shops at lower price and free sale sugar quota is sold
— Textile Export Promotion Council (TEXPROCIL) was by sugar factories at higher prices in the open market.
established by the government to strengthen and facilitate
the textiles exports. Cement Industry
— The Scheme for Integrated Textile Park (SITP) was launched in — The foundation of stable Indian Cement Industry was
July 2005, merging two schemes, i.e. Apparel Parks for Export laid in 1914, when the Indian Cement Company
Scheme (APES) and Textile Centre Infrastructure Development Limited started manufacturing of cement at Porbandar
Scheme (TCIDS). in Gujarat.
— In Global Textiles Exports, India now stands at 2nd position. — India is the second largest producer of cement in the
world.
— India’s share in Global Textiles has increased by 17.5% in
the year 2013.
— The per capita consumption of cement in India is just
68 kg.
Jute Industry
Petrochemical Industry
— Jute industry was started in 1885 and India is the largest
producer and second largest exporter of jute in the world.
— The real thrust to this industry came with the
establishment of Indian Petrochemical Corporation
— Jute Technology Mission was launched on 2nd June, 2006.
Limited at Baroda.
— The revival package of National Jute Manufactures — Petrochemical industry mainly, comprises synthetic
Corporation (NJMC) envisages operationalisation of three jute
fibres, polymers, elastomers, synthetic detergents and
mills viz. Kinnison and Khardah in West Bengal and Rai
performance plastics. The main source of feedstock
Bahadur Hadrut Mill, Katihar (Bihar). Government has
and fuel to this industry are natural gas and naptha.
enacted Jute Packaging Materials (compulsory use in
packing commodities) Act, 1997 to broaden the usage of
— National Policy on Petrochemicals was announced in 2007
jute. with the objective of increasing investment, demand and
achieve environmentally sustainable growth.
Food Processing Industry Fertilizer Industry
— Food processing is one of the most heterogeneous sectors of — The first fertilizer industry was set-up in 1906 in
manufacturing covering marine products dairy products, grain, Ranipat near Chennai.
meat products, fruits, vegetables, sugar, edible oils and
— India meets 85% of its requirement through
beverages. This sector has, however, been one of the fastest
indigenous production, but is largely import
growing segment in manufacturing in the current year
dependent for meeting the phosphorus and potassium
contributing 27% to average industrial growth.
fertilizer.
Mega Food Parks (MFPs) — India is the third largest producer of fertilizer after
— Ten MFPs were approved in the first phase. China and USA and second largest consumer after
— Five MFPs were approved in the second phase. China.
— Proposals have been invited for additional 15 MFPs. — NPK (Nitrogen, Phosphorus, Potassium) consumption
— Each of these MFPs is likely to consist of 30-40 ratio in 2008-09 was 4.6:2:1. The ideal ratio is 4:2:1.
food-producing units in the cluster. — Urea is the only fertilizer under statutory price control.
— With effect from 1st April, 2010, Nutrients Based
Sugar Industry Subsidy (NBS) Policy is implemented. The nutrients
— India is the largest producer of sugar in the world with a 22% covered are NPK and Sulphur.
share.
— It is the second largest agro-based industry in the country.
Automotive Industry
Statutory Minimum Price (SMP) of sugar is fixed by the — India is the 2nd largest manufacturer of motorcycle and
government on the recommendation of Commission for fifth largest manufacturer of commercial vehicles in the
Agricultural Costs and Prices (CACP) and after consulting the world.
State Governments, Association of Sugar Industry and cane — In 2009, India was the forth largest exporter of
growers. passenger cars after Japan, South Korea and Thailand.
114 Magbook ~ Indian Economy

— Automotive industry was delicensed in July 1991, however, — It may be noted that while macroeconomic
passenger cars industry was delicensed in 1993. India is the stabilisers are short-term measures to correct overall
largest manufactures of tractor in the world. imbalances in the system, microeconomic
— India is the 9th largest car manufacturer in the world. adjustments are long-term measures aiming at
improving the level of efficiency and productivity in
Industrial Growth different sectors of the economy.
— As per recently released national accounts data with 2011-12 as Major Structural Adjustment
the base year, industrial growth was much better in 2012-13 and Programme (SAP) in India
2013-14 at 2.4% and 4.5% respectively. — Industrial deregulation, public sector reforms,
— Growth rate, at the base rate 2011-12, in the year of 2015-16 industrial delicensing, removal of restrictions on
and 2016-17 was 7.4% and 5.2% (up to January, 2017) industrial expansion etc.
respectively. — Public private partnership in infrastructure development
and financial sector reforms.
Economic Reforms — Removal of restraints on inter-state movement of
— Economic reforms refer to all those programmes and policies, foodgrains, the restructuring of the Public
which were introduced by the government as a package of New Distribution System (PDS), relaxation of restrictions
Economic Policy (NEP) to restore the growth process, which by under the Essential Commodities Act, introduction of
1991, was scraping the bottom. There are two parameters of forward trading in most agricultural commodities
economic reforms macroeconomic Stabilisation Policy and and removal of some marketing restrictions on crop
Microeconomic Structural Adjustment Programmes. produce.

Macroeconomic Components of New Economic


Stabilisation Policy Policy/Economic Reforms
— Macroeconomic stabilisation measures refer to those set of — New Economic Policy was announced in July 1991.
measures, which affect the entire economy and are therefore, Main components of new economic policy are
pervasive in nature (spreading across all sectors of the economy). Liberalisation, Privatisation and Globalisation (LPG)
of the economy.
— These measures include review of: (a) Monetary Policy, (b)
Fiscal Policy and (c) Exchange Rate Policy. Reasons for Economic Reforms
— The focus of these measures was to cope with the crises of — Mounting Fiscal Deficit
confidence relating to ability of the government to manage the — Adverse Balance of Payments
country’s dwindling Balance of Payment (BoP) status, particularly — Fall in Foreign Exchange Reserves
its ability to repay the loans taken from the rest of the world. — Rise in Prices
Major Stabilisation Measures in India — International Pressures
— Devaluation of the Indian currency in 1991. — Poor Performances of Public Sector Enterprises
— Move of the exchange rate regime from that of a crawling peg Liberalisation
towards a market determined one, though somewhat managed. — Liberalisation of the economy means freedom of the
— Removal of quantitative restrictions on imports. producing units from direct or physical controls by
— Rationalisation of the tariff structure; reduction in the number of the government.
tariff rates and the peak rate of tariff has been reduced from around Measures Taken for Liberalisation
400% to 12.5% for non-agriculture products. — With the exception of 5 industries, industrial
— Current account convertibility with limited capital account licensing has been abolished for all other
convertibility. industries. In 2002, MRTP Act has been
abolished and in its place a much Liberal
Microeconomic Structural Competition Act, 2002 has been enacted.
Adjustment Programmes — Under the policy of liberalisation, industries
(which are not covered under industrial licensing)
— These refer to those measures by the government, which focus are free to expand and produce. They need no prior
on structural changes in the economy and which had specific official approval.
bearing on different sectors of the economy. These measures
— Investment limit of the small industries has been
include reforms in (a) Industrial Policy, (b) Trade Policy, (c)
raised to ` 5 crore so as to enable them to
Public Sector Policy, (d) Price Policy, (e) Tariff Policy etc.
introduce modernisation. Investment limit of tiny
Magbook ~ Indian Industry 115

industries or micro enterprises has also been increased to Measures Adopted for Globalisation
` 25 lakh. — Under economic reforms, limit of foreign capital
— Under the policy of liberalisation, Indian industries will be free investment has been raised. In many industries,
to buy machines and raw materials from abroad in order to foreign direct investment to the extent of 100% has
expand and modernise themselves. been allowed without any restriction and red-tapism.
— Earlier, for regulating foreign exchange transactions, — To achieve the objective of globalisation, partial
government had enacted Foreign Exchange Regulation convertibility of Indian rupee was allowed. It was in
Act—FERA. This act was very restrictive in nature. It involved conformity with economic reforms.
various checks and controls on transactions involving foreign — This convertibility is valid for the following transactions
exchange. — import and export of goods and services;
Privatisation — payment of interest or dividend on investment;
— “Privatisation is the general process of involving the private — remittances to meet family expenses.
sector in the ownership or operation of a state owned — It is called partial convertibility because It does not
enterprise.” It implies parting with government ownership or cover capital transactions.
management of the public sector enterprises. — All restrictions and controls on foreign trade have been
— It may happen in two ways: removed. Open competition has been encouraged.
(i) outright sale of the government enterprises to the private Administrative controls have also been minimised.
entrepreneurs or. — Custom duties and tariffs imposed on imports and
(ii) withdrawal of the government ownership and exports are being reduced gradually.
management from the mixed enterprises (the
enterprises jointly owned and managed by the
government and the private entrepreneurs).
Make in India
— Make in India compaign aims at reviving the job
Measures Adopted for Privatisation creating manufacturing sector, which is being seen as
— Number of industries reserved for the public sector has been the key to taking the Indian economy on a sustainable
reduced from 17 to 3 only. high growth path. It is being seen as the key to taking
— Public sector industrial units are treated in the same way as the Indian Economy on a sustainable high path. It
sick industries of private sector. In this respect, Sick Industrial aims to take manufacturing growth to 10% on a
Companies Act, 1985, has been amended in December 1991. sustainable basis.
— With a view to improve the working of public sector — Some of the important features of make in India are as
enterprises, a system of MoU has been introduced. Under it, follows:
management of public sector enterprises will be given more — It aims to attract foreign companies to set-up factories in
freedom and they will be accountable for the results. India and invest in the country’s infrastructure.
— National Renewal Fund was established for protecting the — It aims to transform the economy from the services-driven
interest of employees on account of privatisation. The growth model to labour-intensive manufacturing-drive
employees were offered Voluntary Retirement under this growth. This is expected to create over 10 million new
jobs annually.
scheme. Upto March 2009, more than 6 lakh employees
had sought voluntary retirement from public sector units. — 25 key sectors have been identified, in which India has
the potential of becoming a world leader. These include
This fund is even used for providing social security
automobiles, chemicals, IT, pharmaceuticals, textiles,
measures to retrenched employees of PSUs.
ports, aviation, leather, tourism and hospitality, wellness,
Globalisation railways etc.
— Globalisation means integrating the economy of a country with — A dedicated new portal (www.makeinindia. com )has been
especially created to answer queries from business
the economies of other countries in an environment of free flow
entities.
of goods and services across the borders.
— Top executives from 3000 top companies and
— It is a process, associated with increasing openness, corporations world wide were invited to be part of the
growing economic inter-dependence and deepening campaign.
economic integration with the world economy. — Indian embassies around the world also became part of
— Owing to globalisation, it was expected that capital and this global campaign.
technology will flow from developed countries of the world into — The Department of Industrial Policy and Promotion (DIPP)
India. Accordingly, India would have access to the fruits of constituted an eight-member expert panel to redress
global growth. grievances and queries and handle queries of global and
domestic investors within 24 hours.
116 Magbook ~ Indian Economy

Start-up India Mission


Digital India — Start-up India campaign is based on an action plan
— Digital India aimed at transforming the country into 4 aimed at promoting bank financing for start-up
digitally empowered social and knowledge economy, as well ventures to boost entrepreneurship and encourage
as to revive the State of Governance in the country. start ups with jobs creation.
— It is an ‘umbrella programme’ weaving together many existing — The campaign was first announced by Prime Minister
schemes under multiple ministries and departments to Narendra Modi in his 15th August, 2015 address from
ensure that it’s services are available to citizens electronically. the Red Fort. The mission was launched on 16th
— In order to achieve to above objective, several projects January, 2016.
products were already launched by the Government and — It is focused on to restrict role of states in policy
Public Sector Enterprises or ready to be launched as domain and to get rid of ‘license raj’ and hindrances
indicated below. like in land permissions, foreign investment proposal,
— Digital Locker System aims to minimise the usage of physical environmental clearances. It was organised by
documents and enable sharing of e-documents across Department of Industrial Policy and Promotion (DIPP).
agencies. — The key points of Start-up India Mission are as follows
— Swachh Bharat Mission (SBM) mobile app would be used by — Single Window Clearance even with the help of a mobile
people and government organisations for achieving the goals application
of Swachh Bharat Mission. — ` 10,000 crore fund of funds
— e-Sign framework would allow citizens to digitally sign a — 80% reduction in patent registration fee
document online using Aadhaar authentication. The Online — Modified and more friendly Bankruptcy Code to ensure 90
Registration System (ORS) under the e-Hospital application days exit window
has been introduced. — Freedom from mystifying inspections for 3 years
— Digitise India Platform (DIP) for large-scale digitisation of — Freedom from Capital Gain Tax for 3 years
records in the country that would facilitate efficient delivery of — Freedom from tax in profits for 3 years
services to the citizens. Bharat Net, a high speed digital — Starting with 5 lakh schools to target 10 lakh children for
highway to connect all 2.5 lakh Gram Panchayats of country. innovation programme
— Centre of Excellence on Internet on Things (IoT) is a joint
Stand-up India Scheme
initiative of Department of Electronics and Information
— Stand-up India scheme facilitates loans between ` 10
Technology (Deity), ERNET and NASSCOM.
lakh and ` 1 crore to at least one Scheduled Caste or
— The BSNL has introduced Next Generation Network (NGN) to Scheduled Tribe borrower and at least one woman
replace 30 years old exchanges. borrower per bank branch for setting up a Greenfield
enterprise. This enterprise may be in manufacturing,
Startup India - Standup India services or trading.
— Startup India compaign has been launched on 16th January, — In case of non-enterprise individual, at least services or
2016 by the honourable Prime Minister of India, Narendra trading and in case of non-enterprise individual, at
Modi. The Startup India, Standup India initiative and unveil least 51% of share holding and controlling stake
the government’s actions plans to support early stage should be held by either an SC/ST or women
startups. entrepreneur. Prime Minister launched the ‘Stand-up
— Startup India, Standup India celebrates the entrepreneurship India Scheme’ and a web portal for the scheme on 6th
spirit of country's youth and those that are using technology January, 2016 at sector 62, Noida.
to change the way we solve problems in India across — The key points of Stand-up India Scheme are as
industries, from medicine and sports to education and the follows
environment. — Composite loan between ` 10 lakh and ` 1 crore, inclusive
of working capital component for setting up any enterprise.
Aim of Scheme Debit Card (Rupay) for drawal of working capital.
— Credit history of borrower to be developed. Refinance
— Organised by Department of Industrial Policy and Promotion window through Small Industries Development Bank of
(DIPP), along with other key Indian startup ecosystem India (SIDBI) with an initial amount of ` 10,000 crore.
players, the Startup India, Standup India initiative aims to Creation of a corpus of ` 5,000 crore for credit guarantee
celebrate the country's entrepreneurial spirit, and create a through National Credit Guarantee Trustee Company
strong ecosystem for fostering innovation and startups in (NCGTC).
India.
Self Check
Build Your Confidence
1. Which one of the following committees recommended 7. Consider the following statements
the abolition of reservation of item of small-scale sector 1. Startup India initiative aims to celebrate the country’s
in industry? [UPPCS 2006] entrepreneurial spirit.
(a) Abid Hussain Committee 2. Income tax has been exempted for 3 years in Startup
(b) Narsimhan Committee India Scheme.
(c) Nayak Committee Which of the statement(s) given above is/are correct?
(d) Rakesh Mohan Committee (a) Only 1 (b) Only 2
2. Which one of the following Industrial Policies has (c) Both 1 and 2 (d) Neither 1 nor 2
abolished (with a few exception) the Industrial 8. Consider the following statements
Licensing? 1. World Investment Report is published by World Bank.
(a) Industrial Policy, 1970 (b) Industrial Policy, 1980 2. Bokaro Steel Limited was established with the
(c) Industrial Policy, 1991 (d) Industrial Policy, 1985 assistance of Soviet Union.
3. A labour intensive industry is one that [UPPCS 2006] Which of the statement given above is/are correct?
(a) require hard manual labour (a) Only 1 (b) Only 2
(b) pay adequate wages to the labour (c) Both 1 and 2 (d) Neither 1 nor 2
(c) employs more hands 9. Consider the following statements
(d) provide facilities to labour
1. The first cement industry in India was the Indian Cement
4. Why is Government of India disinvesting its equity in the Company Limited.
central public sector enterprises? 2. The per capita consumption of cement in India is one of
1. The government intends to use the revenue earned from the highest in world.
the disinvestment mainly to pay back the external debt. Which of the statement(s) given above is/are correct?
2. The government no longer intends to retain the (a) Only 1 (b) Only 2
management control of the CPSEs. (c) Both 1 and 2 (d) Neither 1 nor 2
Select the correct answer using the codes given below 10. Consider the following statements
(a) Only 1 (b) Only 2
1. India had no iron and steel industry at independence.
(c) Both 1 and 2 (d) Neither 1 nor 2
2. The first iron and steel industry in independent India
5. What is/are the recent policy initiatives of Government was set-up in Raurkela.
of India to promote the growth of manufacturing sector? Which of the statement(s) given above is/are correct?
1. Setting-up National Investment and Manufacturing (a) Only 1 (b) Only 2
Zones. (c) Both 1 and 2 (d) Neither 1 nor 2
2. Providing benefits of single window clearance. 11. The Companies Act, 2013 introduces a new type of entity
3. Establishing the Technology Acquisition and to existing list i.e. apart from forming a public or private
Development Fund. limited company. The 2013 Act enables the formation of
Select the correct answer using the codes given below a new entity is
(a) 2 and 3 (b) 1 and 3 (a) one person company
(c) 1 and 2 (d) All of these (b) two person company
6. Consider the following statements (c) more than five person company
(d) None of the above
1. Privatisation of public sector units occurs, when
government sells 5% of its share. 12. In the ‘Index of Core Industries’, which one of the
2. Abid Hussain Committee recommended the abolition of following is given the highest weight overlapping?
reservation of items of small-scale sector in industry. (a) Coal Production [UPSC 2015]
Which of the statement(s) given above is/are correct? (b) Electricity Generation
(a) Only 1 (b) Only 2 (c) Fertilizers Production
(c) Both 1 and 2 (d) Neither 1 nor 2 (d) Steel Production

1. (a) 2. (c) 3. (c) 4. (d) 5. (d) 6. (b) 7. (c) 8. (b) 9. (c) 10. (a)
11. (a) 12. (b)
Chapter thirteen
Services Sector
Among the top 15 countries with highest overall
Current Scenario —
GDP in 2013, India ranked 9th in overall GDP
‘‘The phenomenal — India’s dynamic services sector has and 10th in terms of services GVA in 2014.
expansion of services grown rapidly in the last decade. — Among the top 15 nations in the period of 2001
According to Economic Survey to 2013, the maximum services share to GVA
worldwide led to the 2017-18, the Services Sector, with a was recorded by Spain (9.7% points), followed
services being regarded share of 54.17% in India’s Gross by India (7.8% points) and China (6.8%
as an engine of growth Value Added (GVA), continued to be points).
the key driver of India’s economic
and even as a necessary growth contributing almost 72.5% of Share of Services in Income and
concomitant of GVA growth in 2017-18 and Employment
economic growth.’’ employed 28.6 per cent of the total — While the share of services in employment for
population. Net services exports from many developed countries is very high and in
India grew 14.98 per cent many cases higher than the share of services in
year-on-year to US$ 77,562.89 income, the gap between these shares is
million in 2017-18. relatively less. Except China and India, all the
other BRICS countries also have a similar
Composition of —
pattern.
In the Indian and Chinese cases, there is a
Service Sector wide gap between the two, with gap being
wider for India. China’s share of services in
— While the growth of Service Sector as
both income and employment is relatively low
a whole is expected to be at 8.3% in
due to the domination of the industrial sector,
2017-18. India’s services sector
but the gap is also narrower than that of India.
covers a wide variety of activities
— For instance, share of services in overall GDP
such as trade, hotel and restaurants,
(at factor cost at 2004-05 prices) rose from
transport, storage and
35.9% in 2001 to 50.9% in 2010.
communication, financing,
insurance, real estate, business
— Of the 15 countries, in the last 13-years period
services, community, social and between 2001 and 2014, China had the
personal services, and services highest increase in the share of services
associated with construction. employment (34.4% points) followed by Brazil
(17.2% points) and Spain (14.3% points). For
International India, the increase was by only 4.7% points.

Comparison
— In world GDP of US $ 74.0 trillion in
Services Sectors of
2014, the share of services was India
66%, more or less the same as in — India’s services sectors has emerged as a
2001. In the last 13 years, the share prominent sector in terms of its contribution to
of service in world GDP has declined national and states incomes, trade flows, FDI
by 2.7% points. inflows and employment.
Magbook ~ Services Sector 119

The years, India’s overall and services growth rates


—
have outpaced those of the world. Some Major Services of India
— This sector has been pulling up the growth of the — The service sector is a vital cog in the wheel of Indian economy.
Indian economy with a great amount of stability. The sector accounting for 55.2% of the GVA. The government has
— As per the new method of India’s National Accounts taken many initiatives in the different services which include
statistics, the service sector accounting for 52.9% of digitisation, e-visas, infrastructure status to Logistics, Start-up India,
India’s Gross Value Added (GVA) at basic prices Schemes for the housing sector, etc. which could give a further
(current prices) in 2015-16. fillip to the Services Sector. Major Services’ Sector-wise
performance and some recent government policies to boost the
Growth of India’s Services Sector (GVA at Basic growth of the sector are as follows:
Price)
Tourism
2015-16 2016-17
India’s Tourism sector has been performing well with Foreign Tourist
Total Services 9.7 7.7 Arrivals (FTAs) growing by 9.7% to 8.8 million and Foreign Exchange
Trade, repair, hotels and 10.5 Earnings (FEEs) at 8.8% to US$ 22.9 billion in 2016. FTAs during
restaurants 2017 were 10.2 million, with a growth of 15.6%. FTAs on e-Tourist
Financial Services 6.8 5.7 Visa grew by 143% to 10.8 lakh in 2016, and further grew by 57.2% to
Construction 5.0 1.7 17.0 lakh during 2017. Domestic tourist visits grew by 12.7% in 2016.
Tamil Nadu, Uttar Pradesh, Andhra Pradesh, Madhya Pradesh and
Total GVA/GFC 100.0 (7.9) 100.0 (7.1)
Karnataka were the Top 5 Destination States in 2016.

FDI in India’s Services Sector Various initiatives have been taken by the government to promote
tourism include
— Though there is ambiguity in the classification of FDI
in services, it is the combined FDI share of the top — the introduction of the e-Visa facility under three categories of
10 service sectors such as financial and Tourist,
non-financial services falling under the Department — Medical and Business for the citizens of 163 countries;
of Industrial Policy and Promotion (DIPP)’s services — launch of Global Media Campaign for 2017-18 on various
sectors definition; as well as telecommunications; Channels;
trading; computer hardware and software; — launch of ‘The Heritage Trails’ to promote the World Heritage
construction; hotels and tourism; hospital and Sites in India;
diagnostic centres; consultancy services; sea
— launch of International Media Campaign on various international
transport; and infromation and broadcasting that can
TV channels;
be taken as the best estimate of services FDI. The
share of these services is 56.6 per cent of the — Celebration of ‘Paryatan Parv’ having 3 components namely
cumulative FDI equity inflows during the period April ‘Dekho Apna Desh’ to encourage Indians to visit their own
2000-October 2017 and 65.8 per cent of FDI equity country,
inflows during 2017-18 (April-October). — ‘Tourism for All’ with tourism events at sites across all states in
the country, and
Two Phases of Growth Rate — ‘Tourism and Governance’ with interactive sessions and
of Exports of Services — workshops with stakeholders on varied themes.
The growth rates exports services of India and the world Hotels and Restaurants
show two distinct phases, the first till 1996, when the
two growths had a scissor-like movement and the — Tourism is a major engine of economic growth, an important
second phase after 1996, when the growth of India’s source of foreign exchange earnings and a generator of
services exports was higher than that of the world in employment of diverse kinds in many countries including
almost all the years except 2009. In this second phase, India.
the former was much above the latter in upswings, but — According to the World Travel and Tourism Council (WTTC), the
almost converged with the latter during downswings. total contribution of travel and tourism to world GDP was US $
7.6 trillion (9.8% of GDP) in 2014.
120 Magbook ~ Indian Economy

— The latest World Tourism Barometer of the United Nation's exports grew by 7.6% to US$ 116.1bn from US$ 107.8 billion
World Tourism Organisation (UNWTO) (January, 2016 during the same period. E-commerce market is estimated at
edition) also shows that international tourist arrivals US$ 33bn, with a 19.1% growth in 2016-17. To further
reached 1.2 billion in 2015, a 4.4% increase over the promote this sector, many initiatives have been taken, which
previous year, and for 2016 the forecast is a 3.5-4.5%
include
increase.
— India’s share in ITAs is a paltry 0.7% compared to 7.4% of
— the establishment of BPO Promotion and Common
France, 6.6% of the USA, 5.7% of Spain and 4.9% of Services Centres to help create digital inclusion and
China. equitable growth and provide employment to 1.45 lakh
persons, mostly in the small towns;
— The top five states in domestic tourist visits in 2014 were
(1) Tamil Nadu (327.6 million), (2) Uttar Pradesh (182.8
— setting up a separate Northeast BPO Promotion Scheme
million), (3) Karnataka (118.3 million), (4) Maharashtra with 5000 seats and having employment potential of
(94.1 million) and (5) Andhra Pradesh (93.3 million). 15000 persons;
— preparing the draft Open Data Protection Policy law;
Transport Related Services — besides long-term initiatives like Digital India, Make in
India, Smart Citites, e-Governance, push for digital talent
Some transport related services are as follows:
through skill India, drive towards a cashless economy
Shipping and efforts to kindle innovation through Start-up India.
— Around 95% of India’s trade by volume and 68% in terms
of value is transported by sea. As on 30th November, Real Estate
2015, India had a fleet strength of 1246 ships with Dead The Indian Real Estate sector has begun to show signs of
Weight Tonnage (DWT) of 15.37 million (10.45 million GT) improvement with the total FDI of US$ 257 mn in (First Half)
including Indian controlled tonnage, with the public sector H1 2017, which is more than double the total FDI in 2016 full
Shipping Corporation of India (SCI) having the largest
year. Some of the recent reforms and policies taken by the
share" of around 36%.
Government of India related to Real Estate Sector include –
— As per UNCTAD, India with 11.7 million Twenty-foot
Equivalent Units (TEUs) of container and a world share of — the Pradhan Mantri Awas Yojana (PMAY) with the
1.7%, ranked ninth in 2014 among developing countries in government sanctioning over 3.1 million houses for the
terms of containership operations. affordable housing segment in urban regions. Of this,
about 1.6 million houses have been grounded and are at
Port Services various stages of construction, and about 0.4 million
— Port services are closely connected to shipping services houses have built under the mission.
and merchandise trade. The performance of the latter two — PPP policy for affordable housing was also announced on
is also dependent on the efficiency of ports. 21st September, 2017 for affordable housing segment to
— The Maritime Agenda 2010-20, covers some of these provide further impetus to the ambitious ‘Housing for all
issues like full mechanisation of cargo handling and by 2022’ mission.
movements, having draft of not less than 14 m in major — Credit Linked Subsidy Scheme (CLSS) under PMAY was
ports and 17 m in hub ports and shifting of transshipment extended to the Middle Income Group (MIG) segment,
of Indian containers from foreign ports to Indian ports. which got included in the scheme from 1st January,
— The outlay for shipping sector in 12th Plan includes ` 6960 2017.
crore as global budgetary scheme and ` 21990 crore as — With the enactment of Real Estate (Regulation and
Internal and Extra Budgetary Resources (IEBR). Development) Act, 2016, it is anticipated that
accountability would lead to higher growth across the real
Business Services estate value chain, while compulsory disclosures and
— Business services include services like computer- related registrations would ensure transparency.
services, research and development, accounting services
and legal services and renting of machinery in order of Research and Development
importance (shares) as per India’s National Accounts. — The professional Scientific and Technical activities which
include R & D services grew by 17.5% and 41.1 in
IT-BPM 2014-15 and 2015-16 respectively. India-based R&D
India’s Information Technology – Business Process services companies, which account for almost 22% of
the global market, grew at 12.7%. However, India’s gross
Management (IT-BPM) industry grew by 8.1% in 2016-17 to
expenditure on R & D has been at around 1% of GDP.
US$ 139.9 billion (excluding e-commerce and hardware) from India ranks 60th out of 127 on the Global Innovation
US$ 129.4 billion in 2015-16, as per NASSCOM data. IT-BPM Index (Gll) 2017, improving from 66th rank in 2016.
Magbook ~ Services Sector 121
— Buoyed by the government’s support which includes important — This would also need relaxation in some domestic
schemes of different Ministries/Departments, the R&D sector in regulations and obtaining due recognition to Indian
India is all set to witness robust growth in the coming years. qualifications through Mutual Recognition
According to a study, engineering R&D market in India is Agreements (MRAs). As with legal services, FDI in
estimated to grow at a CAGR of 14% to reach US$ 42 billion accounting services will help to improve the
by 2020. India is also expected to witness strong growth in its competitiveness of the Indian market and link it
agriculture and pharmaceutical sectors as the government is better to global markets.
investing large sums to set up dedicated research centres for
R&D in these sectors. Communication Services
Space Telecom and Related Services
— Telecom services is another sunrise sector, in which
— In the case of Satellite Launching, as on March 2017, PSLV India has made a mark with the second largest
successfully launched 254 satellites. Foreign exchange telephone network in the world, after only China.
earnings of India from export of satellite launch increased
— 12th Plan targets for the telecommunication sector–
noticeably in 2015-16 and 2016-17 to ` 394 crore from ` 149
Provision of 1200 million connections by 2017,
crore in 2014-15.
Broad band connection of 175 million by 2017,
— India’s share in global satellite launch services revenue has Mobile access to all villages and increase rural
also increased to 1.1% in 2015-16 from 0.3% in 2014-15. teledensity to 70% by 2017, To increase domestic
Antrix foresees greater utilisation of PSLV, GSLV and manufactured products in telecom network to the
GSLV-Mk-III launch services by the international community for extent of 60% with value addition of 45% by 2017.
launching their Low Earth Orbit (LEO) satellites.
Postal Services
Legal Services — India Post is the largest postal network in the world
— Legal services have been growing at a steady rate of 8.2% in and provides access to affordable postal services to
each of the years from 2005-06 to 2011-12. The Indian legal all citizens in the country through its vast network.
profession today consists of approximately 1.2 million Out of 1.55 lakh post.offices, 1.39 lakh are in rural
registered advocates. areas and the remaining 15736 in urban areas.
— Though, India’s rankings are better than most of the South Asian — The Department of Posts plays a crucial role in
and some South-East Asian countries in all the three parameters, disbursing wages to Mahatma Gandhi National Rural
there is a need for further improvement particularly in speeding up Employment Guarantee Scheme (MGNREGS)
disposal of cases. beneficiaries. Nearly 6.92 crore MGNREGS accounts
have been opened in post offices up to December,
— The National Legal Services Authority (NALSA) has been
2015.
constituted under the Legal Services Authorities Act, 1987, to
monitor and evaluate implementation of legal aid programmes and — Towards financial inclusion, the number of Post
to lay down policies and principles for making legal services Office Savings Bank (POSB) accounts has increased
available under the act. from 30.86 crore to 33.97 crore and total deposits in
POSB accounts and cash certificates to ` 6.53 lakh
Accounting and Audit Services crore in the last one year.
— The accountancy service providers in India are self-regulated — More than 80 lakh Sukanya Samriddhi Yojana
through a combination of statutory bodies like the Institute of accounts have been opened with a cumulative
Chartered Accountants of India (ICAI), the Institute of Cost and investment of more than ` 2900 crore since the
Work Accountants of India and the Institute of Company launch of the scheme on 22nd January, 2015.
Secretaries of India (ICSI). — More than 1.84 crore Kisan Vikas Patras have been
— Tapping the outsourcing market of the US and other developing sold, attracting an investment of more than ` 16,429
countries in niche areas like actuarial and accountancy crore since launch on 18th November, 2014.
services would depend on the availability of high quality experts — The Pradhan Mantri Suraksha Bima Yojana, Pradhan
in tax, insurance and pension laws of the US and other Mantri Jeewan Jyoti Yojana and Atal Pension Yojana
countries and encouraging setting up of back offices of foreign were rolled out for POSB account holders in Core
firms in India. Tie-ups of domestic firms with foreign firms can Banking Solution (CBS) post offices. So, far more
help to gain expertise and markets, which would otherwise not than 1,35,000 policies have been sold to POSB
be individually available for small domestic accountancy firms. customers.
122 Magbook ~ Indian Economy

— The scheme will include the rural and urban BPL family.
India’s Service Trade ` 1600 will be sent to Pradhan Mantri Jan Dhan Yojana
— India’s share in global exports of commercial services bank as subsidies.
increased to 3.2% in 2013 from 1.2% in 2000. Its ranking
among the leading exporters in 2013 was 6th. Power Tex India Scheme
— The Union Ministry of Textiles has launched Power Tex
— In the 2016, trade of services grew by 11.39%.
India on 3 April 2017, a 3-year comprehensive scheme
for Powerloom Sector Development. The Power Tex India
Challenges of Service Sector Scheme aims to boost common infrastructure and
modernisation of the powerloom sector in the country.
— The immediate challenge for the services sector covering
Power Tex India scheme comprises new research and
myriad activities and areas is growth revival. India’s growth
development in powerloom textiles, new markets,
has been basically a services led growth pulling up overall
branding, subsidies and welfare schemes for the
growth of the economy.
workers.
— While this could be through a business as usual approach,
— Components of the scheme are
a more targeted approach with focus on big-ticket services
— Pradhan Mantri Credit Scheme (PMCS) for powerloom
could lead to exponential gains for the economy. While
weavers.
software and telecom services have led by example, there
— Solar energy Scheme (SEC) for powerlooms.
are some other important services like tourism including
medical tourism and shipping and logistics. — In-situ Upgradation of Plain Powerlooms.
— With world tourist arrivals expected to increase by 43 million — Group Workshed Scheme (GSW).
every year on an average from 2010 to 2030 and FTAs in — Yarn Bank Scheme.
emerging countries expected to grow faster than in advanced — Common Facility Centre (CFC).
economies, a goldmine of opportunity in tourism is waiting — Facilitation, IT, Awareness, Market Development and
for India, which at present has a paltry share of 0.64% in Publicity for Powerloom Schemes,
world tourist arrivals. — Tex Venture Capital Fund.
— Grant-in-Aid and Modernisation. Upgradation of Powerloom
Major Schemes Service Centres (PSCs).

Pradhan Mantri Ujjwal Yojana


— Prime Minister Narendra Modi has launched Pradhan
Saubhagya Yojana
Mantri Ujjwal Yojana on 1st May, 2016 (Labour Day) at Central Government has launched Saubhagya – Pradhan
Ballia (UP) by providing cooking gas connections to 10 Mantri Sahaj Bijli Har Ghar Yojana on 27 September , 2017 for
women. electrifying all the households in rural and urban areas which
— The objective of the scheme is to provide cooking gas are still living without power. The main purpose of Pradhan
connections to 5 million beneficiaries below the poverty line Mantri Sahaj Bijli Har Ghar Yojana – Saubhagya is the
in the next 3 years (till the year 2019). Main objectives of electrification of a large number of rural households which will
this policy are help in the manufacturing sector, growth of social and
— Free LPG gas connection in the name of the female member. It
economic dividends by pushing the demand for power.
will be a cylinder and regulator.
Self Check
Build Your Confidence

1. Which one of the following sub-sector of the services 6. Which one of the following services is included in the
sector in India has contributed the highest per cent in services sector of Indian Economy? [UPPCS 2008]
the annual growth rate of the GDP? [MPPCS 2000] (a) Transport, storage and communication
(a) Real estate, ownership of dwellings and business (b) Financing, insurance, real estate and business services
services (c) Community, social and personal services
(b) Community, social and personal services (d) All of the above
(c) Transport, storage and communication
(d) Trade, hotels and restaurants
7. Consider the following statements
1. In last decade, India is to China, in developing countries in
2. Consider the following statements the terms of Compounded Annual Growth Rate (CAGR) in
1. Software is one sector, in which India has achieved a services sector.
remarkable global brand identity. 2. In the last decade, Russia had higher Compounded Annual
2. The contribution of the services sector to the national Growth Rate (CAGR) in services sector than India.
economy both in terms of value addition and Which of the statement(s) given above is/are correct?
employment generation is growing over the year. (a) Only 1 (b) Only 2
Which of the statement(s) given above is/are correct? (c) Both 1 and 2 (d) Neither 1 nor 2
(a) Only 1 (b) Only 2
(c) Both 1 and 2 (d) Neither 1 nor 2
8. Which one of the following percentage of Foreign Direct
Investment (FDI) is allowed in the Indian entities publishing
3. Consider the following statements newspapers and periodicals dealing with news and current
1. When a country progresses further the affairs? [IAS 2005]
manufacturing sector takes a back seat and give a (a) 26% (b) 49%
way to service sector in terms of both output and (c) 74% (d) 100%
employment.
2. When a country progresses further manufacturing
9. Which one of the following service of the Indian Economy
sector become increasingly service sector.
has the highest percentage of share in the services sector
export?
Which of the statement(s) given above is/are correct?
(a) Computer software
(a) Only 1 (b) Only 2
(b) Financial and non-financial
(c) Both 1 and 2 (d) Neither 1 nor 2
(c) Computer hardware
4. Consider the following statements (d) Legal consultancy
1. Computer software has the highest percentage of
10. Consider the following statements regarding IT sector of
share in service sector export in India.
India.
2. Communication services has highest growth rate in
1. India has achieved brand identity in this sector.
service sector in 11th Five Year Plan.
2. Besides it’s impact on growth, it is also a provider of skill
Which of the statement(s) given above is/are correct?
employment both in India and abroad.
(a) Only 1 (b) Only 2
Which of the statement (s) given above is/are correct?
(c) Both 1 and 2 (d) Neither 1 nor 2
(a) Only 1
5. Various initiatives have been taken by the (b) Only 2
government to promote tourism include (c) Both 1 and 2
1. the introduction of e-visa facility under three (d) Neither 1 nor 2
categories of tourism.
11. In the Index of Eight Core Industries, which one of the
2. Medical and business for the citizens of 163 following is has the highest weight?
countries.
(a) Coal production
Which of the statement(s) given above is/are correct? (b) Electricity generation
(a) Only 1 (b) Only 2 (c) Fertilizer production
(c) Both 1 and 2 (d) Neither 1 nor 2 (d) Steel production

1. (d) 2. (c) 3. (c) 4. (c) 5. (c) 6. (d) 7. (a) 8. (a) 9. (a) 10. (c)
11. (d)
Chapter fourteen
Infrastructure
Definition of Infrastructure in
Infrastructure 12th Plan
Infrastructure refers to — The Rakesh Mohan Committee on — The total investment in the
such core elements of infrastructure titled ‘The India infrastructure sector during the 12th
Infrastructure Report’ 1996, listed Five Year Plan, estimated at ` 56.3 lakh
economic and social
electricity, gas, water supply, telecom, crore (approx, US $ 1 trillion), will be
change, which serve as a roads, industrial parks, railways, ports, nearly double that made during the
support system to the airports, urban infrastructure and storage 11th Five Year Plan. This step up in
production activity in the as infrastructure sector. investment will be feasible primarily
economy. Infrastructure is — A clear understanding of what is covered because of enlarged private sector
under the rubric of infrastructure is participation that is envisaged.
broadly categorised as
necessary for policy formulation, setting of — Unbundling of infrastructure projects,
economic infrastructure targets and monitoring projects to ensure Public Private Partnership (PPP) and
and social infrastructure. consistency and comparability in the data more transparent regulatory
Development of collected and reported by various mechanisms have induced private
agencies. investors to increase their participation
infrastructure is a
in infrastructure sectors, their share in
sine-qua non of economic Economic Infrastructure infrastructure investment increased
development. If proper — It refers to all such elements of economic from 22% in 10th Five Year Plan to
attention is not paid to changes (like power, transport and 35% in the 11th Plan and is expected
communication), which serve as a to be about 48% during the 12th Five
the development of
foundation for the process of economic Year Plan, more than half of the
infrastructure, it is likely resources required for infrastructure
growth.
to act as a severe — In the absence of economic would need to come from the public
constraint on the infrastructure, any efficient system of sector, from the government and their
economic development economic growth would remain a distant portals. This would require not only the
possibility. creation of the fiscal space, but also
process of the country. use of a rational pricing policy.
Social Infrastructure
Infrastructure Targets
— It refers to the core elements of social
changes (like schools, colleges, hospitals in the 12th Plan
and nursing homes), which serve as a — Increase investment in infrastructure as
foundation for the process of social a percentage of GDP to 9% by the end
development of a country. Social of 12th Five Year Plan.
development focuses on human resource — Increase the Gross Irrigated Area from
development, implying the development of 90 million hectare to 103 million
skilled personnel as well as healthy and hectare by the end of 12th Five Year
efficient human beings. Plan.
Magbook ~ Infrastructure 125

— Provide electricity to all villages and reduce AT and C * On 24th July, 2007, India’s first infrastructure
(Aggregate Technical and Commercial) losses to 20% by the index series FTSE-IDFC launched. The series is
end of 12th Five Year Plan. aimed at bringing broad range of investors
— Connect all villages with all weather roads by the end of 12th including retail investors into listed companies
Five Year Plan. Upgrade National and State Highways to the involved in the business of infrastructure.
minimum two-lane standard by the end of 12th Five Year * India Infrastructure Project Development Fund
Plan. (IIPDF) was announced in Budget, 2007-08. The
IIPD fund will contribute up to 75% of the
— Complete Eastern and Western dedicated Freight corridors preparatory expenditure in the form of interest free
by the end of 12th Five Year Plan. loans that will be recovered from the successful
— Increase rural teledensity to 70% by the end of 12th Five bidder. It has been set-up in Department of
Year Plan. Ensure 50% of rural population has access to 55 Economic Affairs.
LPCD (litres per capita per day) piped drinking water supply
and 50% of Gram Panchayats achieve the Nirmal Gram Infrastructure Debt Funds (IDFs)
Status by the end of 12th Five Year Plan. — Infrastructure Debt Fund was launched on 5th March,
2012 by the government, though the announcement for
Funding of Infrastructure it was made in the Union Budget 2011-12. It is a $ 2
billion fund to catalyse long-term lending to core
Public Private Partnerships in Infrastructure
sectors.
— The Planning Commission of India (now NITI Aayog) has — IDFs are investment vehicles which can be sponsored
defined PPP in a generic term ‘‘the PPP is a mode of
by commercial banks and NBFCs in India, in which
implementing government programmes/schemes in
domestic/offshon institutional investors, specially
partnership with the private sector. It provides an
insurance and pension fund can invest, through units
opportunity for private sector participation in financing,
and bonds issued by the IDFs.
designing, construction, operation and maintenance of
— The fund would seek to raise debt capital from
public sector programme and projects’’.
domestic and foreign resources in the form of long-term
— PPPs are expected to augment resource availability as well as pension, insurance funds and sovereign wealth funds.
improve efficiency of infrastructure service delivery. Time and
cost over run in construction of PPP projects are also
expected to be lower compared to traditional public Transport System in India
procurement. The government has also been emphasising — Modes of transport include air, rail, road and water.
the need to explore the scope for PPPs in the development Transportation in India is a large and varied sector of
of the social sectors like health and education. the economy.
— Some of the major PPP projects undertaken, thus so far
are Delhi, Mumbai, Hyderabad and Bengaluru airports; 4 Road Transport
ultra-mega power projects, container terminals at Mumbai, — India has the second largest road network in the world.
Chennai and Tuticorin ports. The road network comprises of National Highways
(NHs), State Highways, major/other district roads and
Viability Gap Funding Scheme village/rural roads.
— With a view to support the infrastructure projects, the — The share of road traffic in total traffic has grown from
viability Gap Funding Scheme for support to PPPs in 12% freight and 31.6% passenger traffic in 1950-51, to
infrastructure announced in 2004 and the modalities to an estimated 60% freight and 87% passenger traffic
operationalise it put is place by 2005. during the 10th Five Year Plan period.
— It provides financial support in the form of grants, one time
or deferred to infrastructure project undertaken through National Highways
public private partnership with a view to make them
— National Highways comprise only 2% of the total length
commercially viable.
of roads, but they carry over 40% of the total traffic
— VGF Scheme provides total viability gap funding up to 20% across the country.
of the total project cost.
— Development and maintenance work of NHs is carried
— The government or statutory entity that owns the project out through 3 agencies, viz National Highways Authority
may, if it so decides, provide additional grants out of its of India (NHAI), State Public Works Departments
budget up to further 20% of the total project cost. (PWDs) and Border Road Organisation (BRO).
126 Magbook ~ Indian Economy

National Highway Development Project (NHDP) Bharat Nirman and Rural Roads
— It is the largest highway project undertaken in the country — Rural roads have been identified as one of the six
and is being implemented by NHAI in Phase I to VII. It is a components of Bharat Nirman and has the goal to
project to upgrade, rehabilitate and widen major highways provide all weather road connectivity to all villages with a
in India to a higher standard. population of 1000 (500 in the case of hilly or tribal
— Phase I and II of NHDP Comprises Golden quadrilateral; areas. In effect, Bharat Nirman proposes to provide new
North-South corridor and East-West corridor. connectivity to a total of 54648 habitations. This would
involve construction of 146184 km of rural roads. In
— Golden Quadrilateral Connecting four metropolitan
addition to new connectivity, Bharat Nirman envisages
cities-Delhi, Mumbai, Chennai and Kolkata having an
upgradation/renewal of 194130 km of existing rural roads.
aggregate length of 5846 km.
— The NSEW corridor comprises a length of 7142 km and Setu Bharatam Programme
connects Srinagar (North) to Kanyakumari (South) and — This programme was launched by Prime Minister
Silchar (East) to Porbandar (West). Narendra Modi on 4th March, 2016 at a budget of ` 102
— NHDP is being implemented in all phases except Phase VI billion (US$1.5billion), with an aim to make all national
at present. highways free of railway crossings by 2019.
— Under the project as may as 208 rail over and under
Special Accelerated Road Development bridges (ROBs/RUBs) would be constructed at
Programme for North-Eastern (SARDP-NE) unmanned railway crossings on national highways and
Region more than 1,500 British-era bridges would be widened,
— The SARDP-NE aims at improving the road connectivity to rehabilitated or replaced in a phased manner at a cost
state capitals, district headquarters and remote places of ` 208 billion (US$3.1 billion) and ` 300 billion (US$4.5
the North-East region. This will ensure connectivity of 88 billion), respectively. ` 50,000 crore are to be spent to
district headquarters in the North-Eastern states to build bridges for safe and seamless travel on national
two-lane NHs/two lane state roads. highways.
State Highways
— The State Highways account for nearly 154.5 thousand km Green Highways Policy 2015
or 3.8% of the total length of roads. These roads connect Union government unveiled its Green Highways policy on
the principal cities and resource centres of the states with September 29, 2015. Under it, bushes and trees will be grown
the NHs. along all the highways in a phased manner.
Pradhan Mantri Gram Sadak Yojana (PMGSY) The purpose of this policy is to promote the greening of
— The PMGSY was launched on 25th December, 2000, to highway corridors with the participation of the local community,
provide single all weather connectivity to eligible including local contractors and the local Forest Department.
unconnected habitations having population of 500 persons Thus, the manifest objective of this policy is plantation and the
and above in plain areas and 250 persons and above in latent objective was to generate employment. The investment
hill states, the tribal (Schedule V) areas, desert areas and in the project would be ` 1000 crore this year, calculated as 1%
LWE affected districts as identified by the Ministry of Home of the ` 100000 crore investment in national highway projects
Affairs. It is a 100% centrally sponsored scheme. in the year.
The policy aims at planting trees along all highways. Private
Pradhan Mantri Bharat Jodo Pariyojana (PMBJP)
and government companies along with Non-governmental
— The ` 40000 crore project connecting all the major cities, organisations are participated. The project will be awarded to
not covered by National Highway Development contractors in small stretches of 8 to 10 km as pilot scheme.
Programme, by four-lane highways, named Pradhan
Mantri Bharat Jodo Pariyojana or ‘PM-BJP’, involves
four-laning of 10000 km of road stretches. It tries to link Railways
up major cities to the NHDP highways.
— First rail steamed off in 1853 between Bombay and
Roads for LWE Districts Thane for a distance of 34 km.
— A programme for development of around 1202 km of — Indian Railways is the country’s biggest nationalised
National Highways and 4362 km of State roads in Left enterprise.
Wing Extremism (LWE) affected areas has been taken up. — In terms of route length, Indian Railway system is the
It is stated for completion by March 2015. 4th largest after USA, Russia and China.
Magbook ~ Infrastructure 127

— Of the two main segments, freights and passengers of the (viii) Railtel Corporation of Indian Limited (Rail Tel)
Indian Railways, the freight segment accounts for roughly (ix) Rail Vikas Nigam Limited (RVNL)
two-third of revenues. (x) Dedicated Freight Corridor Corporation of India Limited
— Railway Budget was separated from the Union Budget in (DFCCIL)
the year 1921, on the recommendation of Acworth (xi) Bharat Wagon and Engineering Corporation of India
Committee. Limited (BWEL)
— The railway network is divided into 17 zones. Divisions are (xii) Burn Standard Company Limited (BSCL)
the basic operating units. (xiii) Braithwaite and Company Limited (BCL)
The 17 zones and their respective headquarters are as under: Rolling Stock and their Places
Zonal Railway Headquarters Zonal Railway Headquarters Rolling Stock Places
Central Mumbai North-Western Jaipur Diesel Locomotive Works Varanasi
Eastern Kolkata Southern Chennai
Chittaranjan Locomotive Works Chittaranjan
East-Coast Bhubaneshwar South-Central Secunderabad
Rail Coach Factory Kapurthala
East-Central Hajipur South-Eastern Kolkata
Integral Coach Factory Perambudur (Chennai)
Northern New Delhi South-East Bilaspur
Central Rail Wheel Factory Bengaluru
North-Central Allahabad South-Western Hubli Diesel Loco Modernisation Works Patiala
North-Eastern Gorakhpur Western Mumbai Research, Design and Standard Organisation Lucknow
North-Frontier Maligaon, West-Central Jabalpur
Guwahati Rail Coach Factory (RCF), rolled out the first proto type air
conditioned Double Decker Coach in March, 2010.
Metro Railway Kolkata
Dedicated Freight Corridor (DFC)
Types of Railway Lines — With the present growth rate in GDP of over 8%, the
— There are four gauges in India. These are as follows: Indian railways expects to carry 85 million tonnes of
incremental traffic per year and about 1100 million tonnes
(i) Broad Gauge (1676 mm) This is the widest gauge in
regular use. It is wider than the 1435 mm standard revenue earning freight traffic by the end of 11th Five Year
gauge and also called Indian Gauge. Plan. It has, thus, become necessary to augment the
freight carrying capacity of the railways.
(ii) Metre Gauge (1000 mm) It is one metre broad. About
14500 route km of railway lines are metre gauge in Railways and the 12th Plan
2009. The metre gauge network is especially dense in
— The 12th Five Year Plan (2012-17) has envisaged an
the West.
integrated approach for the transport sector as a whole.
(iii) Narrow Gauge (762 mm). The rationale for the narrow The vision for transport is to be guided by a modal mix
gauges was economy in building the lines. It considered
that will lead to an efficient, sustainable, economic, safe,
that narrow gauge lines would act as feeder lines to the
reliable, environment friendly and regionally balanced
broad and metre gauges. The most well known line is
transport system, in line with the objectives of the plan.
probably the Kalka-Shimla route.
— Indian Railway aims at developing a strategy to build up
(iv) Narrow Gauge (610 mm) A few places in India have even
narrow 2-feet (610 mm) gauge e.g. New Jalpaiguri, the rail network to be part of an effective multi-modal
Darjeeling, Neral Matheran and the Gwalior branch lines. transport system.
5-digit Train Numbers
The Public Undertakings of the Railways
— It came into effect from 20th December, 2010.
— There are thirteen undertakings under the administrative
Accordingly, all the Superfast, Mail/Express Rajdhani,
control of the Ministry of Railways:
Garib Rath passenger trains and others have now uniform
(i) Rail India Technical and Economic Services Limited, 5 digit numbers, i.e. digit 1 has been prefixed to the
(RITES) existing 4 digit numbers.
(ii) Indian Railway Construction (IRCON) International Limited
(iii) Indian Railway Finance Corporation Limited (IRFC) Railway Vision 2020 Highlights
(iv) Container Corporation of India Limited (CONCOR) — It envisages increasing the railway sector’s share in the
(v) Konkan Railway Corporation Limited (KRCL) GDP from the existing level of 1% to about 3% and its
revenue to grow by 10% annually over the next 10 years.
(vi) Mumbai Rail Vikas Nigam Limited (MRVNL)
— Laying of 25000 km of new lines.
(vii) Indian Railway Catering and Tourism Corporation
Limited (IRCTC)
128 Magbook ~ Indian Economy

— Quadrupling of the 6000 km network with segregation Upgradation of Passenger Amenities


of passenger and freight lines. — Adarsh Stations are being developed with basic facilities such
— Electrification of 14000 km rail route. as drinking water, adequate toilets, catering services etc.
— Completion of gauge conversion. — SIMRAN (Satellite Imaging for Rail Navigation) is being
— Upgradation of speed to 160-200 kmph for passenger carried out. The SIMRAN used real time train tracking
train. through GPS and mobile GSM technologies.
— Construction of 2000 km of high speed rail lines. — Passenger Reservation System (PRS), is the largest
passenger network in the world and is thoroughly
Luxury Trains in India computerised.
— Freight Operations Information System (FOIS), gives an
The Buddha Luxury Train in Uttar Pradesh will be started on
account of all demands, number of loads /rakes / trains and
the lines of Palace on Wheels.
their pipeline, etc. FOIS is being implemented in two phases.
Some luxury trains in India.
Phase I (Rake Management System) and Phase II (Terminal

Deccan Odyssey ◆
Golden Chariot Management System).

Fairy Queen ◆
Royal Rajasthan on Wheels
Green Initiatives Taken in the Railways

Maharaja’s Express ◆
Indian Maharaja
— The year 2011-12, has been declared as the Year of Green
Energy’ by the railways. Some of the green initiatives taken
New Initiatives by Indian Railways
by the railways are as follows:
— Kisan Vision Project It is to encourage setting up of — Free supply of 14 lakh CFLs to railway households and phasing
cold storage and temperature-controlled perishable out of incandescent lamps.
cargo centres through PPP model. Logistics based — Regenerative breaking in Mumbai EMUs.
PSUs including the Container Corporation of India — Windmill at Integral Coach Factory, Chennai.
Limited, Central Warehousing Corporation and Central — Production of locos with ‘hotel land converter’.
Rail-side Warehouse Company Limited have been — Use of bio-diesel, CNG and LNG in locos, workshops etc.
asked to provide infrastructure at six Indian Railways
locations under a pilot project-the Kisan Vision Project. High Level Safety Review Committee
— Indian railways is adopting a multi-pronged strategy to (Railways)
provide safer, faster, cleaner and more comfortable — The committee under Dr Anil Kakodkar recently presented its
passenger trains. Seven corridors have been identified report. The total financial implication of the recommendations
for conducting pre-feasibility studies for running over 5 years would be `100000 crore. Its key
high-speed trains at speeds above 350 km/h. These recommendations and observations are as follows:
trains are popularly known as Bullet trains. — Indian Railways is at the brink of collapse unless some concrete
— Link Holfmann Busch (LHB) Coaches are being measures are taken.
inducted in train services including certain important — There is need for independent mechanism for safety regulation. A
Rajdhani and Mail-express trains. statutory Railway Safety Authority needs to be created.
— A Railway Research and Development Council needs to be
— Till December 2012, LHB coaches has been inducted
set-up.
in about 14 Rajdhani, 12 Shatabdi and 11 AC Duranto
— An advanced signaling system should be adopted for the entire
services. route length within 5 years.
— LHB coaches have higher carrying capacity, better — All level crossing (manned and unmanned) should be eliminated
riding comfort, higher speed potential, longer life, over 5 years.
upgraded amenities, provision of control discharge — Switch over from ICF design coaches to LHB design coaches.
toilet system, lower maintenance requirement,
enhanced safety features and aesthetic interiors.
Some Other Committees on
Indian Railways alongwith the Defence Research and
—
Railway Safety
Development Organisation (DRDO) has developed
environment-friendly bio-toilets for its passenger

Shahnawaj Committee (1954)
coaches. ◆
Kunjaru Committee (1962)
— Eight trains are running with 436 bio-toilets. Railways ◆
Wanchu Committee (1968)
has targeted elimination of direct-discharge passenger ◆
Sikari Committee (1978)
coach toilet system by the end of the 13th Five Year ◆
Khanna Committee (1998)
Plan.
Magbook ~ Infrastructure 129

Metro Rail Projects Water Transport


— The metro railway system and service are operational in — There are two kinds of water transport inland water
10 cities in India. These are Kolkata, Delhi, Bengaluru,
transport or river transport and shipping which includes
Gurugram, Mumbai, Chennai, Jaipur, Kochi, Hyderabad
coastal shipping and overseas shipping.
and Lucknow. The Kolkata Metro rail is the oldest (1984)
metro service in the country. Inland Water Transport (IWT)
— The Delhi metro is rapid transit system serving Delhi, — India has over 14500 km of navigable waterways. Inland
Gurgaon, Noida and Ghaziabad in the NCR. The network Water Transport (IWT), has certain inherent advantages
consist of Eight lanes with a total length of 296 Kilometres namely: fuel efficiency, environment friendliness, cost
with 214 stations. effectiveness and decongestion of road etc.
— Delhi metro is being built and operated by the Delhi Metro — Inland Waterways Authority of India (IWAI), was set-up in
Rail Corporation (DMRC), which is a joint company of 1986, for regulation and development of inland waterways for
Government of India and the Government of Delhi. After the purpose of shipping and navigation.
the Phase IV total length of Delhi metro will be 413 km. It — Out of total transport IWT accounts for 32% in Bangladesh,
20% in Germany, 14% in the US, 9% in China and only
is the second oldest metro in India after the Kolkata
0.15% in India.
metro.
— IWT is best suited for the movement of bulk cargo,
— Metro Railways Amendment Act, 2010, with effect from overdivisional cargo and hazardous goods. It is also an
September, 2009, provides an umbrella ‘statutory’ safety environmentally friendly made.
cover for metro rail work in all the metro cities of India.
Shipping
— The act was extended to National Capital Region,
Bengaluru, Mumbai and Chennai metropolitan areas with
— Shipping plays an important role in the transport sector of
effect from 16th October, 2009. India’s economy. Approximately, 95% of the country’s
trade volume (68% in terms of value) is moved by the
— Metro rail projects have also been taken up on PPP basis
sea.
in Mumbai for Versova-Andheri-Ghatkopar (11.07 km),
— India has one of the largest merchant shipping fleet among
Charkhup to Mankhurd via Bandra (31.87 km) and the developing countries and ranks 16th amongst the
Hyderabad metro (71.16 km) with viability gap funding countries with the largest cargo carrying fleet.
support from the Government of India. — The salient features of India’s shipping policy are the
— The first line of the metro in Chennai is opened in June, promotion of national shipping to increase self-reliance in the
2015. In the North, a metro in Jaipur also started in June carriage of the country’s overseas trade and protection of
2015. The long awaited first phase of Mumbai’s metro stakeholder’s interest in EXIM trade.
launched to the public in 2014. — Government nationalised the fiscal regime for Indian shipping
by introducing Tonnage Tax System from the financial year
— In 2015, a new line for Bengaluru’s Metro was launched. 2004-05, in order to provide Indian Shipping Industry a level
Metros in Kochi in Kerala and in Hyderabad is opened in playing field vis-a-vis international shipping companies and
2017. also facilitate the growth of Indian tonnage. India has allowed
— There are also metro systems planned for many other 100% FDI in the shipping sector.
cities like Ahmedabad in Gujarat, Pune in Maharashtra, — India has a long coastline studded with 13 major ports and 200
non-major ports providing congenial and favourable conditions
Kanpur in Uttar Pradesh, Patna in Bihar etc.
for the development of this alternate mode of transport.
Rapid Rail — Major ports are the direct responsibility of the Central
Government while non-major ports are managed and
— National Capital Region Planning Board (NCRPB) created
administered by the State Governments.
a spearate entity – National Capital Region Transport
— India has the 17th largest coastline in the world.
Company (NCRTC) in 2013. Aim of this entity to provide
fast, safe and comfortable rapid transit in NCR. It is Categories of Shipping
possible by Regional Rapid Transit System (RRTS). The Shipping is divided into two categories
RRTS are expected to have design speed of 180 kmph (i) Coastal (ii) Overseas.
with high acceleration and deceleration.
Coastal Shipping
Bullet Train — This includes shipping within the country along the
— Bullet train project is an initiative by Indian Government to coastline. Due to India’s long coastline, coastal shipping
build a high speed bullet train that would connect Mumbai provides an excellent opportunity for an energy efficient,
to Ahmedabad. This project expected to nearly 18.6 billion cheap mode of transport, especially for large and bulky
dollars and should be operational in about seven years. goods.
130 Magbook ~ Indian Economy

Cruise Shipping — 49% FDI is allowed in Scheduled Air Transport


— To promote the growth of shipping, an Inter-Ministerial Services/Domestic Scheduled Passenger Airline under
Steering Committee with Secretary (Shipping) as Chairman, automatic route. But 100% for NRIs.
was constituted in June, 2010. The Steering Committee has — Air India and Indian Airlines were merged to make them
identified five ports namely, Mumbai, Goa, Chennai, more efficient.
Managalore and Cochin for development of cruise tourism. — Greenfield Airports at Bengaluru and Hyderabad were
completed and Delhi and Mumbai airports were
— Tariff Authority for Major Ports (TAMP) was established in restructured.
1997 to determine tariffs for major ports and to specify the
conditionality governing these tariffs. Non-major ports
however, are free to determine their tariffs as these are National Civil Aviation
deregulated. Policy (NCAP) 2015
Minister of Civil Aviation P. Ashok Gajapathi Raju released
Project Green Ports the Revised Draft National Civil Aviation Policy (NCAP 2015)
The Ministry of Shipping has started ‘Project Green Ports’ on in New Delhi on October 30, 2015.
January 19, 2016 which will help in making the Major Ports across
India cleaner and greener. ‘Project Green Ports’ will have two Aims of the Policy
verticals—Green Ports Initiatives related to environmental issues ◆
To provide a conducive environment and a level playing
and Swachh Bharat Abhiyaan.
field to various aviation sub-sectors, i.e Airlines, Airports,
The Green Port Initiatives include twelve initiatives which will be
Cargo, Maintenance Repairs and Overhaul services,
implemented under strict time bound fashion in order to achieve the
General Aviation, Aerospace manufacturing, Skill
targets. Under Swachh Bharat Abhiyaan, the Ministry has identified
Development, etc.
20 activities with certain time-line to promote cleanliness at the
port premises.

To create an ecosystem to enable 30 crore domestic
ticketing by 2022 and 50 crore by 2027. Similarly,
Regular training will be provided to the staff in order to generate international ticketing to increase to 20 crore by 2027.
awareness and inculcate a positive attitude towards keeping the
environment clean and green. All the Major Ports have already
initiated action on the above mentioned activities and are making Power/Energy Sector
good progress. — Energy is the most important component of economic
infrastructure. Even agriculture needs energy to run
tubewells, tractors and thrashers. In fact, energy has
Air Transport (Civil Aviation) become the lifeline of human existence.
— Air transport is the speediest of all means of transport. It is
Commercial and Non-Commercial
particularly useful for transporting costly and light weight
Energies
consignments. Air transport has its special significance at the
time of war and for providing help in natural calamities. — Energy is broadly classified as commercial and
non-commercial energy. Firewood, agricultural waste
— Air transport is also used in spraying pesticides in agricultural
(straw etc) and animal waste (cowdung) are the
fields. India has many favourable features for the
important components of non-commercial energy.
development of air transport, e.g. vast plains facilitating
landing and take-off operations, vast size of the country, — Coal, petroleum products, natural gas and electricity
appropriate climate etc. However, it is a very expensive are the important components of commercial energy.
means of transport. These goods are largely used for commercial purposes
in the factories and farms. They command a price and
— India has around 486 airports of which 34 are designated as
have an established market of sale and purchase.
international airports. 11 international airports are managed
by AAI while 6 are in private hands. Passenger terminal Conventional and Non-Conventional
capacity in all airports put together is around 230-240 million. Sources of Energy
Recent Initiatives to Promote the — Conventional sources of energy are those, which are
Aviation Industry known as to us and which are popularly in use since a
very long time. Examples are coal, petroleum, natural
— 100% FDI is permitted for greenfield airport project under the
gas and electricity. Non-conventional sources of
automatic route.
energy are those sources, which have been discovered
— Up to 74% FDI is permitted for existing airport projects under or explored only in the recent past and which are yet
the automatic route, above 74% and up to 100% permitted to gain popularity for their use. Examples are solar
under government approval route.
Magbook ~ Infrastructure 131

energy, wind energy, biomass etc. Share of different UJALA YOJANA


sources in Total Energy Production (Supply) (in percentage) — It was launched by Union Minister for State (IC) for
Power, Coal and Renewable Energy Piyush Goyal in
Electricity Bhopal Madhya Pradesh on 30th April, 2016.
— Electricity is the most useful and convenient source of energy — The UJALA scheme is being implemented by Energy
in India. Three main sources of electricity in India are as Efficiency Services Limited (EESL), a joint venture of
follows: PSUs under the Union Ministry of Power.
(i) Thermal Power Stations Generation of electricity by — The scheme will help to reduce electricity bills of
using coal is known as Thermoelectricity. It is an consumers, contribute to the energy security of India and
important source of electricity in India, generating nearly also help in environment protection. The main motive of
two third of the total electricity. Now, there are more than this policy is energy efficiency in the country. Consumers
318 thermal power plants in India. can buy the bulbs from distributor by showing any
(ii) Hydroelectricity Stations Generation of electricity using identification card.
water is known as Hydroelectricity. Whatever, water is
available in India for generation of electricity, we are not Urban Flagship Programme
able to utilise 50% of it. The share of hydroelectricity is — Government’s flagship Pradhan Mantri Awas Yojana
only one-fourth of the total electricity generated. Some (PMAY) housing scheme, alongwith its long-planned
important hydroelectric power stations are Koyna, Bhakra Smart Cities Mission and the Atal Mission for
Nangal, Beas, Hirakud, Kosi, Nagarjuna Sagar, Rejuvenation and Urban Transformation (AMRUT) is an
Malaprabha etc. urban renewal initiative started on 25th June, 2015.
(iii) Atomic Power Stations In the modern times, atomic
energy or nuclear energy is used all over the world as a AMRUT for 500 Cities
source of energy. Its sources are uranium and thorium. — The Narendra Modi Government has renewed the 10
It is a new and cheap source of energy. Atomic Energy years old Jawaharlal Nehru National Urban Renewal
Commission of India is engaged in the research of atomic Mission (JNNURM) and named it after the first BJP
energy. Prime Minister. The renewed scheme is known as Atal
— In India, first atomic power station was established at Mission for Rejuvenation and Urban Transformation
Tarapur in Bombay in 1969. Subsequently, many nuclear (AMRUT). AMRUT for 500 Tier 2 and Tier 3 cities will
power plants were established in India including Kalpakkam also be launched alongwith smart city project.
in Tamil Nadu, Narora in Uttar Pradesh, Surat in Gujarat — For AMRUT as well, states have been asked to
etc. recommend cities which can be included under this
— The nuclear power generation has been increasing in India. scheme. Uttar Pradesh leads the pack, as it can
But, the safety parameters of atomic power plants has, now nominate 64 cities under this project. Here is the
breakdown for different states
to be re-devised considering the Fukushima episode. This
— Tamil Nadu (33)
is why, people are now more worried regarding proposed
— Maharashtra (37)
Jaitapur Power Plant in Maharashtra.
— Gujarat (31)
Major Schemes — Karnataka (21)
— Andhra Pradesh (31)
UDAY Scheme — Rajasthan (30)
— The Union Cabinet chaired by Prime Minister Narendra — West Bengal (28)
Modi approved the Ujwal Discom Assurance Yojana (UDAY) — Bihar (27)
scheme on November 5, 2015 that aimed at bailing out — Odisha (19)
debt ridden State-run power distribution companies — Haryana (19)
(discoms). — Kerala (18)
— The financial restructuring package is launched to — Punjab (17)
financially turn around discoms that have been severely — Telangana (15)
indebted and make them strong. — Chhattisgarh (10)
— According to the package, States can take over by March, Under this scheme, Central Government will provide 30% of
2016, 75% of the discoms’ debts up to September, 2015, the overall cost, if the city has a population of more than 10
by issuing special bonds at a price determined by the lakh; and 50% if under 10 lakh.
Reserve Bank at G-Sec rates plus 5%.
132 Magbook ~ Indian Economy

(iv) Subsidy for Individual Beneficiary-led Construction or


100 Smart Cities Enhancement of Houses Under this, a Central assistance
Government of India has announced an ambitious 100 Smart of ` 1.50 lakh would be provided to each eligible urban
Cities programme. Based on the population and area, a fix poor beneficiary to enable him build his own house or
number has been allocated for each state. Hence, Uttar undertake improvements to existing houses.
Pradesh, the most populated state, will get 13 smart cities,
meanwhile Tamil Nadu will receive 12 smart cities. Heritage City Development and
Maharashtra will get 10 cities, while Karnataka and Gujarat
Augmentation Yojana (HRIDAY)
will get 6 each. West Bengal and Rajasthan has been — Union Government has launched a Heritage City
allocated 4 smart cities; Bihar, Andhra Pradesh and Punjab 3 Development and Augmentation Yojana (HRIDAY) scheme to
each and Odisha, Haryana, Telangana and Chhattisgarh will preserve and rejuvenate the rich cultural heritage of the
get 2 smart cities each. Jammu and Kashmir, Kerala, country. Union Governments ambitious ` 500 crore HRIDAY
Jharkhand, Assam, Himachal Pradesh, Goa, Arunachal project was launched by Urban Development Minister
Padesh and Chandigarh, along with National Capital New Venkaiah Naidu in New Delhi. In the initial phase, 12
Delhi will get one smart city each. heritage cities have been identified which will be rejuvenated
and developed under HRIDAY. The 12 cities are—Amritsar,
Pradhan Mantri Awas Yojana Varanasi, Gaya, Puri, Ajmer, Mathura, Dwarka, Badami,
Velankanni, Kanchipuram, Warangal and Amaravati.
‘Housing for All by 2022’
The aims of HRIDAY are as follows:
— A new integrated national housing mission launched by — It aims to bring urban planning, economic growth and heritage
merging UPA flagship scheme and Rajiv Awas Yojana. conservation together for heritage cities.
— The third project is the ‘Housing for All by 2022’ — It also seeks beautification in an inclusive and integrated manner
scheme, wherein more than 2 crore homes would be with focus on cleanliness, livelihoods, skills, safety, security,
build across all the urban locations in the next 7 years. accessibility and faster service delivery of heritage cities.
— The Cabinet, headed by Prime Minister, accepted the — Heritage Management Plan (HMP) will be prepared for the
recommendations of an Inter-Ministerial Committee to identified cities which will outline heritage resources and develop
approve a substantial increase in interest relief on loan policies to guide their conservation, restoration, future use and
development.
for the urban poor to promote affordable homes.
— It will seek to improve last-mile connectivity heritage sites by
— The committee recommended to increase interest documentation, conservation of areas, providing more facilities
subvention to 6.50% on housing loan for economically for women, senior citizens and differently abled citizens.
weaker sections of society. Women, SC/STs and people — HRIDAY will be dovetailed with the Tourism Ministry’s Pilgrimage
from Economically Weaker Section (EWS) would be the Rejuvenation and Spiritual Augmentation Drive (PRASAD)
main beneficiaries of this urban housing project. scheme which has an outlay of ` 100 crore for augmentation of
There are four components to the National Urban infrastructure at pilgrimage sites across the country.
Housing Mission UDAN Scheme
(i) Redevelopment Plan of Slums Under this, with the — The UDAN RCS (Regional Connectivity Scheme) was
participation of private developers using land as a
launched in October, 2016. The PM Narendra Modi has
resource component, every beneficiary would be
launched the ‘Ude Desh ka Aam Nagrik’ (UDAN) Scheme
provided a Central grant of ` 1 lakh on an average.
from Jubbarhatti, an airport on the outskirts of Shimla on 27
(ii) Affordable Housing through Credit-linked Subsidy
April, 2017.
Scheme Under this, an interest subsidy of 6.50% on
each housing loan to Economically Weaker Section — As per the modalities of the scheme, airfare for a one-hour
(EWS) and Low Income Groups (LIG) beneficiaries journey of 500 km has been capped at an all-inclusive
would be provided by the Central Government. charge of ` 2500.
(iii) Affordable Housing in Partnership with Private and — The scheme has been launched to provide connectivity to
Public Sectors Under this, the Central assistance of un-served and under-served airports of the country through
` 1.50 lakh to each beneficiary would be provided to revival of existing air-served airports of the country through
promote housing stock for urban poor with the revival of existing air-strips and airports.
involvement of private and public sectors. Provided — First flight under this scheme was on the route of Shimla to
35% of houses of the projects are proposed to be Delhi.
earmarked for EWS category.
Self Check
Build Your Confidence

1. Among other things which one of the following was the 6. With what purpose is the Government of India promoting
purpose for which Deepak Parekh Committee was the concept of ‘Mega Food Park'? [UPSC 2011]
constituted? [IAS 2009] 1. To provide good infrastructure facilities for the food
(a) To study the current socio-economic conditions of processing industry.
certain minority communities 2. To increase the processing of perishable items and
(b) To suggest measures for financing the development of reduce wastage.
infrastructure 3. To provide emerging and eco-friendly food processing
(c) To frame a policy on the genetically modified organisms
technologies to entrepreneurs.
(d) To suggest measures to reduce the fiscal deficit in the
Union budget Select the correct answer using the codes given below
(a) Only 1 (b) 1 and 2
2. There has been a persistent deficit budget year after (c) 2 and 3 (d) All of these
year. Which of the following actions can be taken by the
government to reduce the deficit? 7. Name the scheme under which accidental death
1. Reducing revenue expenditure insurance cover for up to ` 2 lakh will be provided to the
2. Introducing new welfare schemes people of age group of 18-70 years?
3. Rationalising subsidies (a) Atal Jeevan Bima Yojana
4. Expanding industries (b) Pradhan Mantri Suraksha Bima Yojana
(c) Pradhan Mantri Jeevan Jyoti Bima Yojana
Select the correct answer using the codes given below:
(d) Atal Pension Yojana
(a) 1 and 3
(b) 2 and 3 8. Consider the following statements
(c) Only 1 1. Golden Quadrilateral is a National Highway
(d) 1, 2, 3 and 4 Development Project connecting Delhi, Pune, Chennai
3. Consider the following components and Kolkata.
1. Bring an additional one crore hectares under assured 2. North-South Corridor which comprises 4-laning of
irrigation. National Highways connecting-Salem.
2. To provide road connectivity to all villages that has a Which of the statement(s) given above is/are correct?
population of 2000. (a) Only 1 (b) Only 2
3. To give telephone connectivity to the remaining villages. (c) Both 1 and 2 (d) Neither 1 nor 2
Which of the component(s) given above is/are included in 9. Which kind of the power accounts for the largest share
the in the Bharat Nirman Scheme? of power generation in India? [UPPCS 2008]
(a) 1, 2 and 3 (b) 1 and 3 (a) Nuclear
(c) 2 and 3 (d) Only 2 (b) Hydro-electricity
4. Which one of the following is not a nuclear power (c) Thermal
centre? (d) Solar
(a) Narora (b) Kakrapara 10. Which one of the following statement is correct?
(c) Chamera (d) Kota [UPPCS 2008]
5. Ten year old JNNURM Scheme named as (a) Singrsauli mines are located in Andhra Pradesh
(a) AMRUT (b) In India, majority of domestic coal supply comes from
(b) Housing for all open cost mines
(c) HRIDAY (c) Reliance is the only private sector refinery in India
(d) UDAN Scheme (d) None of the above
134 Magbook ~ Indian Economy

11. Oil shale is obtained from 13. Ude Desh ke Aam Nagrik (UDAN) is a regional connectivity
(a) coal bed methane scheme launched in
(b) metamorphic rocks containing clathrate hydrates (a) 2014
(c) sedimentary rocks containing kerogen (b) 2015
(d) coastal regions (c) 2016
(d) 2017
12. Ujala Scheme launched for
(a) Power 14. Component(s) of the National Urban Housing Mission is/are
(b) Coal (a) Affordable Housing through Credit Linked Susidy Scheme
(c) Renewable energy (b) Redevelopment plan of slums
(d) All of the above (c) Affordable housing in partnership with private and public sector
(d) All of the above

1. (b) 2. (d) 3. (b) 4. (c) 5. (a) 6. (a) 7. (b) 8. (b) 9. (c) 10. (b)
11. (c) 12. (d) 13. (d) 14. (d)
Chapter fifteen
Poverty and Unemployment
In India, Planning Commission is of the
Poverty opinion that an individual in rural area must
get 2400 Kilo calories and in urban area,
— Poverty is a social phenomenon,
“Goal of sustained 2100 calories per day.
wherein a section of society is unable to
poverty reduction cannot fulfill even its basic necessities of life. — Minimum Consumption Expenditure Criteria
An Expert Committee was appointed in
be achieved unless — The UN Human Rights Council has 1962, by the Planning Commission to
equality of opportunity defined poverty as a human condition determine poverty line, by adopting
characterised by the sustained or Minimum Consumption Expenditure
and access to basic
chronic deprivation of the resources, Criteria. As per this committee, those
services is ensured. Goal capabilities, choices, security and power people will be treated as living below the
of reducing inequality necessary for the enjoyment of an poverty line, whose per-capita consumption
adequate standard of living and other expenditure at 2004, prices is below ` 368
must be explicitly per month in rural areas and below ` 559
civil, cultural, economic, political and
incorporated in policies social rights. per month in urban areas.
and programmes aimed — The number of people living on less Relative Poverty
at poverty reduction.” than $ 1.25 a day has been reduced — Relative poverty refers to poverty on the
from 1.9 billion in 1990 to 836 million basis of comparison of per-capita income
in 2015. World Bank raised the of different countries. The country, whose
international poverty line to $ 1.90 per per-capita income is quite less in
day from the existing $ 1.25 on date not comparison to other countries is treated
given October, 2015. as relatively poor nation.
In poor nations, that part of population,
Types of Poverty —
which is living at the bottom (whose
The poverty has two aspects: income is less), is unable to fulfill the
Absolute Poverty basic requirements of life. In the following
table, India’s percapita income is
— It is a situation, in which the compared with the percapita income of
consumption or income level of people some other countries.
is less than some minimum level
necessary to meet basic needs as per
the national standards. It is expressed Poverty in India
in terms of a poverty line. — There is substantial decline in poverty
— Economists have given many definitions ratios in India from about 45% in
of poverty in this regard, but in a large 1993-94 to about 21.9% in 2011-12. If
number of countries poverty has been the trend continues, people below poverty
defined in the context of per capita line may come down to less than 20% in
intake of calories and minimum level of the next few years.
per capita consumption expenditure. — In a given year in India, official poverty
— Calorie Criteria The energy that an lines finds higher in some states than in
individual gets from the food that he eats others because price levels vary from
everyday is measured in terms of calories.
state to state.
136 Magbook ~ Indian Economy

It is expressed as:
Categories of Poor in India
S = H [ I + (1 − I) G ]
— Although poverty is a relative concept, but where there is an
absolute poverty, we can categorise the poor people by Where, S = Sen index of poverty
defining the poverty line. H = Head count index
— Some are always poor, some are occasionally poor and some I = Poverty gap index and
are never poor. G = Gini co-efficient.
We can categorise poor people in three categories;
—
Multi-Dimensional Poverty Index (MPI)
semicolon chronic poor, trausient poor and non-poor.
— It was developed in 2010, by Oxford Poverty and
Poverty Line Human Development Initiative and the United Nations
Development Programme. It uses different factors to
— It is the line, which indicates the level of purchasing power determine poverty beyond income-based lists. It uses a
required to satisfy the minimum needs of a person. range of deprivations that afflict an individual’s life.
— This line divides the population in two groups, one of those, — The measure assesses the nature and intensity of
who have this purchasing power or more and the other poverty at the individual level in education, health
group of those people, who do not have this much of outcomes and standard of living. The MPI is calculated
purchasing power. as follows:
— The former group is regarded as living ‘Above the Poverty MPI = H × A
Line (APL)’. These people are not regarded as poor. The
Where, H = Percentage of people, who are MPI poor
latter group is considered as living ‘Below the Poverty Line
(incidence of poverty).
(BPL)’. These people are called poor.
A = Average intensity of MPI poverty across
— Asian Development Bank has defined a new poverty line
the poor (%).
taking base of expenditure of US $ 1.35 per day.
According to the Tendulkar Committee Report, which gives
—
state wise poverty estimates, Odisha with 57.2% of BPL Human Poverty Index (HPI)
people is the poorest state followed by Bihar, Madhya — Earlier UNDP set HPI as parameter to measure poverty in
Pradesh and Chhattisgarh. its Human Development Reports but 2010 onwards it
switched over to a new parameter,
Measures of Poverty namely–Multidimensional Poverty Index (MPI).
— The extent of poverty is depicted by the following measures: — The measure assesses the nature and intensity of
poverty at the individual level in education. Health out
Head Count Ratio or Poverty Ratio comes and standards of living.
— It is calculated by dividing the number of people below
poverty line by the total population. It measures the Fisher Price Index (FPI)
proportion of poor in the total population. — It updates the poverty line on the basis of actual
consumption data. This index gives just 60% weightage
Poverty Gap Index (PGI) to food articles.
— It is the difference between the poverty line and the average — The reason why Tendulkar’s method show higher
income of all households living Below Poverty Line (BPL), poverty level is primarily that he has moved away from
expressed as a percentage of poverty line. It indicates the the traditional practice of bench marking poverty by
depth and severity of poverty. certain caloric consumption levels.
Poverty Line − Average Income of BPL
PGI =
Poverty Line
Conditional Cash Transfers (CCTs)
Squared Poverty Gap Index ◆
It is an important mechanism to fight poverty around the
— It is the mean of the squared individual poverty gaps relative world. Here, the government transfers cash to the
to the poverty line. It indicates the severity of poverty as well beneficiaries conditional upon certain action by the
as the inequality among the poor. receiver. These actions could include enrolling children into
school, regular check-up with doctor, institutional delivery,
Sen Index of Poverty receiving vaccination etc.
— It was developed by Professor Amartya Sen. It is based on ◆
It helps to reduce poverty not only by providing cash to the
the head count ratio, poverty gap index and the Gini needy households, but also inducing positive behaviour in
co-efficient. It takes into account the extent and severity of the people through the conditions.
poverty as well as inequality.
Magbook ~ Poverty and Unemployment 137

Two methods of poverty estimation on the basis of


Expert Groups for consumption:
Estimating Poverty Uniform Recall Period (URP)
Here, consumption data is asked for a 30 days recall period
Lakdawala Committee —
for all items. The updated poverty estimates of the Tendulkar
— It was constituted in 1989 by the Planning Commission to Committee have lowered the poverty line from ` 32 a day to
consider methodological and computational aspects of ` 28.
estimation of proportion and number of poor in India.
Mixed Recall Period (MRP)
— The report of this committee was submitted in July, 1993.
— It suggested that the consumption expenditure should be — Here, consumption expenditure is asked for five
calculated based on caloric consumption as earlier. frequently used items clothing, footwear durable goods,
education and institutional medical expenses for a 365
— It also suggested that state specific poverty lines should be
days recall period and other items are asked for a 30
constructed and these should be updated using CPI –
days recall period.
Industrial workers in urban areas and CPI–Agricultural
labour in rural areas.
NC Saxena Committee
Tendulkar Committee Report (For BPL Families in Rural Areas)
— Tendulkar Committee headed by Prime Minister’s Economic — To review the Methodology for conducting BPL Census in
Advisor, Mr Suresh Tendulkar was set-up in March, 2009 to Rural Areas. An expert group headed by Dr NC Saxena
look into the methodology of estimating poverty in India. was constituted by the Ministry of Rural Development to
Tendulkar Committee submitted its report in recommend a suitable methodology for identification of
December, 2009 to the Planning Commission. BPL families in rural areas.
— In its findings, this committee has moved away from just — The expert group submitted its report in August, 2009
calorie criterion definition to a broader definition of poverty and recommended doing away with score-based ranking
that also includes expenditure on health, education, of rural households followed for the BPL Census, 2002.
clothing in addition to food. — The committee has recommended automatic exclusion
— Using this approach, new poverty line for the year of some privileged sections and automatic inclusion of
2004-05, has been raised from ` 356 percapita per certain deprived and vulnerable sections of society and a
month to ` 447 percapita per month in rural areas and survey for the remaining population to rank them on a
from ` 539 per capita per month to ` 579 percapita per scale of 10.
month in urban areas. In daily terms, poverty line has — Based on the above methodology, the committee
been raised from ` 12 to `15 percapita per day in rural estimated the population below the poverty line at 50%
areas and from ` 18 percapita per day to ` 19 percapita of the total population.
per day in urban areas.
— As per Tendulkar Committee Report, 37.2% of Indian SR Hashim Committee
population is living below poverty line using ‘uniform (For BPL Families in Urban Areas)
recall period consumption’ in the year 2004-05, against
the official estimate of 27.5%. According to this report, — On the methodology for identification of BPL families in
41.8% population in rural areas and 25.7% population in urban areas, according to the expert group headed by
urban areas is living below poverty line. renowned economist.
— The new, updated data released by the commission based — SR Hashim to suggest methodology for identifying urban
on the price indices computed from the 66th Round NSS poor, households having three of the four items like
(2009-10) data on Household Consumer Expenditure refrigerator, motorised two-wheelers, landline telephone
Survey, say anyone who has ` 28 to spend daily is out of or washing machines should not be treated as poor.
poverty. — The panel has suggested that the government should
— It has estimated the poverty lines at all India level as MPCE use three-stage approach-automatic exclusion, automatic
(Monthly Per-Capita Consumption Expenditure) of ` 673 for inclusion and scoring index to identify urban poor.
rural areas and ` 860 for urban areas in 2009-10. — Planning Commission had constituted the expert group
— Based on these cut-offs, the percentage of people living to recommend the detailed methodology for the
below the poverty line in the country has declined from identification of families living below poverty line in the
37.2% in 2004-05 to 21.92 in 2011-12. urban areas on 13th May, 2010.
138 Magbook ~ Indian Economy

— Under the automatic inclusion step, homeless families facing States 2004-05 2011-12 Decrease
social and occupation deprivations should be included in the
BPL list. As per the report, a family be defined as poor if any Maharashtra 38.1 17.4 20.7
of its member (including children) is a beggar or rag picker, Manipur 38 36.9 1.1
domestic worker and sweeper or sanitation worker or mali. Meghalaya 16.1 11.9 4.2
The family would also be poor if all its earning adult
Mizoram 15.3 20.4 –5.1
members are either daily wagers or workers with irregular
wages. Nagaland 9 18.9 –9.9
— In the third and final stage, the remaining households should Odisha 57.2 32.6 24.6
be assigned scores from 0 to 12 based on various indicators Puducherry 14.1 9.7 4.4
of residential, social and occupational vulnerabilities. Those Punjab 20.9 8.3 12.6
households with scores from 1 to 12 should be considered
Rajasthan 34.4 14.7 19.7
eligible for inclusion in the BPL list in the increasing order of
the intensity of their deprivations meaning thereby that those Sikkim 31.1 8.2 22.9
with higher scores are more deprived, the report suggested. Tamil Nadu 28.9 11.3 17.6
Tripura 40.6 14.1 26.5
Rangarajan Committee on Poverty Uttar Pradesh 40.9 29.4 11.5
— Planning Commission constituted an expert group headed by Uttarakhand 32.7 11.3 21.4
C Rangarajan to review the Tendulkar Committee
West Bengal 34.3 20 14.3
methodology for estimating poverty in May, 2012.
Perspective of people about poverty has changed, therefore, All India 37.2 21.9 15.3
commission needs to take a fresh look into the methodology
Source: Review Expert Group to Review the Methodology
for estimation of poverty in the country. The committee
for Estimation of Poverty NITI Aayog, Government of India.
submitted its report on 6th July, 2014 to the Planning
Commission.
UN Report on Indian Poverty
— The report observed that the population, living below poverty
line, has decreased from 38.2% in 2009-10 to 29.5% in — In the Millennium Development Report, 2010, it has
2011-12. This report, thus, contested the facts given by the been mentioned that the poverty rate in India was 51%
Tendulkar Committee. The Rangarajan report also revised the in 1990. However, it is expected to fall to a level of
poverty line by increasing it to ` 972/month (or ` 32/day), as 24% by 2015. According to the UNDP, the 8 poorest
against the ` 816/ month suggested by the Tendulkar Indian States— Bihar, Chhattisgarh, Jharkhand,
Committee. For the urban areas, the Rangarajan Committee Madhya Pradesh, Odisha, Rajasthan, Uttar Pradesh
revised the poverty line to ` 1407/month (` 47/day), as and West Bengal have more number of poor, than the
against ` 1000/month of the Tendulkar Committee. 26 poorest African nations.

Statewise Poverty Estimates (% below poverty line) Measurement of Poverty by UNDP


States 2004-05 2011-12 Decrease
— As per UNDP, poverty is a multi-dimensional problem.
Apart from income, it also includes factors like-health
Andhra Pradesh 29.9 9.2 20.7 and nutrition. World Bank has established 2 parameters
Arunachal Pradesh 31.1 34.7 –3.6 to measure poverty at the international level
Assam 34.4 32 2.4 — Those earning less than 1.25 US $ of per day, such
Bihar 54.4 33.7 20.7 persons are referred as suffering from extreme poverty.

Chhattisgarh 49.4 39.9 9.5 — Those spending less than 2 US $ per day. These sections
are referred to as poor.
Delhi 13.1 9.9 3.2
Goa 25 5.1 19.9
— Besides these parameters, Lorenz Curve and Gini
Co-efficient are also used to observed poverty in a
Gujarat 31.8 16.6 15.2
state. The Asian Development Bank (ADB) has set the
Haryana 24.1 11.2 12.9 parameter of US $ 1.35; while Indian Government has
Himachal Pradesh 22.9 8.1 14.8 set this parameter at US $ 1.02.
Jammu and Kashmir 13.2 10.4 2.8
State of Poverty (World Bank Report)
Jharkhand 45.3 37 8.3
— World Bank, on 18th April, 2013, in its report entitled,
Karnataka 33.4 20.9 12.5 where are the poor and most poor, observed that
Kerala 19.7 7.1 12.6 — one-third of the global poor in India.
Madhya Pradesh 48.6 31.7 16.9 — the poor in the India live on less than US$ 1.25 a day.
Magbook ~ Poverty and Unemployment 139

— there are around 120 crore extremely poor persons in Lorenz Curve of Income Distribution
the world today.

Cumulative income share (%)


— between 1981-2010, the developing countries have 100
witnessed a decline in poverty rate from 50% to 21%. 90
80
— despite development in Africa, poverty is still
70
widespread.
60
50 A
Inequality 40
30 B
— Inequality often refers to the income gap between 20
the rich and poor of society. The greater the gap the 10
greater the inequality. It essentially refers to 0
10 20 30 40 50 60 70 80 90100
disparities in the distribution of economic assets and
Cumulative population share (%)
income among individuals and groups within a
nation and nations.
— It may result from the operation of the economic Inequality in India
system, access to assets, education and skills,
— Both overall GDP and per capita GDP have increased rapidly in
social factors like caste and gender etc.
the economic reform period. The number of poor as the
percentage of population have also come down. However,
Adverse Impacts of Inequality inequality in the reform period has increased.
— Growing inequalities can dampen growth due to — Gini co-efficient increased from 0.27 to 0.28 in rural areas and
potential instability; weaken social cohesion. from 0.35 to 0.39 in urban areas between 2004-05 and
— Urban-dominated growth in India has caused social 2009-10. Top 10% wage earners make 12 times more than the
friction as a result of the high levels of migration to bottom 10% compared to 6 times 20 years ago.
cities and a shortage of foreign investment in more * Trickle Down theory says that let business flourish, since
isolated areas. their profits ultimately trickle down to lower income
— In societies, where wealth is concentrated in the individuals and the ease of the economy.
hands of a few, there is danger of policy levers * Typically, India used the Consumer Price Index for
being captured by the rich for their own benefit and Agricultural Labour (CPIAL) and CPIIW for industrial
a weakening of the institutional foundations of the labour.
growth process.

Gini Co-efficient Employment and Unemployment


— The Gini co-efficient (also known as the Gini index Employment
or Gini ratio) is a measure of statistical dispersion — Employment refers to the capacity in which a worker pursues
developed by the Italian statistician and sociologist
gainful activity during the referece period. According to their
Corrado Gini, it measures the inequality among
status of employment, there are three types of employed
values of a frequency distribution (e.g. levels of
workers, self employed workers, casual wage and regular wage
income).
employees.
— Gini co-efficient is commonly used as a measure of
inequality of income or wealth. A Gini co-efficient of Unemployment
zero expresses perfect equality, where all values are
the same (e.g. where everyone has an exactly equal — Unemployment refers to a situation, when a person is able and
income). willing to work at the prevailing wage rate, but does not get the
opportunity to work. The term unemployment is directly related
— A Gini co-efficient of one (100 on the percentile with the concept of labour force, because the people, who are
scale) expresses maximal inequality among values not included in labour force cannot be regarded as unemployed.
(e.g. where only one person has all the income).
— Labour force includes all people in the working age group
— The Gini co-efficient is usually defined (15-59 years), who are able and willing to work. Labour force
mathematically based on the Lorenz curve, which equals the work force plus the number of unemployed people.
plots the proportion of the total income of the So, unemployment refers to only involuntary unemployment.
population (y axis) that is cumulatively earned by
the bottom x% of the population (see diagram). The — Unemployment rate is defined as the number of persons
line at 45 degrees Gini, thus, represents perfect unemployed per 1000 persons in the labour force (which
equality of incomes. included both the employed and the unemployed).
140 Magbook ~ Indian Economy

Types of Unemployment Disguised Unemployment


There are following types of unemployment are as follows:
— It is a situation, in which more persons are employed to
do a job which can be done with equal efficiency by a less
Voluntary Unemployment number of workers.
— It occurs when a person is not willing to work at the — The following features of disguised unemployment are as
prevailing rate of wage or does not desire to work. It is not follows:
taken into consideration while measuring unemployment — Marginal productivity of labour is zero. However, Professor Sen
in an economy. does not agree with this view.
— It is not possible to identify the persons who are actually
Involuntary Unemployment unemployed.
— It is the situation, in which the worker is willing and able — Excess of population and lack of capital is the principal cause.
to work, but he does not get work. It is also called open — Generally associated with agricultural families.
unemployment.
Under Employment
Frictional Unemployment — It is a situation, in which a person is employed in a job,
— It is temporary unemployment which is associated with which is not commensurate with his/her qualification, skill
the changing of jobs in an economy. It can occur due to and experience. Under this condition, capabilities of the
immobility it of labour, shortage of raw materials, lack of workers are not utilised to the optimum level.
knowledge of job opportunities, breakage of machines etc.
— Under employment is another problem in developing
Structural Unemployment countries alongwith unemployment. It measures how well
— It is concerned with the structural pattern of the economy. the labour force is being utilised in terms of skills
It arises in situations such as: experience and availability of work.
— Shortage of factors of production like land, capital etc. Types of Under Employment
— Workers don’t have skills for new industries.
— There are two types of under employment:
— Compared to supply of labour availability of employment is
less. (i) Visible Under Employment It is when people get work for
— When certain industries close down.
less than the normal hours of work e.g. 2 hours a day.
— When there is a change in production technology and nature (ii) Invisible Under Employment It is the situation, in which
of produced products. people work full time, but their income is very low or
— Mainly, it is caused due to a mismatch between the they work in jobs, in which they cannot make full use of
skills of a person and the requirements of the job. their ability e.g. an MA degree holder working as a
driver.
Cyclical Unemployment — Under employment is found when persons engaged in
— It is associated with a general depression in the overall part time work are prepared to do more work than they
economy. It occurs due to economic cycles of boom and are doing or the productivity and income of a person rises
depression. During depression, demand falls and after shifting another type of occupation.
production goes down which leads to fall in level of
employment. Nature and Estimates of
Seasonal Unemployment Unemployment in India
— It arises because some occupations require workers only — Like all the underdeveloped countries, India presently
during certain parts of the year such as in agriculture, suffers mainly from structural unemployment, which exists
sugar industry etc. in open and disguised forms.
— Apart from structural unemployment, there is Keynesian
Technical Unemployment
involuntary unemployment, which can be eliminated by
— It is associated with technical changes. Modern industries increasing effective demand as is done in developed
are capital intensive and workers are being replaced by countries.
machines. Adoption of labour saving technologies renders
some workers unemployed. Causes of Unemployment in India
Educated Unemployment — Rapid population growth.
— It refers to unemployment of those who are normally — Limited land holding.
educated. It is both of open unemployment and under — Seasonal agriculture.
employment type, i.e. those who can’t find work and those — Decline of manufacturing industries.
who work in a job that is not in keeping with their skills, — Inadequate employment planning and its execution.
education or capacity. — Impact of global slowdown.
Magbook ~ Poverty and Unemployment 141

— The GDP from manufacturing increased at


Key Indicators for this Estimation of 9.5% per annum between 2004-05 and
Under Employment in India 2009-10, alongwith same increase in

Labour Force Participation Rate (LFPR) employment in the organised
manufacturing sector. However, the overall
Number of employed persons + Number of unemployed persons employment in manufacturing actually
= × 1000
Total population declined during this period. Between
Number of employed persons 2004-05 and 2009-10, vast majority of

Worker Population Ratio (WPR) = × 1000 new jobs were created in casual
Total population
employment, mainly in construction.
Number of unemployed persons

Proportion Unemployed (PU) = × 1000
Total population
Number of unemployed persons
Poverty Eradication
Unemployment Rate (UR) = × 1000

Number of employed persons + and Employment


Number of unemployed persons Related Programme
— Currently, India is passing through an
unprecedented phase of demographic
Measures of Unemployment in India changes. The ongoing demographic
— On the recommendation of the Bhagwati Committee, since 1972-73, changes are likely to contribute to an ever
National Sample Survey Office (NSSO) comes out with quinquennial increasing size of labour force in the
survey on employment and unemployment. The persons surveyed are country.
classified into various activity categories on the basis of activities pursued
— The government has been attempting to
during certain specified reference periods as Usual Principle Status,
improve basic education through a host of
Current Weekly Status and Current Daily Status.
measures in recent decades and achieving
— Usual Principle and Subsidiary Status (UPSS) A person is considered some success. But, vocational education
working or employed, if the person was engaged for a relatively larger and imparting skills remains a critical area
period (over 182 days) in any one or more work related (economic) of concern.
activities during the reference period of 365 days preceding the date of
survey. Skill Development
Current Weekly Status (CWS) A person is considered working or
—
employed, if the person was engaged for at least 1 hour on any 1 day on
Programme
any work related (economic) activity during the reference period. It also — The Union Cabinet has approved a
measures chronic unemployment. Coordinated Action Plan for Skill
— Current Daily Status (CDS) It is the time rate and measured in man Development, which envisages a target of
days. A person is considered unemployed, if he does not find work even 500 million skilled persons by 2022. The
on a day or some days during the survey week. plan will be executed through 3 tier
strategy of
Salient Features of NSSO (66th Round) on Prime Minister’s National Council on Skill Development
Employment and Unemployment
— Number of young people (15-24 years) in education and therefore, out of National Skill Development Co-ordination Board
workforce, has increased dramatically causing a drop in the labour
participation rate. National Skill Development Corporation
— Between 2004-05 and 2009-10, the overall labour force expanded by only
11.7 million, which is considerably lower than comparable earlier periods. Prime Minister’s National
This is because of larger retention of youth in education; and lower Council on Skill Development
Labour Force Participation Rate among working age women. — The council is chaired by the
— Over the same period (2004-05 to 2009-10), 18 million job opportunities Prime Minister with Ministers for Human
were created on CDS basis. Thus, in absolute terms, unemployment came Resource Development, Finance, Heavy
down by 6.3 million and the unemployment rate, which had increased Industries, Rural Development, Housing
from 6.06% in 1993-94 to 7.31% in 1999-2000 and further to 8.28% in and Urban Poverty Alleviation and Labour
2004-05, came down to 6.60% in 2009-10. and Employment as members.
142 Magbook ~ Indian Economy

— Deputy Chairman, Planning Commission, Pradhan Mantri Kaushal Vikas Yojana


Chairperson of the National Manufacturing
It is a demand-driven, reward-based skill training scheme. PMKVY
Competitiveness Council, Chairperson of the National
is formed to provide skill training to class 10 and 12 dropout youths
Skill Development Corporation and six experts in the
across the country.
area of skill development are its other members.
Under the scheme, besides assessing and certifying 10 lakh youth
— The Prime Minister’s National Council on Skill
for the skills they already possess, around 24 lakh youth will be
Development has endorsed a vision of creating
skilled over the next year.
500 million skilled people by 2022 through skill
systems, which must have high degree of The important characteristics of PMKVY are as follows
inclusiveness in terms of gender, rural or urban, — It will cover 24 lakh persons and skill training would be based

organised or unorganised and traditional or on the National Skill Qualification Framework (NSQF) and
contemporary. industry-led standards.
— It will be implemented by the Union Ministry of Skill
National Skill Development Development and Entrepreneurship through the National Skill
Co-ordination Board (NSDCB) Development Corporation (NSDC) training partners. In addition,
— The NSDCB has been set-up under the Central and State Government affiliated training providers would
chairmanship of Deputy Chairman, Planning also be used for training purposes.
Commission, with secretaries of ministries of human — Skill training would be on the basis of skill gap studies
resource development, labour and employment, rural conducted by the NSDC for the period 2013-17.
development, housing and urban poverty alleviation — Focus of the training would be on improved curricula, better
and finance as members.
pedagogy and better trained instructors.
— Secretaries of four states by rotation, for a period of — Training would include soft skills, personal grooming,
2 years, three distinguished academicians and
behavioural change for cleanliness, good work ethics.
subject area specialists are the other members.
— A monetary reward will be given to trainees on assessment and
Secretary, planning commission is the member
secretary of the board. certification by third party assessment bodies. The average
monetary reward would be around ` 8000 per trainee. It should
National Skill Development be noted that the target for skilling would be associated with
Corporation the recent programmes such as Make in India, Digital India,
— The third tier of the coordinated action on skill Swachh Bharat Abhiyan and National Solar Mission.
development is NSDC, which is a non-profit Skill Loan Scheme
company under the Companies Act with an
— Under the scheme, loans ranging from ` 5000 to ` 1.5 lakh will
appropriate governance structure.
be made available to 34 lakh youth to attend skill development
— The Central Government has created a National Skill programmes. It will be operational between 2015 and 2020.
Development Fund with an initial corpus of ` 995.10
crore for supporting the activities of the corporation. Mahatma Gandhi National Rural
The corpus of the fund is expected to go up to about
—
` 15000 crore as it is intended to garner capital from
Employment Guarantee Scheme
governments, public and private sectors and bilateral (MGNREGS)
and multilateral sources. — The National Rural Employment Guarantee Act (NREGA) was
Objectives of Skill Development enacted in 2005. It was implemented in three phases, starting
Programme in India with 200 districts in 2006 to cover the whole country by 2008.
On 2nd October, 2009 it was renamed as Mahatma Gandhi.
— Create opportunities for all to acquire skills
throughout life and especially for youth, women and Features of MGNREGS
disadvantaged groups. Promote commitment by all
— It seeks to provide at least 100 days (150 days for tribals) of
stakeholders to own skill development. Develop a
guaranteed wage employment in one financial year to at least
high-quality skilled workforce relevant to current and
one adult member of every rural household who volunteer to do
emerging market needs.
unskilled manual work.
— Enable establishment of flexible delivery mechanisms
— Atleast 33% of the beneficiaries are to be women.
that respond to the characteristics for a wide range
of needs of stakeholders. Enable effective — Originally, it promised a wage rate of ` 100 per day from
coordination between different ministries, the centre January, 2011 wages have been linked to increase with
and states and public and private providers. Consumer Price Index for Agricultural Labour (CPI-AL) for each
state.
Magbook ~ Poverty and Unemployment 143

— Focuses on works related to water conservation, drought — NRLM focuses on setting up and strengthening of
proofing, land development, flood control, rural connectivity institutions of the poor in partnership mode.
through all weather roads etc. — Poor will be provided with adequate skills to manage
— Provides time bound employment guarantee and wage their institutions.
payment within 15 days. — NRLM would work towards universal financial inclusion.
— A 60:40 wage to material ratio has to be maintained to — It provides revolving fund, interest subsidy and capital
ensure greater employment generation. No contractor or subsidy to the SHGS.
machinery is allowed for any work. — It will provide infrastructure support for key livelihoods
— Panchayats have been given an important role through and support for marketing.
preparation of perspective plan, approval of shelf of — Funding would be in the ratio of 75:25 between Centre
projects and execution of works atleast to the extent of 50% and States (9:10 for North-East and special category
in terms of cost. states).
— Rights based framework. — It will be implemented in phases to reach all districts of
— Transparency accountability. the country by the end of 12th Plan.
— All states have to transition to NRLM from SGSY within 1
Recent Changes in MGNREGS year of launch.
— 30 new activities such as rural sanitation, live stock, fishery
etc have been added to the permissible list. National Urban Livelihood
— Electronic Fund Management System in all states has been Mission (NULM)
initiated in a phased manner.
— The centrally sponsored scheme of Swarna Jayanti
— Convergence of MGNREGS with total sanitation campaign
Shahari Rozgar Yojana has been restructured as the
has been undertaken.
National Urban Livelihood Mission (NULM) to be
— Provision has been made for seeding in Aadhaar into the implemented in the 12th Plan.
MGNREGS workers records to prevent leakage.
— The new scheme expands the beneficiaries among the
urban poor to include homeless and street vendors.
National Rural Livelihood
— A special provision for funding of 24/7 shelters with all
Mission (NRLM) essential facilities for the urban homeless has been
— The Swarna Jayanti Grameen Swarojgar Yojana (SGSY) has made.
been restructured and launched as the National Rural — Up to 5% of NULM budget has been earmarked to
Livelihood Mission (NRLM). NRLM was recently renamed provide support to urban poor, which will include skill
as Ajeevika. upgradation and development of vendor markets.
Key Features of NRLM — Urban poor, especially women, will be organised into Self
Help Groups (SHGs). This will be the main emphasis of
— It will ensure that atleast one member (preferably a woman) NULM. A provision of ` 1000 has been made for the
from each identified poor rural household is brought under formation and activities of each SHG for the initial 2 years.
a Self Help Group (SHG).
Self Check
Build Your Confidence
1. Consider the following statements 6. Estimates of poverty are made by the Planning
1. Assam has the highest percentage of poverty according Commission based on the data provided by which of the
to the Suresh Tendulkar Committee in India. following?
2. In India, the infant mortality rate per thousand of live (a) NSSO
birth of girls is more than for boys. (b) CSO
Which of the statement(s) given above is/are correct? (c) Ministry of Rural Development
(a) Only 1 (b) Only 2 (d) Finance Ministry
(c) Both 1 and 2 (d) Neither 1 nor 2 7. Disguised unemployment generally means [IAS 2013]
2. Consider the following statements (a) large number of people remain unemployed
1. Gini co-efficient is the measure of income inequality in a (b) alternative employment is not available
country with 1 representing complete inequality and 0, (c) marginal productivity of labour is zero
as complete equality. (d) productivity of workers is low
2. India’s Gini co-efficient is less than the US, but more 8. The Government of India has decided to measure
than China. poverty line in terms of
Which of the statement(s) given above is/are correct? (a) household consumption
(a) Only 2 (b) Only 1 (b) household savings
(c) Noth 1 and 2 (d) Neither 1 nor 2 (c) household investment
3. The foster-greer-thorbecke index is the difference (d) household
between the 9. How does the National Rural livelihood mission seek to
(a) mean income of poor and poverty line improve livelihood options of rural poor? [IAS 2012]
(b) median income of poor and poverty line 1. By setting up a large number of new manufacturing
(c) median income of poor and mean income of poor industries and agribusiness centres in rural areas.
(d) mode of the income of poor and mean income of poor 2. By strengthening self-half groups and providing still
4. The poverty gap index is the difference between the development.
(a) mean income of poor and poverty line 3. By supplying seeds, fertilizers, diesel pump-sets and
(b) median income of poor and mean income of poor micro-irrigation equation free of cost to farmers.
(c) median income of poor and poverty line Select the correct answer using the codes given below
(d) mode of the income of poor and mean income of poor (a) 1 and 2 (b) Only 2
(c) 1 and 3 (d) All of these
5. Match the following
10. Among the following who are eligible to benefit from
List I List II MNREGA? [IAS 2011]
(Committee) (BPL Population) (a) Adult member of only SC and ST households
(b) Adult members of BPL households
A. Tendulkar Committee 1. 50 % (c) Adult members of households of all backward communities
B. NC Saxena Committee 2. 77 % (d) Adult members of any households
C. Arjun Sen Gupta Committee 3. 37 % 11. Disguised unemployment in India is mainly related to
Codes 1. agricultural sector 2. rural sector[MPPCS 2001]
A B C A B C 3. factory sector 4. urban area
Select the correct answer using the codes given below
(a) 3 2 1 (b) 1 2 3
(a) 1 and 2 (b) 1 and 4
(c) 2 3 1 (d) 2 1 3
(c) 2 and 4 (d) 2 and 3

1. (b) 2. (b) 3. (a) 4. (a) 5. (a) 6. (a) 7. (c) 8. (a) 9. (b) 10. (d)
11. (c)
Chapter sixteen
Government Schemes
and Programmes
It is a new direction and a new goal for
Social Welfare Schemes —
the all out of school/dropped out
— Some new social development schemes to students and those studying in
address the critical issues of the country have Madarasas. It is so because they will
Government aided been introduced. These include the following: not be getting formal Class XII and
schemes and Class X Certificates rendering them
programmes are run Skill Development and largely un employed in
organised sector.
by the Central and Employment Scheme
State Governments Solar Charkha Mission USTAD Scheme
for the health and — President Ram Nath Kovind launched Solar
— Union Minister Najma Heptullah
welfare of citizens. launched a welfare scheme,
Charkha Mission on June 27, 2018, in which
Upgradation of Skills and Training in
The people of the the government will be providing a subsidy of
Ancestral arts/crafts for Development
countries are one of ` 550 crore to the thousands of artisans and
(USTAD), aimed at upgrading and
generating employment in the rural areas. The
its most valuable promoting the skills of artisans from the
Ministry of Micro Small and Medium Enterprise
resources. minority community, on May 14, 2015.
(MSME) will cover the 50 indentified clusters
It is launched from Varanasi in order to
across the country including in the Northeast
improve degrading conditions of world
and in each cluster 400 to 2000 artisans will
famous Banaras Saree weavers who
be employed. Along with this mission, the
belong to minority communities.
government also launched a Sampark portal, a
digital platform on which five lakh job seekers — The programme is linked to the ‘Make
can connect with the Ministry of Micro Small in India’ campaign and termed as close
and Medium Enterprise (MSME). to the Prime Minister’s heart. It seeks to
help weavers and artisans connect with
‘Nai Manzil’ Scheme buyers all over the world.
— The Union Minister for Minority Affairs — A portal dedicated to the programme
Dr. Najma Heptulla launched a new Central was also launched to help artisans to
Sector Scheme ‘Nai Manzil’ in Patna on log in and get connected to potential
August 8, 2015. buyers world-over. The National
— The scheme is intended to cover people in Institute of Design along with a host of
between 17 to 35 age group from all minority other bodies would be associated with
communities as well as Madarasa students. this mission and improve designs of the
produce.
— The scheme ‘Nai Manzil’ scheme will address
educational and livelihood needs of minority — The Upgradation of Skills and Training
communities in general and muslims in in Ancestral Arts/Crafts for Development
particular as it lags behind other minority (USTAD) scheme was announced
communities in terms of in 2014 with the Finance Ministry
educational attainments. allocating ` 17 crore for the scheme.
146 Magbook ~ Indian Economy

Van Bandhu Kalyan Yojana — Under the new modified PMAY scheme, the sharing
pattern between the Centre and states is same, but the
— For the welfare of the tribal people ‘Van Bandhu Kalyan
grant would be transferred directly into the bank
Yojana’ is being launched with an initial allocation of `100
account of beneficiaries, who had been selected on the
crore. YKY aims at creating enabling environment for need
basis of socio-economic caste census of 2011.
based and outcome oriented holistic development of the
tribal people. Pradhan Mantri Awas Yojana – Gramin
— The scheme is launched on a pilot basis and will be — PM Narendra Modi launched the scheme on 20
implemented in only 1 block in each of the 10 states being November, 2016 at Agra. Aim of scheme to construct
selected for the scheme. The states which have been picked ` 1 crore Pucca Houses in rural areas in next 3 years
are Andhra Pradesh, Madhya Pradesh, Himachal Pradesh, by financial assistance (for plain areas – ` 1,20,000
Telangana, Odisha, Jharkhand, Gujarat, Chhattisgarh, and for hilly areas – ` 1,30,000).
Rajasthan and Maharashtra.
Shyama Prasad Mukherjee Rurban
Skill Upgradation Programme for Mission (SPMRM)
Minorities — It launched in 2016 to deliver integrated project based
— A programme for the upgradation of skills and training in infrastructure in the rural areas. The scheme will also
ancestral arts for development for the minorities called include development of economic activities and skill
upgradation of traditional skills in arts, resources and goods development. The preferred mode of delivery would be
is to be launched to preserve the traditional arts and crafts, through PPPs while using various scheme funds.
which are rich heritage. — The scheme of Providing Urban Amenities in Rural
— An additional amount of ` 100 crore for Modernisation of Areas (PURA) is merged with Shyama Prasad
Madarsas has been in the budget provided to the Department Mukherjee Rurban Mission.
of School Education.
Benefits of Rurban Mission
The benefits of Rurban mission are as follows
Rural Infrastructure and — Under this scheme, the facilities of cities will be given
Development to villages.
Swachh Bharat Abhiyan — Electricity, water, roads and health facilities will be
provided to villages.
— Total sanitation by 2019, is the slogan of this programme. The
year 2019 also marks the 150th Birth Anniversary of Mahatma — The villages will be developed on the line of the cluster.
Gandhi. Nirmal Bharat Abhiyan was relaunched in name of — Population in a cluster will be 21,000 to 50,000.
Swachh Bharat Abhiyan. — 30% of total expenditure on Center Gap Funding will be
from our Budget.
GOBAR-Dhan Yojana
— The Galvanising Organic Bio-Agro Resources Dhan Sansad Adarsh Gram Yojana
(GOBAR-DHAN) scheme was launched by Haryana’s Chief — It is a rural development and cleanliness programme
Minister Manohar Lal Khattar and Uma Bharti (Union broadly focusing upon the development in the villages,
Minister for Sanitation and Drinking Water) on April 30, which includes social development, cultural
2018. The GOBAR-Dhan scheme is an effort by the development and spread motivation among the people
government to improve the living conditions in the Indian on social mobilisation of the village community. The
villages and make them open-defecation free. A segment of programme was launched by the Prime Minister on the
the Swachh Bharat initiative, this scheme will focus on birth anniversary of Jayaprakash Narayan on 11th
useful conversion of solid waste and cattle dung into manure October, 2014. Prime Minister Narendra Modi adopted
and biogas. Jayapur village under this scheme.
Pradhan Mantri Awaas Yojana (PMAY) Bharat Nirman
— The rural housing scheme Indira Awaas Yojana (IAY), started — This programme, launched in 2005-06 for building
by former Prime Minister Rajiv Gandhi, has been infrastructure and basic amenities in a rural areas, has 6
restructured and renamed as Pradhan Mantri Awaas Yojana components, namely rural housing, irrigation potential,
(PMAY) on 21st September, 2016. The IAY would be drinking water, rural roads, electrification and rural
included by the PMAY. Under the new scheme, the telephone.
government was aiming to construct one crore houses by
2019.
Magbook ~ Government Schemes and Programmes 147

Pradhan Mantri Gram Sadak Rural Sanitation Total Sanitation Campaign


Yojana (PMGSY) (TSC) and Nirmal Bharat Abhiyan
— The PMGSY was launched in December, 2000, as a — Government started the Central Rural Sanitation
fully funded centrally sponsored scheme with the Programme(CRSP) in 1986 primarily with the objective of
objective of providing connectivity to the eligible improving the quality of life of the rural people and also to provide
unconnected habitations in the core network with a privacy and dignity to women.
population of 500 persons and above (as per — The concept of sanitation was expanded to include personal
Census, 2001) in plains areas and 250 persons and hygiene, home sanitation, safe water, garbage disposal, excreta
above in hill states, tribal areas, desert areas. The disposal and waste water disposal. With this broader concept of
government has revised its target year for sanitation, CRSP adopted a demand driven approach with the
completion of phase-I of PMGSY a rural road name Total Sanitation Campaign (TSC) with effect from 1999.
scheme, to 2019 from 2022. Nirmal Gram Puraskar (NGP) is given to Panchayati Raj
Institutions that achieve 100% coverage of sanitation facilities.
Housing and Sanitation — Encouraged by the success of NGP, the TSC is being renamed as
Scheme ‘Nirmal Bharat Abhiyan’ (NBA). The objective is to acelerate the
sanitation coverage in the rural areas so as to comprehensively
— The Central Government has been assisting State cover the rural community through renewed strategies and
Governments by way of various centrally sponsored saturation approach.
schemes through national financial institutions
— Nirmal Bharat Abhiyan (NBA) envisages covering the entire
providing better urban infrastructure, housing and
community for saturated outcomes with a view to create. Nirmal
sanitation in the country. Some of the initiatives in
Gram Panchayats with following priorities:
this area are as follows
— Provision of Individual Household Latrine (IHHL) of both Below Poverty
Atal Bhujal Yojana Line (BPL) and Indentified Above Poverty Line (APL) households within
a Gram Panchayat (GP).
— Atal Bhujal Yojana is Central Government’s ` 6,000
— Gram Panchayats where all habitations have access to water to be
crore ambitious water conservation scheme
taken up. Priority may be given to Gram Panchayats having functional
launched in June, 2018 to deal with the piped water supply.
ever-deepening crisis of depleting groundwater
— Provision of sanitation facilities in government schools and Anganwadis
level. The main objectives of this scheme are to in Government buildings within these GPs.
revitalise groundwater level and create sufficient
— Solid and Liquid Waste Management (SLWM) for proposed and existion
water storage for agricultural purposes; Nirmal Grams.
rejuvenation of the surface water bodies so that — Extensive capacity building of the stake holders like Panchayati Raj
groundwater level can be increased, especially in Institutions (PRIs), Village Water and Sanitation Committees (VWSCs)
the rural areas; recharging sources of groundwater and field functionaries for sustainable sanitation.
and ensure effective use of water by involving — Appropriate convergence with MNREGS with unskilled man-days and
people at local level. skilled man-days.

“Sankalp Se Siddhi” Programme Integrated Low Cost Sanitation Scheme


— “Sankalp Se Siddhi” (Attainment through Resolve) (ILCS)
programme is a new initiative launched by PM — The ILCS aims at conversion of individual dry latrines into pour flush
Narendra Modi on 23 August ,2017 for a New latrines thereby liberating manual scavengers from the age-old,
India movement from 2017 to 2022. This program degrading practice of manually carrying night soil. The allocation for
aims to bring in many changes in the country for the scheme for 2012-13, is ` 25 crore.
the betterment of country’s economy, citizens,
society, governance, security and other verticals.
Education
Neeranchal — India, which had a bottom heavy population is now graduating to
— To give an added impetus to watershed development an economy with middle-heavy population. To reap the benefits of
in the country, a new programme called Neeranchal the demographic dividend to the full, India has to provide
with an initial outlay of 2142 crore in the current education to its population and that too quality education.
financial year. Pashmina Promotion Programme — The draft Twelfth Plan focuses on teacher training and evaluation
(P-3) and a programme for the development of other and measures to enforce accountability. It also stresses the need to
crafts of Jammu and Kashmir is also to be started. build capacity in secondary schools to absorb the pass outs from
For this a sum of ` 50 crore is set aside. expanded primary enrolments.
148 Magbook ~ Indian Economy

Samagra Shiksha Scheme Rashtriya Madhyamik Shiksha Abhiyan


— The Samagra Shiksha Scheme was launched by the Union (RMSA)
Ministry of Human Resource Development (HRD) on May — RMSA was launched in March, 2009, with the objective to
24, 2018, in view of improving the quality of education at enhance access to secondary education and to improve its
school level in India. It is an overarching program which quality.
will incorporate digital technology and introduce skill — Objectives include : improving quality of education imported
development in the school education system. An annual at the secondary level through making all secondary
grant of five thousand to twenty thousand rupees will be schools conform to prescribed norms; removing gender,
provided for strengthening libraries in schools under the socio-economic and disability barriers; providing universal
program. access to secondary level education by 2017 and achieving
universal retention by 2020.
Sarva Shiksha Abhiyan (SSA)/Right to
Education (RTE) — Important physical facilities provided under the scheme
are- additional class rooms, laboratories, libraries, toilet
— The Right of Children to Free and Compulsory Education blocks, art and craft rooms and residential hostels for
(RTE) Act, 2009, legislating Article 21A of the Constitution teachers.
of India, became operational in the country on 1st April,
2010. It implies that every child has a right to elementary Saakshar Bharat (SB) or Adult Education
education of satisfactory and equitable quality in a formal — Saakshar Bharat has been formulated with the objective of
school, which satisfies certain essential norms and standards, achieving 80% literacy level by 2012 at national level by
significant reduction in the number of out-of-school children focusing on adult women literacy seeking to reduce the gap
on account of SSA interventions has been noted. between male and female literacy to not more than 10%
— Top 5 government education schemes are Shiksha Sahayog points. The principal target of the mission is to impart
Yojana, Sarva Shiksha Abhiyan, Saakshar Bharat, Kanya functional literacy to 70 million non-literate adults in the age
Saaksharta Prothsan Yojana and Kasturba Gandhi Balika group of 15 years and beyond.
Vidyalaya Yojana.
Initiatives in Education in the Twelfth
— Rashtriya Madhyamik Siksha Abhiyan (RMSA) has quality
Plan
intervention schemes to ensure that all second any conform
to prescribed worms, removing gender, socio-economic and — A number of initiatives have been taken during the Eleventh
disability barriers. Plan period with focus on improvement of access along
with equity and excellence, adoption of state-specific
Mid-Day Meals (MDM) strategies, enhancing the relevance of higher education
— Under the MDM, cooked mid-day meals are provided to all through curriculum reforms, vocationalisation, networking
children attending Classes I-VIII in government, local body, and use of IT and distance education along with reforms in
government aided and National Child Labour Project governance in higher education. The major initiatives are as
(NCLP) schools. follows:
— During the Eleventh Plan, 16 Central Universities were
— Education Guarantee Scheme (EGS) or alternate and
established, which include conversion of 3 State Universities to
innovative education centres including madarsa /maqtabs
Central Universities.
supported under the SSA across the country are also
— Seven new Indian Institutes of Management (IIMs), 8 new
covered, under this programme. Indian Institutes of Technology (IITs), 10 new National Institutes
— At present, the cooked mid-day meal provides an energy of Technology (NITs), 5 Indian Institutes of Science Education
content of 450 calories and protein content of 12 gm at and Research (IISERs) and 2 Schools of Planning and
primary stage and an energy content of 700 calories and Architectures (SPAs) were also established.
protein content of 20 gm at upper primary stage. — A Scheme of Interest Subsidy on Educational Loans to
Economically Weaker Sections (EWS) students was introduced
— Adequate quantity of micro-nutrients like iron, folic acid
from 2009-10.
and vitamin A are also recommended for convergence with
— To address the increasing skill challenges of the Indian IT
the NRHM. The MDM-MIS has been launched to monitor industry, the government has approved setting up of 20 new
the scheme and annual data entries for about 11.08 lakh Indian Institutes of Information Technology (IIITs) on PPP basis.
schools have been completed. The project is targeted for completion in 9 years from 2011-12
— The MDM-MIS will be integrated with the Interactive Voice to 2019-20. The Government of India also provides financial
Response System (IVRS) meant to capture the information assistance to the states up to a limit of ` 12.30 crore per
from the schools within a span of 1 hour on daily basis to polytechnic to meet the costs of establishing new government
monitor the scheme. polytechnics in un-served districts.
Magbook ~ Government Schemes and Programmes 149

To make withdrawal of money by the beneficiaries easier


Financial Inclusion Mission —
and more accessible and friendly, micro ATMs will be
— To provide all households in the country with banking services, set-up by banks or post offices throughout the country in
a time bound programme is to be launched as Financial an open manner particularly with the help of SHGs,
Inclusion mission on 15th August, in 2014. Community Service Centres (CSCs), post offices, grocery
— It would particularly focus to empower the weaker sections stores, petrol pumps, etc in rural areas and accessible
of the society, including women, small and marginal pockets. This is being done initially in 51 pilot districts
farmers and labourers. Two bank accounts in each across the country from 1st January, 2013 pilots on direct
household are proposed to be opened which will be eligible benefit.
for credit. Brief Descriptions of Major Programmes
Pradhan Mantri Jan-Dhan Yojana — Transfer (DBT) have also been successfully conducted in
the States of Jharkhand, Tripura, and Maharashtra to
The Pradhan Mantri Jan-Dhan Yojana (PMJDY), launched by
transfer monetary benefits related to rural employment,
Prime Minister Narendra Modi on August 28, 2014 was
recognised for opening the most bank accounts (about pension, the IAY and other social welfare schemes. An
12 crore) in one week as part of the financial inclusion important pilot is the fair price shops in East Godavari and
campaign. The benefits of account holders under PMJDY are Hyderabad districts of Andhra Pradesh, which are being
as follows: enabled to carry out online Aadhar authentication.

Accidental Insurance Coverage PAHAL


— Accidental insurance of ` 1 lakh is available to all RuPay — The Direct Benefit Transfer of LPG (DBTL) or PAHAL
card holders in the age group of 18 to 70 where RuPay card (Pratyaksh Hastantrit Labh) scheme was earlier launched
need to be used once in 45 days of receipt. on 1st June, 2013 and finally covered 291 districts.
— Claim intimation should be given his or her bank where — The modified scheme is being relaunched in 54 districts
account is maintained within 30 days from the date of on 15th November, 2015 in the 1st Phase and in the rest
accident. of the country on 1st January, 2015. Under the modified
— The persons should normally be head of the family or an scheme, the LPG consumer can now receive subsidy in
earning member of the family and should be in the age his bank account by two methods. Such a consumer will
group of 18 to 59. be called CTC (Cash Transfer Compliant) in the bank
— Account holder need to have valid RuPay card. The account account. The two options are as follows
can be any bank account including a small account. — Option I (Primary) For joining the scheme, the consumers
have to fill up a form available with distributors and also on
— Only one person in the family will be covered and in case of
www.mylpg.in. The consumers need to provide their Aadhaar
the person having multiple cards or accounts, the benefit
number to LPG distributor and to bank.
will be allowed only under one card i.e. one person per
— Option II (Secondary) If LPG consumer does not have an
family will get a single cover of ` 30000.
Aadhaar number, he can directly receive subsidy in his bank
— The claim of ` 30000 is payable to the nominee(s) of account without the use of Aadhaar number.
account holder who need to submit necessary documents to
the Nodal Branch of the concerned bank.
Health
— Government employees (serving/retired) and their families,
— Improvement in the standard of living and health status of
persons filling Income Tax Return or TDS deductees and
the population has remained one of the important
persons covered under the Aam Adami Bima Yojana, are
objectives for policymakers in India.
ineligible for life insurance under PMJDY.
— In line with the National Health Policy, 2002, the NRHM
Direct Benefit Transfer was launched on 12th April, 2005 with the objective of
— The government has decided to initiate direct transfer of providing accessible, affordable and quality healthcare to
subsidy under various social schemes into beneficiaries’ the rural population.
bank accounts. The transfer will be enabled through a — It seeks to bring about architectural correction in the
payments bridge known as Aadhar Payment Bridge (APB), health systems by adopting the approaches like increasing
wherein funds can be transferred into any Aadhar-enabled involvement of community in planning and management
bank account on the basis of the Aadhar number. of healthcare facilities, improved programme
— This eliminates chances of fraud or error in the cash management, flexible financing and provision of untied
transfer process. The Aadhar number will be linked to the grants, decentralised planning and augmentation of
beneficiary database, so that ghosts or duplicates are human resources.
weeded out from the beneficiary list.
150 Magbook ~ Indian Economy

— The government has launched a large number of Following are the key components of ASHA:
programmes and schemes to address the major concerns — ASHA must primarily be a woman resident of the village,
and bridge the gaps in existing health infrastructure and married, widowed or divorced, preferably in the age group of 25
provide accessible, affordable, equitable healthcare. to 45 years.
— She should be a literate woman with formal education up to
Ayushman Bharat Scheme class VIII. This may be relaxed only if no suitable person with
— It is centrally sponsored health insurance scheme of this qualification is available.
Ministry of Health and Family Welfare. It will cover of ` 5 — Capacity building of ASHA is being seen as a continuous
lakh per family per year, taking care of almost all secondary process. ASHA will have to undergo series of training episodes
care and tertiary care procedures. There will be no to acquire the necessary knowledge, skills and confidence for
performing her spelled out roles.
limitation on family size and age in the scheme. It was
announced in Union Budget 2018. — ASHA will mobilise the community and facilitate them in
accessing health and health related services available at the
Mission Indradhanush Anganwadi or sub-centre or primary health centres, such as
immunisation, Ante Natal Check-up (ANC), Post Natal
— The Union Ministry of Health and Family Welfare launched Check-up supplementary nutrition, sanitation and other
Mission Indradhanush on 25 December, 2014 to achieve services being provided by the government.
full immunisation coverage for all children by 2020. The — She will act as a depot older for essential provisions being
mission aims to cover all those children who are either made available to all habitations like Oral Rehydration
unvaccinated or are partially vaccinated against seven Therapy (ORT), Iron Folic Acid Tablet (IFA), Chloroquine,
vaccine preventable diseses including diphtheria, whooping Disposable Delivery Kits (DDK), Oral Pills and Condoms, etc.
cough, tetanus, polio, tuberculosis, mearles and — At the village level, it is recognised that ASHA cannot
hepatitis-B. function without adequate institutional support. Women’s
committees (like self-help groups or women’s health
National Rural Health Mission (NRHM) committees), village Health and Sanitation Committee of
— The NRHM, which provides an overarching umbrella to the the Gram Panchayat, peripheral health workers especially
existing health and family welfare programmes, was ANMs and Anganwadi workers and the trainers of ASHA
launched in 2005, to improve accessibility to quality and in-service periodic training would be a major source
healthcare for the rural population, bridge gaps in of support to ASHA.
healthcare, facilitate decentralised planning in the health
sector and bring about inter-sectoral convergence. Auxiliary Nurse Midwife (ANM)
— Better infrastructure, availability of manpower, drugs and — The roles of Auxiliary Nurse Midwife (ANM) and ASHA
equipment and augmentation of health human resources in have been integrated in various ways. The ANM will hold
health facilities at different levels have led to improvement in weekly or fortnightly meeting with ASHA and provide on
healthcare delivery services and increase in Out Patient job training by discussing the activities undertaken during
Department (OPD) and In Patient Department (IPD) services. the week or fortnight and provide guidance in case ASHA
— The NRHM is thus also about the health sector reform. The encounters any problem.
architectural correction envisaged under NHRM is organised
around 5 pillars, each of which is made up of a number of
Anganwadi Workers (AWW)
overlapping core strategies. — The responsibilities of Anganwadi Workers (AWW) will be
— Increasing participation and ownership by the community. to their role will be guide ASHA in performing on health
— Improved management capacity.
and integrated with the role of ASHA. AWW will guide
ASHA in performing activities such as organising Health
— Flexible financing.
Day once or twice a month at Anganwadi centre and
— Innovations in human resources development for the health
orientating women on health related issues such as
sector.
importance of nutritious food, personal hygiene, care
— Setting of standards and norms with monitoring.
during pregnancy, importance of immunisation etc.
Accredited Social Health Activist (ASHA) — Anganwadi worker will be depot holder for drug kits and
— One of the key components of the National Rural Health will be issuing it to ASHA. ASHA will support the AWW in
mission is to provide every village in the country with a mobilising pregnant and lactating women and infants for
trained female community health activist ASHA or Accredited nutrition supplement. She would also take initiative for
Social Health Activist elected from the village itself and bringing the beneficiaries from the village on specific days
accountable to it, the ASHA will be work as an interface of immunisation, health check-ups or health days etc to
between the community and the public health system. Anganwadi centres.
Magbook ~ Government Schemes and Programmes 151

India—Selected Health Indicators marginalised groups like reckshaw pullers, street vendors,
railway and bus station coolies, homeless people, street
S.No. Parameters Current Level children, construction site workers.
1. Crude Birth Rate (CBR) 21.8 (2011)
— The centre-state funding pattern will be 75:25 for all the
(per 1000 population)
States except North-Eastern states including Sikkim and
2. Crude Death Rate (CDR) 7.1 (2011)
(per 1000 population)
other special category states of Jammu and Kashmir,
Himachal Pradesh and Uttarakhand for whom the
3. Total Fertility Rate (TFR) 2.5 (2010)
centre-state funding pattern will 90:10. The Programme
(per woman)
Implementation Plans (PIPs) sent by the states are
4. Maternal Mortality Rate (MMR) 212 (2007-9)
apprised and approved by the ministry.
(per 100000 live births)
— It aims to address the public healthcare needs of urban
5. IMR (per 1000 live births) 44 (2011)
population. Addressing the needs of public healthcare
Rural 48
services for urban population would involve revamping or
Urban 29 creation. Upgradation of primary, secondary and tertiary
6. Child (0-4 years) Mortality Rate 13.3 (2010) healthcare services delivery system in urban areas.
(per 1000 children)
Life Expectancy at Birth (2006-10) Janani Suraksha Yojana (JSY)
Total 66.1 — JSY is continuation of the previous delivery allowance
scheme of the Central Government. It is being implemented
Male 64.6
with the objective of reducing maternal and neonatal
Female 67.7
mortality by promoting institutional delivery among poor
pregnant women.
National Urban Health Mission (NUHM) — Under this scheme, pregnant women belonging to below
— The National Urban Health Mission (NUHM) is a poverty line families and SC, ST families will get an
sub-mission of National Health Mission (NHM). NUHM assistance of ` 500 if delivered at home, ` 600 for urban
envisages to meet health care needs of the urban institutional delivery, ` 700 for delivery in health centers in
population with the focus on urban poor, by making rural areas and ` 1500 for cesarean delivery. This benefit is
available to them essential primary health care services and available if delivered in recognised private health institutions
reducing their out of pocket expenses for treatment. other than government hospitals also. The eligibility
— This will be achieved by strengthening the existing health conditions for the beneficiaries are as follows:
care service delivery system, targeting the people living in — The woman delivering at home or admitted to sub-centreor
slums and converging with various schemes relating to government hospital or registered private hospital (general
wider determinants of health drinking water, sanitation, ward), must belong to BPL family.
school education, etc. implemented by the ministries of — Current delivery must be the first or second live delivery.
Urban Development, Housing and Urban Poverty — She should be above 19 years of age and must have got ANC
Alleviation, Human Resource Development and Women check up at-least 3 times.
and Child Development. NUHM would endeavour to — Must have taken Iron and Folic acid tablets and TT injection.
achieve its goal through: — SC or ST women not belonging to BPL families are also entitled
— Need based city specific urban health care system to meet the for this benefit if they are admitted to general ward of
diverse health care needs of the urban poor and other government or registered private hospital.
vulnerable sections. — The JSY launched in 2005, aims to bring down the MMR by
— Institutional mechanism and management systems to meet the promoting institutional deliveries conducted by skilled birth
health-related challenges of a rapidly growing urban population. attendants.
— Partnership with community and local bodies for a more
proactive involvement in planning, implementation and Pradhan Mantri Swasthya Suraksha
monitoring of health activities. Yojana (PMSSY)
— Availability of resources for providing essential primary health — The PMSSY was launched in March, 2006, with aims at
care to urban poor service providers and other stakeholders.
correcting regional imbalances in the availability of
— Partnerships with NGOs for profit and not for profit health.
affordable or reliable tertiary healthcare services and
— NUHM would cover all state capitals, district headquarters augmenting facilities for quality medical education in the
and cities or towns with a population of more than 50000. country.
It would primarily focus on slum dwellers and other
152 Magbook ~ Indian Economy

Ayurveda, Yoga and Naturopathy, Unani, Sukanya Samridhi Yojana


Siddha and Homoeopathy (AYUSH) — Sukanya Samridhi Yojana was launched by Prime Minister
— The Indian system of medicines is also being developed and Narendra Modi under BBBP campaign in January, 2015.
promoted by involvement/integration of the AYUSH system in A small deposit scheme for girl child ‘Sukanya Samridhi
national healthcare delivery through an allocation of ` 990 Yojana’ was launched under the Beti Bachao, Beti Padhao
crore plan outlay in 2012-13. To integrate AYUSH healthcare (BBBP) campaign. With an aim to provide social security
with mainstream allopathic healthcare services, the for girls, the scheme will enable parents to open bank
states are provided financial support for co-location of accounts of girls who are under 10 years of age.
AYUSH facilities at PHCs, CHCs and district hospitals and — Thus, Sukanya Samridhi account can be opened in any
supply of essential drugs to standalone AYUSH hospitals or post office or authorised branches of commercial banks.
dispensaries. — The account will fetch an interest rate of 9.1% and no
income tax will be levied. The account can be opened at
Women Empowerment any time from the birth of a girl child till she attains the
Programmes age of 10 years, with a minimum deposit of `1000. A
maximum of `1.5 lakh can be deposited during the
Some of the important schemes and policy initiatives for financial year.
economic and social empowerment of women and child
— The account will remain operative for 21, years from the
development are as follows:
date of opening of the account. When the girl is 21, years
PMMSK Scheme old, she will get the entire amount. To meet the
requirement of higher education expenses, partial
— Pradhan Mantri Mahila Shakti Kendra Scheme (PMMSK)
withdrawal of 50% of the balance amount will be allowed
was approved by cabinet committe on Economic Affairs in
after the girl child has attended 18, years of age.
November 2017 for a period 2017-18 to 2019-20. PMMSK
Scheme is envisioned as one stop convergence support Pradhan Mantri Ujjwala Yojana
service for empowering rural women with opportunities for
— Prime Minister Narendra Modi has launched Pradhan
skill development, digital literacy, health and nutrition and
Mantri Ujjwala Yojana on 1st May, 2016 (Labour Day) at
employment. This scheme will perform under the patronage
Ballia (UP) by providing cooking gas connections to 10
of Ministry of Women and Child Development.
women.
Beti Bachao, Beti Padhao Yojana — The objective of the scheme is to provide cooking gas
— Government has introduced a new scheme called Beti connections to 5 million beneficiaries below the poverty
Bachao, Beti Padhao, which will help in generating line in the next 3 years (till the year 2019). Main
awareness and improving the efficiency of delivery of welfare objectives of this policy are
services meant for women with an initial corpus of ` 100 — Free LPG gas connection in the name of the female
crore. The government would focus on campaigns to member. It will be a cylinder and regulator.
sensitise people of this country towards the concerns of the — The scheme will include the rural and urban BPL family.
girl child and women. ` 1600 will be sent to Pradhan Mantri Jan Dhan Yojana bank
as subsidies.
— The process of sensitisation must being early and therefore
the school curriculum must have a separate chapter on Rajiv Gandhi Scheme Empowerment of
gender main streaming. Adolescent Girls (RGSEAG) Sabla
National Nutrition Mission (POSHAN — Sabla now operational in 205 selected districts, aims at
Abhiyan) all-round development of adolescent girls in the age group
11 to 18 years and making them self-reliant with a special
— National Nutrition Mission was launched as an expansion of
focus on out-of-school girls.
Beti Bachao Beti Padhao programme by Prime Minister
— The scheme has 2 major components, nutrition and
Narendra Modi at Jhunjhunu in Rajasthan on the occasion
non-nutrition. Nutrition is being given in the form of ‘take
of the International Women’s Day on March 8, 2018. The
home rations’ or ‘hot cooked meals’ to out-of -school 11 to
main objectives of this scheme are to attain proper
14 years old girls and all adolescent girls in the 14 to 18
nutritional status among children from 0-6 years,
age group.
adolescent girls, pregnant women and lactating mothers in
a timely manner; reduce stunting, under-nutrition, and — The non-nutrition component addresses the
anaemia among young children, women, and adolescent developmental needs of 11 to 18 years old adolescent
girls; and lowering low birth weight by at least 2% per girls, who are provided iron-folic acid supplementation,
annum. health check-up and referral services, nutrition and health
Magbook ~ Government Schemes and Programmes 153

education, counselling or guidance on family welfare, skill Rashtriya Mahila Kosh (RMK)
education, guidance on accessing public services and
— The RMK provides micro-credit in a quasi-informal
vocational training. The target of the scheme is to provide
manner, lending to Intermediate Micro-credit
nutrition to 1 crore adolescent girls in a year.
Organisations (IMOs) across states. It focuses on poor
Indira Gandhi Matritva Sahyog Yojana women and their empowerment through the provision
(IGMSY) of credit for livelihood-related activities.
— IGMSY, Conditional Maternity Benefit (CMB) is a centrally Other Women Empowerment
sponsored scheme for Pregnant and Lactating (P & L) women Programmes
to improve their health and nutrition status to better enabling
— Support to Training and Employment Programme for
environment by providing cash incentives to pregnant and
Women (STEPW) (set-up in 2003-04) To increase the
nursing mothers. It is being implemented using the platform
self-reliance and autonomy of women by enhancing their
of ICDS.
productivity and enabling them to take up income
— The scheme attempts to partly compensate for wage loss to generation activities.
pregnant and lactating women both prior to and after delivery
— Swayamsiddha (set-up in 2001) Aims at organising
of the child. The scheme envisages providing cash directly to
women into self help groups to form a strong institutional
the beneficiary through their Bank Accounts or Post Office
base.
Accounts, in response to individual fulfilling specific
conditions. — Swadhar (set-up in 1995) Aims to support women to
become independent in spirit, in thought, in action and
— The scheme covers all pregnant and lactating women above
have full control over their lives rather than be the victim
19 years of age and above for first 2 live births are entitled for
of others actions.
benefits under the scheme except all government or PSUs
(central and state). — Development of Women and Children in Rural Areas
(DWCRA) (set-up in 1982) To improve the
— The IGMSY is a conditional cash transfer scheme for pregnant
socio-economic status of the poor women in the rural
and lactating women implemented initially on pilot basis in 53
areas through creation of groups of women for
selected districts in the country from October, 2010. As on
income-generating activities on a self-sustaining basis.
31st December, 2012, more than 3 lakh beneficiaries had
been covered and ` 27 crores released to states. The scheme — Dhana Laxmi (set-up in March, 2008) Conditional cash
is now covered under the Direct Benefit Transfer (DBT) transfer scheme for the girl child to encourage families to
programme with 9 districts being included in the first phase. educate girl children and to prevent child marriage.
— Ujjawala (set-up in 4th December, 2007) A
Integrated Child Development Services comprehensive scheme for prevention of trafficking with
(ICDS) Scheme five specific components-prevention, rescue,
— Launched on 2nd October, 1975, ICDS scheme represents rehabilitation reintegration and repatriation of victims.
one of the world’s largest and most unique programmes for — Gender Budgeting Scheme (GBS) (set-up in 2004) with
early childhood development. ICDS is the foremost symbol of a view to empower women.
India’s commitment to her children– India’s response to the — Swawlamban (NORAD) (Norwegian Agency for
challenge of providing pre-school education on one hand and International Development) Scheme is being
breaking the vicious cycle of malnutrition, morbidity, reduced implemented by the Department of Women and Child
learning capacity and mortality, on the other. Development, Government of India with partial
assistance from Norway since, 1982.
National Mission for Empowerment of
— Its basic objective is to provide training and skill to
Women (NMEW) women to facilitate them obtain employment or
— This initiative for holistic empowerment of women through self-employment on a sustained basis.
better convergence and engendering of policies, programmes — The target group under the scheme are the poor and
and schemes of different ministries was operationalised in
needy women, women from weaker sections of the
2010-11.
society, such as scheduled castes, scheduled tribes etc.
— Under the mission, institutional structures at state level — Financial assistance is provided to undertake training
including State Mission authorities headed by Chief
programmes for women in both traditional as well as
Ministers and State Resource Centres for Women (SRCWs)
non-traditional trades.
for spearheading initiatives for women’s empowerment have
been established across the country.
154 Magbook ~ Indian Economy

Social Defence National Social Assistance Programme


(NSAP)
Major social defence programmes are given below:
— The National Social Assistance Scheme (NSAS) or National Social
Pradhan Mantri Jeevan Jyoti Assistance Programme (NSAP) is a flagship welfare programme
Bima Yojana (PMJJBY) of the Government of India initiated on 15th August, 1995.
— PMJJBY is one of the several ambitious social — Article 41 of the Indian Constitution directs the State to provide
security programmes initiated by Narendra Modi. public assistance to its citizens in case of unemployment, old age,
sickness and disablement and in other cases of undeserved want
— It is basically a term life insurance policy that can be
within the limit of its economic capacity and development. The
renewed either on a yearly basis or for a longer
scheme is a ‘giant step’ towards achieving the directive principle
period of time. It will provide life insurance coverage
in the Constitution.
on the death of the policy holder.
— NSAP at present comprises of Indira Gandhi National Old Age
— The Pradhan Mantri Jeevan Jyoti Bima Yojana willl be
Pension Scheme (IGNOAPS), Indira Gandhi National Widow
made available to anyone between the age group of
Pension Scheme (IGNWPS), Indira Gandhi National Disability
18 to 50 years.
Pension Scheme (IGNDPS), National Family Benefit Scheme
— The policy holders will need to pay INR 330 per year.
(NFBS) and Annapurna Scheme.
The risk coverage being provided in the Pradhan
Mantri Jeevan Jyoti Bima Yojana is INR 2 lakh. National Old Age Pension Scheme
— Life Insurance Corporation of India (LIC) will be — The National Old Age Pension scheme provides a pension for the
offering the plan. However, other life insurers, who elderly who live below the poverty line.
are eager to take part in the programme, can join it — The age of the applicant (male or female) should be 60 or above
through tie-ups with specific banks. (revised from 65 in 2009).
— The applicant may reside in either rural or urban areas, but must be
Atal Pension Yojana (APY) living under the poverty line.
— The idea of APY is to provide a definite pension to all — The amount of old age pension in ` 300 per month for applicants
Indians, However, in order to get pension during your aged 60 to 79. For applicants aged above 80 years, the amount has
old age, you need to contribute accordingly. been revised in ` 500 a month according to the 2012 budget.
— An Indian national within the age group of 18 to 40 — The ceiling on the total number of old-age pensions for purpose of
years is eligible to contribute under APY. claiming central assistance will be specified for the States and Union
Territories from time-to-time.
— There is also a policy under the scheme, wherein if
— The benefit under NOAPS should be disbursed in not less than two
the pension account holder dies, the contributions
installments in a year and, if possible, the benefit may be disbursed in
would go to the family or the nominee of the account.
more installments as per direction of State Government.
Pradhan Mantri Suraksha Bima — The death of a pensioner is immediately reported to the appropriate
Yojana (PMSBY) sanctioning authority by the Village Panchayats and municipalities
and the payments are promptly stopped by the same.
— PMSBY aims to reach poor people with it’s benefied
insurance scheme after the successful performance — Any pension amount sanctioned on the basis of false or mistaken
of Jan Dhan Yojana. information about eligibility can also be stopped or recovered by
the sanctioning authority.
— Benefits
— The death benefits are up to 2 lakh. National Family Benefit Scheme (NFBS)
— In case of irrecoverable and total loss of both hands, — In case of the death of the ‘primary breadwinner’ of a household
both eyes or sight or one leg or foot, the insurance living below poverty line conditions, a lump sum grant of ` 20000
cover would be up to 2 lakh. (from fiscal 2012-13) is provided to the household. The primary
— In case of lost of one leg, hand, foot eye or sight, the
breadwinner as specified in the scheme, whether male or female,
sum assured would be ` 1 lakh.
had to be a member of the household whose earning contributed
— The premium is just ` 12 per annum for each substantially to the total household income.
member.
— The death of such a primary breadwinner occurring whilst he or
— The aspirants should have completed 18 years of age she in the age group of 18 to 64 years i.e. more than 18 years of
or should not be more than to years of ager in order age and less than 65 years of age, makes the family eligible to
to get benefit of PMSBY. receive grants under this scheme.
Magbook ~ Government Schemes and Programmes 155

Indira Gandhi National Widow — The scheme also provides an add-on benefit of
scholarship of ` 100 per month per child paid on
Pension Scheme (IGNWPS)
half-yearly basis to a maximum of two children per
— A pension of ` 300 per month (From fiscal 2012-13) to be member studying in classes 9 to 12 (including ITI
granted to widows aged 40 to 59 living below poverty-line courses).
conditions. Pradhan of Gram Panchayat shall review the list
of widows and report in case of any re-marriage. Rashtriya Swasthya Bima Yojana
Indira Gandhi National Disability Pension (RSBY)
Scheme (IGNDPS) — The RSBY was launched on 1st October, 2007, to provide
smart card-based cashless health insurance cover of
— It is a component of National Social Assistance Programme
` 30000 per family per annum on a family floater basis to
(NSAP). Under IGNDPS, central assistance of ` 300 pm per
BPL families (a unit of five) in the unorganised sector.
beneficiary is provided to persons with severe or multiple
disabilities in the age group of 18 to 79 years and belonging — The scheme became operational from 1st April, 2008. The
to a household living Below Poverty Line (BPL) as per criteria premium is shared on 75:25 basis by the Centre and State
prescribed by Government of India. Governments. In the case of the North-Eastern states and
Jammu and Kashmir, the premium is shared in a 90:10
ratio. As on 31st December, 2012, the scheme is being
Other Social Protection implemented in 27 States or UTs with more than 3.34
Programmes crore smart card issued.
— Keeping in view the importance of the informal sector’s share
in total workforce, the government has been focusing on
Pradhan Mantri Vaya Vandana Yojana
expanding the coverage of social security schemes so as to (PMVVY)
provide a minimum level of social protection to workers in — Government has launched the ‘Pradhan Mantri Vaya
the unorganised sector and ensure inclusive Vandana Yojana (PMVVY)’ to provide social security
development.These include the following: during old age and to protect elderly persons aged 60
and above against a future fall in their interest income
Rashtriya Vayoshri Yojana due to uncertain market conditions. The scheme enables
— Rashtriya Vayoshri Yojana is a scheme for providing old age income security for senior citizens through
physical aids and assisted-living devices for senior citizens provision of assured pension/return linked to the
belonging to BPL category. It is launched in Nellore, subscription amount based on government guarantee to
Andhra Pradesh on 1st April, 2017. It is the first-of-its-kind Life Insurance Corporation of India (LIC).
Central Sector Scheme (CCS) in India, to be fully funded by
the Central Government. National Social Security Fund
— A National Social Security Fund for unorganised sector
Aam Admi Bima Yojana (AABY) workers with initial allocation of ` 1000 crores has been
— The Janashree Bima Yojana (JBY) has now been merged set-up. This fund will support schemes for weaver, toddy
with the AABY to provide better administration of life tappers, rickshaw pullers, beedi workers etc.
insurance cover to the economically backward sections of
society. Annapurna Scheme
— The scheme extends life and disability cover to persons — On 1st April, 2000 a new scheme known as Annapurna
between the ages of 18 and 59 years living below and scheme was launched. This scheme aimed at providing
marginally above the poverty line under 47 identified food security to meet the requirement of those senior
vocational and occupational groups, including ‘rural landless citizens who, though eligible, have remained uncovered
households’. under the other government scheme. Under the
Annapurna scheme, 10 kg of food grains per month are
— It provides insurance cover of a sum of ` 30000 on natural
provided free of cost to the beneficiary. The number of
death, ` 75000 on death due to accident ` 37500 for partial
persons to be benefited from the scheme are in the first
permanent disability due to accident and ` 75000 on death
instance, 20% of the persons eligible to receive pension in
or total permanent disability due to accident.
States or UTs.
156 Magbook ~ Indian Economy

Employment, Poverty, Rural and Urban Development Programmes till 2015


Name of the Programmes Years of Beginning Objectives/Descriptions
Training Rural Youth for Self-Employment (TRYSEM) 1979 Programme for training rural youth for self-employment.
Integrated Rural Development Programme (IRDP) 1980 All-round development of the rural poor through a
programme of asset endowment for self-employment.
National Rural Employment Programme (NREP) 1980 To provide profitable employment opportunities to the rural
poor.
Rural Landless Employment Guarantee Programme 1983 For providing employment to landless farmers and labourers.
(RLEGP)
Jawahar Rozgar Yojana 1989 For providing employment to rural unemployed.
Nehru Rozgar Yojana 1989 For providing employment to urban unemployed.
Scheme of Urban Wage Employment (SUWE) 1990 To provide wages employment after arranging the basic
facilities for poor people in the urban areas, where population
is less than 1 lakh.
Employment Assurance Scheme (EAS) 1993 To provide employment of at least 100 days in a year in
village.
Swarna Jayanti Shahari Rozgar Yojana (SJSRY), it has 1997 To provide gainful employment to urban unemployed and
been revamped with effect from April, 2009. The under employed poor through self employment of wage
revamped scheme has five components: Urban employment.
Self-Employment Programme (USEP); Urban Women
Self Help Programme (UWSP); Skill Training for
Employment Promotion amongst Urban Poor
(STEP-UP); Urban Wage Employment Programme
(UWEP); and Urban Community Development Network
(UCDN)
Swaranajayanti Gram Swarozgar Yojana (SGSY), it 1999 For eliminating rural poverty and unemployment and
replaced IRDP, DWCRA, Ganga Kalyan Yojana (1997), promoting self employment through establishing micro
Million Wells Scheme (1989) and Supply of Improved enterprises in rural areas. Targets to cover 50% SCs/STs,
Tool kits to Rural Artisans (1992) 40% women, 15% minorities and 3% disabled.
Jai Prakash Narayan Rozgar Guarantee Yojana Proposed in 2002-03 Employment guarantee in most poor districts.
(JPNRGY) Budget
Pradhan Mantri Gramodaya Yojana (PMGY) 2000 Focus on village level development in 5 critical areas, i.e.
primary health, primary education, housing, rural roads and
drinking water and nutrition with the overall objective of
improving the quality of life of people in rural areas.
Food for Work Programme 2001 To give food through wage employment in the drought
affected areas in eight states. Wages are paid by the State
Governments, partly in cash and partly in foodgrains.
MGNREGS (Mahatma Gandhi National Rural 2nd February 2006 It aims at enhancing livelihood security of households in rural
Employment Guarantee Scheme). The scheme was areas of the country by providing at least 100 days of
notified throughout the country with effect from 1st April, guaranteed wage employment in a financial year to every
household, whose adult members volunteer to do unskilled
2008. Renamed as MGNREGS from 2nd October, 2009.
manual work. It also mandates 33% participation for women.
SGRY and Food for Work Programme merged into it. The primary objective of the scheme is to augment wage
employment.
Prime Minister’s Employment Generation Programme 2008 To generate employment opportunities in rural as well as
(PMEGP) urban areas through setting up of new self-employment
ventures or projects or micro enterprises.
Atal Mission for Rejuvenation and Urban Transformation 2015 To upgrade urban infrastructure across 500 towns and
(AMRUT) cities.

Rural Development Programmes till 2015


Name of the Programmes Years of Beginning Objectives/Descriptions
Council for Advancement of People’s Actions and 1986 To provide assistance for rural prosperity.
Rural Technology (CAPART)
Integrated Wasteland Development Programme 1989-90 For the development of wasteland and degraded lands.
(IWDP)
District Rural Development Agency (DRDA) 1993 To provide financial assistance for rural development.
Member of Parliament Local Area Development 1993 It provides for ` 2 crore to each MP to undertake development
Programme (MPLAD) activities in its constituency. The amount has been raised to ` 5 crore
from 2011.
Magbook ~ Government Schemes and Programmes 157

Name of the Programmes Years of Beginning Objectives/Descriptions


Indira Awas Yojana 1999 To help construction of new dwelling units as well as conversion of
unserviceable kutcha houses into pucca or semi-pucca by members of
SC or STs, rural poor below the poverty line by extending them
grant-in-aid.
Total Sanitation Campaign (TSC) 1999 It follows a community led and people-centred approach and places
emphasis on Information, Communication and Education (ICE) for
demand generation of sanitation facilities.
Pradhan Mantri Gram Sadak Yojana (PMGSY) 2000 To line all villages with pakka road.
Annapurna Scheme 2000 To ensure food security for all, create a hunger free India in the next
5 years and to reform and improve the Public Distribution System,
so as to serve the poorest of the poor in rural and urban areas.
Valmiki Ambedkar Awas Yojana (VABAY) 2001 Facilitates construction and upgradation of dwelling units for slum
dwellers.
Nirmal Gram Puruskar (NGP) 2003 It is an incentive scheme to encourage PRIs to take up sanitation
promotion.
Bharat Nirman Programme 2005 Development of rural infrastructure including 6 components :
irrigation, water supply, housing, road, telephone and electricity.
Jawaharlal Nehru National Urban Renewal Mission 2005 To assist cities and towns in taking up housing and infrastructural
(JNNURM), it has 2 components. facilities for the urban poor in 63 cities (now 65 cities) in the
(a) Basic services to urban poor and country.
(b) Integrated Housing and Slum Development
Programme
National Rural Drinking Water Programme 2009 Aims to move forward from achieving habitation level coverage
(NRDWP)-previously called Accelerated Rural towards household level drinking water coverage through resorting to
Water Supply Programme. multiple sources like ground water and surface water etc.
Affordable Housing in Partnership (AHIP) 2009 Aims at constructing one million houses for the EWS or LIG or MIG
with at least 25% for EWS category seeks to operationalise National
Habitat Policy, 2007.
Saansad Adarsh Gram Yojana 2014 Rural development and cleanliness programme
Shyama Prasad Mukherji Rurban Mission 2015 To deliver integrated project based infrastructure in the rural areas.
Deendayal Upadhyaya Gram Jyoti Yojana 2015 Power supply to the rural areas
Deendayal Upadhyaya Grameen Kaushalya Yojana 2015 Skill upgradation of skill youth

Women Empowerment Programmes till 2015


Name of the Programmes Years of Beginning Objectives/Descriptions
Swadhar 1995 To support women to become independent in spirit, in thought, in
action and have full control over their lives rather than be the victim of
other’s actions.
Swayam Siddha 2001 At organising women into Self-Help Groups to from a strong
institutional base.
Support to Training and Employment Programme 2003-04 To increase the self-reliance and autonomy of women by enhancing
for Women (STEP) their productivity and enabling them to take up income generation
activities.
Ujjwala 2007 A comprehensive scheme for prevention of trafficking with 5 specific
components: prevention, rescue, rehabilitation, reintegration and
repatriation of victims.
Dhan Laxmi 2008 Conditional cash transfer scheme for the girl child to encourage
families to educate girl children and to prevent child marriage.
Indira Gandhi Matritva Sahyog Yojana (IGMSY) 2010 To improve the health and nutrition status of pregnant, lactating
women and infants.
National Mission for Empowerment of Women 2010 To achieve empowerment of women socially, economically and
(NMEW) educationally by securing convergence of schemes.
Rajiv Gandhi Scheme for Empowerment of 2010 It aims at empowering adolescent girls of 11 to 18 years by improving
Adolescent Girls (RGSEAG)- ‘Sabla’ their nutritional and health status, upgradation of home skills, life
skills and vocational skills.
158 Magbook ~ Indian Economy

Child Welfare Programmes till 2015


Name of the Programmes Years of Beginning Objectives/Descriptions
Reproductive and Child Health 1951 To provide quality integrated and sustainable Primary Health Care services to
Programme the women in the reproductive age group and young children and special focus
on family planning and immunisation.
Integrated Child Development Services 1975 It is aimed at enhancing the health, nutrition and learning opportunities of infants,
(ICDS) young children (0 to 6 years) and their mothers.
Rajiv Gandhi National Creche Scheme 2006 Overall development of children, childhood protection, complete immunisation,
for the Children of Working Mothers awareness generation among parents on malnutrition, health and education.
Scheme for Welfare of Working 2008-09 Provides for non-formal education, vocational training, etc to working children
Children in Need of Care and Protection to facilitate, their entry or re-entry into mainstream education.
Integrated Child Protection Scheme 2009-10 Providing a safe and secure environment for comprehensive development of
(ICPS) children, who are in need of care and protection as well as children in conflict
with law.
Bal Bandhu Scheme 2011 Provides for protection of children is areas of civil unrest. It is implemented by
NCPCR with grant from PM’s National Relief Fund.
Beti Bachao Beti Padhao Yojana 2014 Generating awareness and improving the efficiency of delivery of welfare
services meant for women.

Education Oriented Programmes till 2018


Name of the Programmes Years of Beginning Objectives/Descriptions
Mid-Day Meal Scheme (largest feeding 1995 Improving the nutritional status of children in classes I-VII in government, local
school programme in the world.) body and government aided schools and EGS and AIE centres with the end
objective of enabling disadvantaged and poor children to attend school regularly.
Sarva Shiksha Abhiyan (SSA) 2001 All children (6 to 14) complete 5 years of primary schooling by 2007; all children
complete 8 years of elementary schooling by 2010; bridge all gender and social
category gaps at primary stage by 2007 and at elementary education level by
2010; universal retention by 2010.
National Programme for Education of Girls 2003 Focused intervention to reach the ‘Hardest to Reach’ girls and provides for ‘Model
at Elementary Level (NPEGEL)- important School’ in every cluster with more intense community mobilisation and supervision
component of SSA of girls enrollment in schools.
Kasturba Gandhi Balika Vidyalayas (KGBVs) 2004 To set-up residential schools at upper primary level for girls belonging to SC, ST,
(with effect from) 1st April, 2007, merged OBC or Minority communities. The scheme is being implemented in rural areas
with SSA) and urban areas with female literacy below 30% and national average
respectively.
Rashtriya Madhyamik Shiksha Abhiyan 2009 Aims at raising the enrolment rate at secondary stage from 52.26% in 2005-06 to
(RMSA) or Scheme for Universalisation of 75% in next 5 years by providing a secondary school within a reasonable distance
Access for Secondary Education (SUCCESS) of 5 km of any habitation; ensure universal access by 2017 and universal retention
by 2020.
Saakshar Bharat 2009 National Literacy mission has been recased as ‘Saakshar Bharat’. The aim is to
cover all adults, is the age group of 15 and above, with its primary focus on
women.
Inclusive Education for the Disabled at 2009-10 Provides 100% central assistance for inclusive education of disabled children
Secondary Stage (IEDSS)-replaced Integrated studying in class IX-XII in government, local body and government-aided schools.
Education for Disabled Children (IEDC)
Atal Innovation Mission 2018 To promote innovation and entrepreneurship across the length and breadth of the
country. The other sections of AIM are the Atal Tinkering Lab and Atal Incubation
centre.

Health Oriented Programmes till 2018


Name of the Programmes Years of Beginning Objectives/Descriptions
National Rural Health Mission 2005 To provide effective healthcare to rural population with special focus on 18
(NRHM) states with weak health indices or infrastructure; to raise public spending on
health from 0.9% of GDP to 2.3% of GDP reduction of IMA and MMR; and
universal access to healthcare with emphasis on women.
Janani Suraksha Yojana (JSY)-a core 2005 Focus on demand promotion for institutional deliveries in states and regions
component of NRHM and targets lowering of MMR; it is a conditional cash transfer programme to
increase births in health facilities.
Pradhan Mantri Swasthya Suraksha 2010 To correct regional imbalance in tertiary healthcare and augmenting facilities
Yojana (PMSSY) for quality medical education in the country; and setting up 6 AllMS-like
institution in phase-1 and in phase-2, 2 more AllMS like institutions.
Ayushman Bharat or Pradhan Mantri 2018 Health Insurance Scheme for approx 10 crore families identified through socio
Jan Arogya Yojana economic caste census database. The scheme guarantees eligible families
expenses of upto ` 5 lack per year in any government or empanelled private
hospitals all over India.
Self Check
Build Your Confidence

1. Consider the following schemes launched by the Union 4. To provide an adequate quantities of essential fatty
Government [IAS 2001] acids and medicine for the common cold and
1. Antyodaya Anna Yojana diarrhoea.
2. Beti Bachao, Beti Padhao Yojana Which of the norms given above are correct about the
3. Sukanya Samridhi Yojana Mid-day Meal Scheme?
4. Jawahar Gram Samridhi Yojana (a) 1, 3 and 4 (b) 1, 2 and 3 (c) 1, 2 and 4 (d) All of these
Which of these were announced in the year 2014-15? 6. Consider the following statements
(a) 1 and 2 (b) 1, 2 and 3 (c) 2 and 3 (d) 3 and 4 1. Accidental insurance of 1 lakh is available to all RuPay
2. Consider the following statements [IAS 2000]
Card holder in the age group of 18-60 where RuPay
Card need to be usedonce in 2 months of recipt.
1. The National Heritage City Development and
2. Accidental insurance claim intimation should be given
Augmentation Yojana (HRIDAY) aims to bring together
his or her bank where account is maintained within
urban planning, economics growth and heritage
30 days from the date of accident.
conservation.
Which of the statement(s) given of accidental insurance
2. As per the announcement, the scheme will involve a
coverage under Pradhan Mantri Jan-Dhan Yojana is/are
partnership and local communities offer dable
correct?
technologies.
(a) Only 1 (b) Only 2
Which of the statement(s) given above is/are correct?
(c) Both 1 and 2 (d) Neither 1 nor 2
(a) Only 1 (b) Only 2
(c) Both 1 and 2 (d) Neither 1 nor 2 7. Consider the following [IAS 2012]

3. Consider the following statements about the Rashtriya 1. Hotels and restaurants
Madhyamik Shiksha Abhiyan (RMSA) 2. Motor transport undertakings
1. It aims to enhance access to secondary education in 3. Newspaper establishments
India. 4. Private medical institutions
2. It has quality intervention schemes to ensure that all The employees of which of the above can have the ‘Social
secondary schools conform to prescribed norms, Security’ coverage under Employees State Insurance
removing gender, socio-economic and disability scheme?
barriers. (a) 2 and 4 (b) 1, 2 and 3
3. It has the target to achieve the universal retention by (c) 2 and 3 (d) All of these
2020. 8. How do District Rural Development Agencies (DRDAs)
Which of the statements given above are correct? help in the reduction of rural poverty in India? [IAS 2012]
(a) 1 and 2 (b) 1 and 3 (c) 2 and 3 (d) All of these 1. DRDAs act as Panchayati Raj institutions in certain
4. Which one of the following areas of the country is mainly specified backward regions of the country.
emphasised in the Bharat Nirman project Launched by 2. DRDAs undertake area-specific scientific study of the
the Indian Government? causes of poverty and malnutrition and prepare
(a) Building infrastructure and basic amenities in the urban detailed remedial measures.
slum areas 3. DRDAs secure inter-sectoral and inter-departmental
(b) Building infrastructure and basic amenities in the rural areas co-ordination and co-operation for effective
(c) Building infrastructure and basic amenities in the implementation of anti-poverty programmes.
semi-urban areas 4. DRDAs watch over and ensure effective utilisation of
(d) Both ‘a’ and ‘c’ the funds intended for anti-poverty programmes.
Which of the statements given above are correct?
5. Consider the following norms of the Mid-day Meal
(a) 2 and 4 (b) 1, 2 and 3
Scheme
(c) 2 and 3 (d) All of these
1. To provide a minimum of 450 calories of food.
2. To provide a minimum 12 gram of protein. 9. Which one of the following statements is/are correct
3. To provide an adequate quantities of micro-nutrients about the Pradhan Mantri Gram Sadak Yojana
like iron, folic acid, vitamin-A. (PMGSY)?
160 Magbook ~ Indian Economy

(a) It provides connectivity to all the unconnected 16. Consider the following statements on National
habitations of more than 1000 persons in the rural areas Programme of Education for Girls at Elementary Level
by good quality all weather roads (NPEGEL)
(b) It was launched in 2005 as a fully funded centrally 1. It addresses issues at micro-level.
sponsored scheme 2. It is implemented in Educationally Backward Blocs .
(c) It provides connectivity to all unconnected habitations of
3. It addresses the needs of girls’s ‘in school.
more than 250 persons in the hilly and desert areas by
Which of the statement(s) given above is/are correct?
good quality all weather roads
(d) Both ‘a’ and ‘c’ (a) Only 1 (b) Only 2
(c) Only 3 (d) All of these

10. Which one of the following schemes subsumed the


Valmiki Awas Yojana? 17. Which of the following are the objectives of Nirmal Gram
Puruskar?
(a) Integrated Housing and Slum Development Programme
(IHSDP) 1. Bringing sanitation to the forefront of rural development
(b) Sampoorna Grameen Swarozgar Yojana (SGSRY) discourse.
(c) Rajiv Awas Yojana (RAY) 2. Incentivising the PRIs.
(d) Integrated Rural Development Programme (IRDP) 3. Increasing social mobilisation.
Select the correct answer using the codes given below
11. Which one of the following descriptions is correct about
the Jawaharlal Nehru National Urban Renewal Mission (a) 1 and 2 (b) 2 and 3
(JNNURM) of the Indian Government? (c) 1 and 3 (d) All of these
(a) JNNURM is the single largest initiative ever launched in 18. Which of the following are the major strategic shifts
the country to address the issues of urban infrastructure under the proposed National Rural Livelihood mission?
and basic services to the urban poor 1. Focus of allocation- based strategy.
(b) JNNURM will be implemented over a period of 7 years 2. Making SHGs the prime movers of rural development.
from 2005 to 2012 3. Making SHGs a subordinate banking institutions for
(c) The National Building organisation is the model agency making available easy credit.
for co-ordination of sanction, monitoring and review of
projects under BSUP and IHSDP Select the correct answer using the codes given below
(d) All of the above (a) 1 and 2 (b) 2 and 3
(c) 1 and 3 (d) None of these
12. Consider the following statements
1. MNREGA was launched in the Eleventh Five Year Plan.
19. Consider the following statements on NREGA (National
Rural Employment Guarantee Act) of 2005.
2. Indira Awas yojana was launched in the Ninth Five Year
Plan. 1. It is not a legally enforceable right to 60 days of
employment.
Which of the statement(s) given above is/are correct?
2. It provided economic and not social security to rural
(a) Only 1 (b) Only 2
workers.
(c) Both 1 and 2 (d) Neither 1 nor 2
3. It is not universal in the sense,; it is applicable only to
13. Consider the following statements about Pradhan BPL families.
Mantri Kaushal Vikas Yojana Which of the statement(s) given above is/are correct?
1. Pradhan Mantri Kaushal Vikas Yojana (PMKVY) is a (a) Only 1 (b) Only 2
flagship outcome-based skill training scheme. (c) Only 3 (d) None of these
2. It was unveiled anveited by Prime Minister Narendra
Modi on 20th March, 2015
20. Consider the following statements
1. Rashtriya Krishi Vikas Yojana (RKVY) was launched to
Which of the statement(s) given above is /are correct?
incentivise the states to enhance public investment to
(a) Only 1 (b) Only 2
achieve 4% growth rate in Agriculture and Allied sectors
(c) Both 1 and 2 (d) Neither 1 nor 2
during the Eleventh Five Year Plan period.
14. In order to provide pensions to workers of the 2. Rashtriya Krishi Vikas Yojana (RKVY) was launched in
unorganised sector, which one among the following 2006-07.
schemes has been launched by the government? Which of the statement(s) given above is/are correct?
(a) Swabhiman (b) Swavalamban (a) Only 1
(c) Swadhar (d) Aadhaar (b) Only 2
(c) Both 1 and 2
15. Janani Suraksha Yojana focuses on
(d) Neither 1 nor 2
1. reducing maternal mortality rates.
2. reducing infant mortality rates. 21. Which one of the following schemes is not included in
3. increasing life expectancy among women. the Rashtriya Krishi Vikas Yojana (RKVY)?
Which of the statement(s) given above is/are correct? (a) National Mission for Protein Supplements
(a) 1 and 2 (b) 2 and 3 (b) Saffron Mission
(c) Only 1 (d) All of these (c) Accelerated Fodder Development Programme
(d) All of the above
Magbook ~ Government Schemes and Programmes 161

22. Consider the following statements (a) To prevent the trafficking, rescue, rehabilitation, reintegration
1. Jawahar Rozgar Yojana was launched in the 7th Five and repatriation of women and children
Year Plan. (b) To assist women and children below 6 years to attain
self-sufficiency in the health-care and nutrition
2. Small-scale and food processing industries were given
(c) To prevent women from becoming victims to maternal
new impetus in 7th Five Year Plan.
mortality and infant mortality in the society
Which of the statement(s) given above is/are correct? (d) None of the above
(a) Only 1
(b) Only 2 28. Consider the following statements
(c) Both 1 and 2 1. Mahatma Gandhi National Rural Employment Guarantee
(d) Neither 1 nor 2 Scheme (MGNREGS) aims at enhancing livelihood
security of households in both rural and urban areas of
23. With regard to household saving, the budget proposes the country.
1. under the Rajiv Gandhi Equity Saving Scheme, the 2. The primary objective of the Mahatma Gandhi National
income limit for the first time investors is being raised Rural Employment Guarantee Scheme (MGNREGS) is to
from ` 10 lakh to ` 15 lakh. augment wage employment.
2. persons taking loans for first home up to ` 25 lakh will Which of the statement(s) given above is/are correct?
be entitled to an additional deduction of interest of upto (a) Only 1
` 1 lakh. (b) Only 2
3. inflation indexed bonds will be introduced to protect (c) Both 1 and 2
saving from inflation. (d) Neither 1 nor 2
Which of the statements given above are correct?
(a) 1 and 2 (b) 2 and 3
29. Which one of the following statements is correct about
(c) 1 and 3 (d) All of these
the Rajiv Gandhi Scheme for Empowerment of Adolescent
Girls SABLA launched by the Indian Government?
24. Consider the following statements about Rashtriya (a) The scheme will be implemented using the platform of
Swasthya Bima Yojana (RSBY) Anganwadi Centres (AWCs) of the Integrated Child
1. Under RSBY the premium is shared on 85 : 15 basis by Development Services (ICDSs) Scheme
the Centre and State Governments. (b) The cost sharing of the scheme would be on a 50 : 50
2. In the case of the North-Eastern States and Jammu and basis between the Centre and States
Kashmir, under RSBY the premium is shared in a 90 : 10 (c) The scheme is implemented by the Ministry of Rural
ratio. Development
Which of the statement(s) given above is/are correct? (d) Both ‘a’ and ‘b’
(a) Only 1 (b) Only 2
(c) Both 1 and 2 (d) Neither 1 nor 2
30. Consider the following statements about the Sarva
Shiksha Abhiyan
25. Consider the following statements 1. To achieve the universalisation of elementary education
1. Mahila Samakhya Scheme is implemented in 9 states. in a time bound manner.
2. Mahila Samakhya Scheme recognises the centrality of 2. To make free and compulsory education for children of
education in empowering women to achieve equality. ages 6-16.
Which of the statement(s) given above is/are correct? Which of the statement(s) given above is/are correct?
(a) Only 1 (b) Only 2 (a) Only 1 (b) Only 2
(c) Both 1 and 2 (d) Neither 1 nor 2 (c) Both 1 and 2 (d) Neither 1 nor 2
26. Consider the following about the Rajiv Gandhi Scheme 31. How does the National Rural Livelihood Mission seek to
for Empowerment of Adolescent Girls SABLA improve livelihood options of rural poor? [UPSC, 2012]
1. To enhance the nutritional status of adolescent girls. 1. By setting up a large number of new manufacturing
2. To empower women in achieving equality. industries and agribusiness centres in rural areas.
Which of the statement(s) given above is/are the aims of 2. By strengthening ‘self-help groups’ and providing suill
the SABLA Scheme? development.
(a) Only 1 (b) Only 2
3. By supplying seeds, fertilizers, diesel, pump-sets and
(c) Both 1 and 2 (d) Neither 1 nor 2
micro irrigation equipments free of cost to farmers.
27. Which one of the following is the main objective of the Select the correct answer using the codes given below
Ujjawala Scheme launched for women and children in (a) 1 and 2 (b) Only 2
India? (c) 1 and 3 (d) All of these

1. (c) 2. (d) 3. (d) 4. (b) 5. (b) 6. (b) 7. (a) 8. (d) 9. (c) 10. (a)
11. (d) 12. (b) 13. (c) 14. (b) 15. (a) 16. (d) 17. (d) 18. (b) 19. (d) 20. (c)
21. (d) 22. (c) 23. (b) 24. (c) 25. (c) 26. (c) 27. (a) 28. (b) 29. (a) 30. (a)
31. (b)
Chapter seventeen
International Financial and
Economic Organisation
(ii) The creation of International
International Bank for Reconstruction and
Organisations Development (IBRD)
also known as World Bank.
At the Bretton Woods — An international organisation has been
Conference in 1944 it was defined as a forum of co-operation of International
decided to establish new sovereign states based on multilateral
international agreement and comprising of Monetary Fund (IMF)
monetary order that would a relatively stable range of participants. — The International Monetary Fund
expand international trade, — The fundamental feature of this is the (IMF) is the inter-governmental
promote international existence of permanent organs with definite organisation that overseas the
competences and powers acting for the global financial systems by
capital flow and contribute
carrying out of common aims. following the macro-economic
to monetary stability. policies of its member countries, in
Role of International particular, those with an impact on
exchange rate and the Balance of
Organisation Payments (BoP). Its headquarter is
There are various functions of the International Washington DC, in United States.
Organisations (IOs), which are as follows : — The IMF was formally organised on
— One of the main functions of International 27th, December 1945, when the
Organisations (IOs), is that it keeps infact first 29 countries signed its Articles
the sovereignty of states and despite their of Agreement.
different social systems, establish and — Presently, the IMF has 189
expand peaceful co-operation among them. member countries. Nauru has
— The second main function is to ensure that become the 189th member of the
the competition among the individual states IMF on 12th April, 2016. It is a
remains peaceful. specialised agency of the United
Nations, but has its own charter,
governing structure and finances.
Bretton Woods Its members are represented
Conference through a quota system broadly
based on their, relative size in the
— The Bretton Woods Conference, officially
global economy.
known as the United Nations Monetary
and Financial Conference, was a gathering — A Global Financial Stability Report
of delegates to agree upon a series of new (GFSR) is developed and published
rules for the Post World War II International by the International Monetary
Monetary System. Fund. The report is issued twice a
year and provides updates on
— The two major accomplishments of the
current economic conditions and
conference were as follows :
financial markets worldwide. IMF’s
(i) The creation of the International
other publication is the World
Monetary Fund (IMF).
Economic Outlook.
Magbook ~ International Financial and Economic Organisation 163

IMF Lending — China is not yet the biggest economy in the world, its currency
is alleged to be manipulated by the government and China
— IMF loans are meant to help member countries tackle
limits the amount of its bonds that foreigners can hold.
Balance of Payments (BoP) problems, stabilise their
— There are however, many reasons to find an alternative to
economies and restore sustainable economic growth.
the US dollar as the global reserve currency. Firstly, it
— Today, IMF lending serves three main purposes : creates a global recessionary bias, both during and after
(i) First, it can smoothen adjustment to various shocks, financial crisis. Secondly, if the US succeeds in reigning in
helping a member country avoid disruptive economic its large budget and current account deficits, the global
adjustment or sovereign default, something that would liquidity will shrink. Thirdly, the large accumulation of
be extremely costly, both for the country itself and dollars by countries around the world like China has led to
possibly for other countries through economic and global imbalances.
financial ripple effects (known as contagion). — The SDR of IMF is not a currency. It is a potential claim
(ii) Second, IMF programmes can help to unlock other on the currencies of the member countries. To increase
financings, acting as a catalyst for other lenders. the importance of SDRs internationally, it should be issued
(iii) Third, IMF lending can help prevent crisis. The in larger quantities annually and it should be made the
experience is crisis capital account crisis typically inflicts main or only means of IMF financing.
substantial costs on countries themselves and on other
IMF Reforms
countries through contagion.
— On 15th December, 2010, IMF’s Board of Governors
IMF Quota approved a package of reforms of the fund’s quotas and
— When a country joins the IMF, it is assigned an initial governance completing the 14th General Review of Quotas.
quota in the same range as the quotas of existing — The reform package builds on the 2008 reforms, which
members that are broadly comparable in economic size became effective on 3rd March, 2011.
and characteristics. — The 14th General Review of quotas will :
— The quota determines the country’s financial contribution — double quotas from approximately SDR 238.4 billion to SDR
to the IMF, its voting power and ability to access IMF 476.8 billion (about US $ 720 billion).
— shift more than 6% quota share from over-represented
financing. Quota subscriptions generate most of the IMF’s
countries to under-represented countries.
financial resources.
— shift more than 6% quota share to dynamic emerging market
Special Drawing Rights (SDRs) and developing countries.
— realign quota share to make all 4 BRIC countries, among the
— SDRs is an international reserve asset, created by the IMF
top 10 largest shareholders in the fund.
in 1969, to supplement its member countries’ official
— preserve the quota and voting share of the poorest member
reserves. Its value is based on a basket of five key countries.
international currencies (US dollar, Japanese yen, pound — Subsequent to the 2010 reforms implementation, India’s
sterling, euro and Chinese renminbi) and SDRs can be quota share will go upto 2.79%, giving it a rank of 8th.
exchanged for freely usable currencies. Chinese renminbi IMF quota changes have to be approved by 85% votes
came into effect on 1st October, 2016. alongwith consent of the country, whose share is changed.
SDRs (Special Drawing Rights) as Global Reserve — However, repayment of all the loans taken from the IMF
Currency has been completed on 31st May, 2000. India is now a
— In recent times, there has been a call to make the SDRs contributor to the IMF.
i.e. the global reserve currency. In this light, there are — The quotas determine the amount of foreign exchange a
several issues related to reserve currency that we need to member may borrow from the IMF and its voting power on
understand. IMF policy matters. Quotas are denominated in SDRs.
— A reserve currency is one, which is held widely by Central Debt Relief
Banks and other financial institutions internationally and is
— In addition to concessional loans, some low- income
used for most international trade and other transactions.
countries are also eligible for debts to be written off under
— Having a currency as one of the global reserve currencies, two key initiatives. The Heavily Indebted Poor Countries
enables the issuer to import at lower rates than other (HIPC) Initiative and the Multilateral Debt Relief Initiative
countries, since they don’t need to pay transaction costs. (MDRI).
It also provides the ability to import according to the
needs, if needed, by simply printing more money. World Bank
— A currency to be a global currency should be widely
available in the international market, it should be freely
— The World Bank Group (WBG) is a family of five
convertible. Also, reserve currencies are generally held as international organisations that provide leveraged loans,
government bonds and not as hard cash. For these generally to poor countries. The bank came into formal
reasons, the Chinese Yen cannot yet become the global existence on 27th December, 1945 following international
reserve currency. ratification of the Bretton Woods Agreements, which
164 Magbook ~ Indian Economy

emerged from the United Nations Monetary and Financial International Finance Corporation (IFC)
Conference (1st to 22nd July, 1944). — The International Finance Corporation (IFC) promotes
— It also provided the foundations of the Osiander-Committee sustainable private sector investment in developing
in 1951, responsible for the preparation and evaluation of countries. It is a member of the World Bank Group and is
the World Development Report. headquartered in Washington DC.
— Commencing operations on 25th June, 1946 it approved — It shares the primary objective of all World Bank Group
its first loan on 9th May, 1947 ($ 250 million to France institutions: to improve the quality of the lives of people in
for (post war reconstruction), in real terms the largest its developing member countries. It has 184 member
loan issued by the bank (to date). countries.
— The World Bank itself comprises of two major — It promotes sustainable private sector development
organisations are as follows : primarily by
— Make the underlined organisations as bold. — financing private sector projects and companies located in the
(i) International Bank for Reconstruction and Development. developing world.
(ii) International Development Association. — helping private companies in the developing world mobilise
(Associated with the World Bank, but legally and financially financing in international financial markets.
separate are three other organisations). — providing advice and technical assistance to businesses and
— International Finance Corporation. governments.
— International Centre for Settlement of Investment Disputes. — IFC currently has 184 member countries.
— Multilateral Investment Guarantee Agency.
Multilateral Investment Guarantee Agency (MIGA)
— On 1st July, 2012, Jim Yong kim became the 12th President
of the World Bank Group. Kim’s new term will begin from — The Multilateral Investment Guarantee Agency (MIGA) is a
1st July, 2017. member organisation of the World Bank Group that offers
political risk insurance. It was established to promote
World Bank Groups foreign direct investment into developing countries.
International Bank for Reconstruction and Development — It promotes foreign direct investment into developing
(IBRD) countries by insuring investors against political risk,
— Founded in 1944, to help Europe recover from World advising governments on attracting investment, sharing
War-II, the International Bank for Reconstruction and information through online investment information services
Development (IBRD) works with middle-income and and mediating disputes between investors and
credit worthy poorer countries to promote sustainable, governments.
equitable and job-creating growth, reduce poverty and — 181 member countries comprise MIGA shareholders.
address issues of regional and global importance. Bhutan is the most country to have joined MIGA in
— IBRD is owned and operated for the benefit of its December, 2014.
189 member countries. Delivering flexible, timely and Objectives of MIGA
tailored financial products, knowledge and technical — Raising FDI inflows to the developing countries.
services and strategic advice helps its members achieve
— Reducing poverty.
results.
— Achieving higher economic growth.
International Development Association (IDA) — Increasing standard of livings.
— The International Development Association (IDA) is the International Centre for Settlement
part of the World Bank that helps the world’s poorest
of Investment Disputes (ICSID)
countries. Established in 1960, it aims to reduce poverty
by providing interest-free credits and grants for — The International Centre for Settlement of Investment
programmes that boost economic growth, reduce Disputes (ICSID), an institution of the World Bank Group
inequalities and improve people’s living conditions. based in Washington DC was established in 1966,
pursuant to the convention on the settlement of investment
— It serves middle-income countries with capital
disputes between states and nationals of other states (the
investment and advisory services. It is one of the largest
ICSID Convention or Washington Convention).
sources of assistance for the world’s 79 poorest
countries, 39 of which are in Africa. It is the single — ICSID has an administrative council, chaired by the World
largest source of donor funds for basic social services in Bank’s President and a Secretariat. It provides facilities for
the poorest countries. Presently, the IDA has 173 the conciliation and arbitration of investment disputes
countries. between member countries and individual investors.
— It lends money (known as credits) on concessional terms. — As of 2012, 161 countries had signed the ICSID
This means that IDA credits have no interest charge and convention.
repayments are stretched over 35 to 40 years, including * The two major accomplishments of the Brettan woods
a 10-years grace period. conference were the creation of IMF and IBRD.
Magbook ~ International Financial and Economic Organisation 165

* Apart from the grants to countries at risk of GENERAL AGREEMENT ON TARIFFS


debt distressed, IDA provides additional
consessional loans and grants and significant AND TRADE (GATT)
level of edli of through the HIPC and MDRI. — It was a multilateral agreement regulating international trade.
* Ease of doing business ranking is annually According to its preamble, its purpose was the ‘substantial
released by the World Bank. reduction of tariffs and other trade barriers and the elimination
of preferences, on a reciprocal and mutually advantageous
basis’. It was negotiated during the United Nations Conference
IMF vs World Bank on Trade and Employment and was the outcome of the failure
Similarities Both are global institutions since most countries of negotiating government to create the International Trade
of the world are their members. Both are concerned with Organisation (ITO). GATT was signed in 1947 and lasted until
strengthening the economies of member nations. 1994, when it was replaced by the World Trade Organisation in
Differences World Bank’s purpose is to promote economic and 1995. The original GATT text (GATT 1947) is still in effect under
social progress in developing countries. IMF’s purpose is to the WTO framework subject to the modifications of GATT 1994.
oversee and maintain an orderly system of payments and
receipts between nations.
World Trade Organisation (WTO)
— The WTO is an intergovernmental organisation which regulates
international trade.
The United Nations — The WTO officially commenced on 1st January, 1995 under the
Conference on Trade and Marrakesh Agreement, signed by 123 nations on 15th April,
1994. The WTO has 164 members. Afghanistan became the
Development (UNCTAD) 164th member on 29th July, 2016.
— The United Nations Conference on Trade and
Development (UNCTAD) was established in 1964, as Structure
a permanent inter-governmental body. It is the
principal organ of the United Nations General Highest Level Ministerial Conference
Assembly dealing with trade, investment and — The Ministerial Conference is the topmost body of the WTO,
development issues. which meets every 2 years. It brings together all the members
— The organisation’s goals are to “Maximise the trade, of WTO.
investment and development opportunities of
Second Level General Council
developing countries and assist them in their efforts
to integrate into the world economy on an equitable — The General Council of the WTO is the highest level
basis.” decision-making body in Geneva, which meets regularly to
carry out the functions of WTO.
Objectives of UNCTAD Third Level Councils for Trade
— The primary objective of the UNCTAD is to formulate — The workings of GATT, which covers international trade in
policies relating to all aspects of development goods, are the responsibility of the Council of Trade.
including trade, aid, transport, finance and
technology. Fourth Level Subsidiary Bodies
— There are subsidiary bodies under the various councils dealing
UNCTAD Conferences with specific subjects such as agriculture, subsidies, market
access etc.
— The highest decision-making body of UNCTAD is the
quadrennial (every 4 years) conference, at which Functions
member states make assessments of current trade
and development issues, discuss policy options and — Regulating WTO trade agreements.
formulate global policy responses. — Establish form for trade negotiations.
— The conference also sets the organisation’s mandate — Monitoring trade policies and handling trade disputes.
and work priorities. — Provide Technical Assistance.
— The conference is a subsidiary organ of the United — To co-operate with other international or trade organisations.
Nations General Assembly.
— The conferences serve an important political function:
Ease of Doing Business Index
they allow inter-governmental consensus building — The ease of doing business index is an index created by
regarding the state of the world economy and Simeon Djanhov at the world Bank group. Higher ranking
development policies and they play a key role in indicate better usually simpler, regulations for businesses and
identifying the role of the United Nations and UNCTAD in stronger protections of property rights. A nation’s ranking on
addressing economic development problems. the index is based on the average of 10 Subindexs - starting a
166 Magbook ~ Indian Economy

business, Dealing with construction permits, Getting Trade Related Aspects of Intellectual
electricity, registering property, Getting credit, Protecting Property Rights (TRIPS)
investors, paying taxes, trading across borders,
— The 1995, TRIPS Agreement provided for both product
enforcing contracts and Resolving inslovency.
patents and process patents. Product patents are meant to
protect the individual product, while process patent protects
Agreement on Agriculture the process used to create the product. The agreement gave
and its Implication developing countries 10 years to enact laws to protect
— The Uruguay Round produced the first multilateral intellectual property. Thus, India enacted its Patents
agreement dedicated to the sector. It was a significant (Amendment) Act in 2005, to conform to the agreement.
first step towards order, fair competition and a less Developed countries, on the other hand, had to enact laws
distorted sector. The Uruguay Round agreement in 1995 itself. Under the agreement, protection to patents
included a commitment to continue the reform through had to be provided for 20 years.
new negotiations. Non-Agricultural Market Access (NAMA)
— The objective of the agriculture agreement is to reform It refers to all products not covered by the agreement on
trade in the sector and to make policies more agriculture. In other words, in practice, it includes
market-oriented. This would improve predictability and
manufacturing products, fuels and mining products, fish and
security for importing and exporting countries alike.
fish products and forestry products.
— The new rules and commitments apply to
— Market access various trade restrictions confronting — The Uruguay round produced significant improvements in
imports. market access for NAMA products in the developed country
— Domestic support subsidies and other programmes, including markets, as tariff averages were reduced from 6.3% to 3.8%.
those that raise or guarantee farmgate prices and farmers’ — Binding coverage for NAMA products in developing
incomes. countries increased from 21% to 73%, which has
— Export subsidies and other methods used to make considerably increased the predictability of trade.
exports artificially competitive.
Boxes in WTO
— Least developed countries do not have to make
commitments to reduce tariffs or subsidies. — In WTO terminology, subsidies in general are identified by
‘Boxes’ which are given the colours of traffic lights‘: green
— The base level for tariff cuts was the bound rate before
(permitted), amber (slow down - i.e. be reduced), red
1st January, 1995 or for unbound tariffs, the actual rate
(forbidden). In agriculture, things are, as usual, more
charged in September, 1986 when the Uruguay Round
complicated.
began.
— The Agriculture Agreement has no Red Box, although
— The other figures were targets used to calculate
domestic support exceeding the reduction commitment
countries legally-binding ‘schedules’ of commitments.
levels in the Amber Box is prohibited; and there is a Blue
Box for subsidies that are tied to programmes that limit
Key Agreements of World Trade production. There are also exemptions for developing
Organisation countries (sometimes called an ‘S&D Box’).
World Trade Organisation agreements are as follows — The Green Box contains fixed payments to producers for
environmental programmes, so long as the payments are
General Agreement of Tariffs and Trade ‘decoupled’ from current production levels. All domestic
(GATT) support measures considered to distort production and trade
— It all began with trade in goods. From 1947 to 1994, (with some exceptions) fall into the Amber box. The
GATT was the forum for negotiating lower customs duty provision excepts 5% of agricultural production for
rates and other trade barriers, the text of the General developed countries, 10% for developing countries.
Agreement spelt out important rules, particularly — The Amber box subsidies with conditions designed to reduce
non-discrimination. distortion are placed in Blue box. They include the direct
payment to the farmers to reduce production. Blue box
General Agreement on Trade in Services
contains subsidies which can be increased without limit, so
(GATS) long as payments are linked to production-limiting
— Banks, insurance firms, telecommunications programmes.
companies, tour operators, hotel chains and transport
companies looking to do business abroad can now Intellectual Property
enjoy the same principles of freer and fairer trade that
originally only applied to trade in goods. These — The WTO’s Agreement on Trade-Related Aspects of
principles appear in the new General Agreement on Intellectual Property Rights (TRIPS), negotiated in the
Trade in Services (GATS). WTO members have also 1986-94 Uruguay Round, introduced intellectual property
made individual commitments under GATS. rules into the multilateral trading system for the first time.
Magbook ~ International Financial and Economic Organisation 167

The agreement covers five broad issues are as follows :


—

(i) How basic principles of the trading system and other


Organisation of Petroleum
international intellectual property agreements should be Exporting Countries (OPEC)
applied. — The Organisation of the Petroleum Exporting Countries
(ii) How to give adequate protection to intellectual property (OPEC) is a permanent, inter-governmental
rights. organisation, created at the Baghdad Conference
(iii) How countries should enforce those rights adequately in during 10th-14th September, 1960 by Iran, Iraq,
their own territories. Kuwait, Saudi Arabia and Venezuela.
(iv) How to settle disputes on intellectual property between — OPEC had its headquarters in Geneva, Switzerland, in
members of the WTO. the first 5 years of its existence. This was moved to
(v) Special transitional arrangements during the period when Vienna, Austria, on 1st September, 1965.
the new system is being introduced. — The current members are Algeria, Angola, Ecuador,
Iran, Iraq, Kuwait, Libya, Nigeria, Qatar, Saudi Arabia,
Other Important UAE and Venezuela. Currently, the organisation has a
total of 14 member countries.
International Organisations — OPEC’s objectives are to co-ordinate and unify
petroleum policies among member countries, in order
G-20 to secure fair and stable prices for petroleum
— The G-20 comprises 19 countries namely Argentina, producers; an efficient, economic and regular supply of
Australia, Brazil, Canada, China, France, Germany, India, petroleum to consuming nations and a fair return on
Indonesia, Italy, Japan, Mexico, Russia, Saudi Arabia, South capital to those investing in the industry.
Africa, the Republic of Korea, Turkey, the United Kingdom,
the United States of America and the European Union, Asia-Pacific Economic
which is represented by the rotating council presidency and Co-operation (APEC)
the European Central Bank as the 20th member. — Asia-Pacific Economic Co-operation or APEC, is the
— It represents 90% of the global gross national product, 80% premier forum for facilitating economic growth,
of the world’s trade and two-third of the world’s population. co-operation, trade and investment in the Asia-Pacific
region. APEC is the only inter-governmental grouping
India and G-20 in the world operating on the basis of non-binding
— India is a member of the G-20, since it was established as commitments, open dialogue and equal respect for the
Finance Ministers Forum in 1999. views of all participants.
— Currently, India is co-chair of the working group on G-20 — APEC has 21 members, referred to as Member
framework for strong, sustainable and balanced growth Economies, which account for approximately 40.5% of
alongwith Canada. the world’s population, approximately 54.2% of world
— India is contributing to various thematic issues being GDP and about 43.7% of world trade.
deliberated in G-20 such as : — APEC’s 21 member economies are Australia, Brunei
— Financial sector regulatory reforms Darussalam; Canada, Chile, People’s Republic of
— Climate change China, Hong Kong, Indonesia, Japan, Republic of
— IFIs (International Financial Institutions) reform Korea, Malaysia, Mexico, New Zealand, Papua New
— Growth and fiscal consolidation Guinea, Peru, the Republic of the Philippines, the
— Enhancing shareholding in forums such as FSB, IASB Russian Federation, Singapore, Chinese Taipei,
— Issues pertaining to Non-Cooperative Jurisdiction (Global Forum, Thailand, United States of America, Vietnam.
FATF etc)
Purpose and Goals
G-8 (Formerly G-7) — APEC was established in 1989, to further enhance
— Members Canada, France, Germany, Italy, Japan, Russia, UK economic growth and prosperity for the region and to
and USA. G-7 was organisation of 7 non-socialist countries, strengthen the Asia-Pacific Community. Since, its
which were highly industrialised in the world. inception, APEC has worked to reduce tariffs and other
— G-7 includes USA, Canada, Germany, Britain, France, Italy trade barriers across the Asia-Pacific region, creating
and Japan. After adopting free market policies in economy, efficient domestic economies and dramatically
Russia was also made member of the organisation in increasing exports.
June, 1997. At present, it is known as G-8. — Key to achieving APEC’s vision are, what are referred to
as the ‘Bogor Goals’ of free and open trade and
— G-8+5 This is a political term used to describe an
investment in the Asia-Pacific by 2010 for industrialised
international group, which consists the G-8 nations, plus the
economies and 2020 for developing economies. These
five leading emerging economies (Brazil, China, India,
goals were adopted by leaders at their 1994 meeting in
Mexico, South Africa).
Bogor, Indonesia.
168 Magbook ~ Indian Economy

The G-20 gave FSB more power in surveillance role. It


Organisation for Economic —
consists of representatives from National Financial Institu-
Co-operation and tions (like Central Banks, Finance Ministries, Regulatory
Development (OECD) Institutions) and International Financial Institutions.
— The Organisation for Economic Co-operation and Financial Action Task Force (FATF)
Development (French: Organisation de co-opération et de
development economics, OCDE) is an international — FATF is an international inter-governmental body
economic organisation of 34 countries founded in 1961, responsible for setting global standards for anti-money
to stimulate economic progress and world trade. laundering and combating financing of terrorism. Its
secretariat is located at the headquarters of OECD in Paris.
— It defines itself as a forum of countries committed to
democracy and the market economy, providing a platform — India became an observer of FATF in 2006 and
to compare policy experiences, seeking answers to subsequently the 34th full member in 2010. India agreed
common problems, identifying good practices and to implement short, medium and long range reforms
co-ordinating domestic and international policies of its related to anti-laundering and financing of terrorism to get
members. the membership. Membership will help India to build
capacity to fight terrorism, terrorist financing and to
— The OECD originated in 1948 as the Organisation for
successfully investigate and prosecute money laundering
European Economic Co-operation (OEEC), led by Robert
cases.
Marjolin of France, to help administer the Marshall Plan
for the reconstruction of Europe after World War-II. Asian Clearing Union (ACU)
— Later, its membership was extended to non-European
states. In 1961, it was reformed into the Organisation for
— ACU is an organisation with its headquarters at Tehran,
Economic Co-operation and Development by the Iran, established to promote regional co-operation. It was
Convention on the Organisation for Economic Co-operation established in 1974, at the initiative of United Nations
and Development. Economic and Social Commission for Asia and Pacific
(ESCAPs).
— Most OECD members are high-income economies with a
High Human Development Index (HDI) and are regarded
as developed countries (Chile being the only OECD
International Labour
member, which is also a member in the organisation of Organisation (ILO)
developing countries, the group of 77). — The International Labour Organisation emerged with the
League of Nations from the Treaty of Versailles in 1919.
Bank for International It was founded to give expression to the growing concern
Settlements (BIS) for social reform after World War I and the (convention)
that any reform had to be conducted at an international
— It was founded in 1930. The Bank for International level.
Settlements (BIS) is one of the oldest international
— The ILO has generated such hallmarks of industrial
financial organisations. Since then, the bank has operated
with the objective of fostering international monetary and society as the eight-hour working day, maternity
financial strength. The Bank for International Settlements protection, child labour laws and a range of policies,
which promote workplace safety and peaceful industrial
is also called ‘a Bank for all the Central Banks.’
relations.
— The statutes-of-the bank determine its governance. The
— The ILO has four principle strategic objectives are as
three major decision-making bodies of the bank are as
follows :
follows :
(i) To promote and realise standards and Fundamental
(i) Members of the Central Banks
Principles and Rights at Work.
(ii) Board of Directors
(iii) The Bank’s Management (ii) To create greater opportunities for women and men to
secure decent employment and income.
— From time-to-time, the BIS has rendered financial services
(iii) To strengthen tripartism and social dialogue.
to assist Central Banks. BIS is situated in Basel,
Switzerland. (iv) To enhance the coverage and effectiveness of social
protection for all.
Financial Stability Board (FSB)
— FSB was established in 2009, following the G-20 London
IBSA (India-Brazil-South Africa)
Summit and includes all G-20 major economies including — It is a international group free promoting international
India. It is based in Basel, Switzerland. co-operation between India, Brazil and South Africa.
— It was established to address vulnerabilities and to — It was created to promote South co-operation and build
develop and implement strong regulatory, supervisory and consensus on the issue of international importance.
other policies in the interest of financial stability. — It represents an important pole for galvanising.
Magbook ~ International Financial and Economic Organisation 169

It promote the welfare of the people of South Asia and to


BRICS (Brazil-Russia-India-China-South Africa) —
improve their quality of life.
— BRICS is the acronym for an association of 5 major emerging — It strengthen co-operation among themselves in
national economies : Brazil, Russia, India, China and South international forums on matters of common interest.
Africa.
— It was established to contribute to mutual trust,
— The BRICS first summit was held in yekaterinburg, Russia understanding and appreciation of one another’s
on 16th June, 2009 and the term was coined by economist problems.
Jim O’Neill.
— 10th summit of BRICS was held at South Africa in 2018 and SAARC Summits
11th coming summit is to be held at Brazil in 2019. Dates Hosts
European Union (EU) 7th-8th December, 1985 Dhaka (Bangladesh)
— European Union is an economic and Political Union of 16th-17th November, 1986 Bangalore (India)
28 member states that are located primarily in Europe. 2nd-4th November, 1987 Kathmandu (Nepal)
— The EU received the 2012 Nobal Piece Prize for having 29th-31st December, 1988 Islamabad (Pakistan)
“contributed to the advancement of peace and
21st-23rd November, 1990 Male (Maldives)
reconciliation, democracy and Human Rights in Europe”.
21st December, 1991 Colombo (Sri Lanka)
— On 1st July, 2013, Croatia became the 28th EU member.
10th-11th April, 1993 Dhaka (Bangladesh)
Brexit 2nd-4th May, 1995 New Delhi (India)
— Brexit is an acronym for british Exit. It is used for the 12th-14th May, 1997 Male (Maldives)
separation of the UK from European Union. In the Brexit 29th-31st July, 1998 Colombo (Sri Lanka)
referendum; the ‘leave’ side won decisively by securing 4th-6th January, 2002 Kathmandu (Nepal)
51.9% of the total votes.
2nd-6th January, 2004 Islamabad (Pakistan)
The Euro 12th-13th November, 2005 Dhaka (Bangladesh)
— It was established by the provisions in the year 1992, 3rd-4th April, 2007 New Delhi (India)
Maastricht Treaty. 1st-3rd August, 2008 Colombo (Sri Lanka)
— The Euro is the currency used by the institutions of the 28th-29th April, 2010 Thimphu (Bhutan)
European Union and is the official currency of the 10th-11th November, 2011 Addu (Maldives)
Eurozone, which consists of 18 of the 28 Member States of
26th-27th November, 2014 Kathmandu (Nepal)
the European Union.
9-10 November, 2016 Pakistan (cancelled)
Eurasian Economic Union (EEU) 2019 (may be) Sri Lanka
— The Eurasian Economic Union, which comes into force in
January, 2015, offers a unique chance for co-operation SAFTA
between Western Europe and the Asia-Pacific Region, the
lawmaker said. The EEU members are currently Russia, — The South Asian Free Trade Area (SAFTA) is an
Belarus, Kazakhstan and Armenia. Kyrgyzstan also joined agreement reacher on 6th January, 2004 at the
the union. Its headquarter is in Moscow (Russia). 12th SAARC summit in Islamabad Pakistan, to reduce
the East on duties of all goods to zero by the year 2016.
Eurasian Development Bank
— Its mission is to facilitate the development of market SAPTA
economies, economic growth and the expansion of trade its — The South Asian Preferential Agreement (SAPTA) is a
member states through investments. The bank’s objectives Inter-Governmental Group of SAAR couations. It was
also include financing projects that support Eurasian established on 11th April, 1993 and enter into force on
integration. The bank has provided financing totaling more 7th December, 1995.
than $ 4.5 billion to investment projects in its members
states. ASEAN
SAARC (South Asian Association for (Association of South-East Asian Nations)
Regional Co-operation) — It was established in 1967 in Bangkok, Thailand by
Indonesia, Malaysia, Philippines, Singapore and
— It was founded in 1985 and dedicated to economic, Thailand. Today it has 10 member (Apart from founding
technological, social and cultural development emphasising members Brunei, Darussalam, Vietnam, Lao PDR,
collective self-reliance. Myanmar and Cambodia)
170 Magbook ~ Indian Economy

— From to promote active collaboration and mutual assistance — The New Development Bank will mobilise resources for
on matters of common interest in the economic, social, infrastructure, sustainable development projects in
central, technical, scientific and administrative field. BRICS and other emerging economy and developing
— To promote South-East Asian studies. countries, to supplement existing efforts of multilateral
— To promote regional peace and stability through abiding regional financial institutions for global growth and
respect for justice and the rule of law in the relationship development.
among countries of the region and adherence to the
principles of the UN Charter. Asian Infrastructure Investment
Last Few Summits of ASEAN
Bank (AIIB)
— It is a Multilateral Development Bank (MDB) conceived
Dates Places for the 21st century. Chinese President Xi Jinping and
April, 2010 Hanoi (Vietnam)
Premier Li Keqiang announced the AIIB initiative
during their respective visits to South-East Asian
October, 2010 Hanoi (Vietnam) countries in October 2013.
May, 2011 Jakarta (Indonesia) — Representatives from 22 countries signed the October
November, 2011 Bali (Indonesia) 2014 memorandum of Understanding (MOU) to
2012 Naypyidaw (Myanmar) establish the AIIB and Beijing was selected to host
2012 Naypyidaw (Myanmar)
bank headquarters.
— By the deadline of March 31st for submission of
April, 2013 Brunei
membership applications, the Prospective Founding
October, 2013 Brunei Members had increased to 57, and the 4th CNM was
May, 2014 Myanmar organised in Beijing in April 2015, after ratifications
November, 2014 Myanmar were received from 10 member states holding a total
April, 2015 Malaysia
number of 50% of the initial subscriptions of the
Authorised Capital Stock. At present total member of
November, 2015 Manila, Philippines AIIB is 97.
April, 2016 Laos, Vientiane
November, 2017 Manila, Philippines Asian Development Bank (ADB)
April, 2018 Singapore — The Asian Development Bank was established following
June, 2019 Thailand the recommendations of the United Nations Economic
and Social Commission for Asia and the Pacific.
It was formed to foster economic growth and
Regional Financial Institutions —
co-operation in the region of Asia and the Pacific and
New Development Bank to contribute to the acceleration of economic
development of the developing countries of the region.
— The New Development Bank (NDB) formerly referred to as — The Asian Development Bank (ADB), an international
the BRICS Development Bank, is a multilateral development partnership of 67 member countries, was established
bank established by the BRICS states (Brazil, Russia, India, in 1966, with its headquarter is at Manila, Philippines.
China and South Africa). The bank is headquartered in India is a founder member.
Shanghai, China. The first regional office of the NDB will be
— The bank is engaged in promoting economic and social
opened in Johannesburg South Africa.
progress of its developing member countries in the
— The idea for setting up the bank was proposed by India at Asia and the Pacific region.
the 4th BRICS summit in 2012 held in Delhi. The creation of
— Its principle functions are as follows :
a new development bank was the main theme of the
— To make loans and equity investments for the economic
meeting. BRICS leaders agreed to set-up a development
and social advancement of its developing member
bank at the 5th BRICS summit held in Durban, South Africa countries.
on 27th March, 2013. — To provide Technical Assistance (TA) for the preparation
According to Devex, the name of the bank is believed to have and execution of development projects and programme
been proposed by Indian Prime Minister Narendra Modi. and advisory services.
— On 15th July, 2014, the first day of the 6th BRICS summit — To respond to the requests for assistance in co-ordinating
held in Fortaleza, Brazil, the BRICS states signed the development policies and plans in developing member
Agreement on the New Development Bank, which makes countries.
provisions for the legal basis of the bank. On 11th May, — To respond to the requests for assistance, co-ordinating
2015, K V Kamath was appointed as the President of the development policies and plans of developing member
bank. countries.
Self Check
Build Your Confidence
1. India is the member of the following international Select the correct answer using the codes given below
organisation [IAS 2009] (a) Only 1 (b) 1 and 3
1. Indian Ocean Rim Association for Regional Co-operation. (c) 2 and 3 (d) All of these
2. Organisation for Economic Co-operation and 8. In the context of International Trade Dumping refers to
Development.
(a) selling a commodity cheaper in foreign market and costly
Which of the statement(s) given above is/are correct? in domestic market
(a) Only 1 (b) Only 2 (b) sending cheap goods to developing countries by
(c) Both 1 and 2 (d) Neither 1 nor 2 developed countries
2. Consider the following statements (c) free distribution of used products by developed countries
in developing countries
1. IMF and World Bank both are Bretton Wood Heir.
(d) All of the above
2. World Bank provides long-term loan for promoting
balance economic development. 9. Consider the following statements.
3. IMF provides loans to eliminate BoP disequilibrium. 1. Gilt-edged market deals in bullion.
Which of the statements given above are correct? 2. World Development Report is an annual publication of
(a) 1 and 2 (b) 2 and 3 UNDP.
(c) 1 and 3 (d) All of these Which of the statement(s) given above is/are correct?
3. Consider the following statements (a) Only 1 (b) Only 2
(c) Both 1 and 2 (d) Neither 1 nor 2
1. IMF was established in 1944.
2. IMF was 188 member countries. 10. Consider the following statements
Which of the statement(s) given above is/are correct? 1. The Eurasian Commission was modelled on the
(a) Only 1 (b) Only 2 European Commission.
(c) Both 1 and 2 (d) Neither 1 nor 2 2. The Eurasian economic union offers unique chance for
co-operation between Western Europe and the
4. Which of the following statements is true for IMF?
Asia-Pocific Region.
(a) It is not an agency of UNO
Which of the statement (s) given above is/are correct?
(b) It can grant loan to any country
(a) Only 1 (b) Only 2
(c) It can grant loan State Governments of India
(c) Both 1 and 2 (d) Neither 1 nor 2
(d) It can grants loan only to member nations
11. The headquarters of World Trade Organisation are at
5. Choose the correct statements regarding the IBRD of the [UPPCS 2011]
World Bank Group.
(a) Doha (b) Geneva
1. IBRD gets its money from the capital markets. (c) Rome (d) New York
2. It has triple A' rating since 1959.
Which of the statement(s) given above is/are correct? 12. Regarding the International Monetary Fund (IMF),
which one of the following statements is correct?
(a) Only 1 (b) Only 2
[IAS 2011]
(c) Both 1 and 2 (d) Neither 1 nor 2
(a) It can grant loans to any country
6. Choose the correct option related to World Bank (b) It can grant loans to only developing countries
Organisations IBRD and IDA. (c) It can grant loans to only member countries
1. India use IDA funds on social sector. (d) It can grant loans to the Central Bank of a country
2. India use IBRD funds on infrastructure development. 13. Which one of the following issues the ‘Global Economic
3. IBRD loan is cheaper then IDA loans. prospects’ report Periodically?
Which of the statement(s) given above is/are correct? (a) The Asian Development Bank
(a) Only 1 (b) 1 and 3 (c) 1 and 2 (d) 2 and 3 (b) The European Bank for Reconstruction and
7. According to the UNCTAD which of the above is/are Development
categorised as ‘emerging economy, (c) The US Federal Reserve Bank
(d) The World Bank
1. Brazil 2. Mexico 3. South Africa

1. (a) 2. (d) 3. (c) 4. (b) 5. (c) 6. (c) 7. (d) 8. (a) 9. (c) 10. (c)
11. (b) 12. (c) 13. (d)
Economic Survey
2018-19
Economic Survey is document which serves as advice to the government.
The Department of Economic
It suggests problems faced by our economy and possible solution to them.
Affairs, Finance Ministry of India
presents the Economic Survey in Economic Survey 2018-2019
the Parliament every year, just Economic Survey 2018-19 aims to facilitate creation of a self-sustaining virtuous
before the Union Budget. The cycle which increases investment and growth. To achieve the objective of
Union Minister for Finance and becoming a $5 trillion economy by 2024-25 a sustained real GDP growth rate of
Corporate Affairs, Nirmala 8% is required to India.
Sitharaman presented the Overview of Indian Economy 2018-19
Economic Survey 2018-19 in the
— Growth of GDP moderated to 6.8 per cent in 2018-19 from 7.2 per cent in
Parliament on 4th July, 2019. The 2017-18. However, India was still the fastest growing major economy. This
Economic Survey 2018-19, moderation in growth momentum is mainly on account of lower growth in
prepared by Chief Economic ‘agriculture & allied’, ‘trade, hotel, transport, storage, communication and
services related to broadcasting’ and 'Public administration & Defence’
Adviser Krishnamurthy sectors.
Subramanian with his team. — Growth in investment, which had slowed down for many years, has bottomed
out and has started to recover since 2017-18. Growth in fixed investment
picked up from 8.3 per cent in 2016-17 to 9.3 per cent in 2017-18 and
further to 10.0 per cent in 2018-19.
— India maintained its macroeconomic stability by containing inflation within
4 per cent and by maintaining a manageable current account deficit to GDP
ratio. Inflation contained at 3.4% in 2018-19.
— Fiscal deficit of Central Government stood at a 3.4 per cent of GDP in
2018-19. Current account deficit manageable at 2.1% of GDP.
— Non-Performing Assets as percentage of Gross Advances reduced to
10.1 per cent at end December 2018 from 11.5 per cent at end
March 2018. 0utlook of Indian economy appears bright with prospects of
pickup in growth in 2019-20 on back of pick up in private investment and
robust consumption growth.

Fiscal Development
— The revised fiscal glide path envisaged achieving fiscal deficit of 3 per cent
of GDP by FY 2020-21 and Central Government debt to 40 per cent of GDP
by 2024-25.
— It aimed to reach the fiscal deficit target of 3.3 per cent of GDP in 2018-19
BE, with projections for 2019-20 and 2020-21 at 3.1 per cent and 3.0 per
cent, respectively.
— The FY 2018-19 has ended with debt to GDP ratio of 44.5 per cent.
The revised fiscal glide path envisages achieving debt to GDP ratio of 40%
by 2024-25.
Magbook ~ Economic Survey 2018-19 173

Prices and Inflation Services Sector


— Consumer Price Index (Combined) has been — As per the provisional estimates for Gross Value Added (GVA),
declining continuously for the last five years. services sector growth moderated in 2018-19 to 7.5 per cent
Headline CPI inflation declined to 3.4 per cent in rate from 8.1 per cent in 2017-18.
2018-19 from 3.6 per cent in 2017-18. Headline — FDI equity inflows into the services sector accounted for more
inflation based on CPI-C continued its declining trend than 60 per cent of the total FDI equity inflows into India.
for fifth straight financial year. It has remained below
4.0 per cent in the last two years. External Sector
— Mani contributors of headline inflation based on — As per WTO, World Trade Growth has slowed down to 3 per
CPI-C during FY 2018-19 are housing, fuel, light
cent in 2018, much below the growth rate of 4.6 per cent in
groups and miscellaneous. Relative importance of
2017 .
services in shaping up headline inflation has
increased. — India’s external debt was US$521.1 billion in the end of
December 2018 .
Sustainable Development and — The significant developments in this industry led to an increase
in the ranking of India in overall logistics performance,
Climate Change according to the Global Ranking of the World Bank’s 2016
— India follows a holistic approach towards its 2030 Logistics Performance Index. In 2018, India stood at 44th
SDG targets by launching various schemes. India’s rank.
SDG Index Score ranges between 42 and 69 for
States and between 57 and 68 for UTs. Social Infrastructure, Employment
— Sustainable Development Goals (SDGs) on education and Human Development
requires ensuring equitable, inclusive and quality
education along with promotion of lifelong learning — The government has taken many steps to ensure a sustainable
opportunities for all by 2030. growth by enhancing the efficiency of the expenditure incurred
on human capital through the convergence of schemes.
— Kerala and Himachal Pradesh are the front runners
amongst all the states with a score of 69, Chandigarh — A crucial step towards the achievement of Sustainable
and Puducherry are the front runners with a score of Development Goals(3) is India's National Health Policy, 2017.
68 and 65 respectively among the UT’s. — World Bank estimates that the lack of sanitation facilities costs
— Namami Gange Mission launched as a key policy India over 6% of GDP.While UNICEF estimated that a
priority towards achieving SDGs with a budget outlay household in an ODF village saves approx ` 50,000.
of ` 20 thousand crore for the period of 2015-20. — Gender gap in labour force participation in India is as high as
50% which adversely affects the growth potential of the
Industry and Infrastructure country. The disadvantaged position of women workers is due
to their low levels of skills and engagement in low productivity
— The industrial growth in terms of Index of Industrial and low paying work.
Production (IIP) registered 3.6 per cent in 2018-19 In India the number of schools with SCR more than
as compared to 4.4 per cent growth rate in 2017-18. 30 declined from 43% in 2009-10 to 25.7% in 2015-16 with
— The overall Index of eight core industries registered a varied improvement in almost all states.
growth rate of 4.3 per cent in 2018-19 similar to the
increase achieved in 2017-18.
— Building sustainable and resilient infrastructure has NUDGE : The Newer Chapter in Economic Survey
been given due importance with the formulation of ◆
First time in Economic Survey 2018-19 ‘NUDGE’ (Behavioural
sector specific programmes such as Saubhagya Economics) is incorporated. Drawing on the psychology of human
Scheme, PMAY etc.
behaviour, behaviour economics provides insight to ‘NUDGE’
— In April, 2018 electricity finally reached every village people towards desirable behaviour.
in India. The construction of National Highways (NH) ◆
Swachh Bharat Mission (SBM) and Beti Bachao-Beti Padhao
proceeded at a rapid pace with more than 20% of (BBBP) have successfully employed behavioural insight.
existing highways length of 132000 km.
Union Budget
2019-20

Introduction History of Budget


Finance Minister Nirmala
— The budget process of India predates
Sitharaman, in her maiden — The Budget of the Government of India independence. Budget was first
Budget on 5th July, 2019, provides a complete picture of introduced on 7th April, 1860, 2 years
estimated receipt and expenditure for
sought to tackle several key after the transfer of Indian
the next fiscal year.
points in the economy administration from East India
— In the budget document, there are Company to British Crown. James
through incremental steps. three sets of figures which are as Wilson was the first ‘Finance Member’ to
Budget 2019-20 reflects follows
present the budget.
Government’s firm 1. Actual receipts and expenditure of
the preceding year.
— After independence, India’s first
commitment to substantially Finance Minister RK Shanmukham
2. Revised estimate of the receipt and
Chetty, presented the first budget on
boost investment in expenditure of the current year.
26th November, 1947. Morarji Desai
Agriculture, Social sector, 3. Estimated receipt and expenditure has presented the maximum budgets
for the following year.
Education and Health, (eight annual and two interim).
Infrastructure Digital — The budget is divided into two parts — Referred to as the Annual Financial
1. Revenue budget Statement in Article 112 of the
Economy and MSME for job 2. Capital budget Constitution of India. Union Finance
creation. Government Minister presents it on the first day of
— Revenue Budget contains all the
aspires to make India a receipt and expenditure of the current February. Until 2016 it was presented
$5 trillion economy up to nature. on the last working day of February.
the year 2024-25. Finance — Revenue receipt is further divided into — It is the most extensive account of the
two parts government’s finances, with two basic
Minister has stated that 1. Tax revenue elements being the revenues and
Indian economy will become 2. Non-tax revenue expenses of the Union Government
$ 3 trillion economy in the — Tax Revenue is taken as all the income employees directly involved in
from various taxes net to centre. preparing and printing the budget
current year. papers are kept in isolation for a few
Important items of non-tax revenue are
interest, dividend, currency and days till it is presented.
coinage etc. — The First Union Budget of independent
— Capital Budget consists of capital India was presented by RK
Shanmukham Chetty on 26th
receipt and capital expenditure,
November, 1947.
sub-heads of capital budget are as
under Capital receipt includes net — Indira Gandhi, the then Prime Minister
recoveries of loan and advances to of India, took over the Ministry of
States/UT and Public sector Finance to become the only woman to
undertaking small saving collection, hold the post of the Finance Minister.
borrowing and other receipt. — Pranab Mukherjee the first Rajya Sabha
— Capital Expenditure include loan to Member to hold the Finance portfolio
States/Union Territories and presented the Annual Budgets for
expenditure of capital nature. 1982-1983-84 and 1984-85.
Magbook ~ Union Budget 2019-20 175

Union Budget 2019-20 Towards a $5 Trillion Economy


The Budget Revised Estimates for expenditure in 2019-20 are ` — Union Finance Minister Nirmala Sitaraman, in her
2786349 crore. This is substantiated by increase in expenditure of speech said that government of India aspires to make
` 3,29,114 crore over revised estimates (2018-19) while keeping India a $5 trillion economy till 2024-25. Indian
the fiscal deficit at 3.3% of GDP. economy to become a $3 trillion economy in the
current year (2019-20).
Budget at a Glance (In ` Crore) — To achieve the target to becoming a $5 trillion
2018-19 2018-19 2019-20 economy, the economic growth rate of India would be
2017-18 over 8% per year till 2020-24.
Budget Revised Budget
Actual
Estimates Estimates Estimates
Infrastructure
1. Revenue Receipts 1435233 1725738 1729682 1962761
— Upgradation of 1,25,000 km of road length over the
2. Capital Receipts 706740 716475 727553 823588
next five years under the Pradhan Mantri Gram
3. Total Receipts 2141973 2442213 2457235 2786349 Sadak Yojana III.
4. Total Expenditure 2141973 2442213 2457235 2786349 — 1.95 crore houses to be provided to eligible,
5. Revenue Deficit 443600 416034 410930 485019 beneficiaries during 2019-20 to 2021-22, under the
6. Effective Revenue 252566 220689 210630 277686
phase two of Pradhan Mantri Awas Yojana (Gramin).
Deficit — Credit Guarantee Enhancement Corporation to be set
7. Fiscal Deficit 591062 624276 634398 703760 up in 2019-20 for infrastructure financing.
— Government to organise an annual Global Investors
8. Primary Deficit 62110 48481 46828 43289
Meet in India, using National Infrastructure
*Figures have been rounded. Investment Fund (NIIF) as an anchor to get all three
Rupees Comes From (Budget 2019-20) sets of global players (Pension, Insurance and
Sovereign Wealth Funds).
Non-Debt Borrowing and
Capital Receipts Other Liabilities — Cumulative resources garnered through new
(20 p.)
(3 p.) Financial Instrument Trusts (Inv ITs), Real Estate
Non-Tax Investment Trusts (REITs) as well as models like
Revenue
(3 p.) Corpoation Toll-Operate and Transfer (TOT) exceed ` 24,000
Tax crore.
(21 p.)
Goods and — To expand Swachh Bharat Mission to undertake Solid
Service Tax
(19 p.) Waste Management in all village.
— To permit investments made by FIIs/FPIs in debt
Income-Tax
(16 p.) securities issued by IDF-NBFCs to be transferred/sold
Union Excise
Duties Customs to any domestic investor within the specified lock-in
(8 p.) (4 p.) period.
Rupees Goes To (Budget 2019-20) Rural India/Urban India
Centrally Sponsored
Scheme (9 p.)
— Electricity and clean cooking to all willing families by
Other Expenditure (8 p.) 2022.
Central Sector
Scheme — Pradhan Mantri Awas Yojana-Gramin (PMAY-G) aims
Pensions (5 p.) (13 p.) to achieve ‘Housing for All’ by 2022.
— Pradhan Mantri Gram Sadak Yojana (PMGSY) :
States Share
of Taxes
Interest 30,000 kilometres of PMGSY roads have been built
Payments using Green Technology, Waste Plastic and Cold Mix
and Duties
(18 p.)
(23 p.) Technology, thereby reducing carbon footprint.
— Pradhan Mantri Matsya Sampada Yojana (PMMSY),
Finance a robust fisheries management framework through
Commission Defense (9 p.)
and Other
PMMSY to be established by the Department of
Subsidies (8 p.) Fisheries under Ministry of Agriculture & Farmer’s
Transfers (7 p.)
Welfare.
Note : Here, P → Paise.
176 Magbook ~ Indian Economy

— Government has proposed zero budget farming. Zero Railways


budget farming is technically known as Zero Budget — ` 50 lakh crore investment needed in Railway
Natural Farming (ZBNF), as the Food and Agriculture Infrastructure during 2018-2030.
Organisation of the United Nations calls it. Zero budget
— Public-Private-Partnership proposed for development and
farming is a set of farming methods that involve zero credit
completion of tracks, rolling stock manufacturing and
for agriculture and no use of chemical fertilisers.
delivery of passenger freight services. 657 kilometres of
— Jal Jeevan Mission to achieve Har Ghar Jal (piped water Metro rail network has become operational across to the
supply) to all rural household by 2024. country.
— 1592 critical and over exploited Blocks spread across 256 — Railways to be encouraged to invest more in suburban
District being identified for the Jal Shakti Abhiyan. railways through SPV (Special Purpose Vehicle) structure
— New Jal Shakti Mantralaya to look at the management of like Rapid Rail Transport System (RRTS) proposed on the
our water resources and water supply in an integrated and Delhi-Meerut route.
holistic manner. — A massive program of railway station modernisation to be
— Swachh Bharat Mission to be expanded to undertakes launched.
sustainable solid waste management in every village.
— Pradhan Mantri Gramin Digital Saksharta Abhiyan will
Micro, Small and Medium Enterprises (MSMEs)
provide Internet connectivity in local bodies in every — ‘‘Make in India’’, with particular emphasis on Micro, Small
Panchayat under BharatNet to bridge rural-urban divide. and Medium Enterprises, is one of the major focus areas
of the Union Budget this year. For ease of access to
— Universal Obligation Fund under a Public-Private
credit of MSMEs, Government has introduced scheme for
Partnership (PPP) arrangement to be utilised for speeding
providing of loans upto ` 1 crore within 59 minutes
up BharatNet.
through a dedicated online portal.
— In ‘Pradhan Mantri Awas Yojna (Urban)’ over 81 lakh
— Government has decided to extend pension benefit to
houses with an investment of about ` 4.83 lakh crore
about 3 crore retail traders and small shopkeepers whose
sanctioned, of which construction started in about 47 lakh
annual turnover is less than ` 1.5 crore under a new
houses.
scheme named ‘Pradhan Mantri Karam Yogi Maandhan
— More than 95% of cities also declared Open Defecation Scheme’.
Free (ODF). Target of achieving Gandhiji’s resolve of
— Enrolment to be kept simple, requiring only Aadhaar,
Swachh Bharat to make India ODF by 2nd October, 2019.
bank account and self declaration. Apart from that,
— Gandhipedia being developed by National Council for among other measures related to the MSMEs, ` 350 crore
Science Museums to sensitise youth and society about allocated for FY 2019-20 for 2% interest subvention (on
positive Gandhian value. fresh or incremental loans) to all GST registered MSMEs,
Education under the Interest Subvention Scheme for MSMEs.
— An amount of ` 400 crore has been provided for FY — Enrolment into the scheme will be kept simple requiring
2019-20 to create ‘‘World Class Institution’’ in the field of only Aadhaar and a bank account and rest will be on
education which is more than three times the revised self-declaration.
estimates for the previous year. Taxation
— New National Education Policy to be brought which — GST on electric vehicles cut to 5%.
proposes major changes in both school and higher
— Additional income tax deduction of ` 1.5 lakh on interest
education.
on loans taken to buy electric vehicles.
— Draft legislation to set up Higher Education Commission of
— Customs duty on gold and other precious metals
India (HECI), to be presented in the budget.
increased from 10% to 12.5%.
— National Research Foundation (NRF) proposed to fund
— Government announced enhancement of surcharge of
and coordinate in research.
3% on individuals having taxable income from ` crore to `
National Sports Education Board (NSEB) 5 crore and 7% for those with taxable income of ` 5 crore
— Setting up of the National Sports Education Board (NSEB) and above. At the same time, several measures are
is the new addition in the Union Budget that was for the announced to make tax administration and tax payment
development of sportspersons to be set up under Khelo easier. Those without Pan Card are now allowed to file
India, to popularise sports at all levels. income tax returns by quoting their Aadhaar number.
Magbook ~ Union Budget 2019-20 177
— The budget proposes a dispute resolution-cum-amnesty — Business establishment with annual turnover more than
scheme, called ‘‘the Sabka Vishwas Legacy Dispute ` 50 crore shall offer low cost digital modes of payment to
Resolution Scheme, 2019’’ that will allow quick closure their customers and no charges or Merchant Discount Rate
of tax litigations. shall be imposed on customers as well as merchants.
— A Scheme of Faceless e-assessment in electronic mode — To facilitate on-shoring of international insurance
involving no human interface is being launched this year transaction.
in a phased manner. To start with, such e-assessments
Disinvestment
shall be carried-out in cases requiring verification of
certain specified transactions or discrepancies. — Target of ` 1,05,000 crore of disinvestment receipts set for
the financial year 2019-20.
— Tax rate reduced to 25% for companies with annual
turnover up to ` 400 crore. — Government to reinitiate the process of strategic
disinvestment of Air India, and to offer more Central Public
— Direct tax revenue increase by over 78% in past 5 years
Sector Enterprises (CPSEs) for strategic participation by the
to ` 11.37 lakh crore.
private sector.
— In the Union Budget, Affordable housing gets further
— There is 100% FDI allowed in aviation, but foreign airlines
encouragement in form of additional tax deduction of
can only invest up to 49% in an India airlines.
` 1.5 lakh beyond ` 2 lakh of interest paid on loans
borrowed upto 31st March, 2020 for purchase of an — Government to consider going to an appropriate level below
affordable house valued up to ` 45 lakh. 51% in PSUs where the government control is still to be
retained, on case to case basis.
Banking and Financial Sector — Government to offer an investment option in ETFs
— Public Sector Banks are to be further provided ` 70,000 (Exchange Traded Fund) on the lines of Equity Linked
crore capital to boost credit for a strong impetus to the Savings Scheme (ELSS).
economy. To further improve ease of living, they will — Government to meet public shareholding of 25% for all
leverage technology, offering online personal loans and listed PSUs and raise the foreign shareholding limits to
doorstep banking, and enabling customers of one Public maximum permissible sector limits for all PSU companies
Sector Bank to access services across all Public Sector which are part of Emerging Market Index.
Banks.
— NPAs (Non Performing Assets) of commercial banks Miscellaneous
reduced by over ` 1 lakh crore over the last year. — A Public Sector Enterprise, New Space India Limited (NSIL)
— Return of regulatory from NHB (National Housing Bank) has been incorporated as a new commercial arm of
to RBI proposed over the housing finance sector. Department of Space to tap the benefits of the Research
and Development carried out by ISRO.
— Reduction of Net Owned Fund requirement from ` 5,000
crore to ` 1,000 crore proposed. — Stand-up India Scheme to be continued for the period of
2020-25. The banks to provide financial assistance for
— Domestic credit growth increased to 13.8%.
demand based busineses.
— For purchase of high-rated pooled assets of financially
— ` 1 lakh loan under MUDRA Scheme for one women in
sound NBFCs, amounting to a total of ` 1 lakh crore
every SHG (Self Help Group).
during the current financial year, Government will
provide one time six months’ partial credit guarantee to — Achieving a healthy society via Ayushman Bharat, well
Public Sector Banks for first loss of up to 10%. nourished women and children, safety of citizens.
— ` 100 lakh crore investment in infrastructure intended — Defence equipment not manufactured in India exempted
over the next five years in the field of Non-Banking from basic custom duty (India had spent 2.4% of GDP on
Financial companies. defence).
— TDS (Tax Deducted at Source) of 2% on cash — 100% FDI to be permitted for insurance intermediaries.
withdrawal exceeding ` 1 crore in a year from a bank — Minimum public shareholding in listed companies to be
account. increased from 25% to 35%.
178 Magbook ~ Indian Economy

— 8 crore more free LPG connections to be given under


Ujjwala Yojana. Ten Points of the Government’s ‘Vision
— National Ganga Plan and Ghat works shows only for the Decade’ in the Union Budget
` 750 crore for 2019-20, a sharp fall from ` 2,250 crore 2019-20
allotted last year. ◆
Building physical and social infrastructure.
— 17 Iconic Tourism Sites being developed into model ◆
Making Digital India reach every sector of the economy.
world class tourist destinations. ◆
Achieving Green Mother Earth and Blue Skies through a
— Overdrafts of ` 5,000 to be allowed for every verified pollution-free India.
women SHG member having a Jan Dhan Bank ◆
Make in India with particular emphasis on MSMEs, start-ups,
Account. Defence manufacturing, Automobiles, Electronics, fabs and
— Social stock exchange an electronic fund raising batteries and Medical devices.
platform under the regulatory ambit of SEBI. ◆
Water, water management, clean rivers.
— About 30 lakh workers joined the Pradhan Mantri ◆
Blue Economy.
Shram Yogi Maandhan Scheme that provides ` 3,000 ◆
Space programmes, Gaganyan, Chandrayan and Satellite
per month as pension on attaining the age of 60 to programmes.
workers in unorganised and informal sectors. ◆
Self sufficiency and export of foodgrains, pulses, oilseeds,
— Blueprints to be made available for gas grids, water fruits and vegetables.
girds, i-ways and regional airports. ◆
Healthy society–Ayushman Bharat, well nourished women and
— Jal Marg Vikahs Project; Navigational Capacity of Ganga children, safety of citizens.
to be enhanced via multi model terminals at Sahibganj ◆
Team India with Jan Bhagidari, Minimum Government
and Haldia and a navigational lock at Frarakka by Maximum Governance.
2019-20.
Self Check
Build Your Confidence
1. NPAs of commercial banks reduced by over ` ……… in the 11. How much money has been provided for ‘World Class
last year according to Budget 2019-20. Institutions’ for FY 2019-20?
(a) 3 lakh crore (b) 4 lakh crore (a) 550 Crore (b) 400 Crore (c) 350 Crore (d) 600 Crore
(c) 1 lakh crore (d) 10 lakh crore
12. An encyclopedia is being developed on which among the
2. How many Iconic tourism sites will be develope into following freedom fighters?
model world class tourist destinations according to (a) Vallabhbhai Patel (b) Subhas Chandra Bose
Budget? (c) Chandra Shekhar Azad (d) Mahatma Gandhi
(a) 16 (b) 20
13. Economic Survey sees FY20 GDP growth at…………
(c) 17 (d) 25
(a) 4% (b) 5% (c) 6% (d) 7%
3. Which Bill was proposed to be passing to solve
problems in infrastructure contracts, PPP and public 14. The fiscal deficit is what percent of GDP in the Budget
utilities. 2019-20.
(a) Skill Development Programme (a) 3.0% (b) 3.3% (c) 3.4% (d) 3.5%
(b) Companies Amendment Bill 15. In budget 2019-20, the government set the target to
(c) Public Utility Resolution of Disputes Bill distribute ……… LPG connections under the Ujjawal
(d) Goods and Services Tax Yojana.
4. How much amount has been allocated for the Mahatma (a) 6 crore (b) 10 crore (c) 5 crore (d) 8 crore
Gandhi National Rural Employment Guarantee Scheme 16. India needs to grow at ……… % per year to be $5 trillion
(MGNREGS) is the Union Budget 2019-20? economy by FY 25?
(a) 60,000 Crore (b) 80,000 Crore (a) 7.3% (b) 8% (c) 9% (d) 8.2%
(c) 70,000 Crore (d) 75,000 Crore
17. Which statement is not correct about budget 2019-20?
5. How much amount has been allocated for the Pradhan (a) The number of new income-tax filers increased in
Mantri Shram Yogi Maan Dhan Yojana in the Union 2017-18, taking the total income tax returns filed to 6.84
Budget 2019-20?
crore.
(a) ` 5,000 Crore (b) ` 500 Lakh (b) Payment of gratuity has been increased from ` 10 lakh to
(c) ` 50 Lakh (d) ` 500 Crore ` 20 lakhs.
6. In the budget 2019-20, what amount has been given as (c) Government has introduced the Prime Minister ‘Kisan
budgetary support to the ailing national carrier Air India? Samman Nidhi Yojana’. This scheme will be in effect from
(a) 30 Crore (b) 100 Crore 1st January, 2019.
(c) 1 Lakh Crore (d) 500 Crore (d) The amount allocated for the defense sector has
exceeded 3 lakh crore for the first time.
7. Under the Jal Marg Vikas Scheme which place has been
chosen for the development of multi model terminal and 18. According to the survey some states will start
navigational lock in budget 2019-20 ? transitioning to an ageing society by the ………… .
(a) Sahibganj (b) Haldia (a) 2030 (b) 2040
(c) Farakka (d) All of these (c) 2035 (d) 2045

8. What amount of money has been infused for the 19. National Total Fertility Rate is expected to be below
recapitalisation of public sector banks ? replacement rate by which year?
(a) 60,000 Crore (b) 75,000 Crore (a) 2022 (b) 2021 (c) 2023 (d) 2025
(c) 70,000 Crore (d) 45,000 Crore 20. India’s working-age population will grow by roughly
9. What is the value of surcharge an income over 2 crore to ……… million per year during 2021-31?
5 crore ? (a) 9.7m (b) 8.8m (c) 4.2m (d) 6.6m
(a) 3% (b) 5% 21. Under Swachh Bharat Mission % of the households have
(c) 8% (d) 2% access to toilets?
10. What is the disinvestment target for financial year (a) 93 % (b) 87% (c) 90 % (d) 89%
2019-20? 22. As on 14th June, 2019 how many States/UTs are 100 per
(a) 80,000 Crore (b) 90,000 Crore cent covered with Individual Household Latrine (IHHL)?
(c) 1,00,000 Crore (d) 1,05,000 Crore (a) 26 (b) 30 (c) 32 (d) 24
180 Magbook ~ Indian Economy

23. The country is required to raise per capita energy 36. What was the rate of share of service sector in
consumption by at least ……… times to increase per employment in 2017?
capita income by $5000? (a) 23 % (b) 34 % (c) 40 % (d) 42 %
(a) 2 times (b) 3 time
37. Food inflation based on Consumer Food Price Index
(c) 2.5 times (d) 3.2 times
(CFPI) has remained below ……… per cent for the last two
24. 4 times increase in per capita energy consumption consecutive years?
needed for India to achieve Human Development Index (a) 2% (b) 2.5 % (c) 3% (d) 2.9 %
score?
38. Overall Index of Eight Core Industries registered a
(a) 0.10 (b) 0.8 (c) 0.7 (d) 0.6
growth rate of ……… per cent in 2018-19?
25. What is the rank of India in terms of wind power? (a) 3.5 % (b) 4.3% (c) 2.9 % (d) 4%
(a) Fifth (b) Sixth (c) Third (d) Fourth
39. What was the rank of India in the World Bank Doing
26. Share of renewable (excluding hydro above 25 MW) in Business (DB) Report,2019?
total electricity generation increased from 6% in (a) 68th (b) 77th (c) 88th (d) 75th
2014-15 to ………… % in 2018-19?
40. Consider the following statements and choose the
(a) 11% (b) 15 % (c) 10 % (d) 14 %
correct ones?
27. In India market share of electric vehicles was at ……… per 1. As per the EPFO, Net employment generation in the
cent compared to 2 per cent in China and 39 per cent in formal sector was higher at 8.15 lakh in March, 2019 .
Norway? 2. EPFO comes under Ministry of Home Affairs.
(a) 1% (b) 0.06 % (c) 0.10 % (d) 0.70 % 3. Gol in 2014 launched Universal Account Number for
28. The survey pointed out that one in every ………… wage Employees covered by EPFO to enable PF number
workers in India is not protected by the minimum wage portability.
law? (a) 1 & 3 (b) All are correct
(a) Five (b) Ten (c) Three (d) Six (c) 1 & 2 (d) Only 1

29. India’s Growth of GDP moderated to ……… per cent in 41. Consider the following statements with respect to
2018-19from 7.2 per cent in 2017-18? Pradhan Mantri Gram Sadak Yojana?
(a) 6.5 % (b) 6.2% (c) 6.1% (d) 6.8% 1. It is a Central sector scheme.
2. It was launched in 2000.
30. The world output growth declined from 3.8 per cent in 3. As per the economic survey 2019, around 1,90,000 km of
2017 to ……… per cent in 2018? rural roads were constructed under it since 2014.
(a) 3% (b) 2.9% (c) 3.6% (d) 3.2% (a) All are true (b) 1 & 2 are true
31. Current account deficit increased to ……… per cent of (c) 2 & 3 are True (d) 1 & 3 are True
GDP in 2018-19? 42. Consider the following statements:
(a) 2.1% (b) 3% (c) 2.6 % (d) 3.1 % 1. India is the second largest fish producing country in the
32. The government has set a target to reduce Central world as it accounts for 6.3 per cent of the global fish
Government debt to ……… per cent of GDP by 2024-25? production.
(a) 30 % (b) 40% (c) 50 % (d) 25 % 2. Siamese fighting fish commonly known as the betta was
declared national aquatic animal of Thailand.
33. Headline CPI inflation declined to per cent in 2018-19 (a) Only 1 (b) Only 2
from 3.6 per cent in 2017-18? (c) Both are correct (d) None is correct
(a) 3% (b) 3.4 % (c) 3.2 % (d) 2.9%
43. Which two states are the front runners with a score of 69
34. Services sector (excluding construction) has a share of amongst states in India’s SDG Index Score?
per cent in India’s GVA and contributed more than half of (a) Kerala and Himachal Pradesh
GVA growth in 2018-19? (b) Gujarat and Andhra Pradesh
(a) 54.3% (b) 55.2 % (c) 53.7 % (d) 49.2 % (c) Tamil Nadu and Kerala
35. The services sector growth declined marginally to what (d) Uttarakhand and Madhya Pradesh
per cent in 2018-19 from 8.1 per cent in 2017-18? 44. Around what per cent of pending cases are in the District
(a) 6.9 % (b) 7.2% and subordinate courts as per Economic Survey 2019?
(c) 8% (d) 7.5 % (a) 87.5% (b) 76.3% (c) 59.4% (d) 80%

1. (c) 2. (c) 3. (c) 4. (a) 5. (d) 6. (c) 7. (d) 8. (c) 9. (a) 10. (d)
11. (b) 12. (d) 13. (d) 14. (b) 15. (d) 16. (b) 17. (c) 18. (a) 19. (b) 20. (a)
21. (a) 22. (b) 23. (c) 24. (b) 25. (d) 26. (c) 27. (b) 28. (c) 29. (d) 30. (c)
31. (a) 32. (b) 33. (b) 34. (a) 35. (d) 36. (b) 37. (a) 38. (b) 39. (b) 40. (a)
41. (c) 42. (c) 43. (a) 44. (a)
Practice Set
1. Which of the following statement(s) is true about HDI?
1. Since 2010, UNDP began using the new method of
ANALYSE YOURSELF
(a) Only 1
(c) Both 1 and 2
1
(b) Only 2
(d) Neither 1 nor 2
calculating HDI. 7. Consider the following regarding the functions of
2. It is based on life expectancy, per capita income mean WTO
years of schooling and expected years of schooling. 1. To provide facilities for implementation, administration
3. HDI is the arithmatic mean of the Life Expectancy Index, and operation of multilateral agreements of the World
Education Index and Income Index. Trade.
Select the correct answer using the codes given below 2. To provide a platform to member countries to decide
(a) Only 1 (b) 2 and 3 future strategies related to trade and tariff.
(c) 1 and 2 (d) All of these Which of the statement(s) given above is/are correct?
2. Deficit financing leads to inflation in general, but it can (a) Only 1 (b) Only 2
be checked if (c) Both 1 and 2 (d) Neither 1 nor 2
(a) government expenditure leads to increase in aggregate 8. Consider the following statements
supply in ratio of aggregate demand 1. Geographical indication is granted to a community or
(b) only aggregate demand is increased group or an institution that represents the interests of
(c) all the expenditure is devoted to the payment of national the world.
debt 2. It is given to a product for a specific period of time
(d) All of the above (10 years in India).
3. Name the bank announced to be set-up in the budget Which of the statement(s) given above is/are correct?
for regulating and refinancing all Micro-Finance (a) Only 1 (b) Only 2
Institutions (MFI) which are in the business of lending (c) Both 1 and 2 (d) Neither 1 nor 2
to micro/small business entities engaged in
9. The GST Bill comprises -
manufacturing, trading and services activities?
1. The CGST Bill - The Central Goods and Services Tax
(a) Micro Bank (b) MUDRA Bank
Bill, 2017
(c) SIDBI Bank (d) Micro-Finance Bank
2. The IGST Bill - The Integrated Goods and Services Tax
4. Which of the following are the main causes of slow rate Bill, 2017
of growth of per capita income in India? 3. The UTGST Bill- The Union Territory Goods and
1. High level of fiscal deficits. Services Tax Bill, 2017
2. High capital output ratio. 4. The Compensation Bill - The Goods and Services Tax
3. High rate of household savings. (Compensation to the States) Bill, 2017
4. High rate of growth of population. Select the correct answer using the codes given below
5. High rate of capital formation.
Select the correct answer using the codes given below (a) Only 1 (b) 1 and 2
(a) 1, 2 and 4 (b) 2 and 4 (c) 3 and 4 (d) 2, 3 and 5 (c) 1,2 and 3 (d) 1, 2 , 3 and 4

5. Which of the following statements are true regarding 10. After a gap of over 20 years, ` 1 note has been
the TRIPS agreement of the WTO? released in the country on 6th March, 2015 by Finance
Secretary Rajiv Mehrishi. The new ` 1 note bears the
1. TRIPS is related to protecting the intellectual property
signature of
rights.
(a) Finance Minister (b) RBI Governor
2. Developing countries were supposed to pass TRIPS
(c) Finance Secretary (d) Home Secretary
Practice Set 1

Complaint National Legislation by 2005. While for


developed countries the time limit was till 1995. 11. The capital account convertibility of the Indian
Select the correct answer using the codes given below Rupee implies
(a) Only 2 (b) Only 1 (a) that the Indian Rupee can be exchanged by the
(c) Both 1 and 2 (d) Neither 1 nor 2 authorised dealers for travel
6. Consider the following statements regarding the OPEC (b) that the Indian Rupee can be exchanged for any major
currency for the purpose of trade in goods and
1. At present, OPEC has a total 14 members countries.
services
2. The objective of OPEC was to control production and
(c) that the Indian Rupee can be exchanged for any major
price of petroleum so as to safeguard the interests of oil
currency for the purpose of trading financial assets
exporting countries.
(d) None of the above
Which of the statement(s) given above is/are correct?
182 Magbook ~ Indian Economy
12. World Bank provides long-run capital to member (b) An index showing the rising and falling prices of
countries for economic reconstruction and development. shares and equities in the Baltic region
World Bank provides capital mainly for the following (c) An assessment showing the prices of the crude oil
purposes produced in the Baltic region
1. To rehabilitate war ruined economics. (d) None of the above
2. To finance productive efforts according to peacetime 19. Which one of the following items has the highest
requirements. share of India in the world exports?
3. To develop resources and production facilities in (a) Coffee, Tea, Mate, Spices etc
underdeveloped countries. (b) Pearls, Precious stones, Metals, Coins etc
Which of the statement(s) given above are correct? (c) Lac, Gums, Resins, Vegetable Saps and extracts etc
(a) 1 and 2 (b) 2 and 3 (c) 1 and 3 (d) All of these (d) Vegetable Plaiting Materials, Vegetable Products etc
13. Consider the following statements 20. The fair and remuneration price of sugarcane is
1. Ministerial conference is the highest decision-making approved by the
body of the WTO, which usually meets once every 2 years. (a) Cabinet Committee on Economic Affairs
2. Its headquarters is in Geneva. (b) Commission for Agricultural Costs and Prices
Which of the statement(s) given above is/are correct? (c) Directorate of Marketing and Inspection, Ministry of
(a) Only 1 (b) Only 2 Agriculture
(c) Both 1 and 2 (d) Neither 1 nor 2 (d) Agricultural Produce Market Committee
14. Consider the following statement(s) regarding the 21. Which one of the following factors is taken account
International Monetary fund to calculate the Balance of Payment (BOP), of a
1. IMF is controlled and managed by a Board of Governors. country?
2. Each Governor has the right of 250 votes on the basis of (a) Current account
membership and one additional vote for each SDR 1 lakh (b) Changes in the foreign exchange reserves
quota. (c) Errors and omissions (d) All of these
Which of the statement(s) given above is/are correct?
22. Which one of the following international organisations
(a) Only 1 (b) Only 2 (c) Both 1 and 2 (d) Neither 1 nor 2
offers political risk insurance guarantees to help
15. Consider the following statements investors to protect foreign direct investments made in
1. General agreement on trade in services is a treaty of the developing countries against political risk?
World Trade Organisation (WTO) that entered into force in (a) IDA (b) IFC (c) IMF (d) MIGA
January, 1995.
23. Examine the following statements in the context of
2. All members of the WTO are signatories to the general
the Base Rate system of interest rates
agreement on trade in services.
1. Corporate entities were able to get loans at
Which of the statement(s) given above is/are correct?
interest rates lower than PLR while it is not
(a) Only 1 (b) Only 2 possible with Base Rate.
(c) Both 1 and 2 (d) Neither 1 nor 2
2. Monetary policy is transmitted better through Base
16. IMF was established to meet which of the following Rate system.
objectives? 3. Base Rate is applicable only to loans given after 1st
1. Promoting International Monetary Co-operation. July, 2010.
2. Expanding International Trade. Which of the statement(s) given above is/are correct?
3. Lessening the disequilibrium in balance of trade. (a) Only 1 (b) 2 and 3 (c) Only 2 (d) All of these
4. Avoiding competitive exchange depreciations. 24. Consider the following statements
Select the correct answer using the codes given below 1. Regional Rural banks were set-up in 1975, under an
(a) 3 and 4 (b) 1, 2 and 3 (c) 2 and 3 (d) All of these Act of Parliament to exclusively cater to the credit
17. Consider the following statements about ‘Mega Food Park’ needs of rural population.
1. To provide good infrastructure facilities for food 2. The sponsor banks have the largest share in the
processing industries. equality of Regional Rural banks.
2. To increase processing perishable items and reduce Which of the statement(s) given above is/are correct?
Practice Set 1

wastage. (a) Only 1 (b) Only 2


3. To provide emerging and eco friendly food processing (c) Both 1 and 2 (d) Neither 1 nor 2
technology to entrepreneurs. 25. Which of these statements is true regarding the
Which of the statement(s) given above are correct? Special Drawing Rights (SDRs) of IMF?
(a) 2 and 3 (b) 1 and 2 (c) 1 and 3 (d) All of these 1. The SDRs are equivalent to a currency and represent
a claim on the IMF.
18. Which one of the following is related with the Baltic Dry
index? 2. The value of SDRs in terms of its component
currencies is fixed annually.
(a) An assessment of the price of moving the major raw
3. The basket of currencies of SDRs which presently
materials by sea
consist of US Dollar, Euro, the chinese renminbi,
Pound and Yen is revised every 5 years.
Magbook ~ Practice Set 1 183
Select the correct answer using the codes given below 32. Which of the following statements is true regarding a
(a) Only 3 (b) Only 1 Non-Banking Financial Company (NBFC)?
(c) Both 1 and 2 (d) All of these 1. NBFC cannot accept demand deposits.
26. Which of the following statements are true regarding 2. For a NBFC financial assets must be 50% more than its
Hedge Funds? total assets.
1. Hedge funds are open to only select group of investors. 3. Normal banks do all of the activities normally done by an
NBFC.
2. Hedge funds have been allowed to function in India
recently after registering with SEBI. Select the correct answer using the codes given below
Select the correct answer using the codes given below (a)Only 1 (b) 2 and 3
(c) 1 and 2 (d) All of these
(a) Only 1 (b) Only 2
(c) Both 1 and 2 (d) Neither 1 nor 2 33. The Competition Commission of India (CCI) wants the
27. Consider the following statements rules tightened to bring a larger number of Mergers
and Acquisitions (M and As) under its scrutiny. In this
1. NABARD is the nodal agency for implementing the
context, consider the following statements about CCI
schemes in respect of co-operative credit and RRBs.
1. Competition Commission of India (CCI) was established
2. NABARD was the nodal agency for implementing the
with effect from14th October, 2003.
agricultural debt waiver and Debt Relief Scheme, 2008.
2. The goal is to curtail the monopoly tendencies and
Which of the statement(s) given above is/are correct?
foster the competition among public sector enterprises.
(a) Only 1 (b) Only 2
Which of the statement(s) given above is/are correct?
(c) Both 1 and 2 (d) Neither 1 nor 2
(a) Only 1 (b) Only 2
28. Which of the following statements are true regarding (c) Both 1 and 2 (d) Neither 1 nor 2
the Agreement on Agriculture of the WTO?
34. Which of the following statements is true regarding the
1. Green box, blue box and amber box refer to domestic
innovation index measure?
subsidies in agriculture.
1. It is developed by INSEAD, France.
2. There is no limit on green box subsidies for both
developing and developed economies. 2. Japan is the top country in this index.
3. In the long-run, the agreement on agriculture is 3. The National Innovation Council (NIC) was formed to
expected to benefit the developing countries. foster innovation in India especially in the area of
growth.
Select the correct answer using the codes given below
Select the correct answer using the codes given below
(a) Only 1 (b) Only 2
(a) 2 and 3 (b) 1 and 2
(c) Both 1 and 2 (d) All of these
(c) 1 and 3 (d) All of the above
29. Which State Government launched ‘Bhagyashree’
scheme in place of the Sukanya scheme which caters 35. Examine these statements in the context of the
to girls from Below Poverty Line (BPL) families? financial instruments known as futures.
1. Futures are agreements between two parties to buy or
(a) Gujarat (b) Kerala (c) Maharashtra (d)
sell shares at a future date.
Haryana
2. Options are a class of futures where there is a right, but
30. Which of the following are the objectives of the not the obligation to buy or sell at a future date.
Commission for Agricultural Costs and Prices 3. Forwards are a particular class of futures.
(CACP)? Which of the statement(s) given above is/are correct?
1. To stabilise agricultural prices. (a) 1 and 3 (b) Only 1
2. To ensure meaningful real income levels to the (c) Only 2 (d) All of these
farmers.
3. To protect the interest of the consumers by providing
36. Which one of the following is likely to the most
essential agricultural commodities at reasonable rate
inflationary, in it’s effect?
through PDS. (a) Repayment of public debt
Select the correct answer using the codes given below (b) Borrowing from the public to finance a budget deficit
(a) 1 and 2 (b) 2 and 3 (c) Borrowing from banks to finance a budget deficit
(c) 1 and 3 (d) All of the above (d) Creating new money to finance a budget deficit
Practice Set 1

31. Which of these statements is true regarding the 37. Consider the following statements regarding Reserve
lending provisions of the IMF? Bank of India
1. IMF lends to only member countries. 1. It is a banker to the Central Government.
2. IMF lends only to help with Balance of Payments (BoP) 2. It formulates and administers monetary policy.
problems. 3. It acts as an agent of the government in respect of India.
3. IMF lending is also possible for specific projects 4. It handles the borrowing programme of Government of
relating to economic restructuring of an economy. India.
Select the correct answer using the codes given below Which of the statement(s) given above are correct?
(a) 1 and 3 (b) Only 1 (a) 1 and 2 (b) 2, 3 and 4
(c) 1 and 2 (d) Only 3 (c) 3 and 4 (d) All of the above
184 Magbook ~ Indian Economy
38. Consider the following regarding World Trade Select the correct option from the codes given below :
organisation (a) 1, 2 and 3 (b) 3, 4 and 5
1. The Uruguay round of GATT (1986-93) gave birth to WTO. (c) 1 ,2, 3, 4 and 5 (d) All of these
2. WTO was officially constituted as a permanent body on
45. Hawala transactions relate to payments
1st January,1995 as an effective formal organisation.
(a) received in rupees against overseas currencies and
Which of the statements given above is/are correct?
vice versa without going through the official channels
(a) Only 1 (b) Only 2
(b) received for sale or transfer of shares without going
(c) Both 1 and 2 (d) Neither 1 nor 2
through the established stock exchanges
39. Under the WTO agreement, Agreement on Agriculture (c) received as commission for services rendered to
(AoA), developing countries won over with some overseas investors, buyers or sellers in assisting them
concessional features and flexibilities. Its three pillars to get over the red tape and in getting preferential
are treatment
1. domestic support 2. export subsidies (d) made to political parties or to individuals for meeting
3. market access 4. non-tariff barriers election expenses
Which of the statement(s) given above are correct? 46. The Reserve Bank of India (RBI) cancelled the
(a) 1, 2 and 3 (b) 2, 3 and 4 certificate of registration of the 26 Non- Banking
(c) 1, 3 and 4 (d) All of the above Financial Companies (NBFCs) including
40. Consider the following regarding WTO (a) Mathura Financial Services Limited
1. WTO negotiations proceed not by consensus of all (b) SJ Finance & Consultants Private Limited
members, but by a process of informal negotiations (c) Modern Vintrade Private Limited
between small groups. (d) All of the above
2. Such negotiations are often called green room. 47. The main cause of cyclical unemployment is that
Which of the statement(s) given above is/are correct? (a) firms engage in race, gender and sex discrimination
(a) Only 1 (b) Only 2 in their hiring practices
(c) Both 1 and 2 (d) Neither 1 nor 2 (b) some individuals do not have marketable job skills
(c) the level of overall economic activity fluctuates
41. Personal Income Tax limit was not changed in Union
(d) workers often voluntarily quit a job look for a better job
Budget 2016-17. What is the present Personal I-T
exemption limit? 48. If the Cash Reserve ratio is lowered by the RBI, its
(a) ` 2.5 lakh (b) ` 2.75 lakh (c) ` 3 lakh (d) ` 5 impact upon credit creation will be to
lakh (a) decrease it (b) increase it
(c) no impact (d) None of these
42. Which one of the follwing items has gained the highest
growth rate in the import composition of the Indian 49. Which of the following is true about money market in
economy in the last decade? India?
(a) Pearls, precious and semiprecious stones 1. The most active segment of money market is overight
(b) Gold and silver call market.
(c) Oil-cake and other solid residues 2. The money market instruments have maturity of less
(d) None of the above than 1 year.
43. Consider the following regarding the objectives of WTO 3. The money market instruments in India are regulated
by RBI and SBI.
1. To improve the standard of living of people in member
countries. Select the correct answer using the codes given below
2. To ensure full employment and increase in effective (a) 1 and 2 (b) Only 2
demand. (c) 2 and 3 (d) All of the above
3. To enlarge production and trade of goods and services. 50. With regard to proposed GST, which of the following
Which of the statement(s) given above are correct? is correct?
(a) 1 and 2 (b) 2 and 3 (c) 1 and 3 (d) All of these 1. Taxes on items containing alcohol and petroleum
44. The Indian banking system consists of product are kept out of GST.
Practice Set 1

1. 26 public sector banks 2. 20 private sector banks 2. Entertainment tax, luxury tax and taxes on gambling
are part of GST.
3. 43 foreign banks 4. 56 regional rural banks
5. 1589 urban cooperative banks Select the correct answer using the codes given below
6. 93550 rural cooperative banks (a) Only 1 (b) Only 2
(c) Both 1 and 2 (d) Neither 1 nor 2
Answers
1. (c) 2. (a) 3. (b) 4. (b) 5. (c) 6. (c) 7. (c) 8. (c) 9. (d) 10. (c)
11. (c) 12. (d) 13. (c) 14. (c) 15. (c) 16. (b) 17. (d) 18. (a) 19. (c) 20. (a)
21. (d) 22. (d) 23. (d) 24. (b) 25. (a) 26. (c) 27. (d) 28. (c) 29. (d) 30. (a)
31. (a) 32. (c) 33. (c) 34. (c) 35. (c) 36. (d) 37. (a) 38. (c) 39. (a) 40. (c)
41. (a) 42. (b) 43. (d) 44. (d) 45. (a) 46. (d) 47. (c) 48. (b) 49. (d) 50. (c)
Practice Set
1. To obtain full benefits of demographic dividend,
what should India do?
ANALYSE YOURSELF 2
(d) a special tax levied by the states on products from other
states
(a) Promoting skill development 7. Consider the following statements
(b) Introducing more social security schemes 1. EXIM Bank was set-up in 1982.
(c) Reducing infant mortality rate
2. It provides financial assistance to importers and
(d) Privatisation of higher education exporters of India as well as of other countries.
2. Which of the following would cause consumption to Which of the statement(s) given above is/are correct?
rise? (a) Only 1 (b) Only 2 (c) Both 1 and 2 (d) Neither 1 nor 2
(a) The GDP Deflator rises 8. Which one of the following schemes and programmes
(b) A greater proportion of the population is between age are mainly aimed to bring inclusive growth in India?
20 and 30
(a) Swabhiman (b) MNREGA
(c) Transitory income increases
(c) Right to Information (d) Both ‘a’ and ‘b’
(d) Income is taken from poor people and given to rich
people 9. Consider the following statements about Rashtriya
Swasthya Bima Yojana (RSBY)
3. With reference to India, consider the following
statements. 1. Under RSBY the premium is shared on 85 : 15 basis by the
Centre and State Governments.
1. WPI is available on a monthly basis only.
2. In the case of the North-Eastern States and Jammu and
2. As compare to Consumer Price Index for the
Kashmir, under RSBY the premium is shared on 90:10
Industrial Worker (CPI-IW), the WPI gives less
ratio.
weightage to food articles.
Which of the statement(s) given above is/are correct?
Which of the statement(s) given above is/are correct?
(a) Only 1 (b) Only 2
(c) Both 1 and 2 (d) Neither 1 nor 2
(a) Only 1 (b) Only 2
(c) Both 1 and 2 (d) Neither 1 nor 2 10. RBI issues currency notes against which of the
following?
4. Consider the following
(a) Gold (b) Foreign reserve
1. Finance and non-finance.
(c) Government of India Security (d) All of these
2. Telecommunication.
3. Computer software and hardware. 11. Under which scheme, Union Government proposed to
4. Hospital and diagnostic centres and services. set-up an IT based student Financial Aid System to
5. Construction activities. ensure that no student misses out on higher education
due to lack of fund?
Which of the services sector given above are included
in the Top 5 Services Sector of the Indian economy? (a) Pradhan Mantri Vidya Laxmi Scheme
(a) 1, 2, 4 and 5 (b) 1, 3, 4 and 5 (b) Vidya Kanya Laxmi Scheme
(c) 1, 2, 3 and 4 (d) All of these (c) Suknya Vidyadhan Yojana
(d) Sukanya Vidya Laxmi Scheme
5. Which two of the following nationalised banks have
been merged as per the recommendation of the 12. Consider the following regarding the Asian
Narasimham Committee ? Development Bank
(a) New Bank of India and Bank of India 1. ADB was established in 22nd August, 1966 on the
(b) Central Bank of India and Corporation Bank recommendations of economic commission for Asia and
Far East.
Practice Set 2

(c) Parur Central Bank of India and Bank of India


(d) Punjab National Bank and New Bank of India 2. The Aim of this bank is to accelerate economic and social
development in Asia and Pacific region.
6. Value added tax is 3. Its headquarter is located in Manila.
(a) an advolerum tax on domestic, final consumption Which of the statements given above are correct?
collected at all stages between production and point (a) 1 and 2 (b) 2 and 3 (c) 1 and 3 (d) All of these
of final sale
(b) an advolerum tax on final consumption collected at 13. Consider the following statements
the manufacturing level India continues to be dependent on imports to meet the
(c) tax on final consumption collected at the consumption requirement of oilseeds in the country because
rate 1. farmers prefer to grow food grains with highly
remunerative support prices.
186 Magbook ~ Indian Economy
2. most of the cultivation of oilseed crops continues to be (d) As per the new dispensation, Enforcement
dependent on rainfall. Directorate can arrest and prosecute the people
3. oils from the seeds of free origin and rice bran have remained for the violation of foreign exchange rule
unexploited.
20. Consider the following statements
4. It is far cheaper to import oil seeds than to cultivate the oil seed
The price of any currency in international
crops.
market is decided by the
Which of the statements given above are correct?
1. world bank.
(a) 1 and 2 (b) 1, 2 and 3 (c) 3 and 4 (d) All of these
2. demand for goods and services provided by the
14. GST would subsume which of the following taxes country concerned.
1. Excise duty 2. Customs duty 3. stability of the government of the concerned
3. Sales tax 4. Service tax country.
Select the correct answer using the codes given below Which of the statements given above are correct?
(a) 1 and 4 (b) 1, 2 and 4 (c) 1, 3 and 4 (d) All of these (a) 1 and 2 (b) 2 and 3
(c) 1 and 3 (d) All of these
15. Consider the following statements
In India, stamp duties on financial transactions are 21. Consider the following statements
1. Collect and Appropriated the State Government. 1. Gini co-efficient is the area between the line of
2. Levied by the Union Government. perfect equality and Lorenz curve.
Which of the statement(s) given above is/are correct? 2. Lorenz curve is a percentage of the area between
(a) Only 1 (b) Only 2 the line of perfect equality and the line of perfect
(c) Both 1 and 2 (d) Neither 1 nor 2 inequality.
Which of the statement(s) given above is/are
16. The problem of international liquidity is related to the correct?
non-availability of (a) Only 1 (b) Only 2
(a) goods and services (c) Both 1 and 2 (d) Neither 1 nor 2
(b) gold and silver
(c) dollars and other hard currencies 22. The recently announced Paramparagat Krishi
(d) exportable surplus Vikas Yojana aims to boost
(a) organic Farming (b) drip Irrigation
17. Consider the following statements (c) horticulture Crops (d) vegetable
1. Poverty Reduction and Growth Facility (PRGF) has been Production
established by the International Development Association
(IDA) to provide further assistance to low income countries 23. Consider the following norms of the Mid-day
facing high level of indebtedness. Meal Scheme
2. Singapore Regional Training Institute (STI) is one of the 1. To provide a minimum of 550 calories of food.
institute that provides training in Macro Economic analysis and 2. To provide a minimum 12 gram of protein.
policy and related subject as a part of programme of the IMF 3. To provide an adequate quantities of
institute. micronutrients like iron, folic acid, vitamin-A.
Which of the statement(s) given above is/are correct? 4. To provide an adequate quantities of essential
(a) Only 1 (b) Only 2 fatty acids and medicine for the common cold
(c) Both 1 and 2 (d) Neither 1 nor 2 and diarrhoea.
Which of the norms given above are correct?
18. The principal functions of Asian Development Bank is/are
(a) 1, 3 and 4 (b) 1, 2 and 3
1. to make loans and equity investments for the economic and
(c) 1, 2 and 4 (d) All of these
social advancement of its developing member countries.
2. to provide technical assistance for the preparation and 24. An increase in the bank rate generally indicates
execution of development projects and programmes and that the
advisory services. (a) Market rate of interest is likely to fall
Which of the statement(s) given above is/are correct? (b) Central house is no longer making loans to
(a) Only 1 (b) Only 2 commercial banks
(c) Both 1 and 2 (d) Neither 1 and 2 (c) Central Bank is following an easy money policy
Practice Set 2

19. Which one of the following statements is correct with (d) Central Bank is following a tight money policy
reference to FEMA in India? 25. Which of the following 2 programmes are the
(a) The Foreign Exchange Regulating Act (FERA) was replaced by parts of the National Social Assistance
Foreign Exchange Management Act (FEMA) in the year 2001 Programme of the Government of India?
(b) FERA was given a sunset clause of 1 year till 31st May, 2002 to 1. Indira Awas Yojana and Pradhan Mantri Gram
enable Enforcement Directorate to complete the investigation Sadak Yojana.
of pending issues 2. National Food for work and Mid-Day Meal
(c) Under FEMA, violation of foreign exchange rules has ceased to Scheme.
be a criminal offence 3. National old age Pension scheme and National
Family Benefit Scheme.
Magbook ~ Practice Set 2 187
Select the correct answer using the codes given below 31. Which among the following sectors is the largest
(a) 1 and 2 (b) 1 and 3 receiver of Foreign Direct Investment in India?
(c) Only 3 (d) Only 2 (a) Services (b) Telecom
26. Aadhar has huge potential for improving operations (c) Automobile (d) Pharmaceuticals
and delivery of services. Its potential application in 32. Recently, which one of the following schemes of Union
various significant public services delivery and social Government has entered into Guinness Book of World
sector programme is immense. Record?
In this context, consider the following statements (a) MGNREGA (b) Swachh Bharat Abhiyan
1. The UID database can be used for the authentication of (c) Indira Awaas Yojana
beneficiaries under the public distribution system. (d) Prime Minister Jan Dhan Yojana
2. It will help in enhancing the efficiency of social audit of
programmes.
33. WTO allows departures from the MFN (Most Favoured
Nations) under the following headings
3. Enable greater commitment towards government
financing of public health and primary health care. 1. Imports from poor countries are allowed at lower or
zero tariffs (Generalised system of preference).
Which of the statement(s) given above is/are correct?
2. Preferential and free trade arrangement among
(a) 1 and 2 (b) 1 and 3 countries of a region and others are allowed at
(c) 2 and 3 (d) All of these concessional rates.
27. Consider the following statements Which of the statement(s) given above is/are correct?
1. Gini co-efficient is commonly used to measure the (a) Only 1 (b) Only 2
inequality of wealth. (c) Both 1 and 2 (d) Neither 1 nor 2
2. The Gini co-efficient with a value of 1 expressing total 34. The economist who was associated with the WTO
equality and a value of O expressing maximum draft document?
inequality.
(a) A K Sen (b) T N Srinivasan
Which of the statement(s) given above is/are correct? (c) J N Bhagwati (d) Avinash Dixit
(a) Only 1 (b) Only 2
(c) Both 1 and 2 (d) Neither 1 nor 2
35. Consider the following statements
1. Product patents provide for absolute protection of the
28. Which one of the following services of the Services product exhausting all the process that may lead to the
sector in the Indian economy contributed the highest product.
percentage in the Gross Domestic Product at the 2. Process patents provide protection in respect of the
Factor Cost in the last 10 years? technology and the process or method of manufacture.
(a) Hotels and restaurants (b) Railways transport Which of the statement(s) given above is/are correct?
(c) Banking and insurance (d) Trade (a) Only 1 (b) Only 2
29. Consider the following statements (c) Both 1 and 2 (d) Neither 1 nor 2
1. Asian Development Bank (ADB) is a regional 36. Name the scheme that will replace the existing
development bank established in 1966. Swavalamban Scheme with effect from 1st June, 2015?
2. Asian Development Bank (ADB) admits the members of (a) National Pension Scheme
the United Nations Economic and Social Commission (b) Atal Pension Yojana
for Asia and the Pacific. (c) Pradhan Mantri Vradha Pension Scheme
Which of the statement(s) given above is/are correct? (d) All of the above
(a) Only 1
37. Consider the following regarding the special products
(b) Only 2 under WTO.
(c) Both 1 and 2
1. It was decided at the Doha Development round of
(d) Neither 1 nor 2
WTO negotiations that SPs would attract lower levels
30. Consider the following statements of tariff reduction commitment than other
1. The World Intellectual Property Organisation (WIPO) is a agricultural products.
specialised agency of United Nations System of 2. The rationale is that higher levels of protection on SP
Practice Set 2

Organisation. will allow developing countries to sustain and develop


2. WIPO has its headquarters at Rome. domestic production of these products.
3. The Trade Related Aspects of Intellectual Property Rights Which of the statement(s) given above is/are correct?
(TRIPS) Agreement is binding on all WTO members. (a) Only 1 (b) Only 2
4. Least developed country members of WTO are not (c) Both 1 and 2 (d) Neither 1 nor 2
required to apply the provisions of TRIPS Agreement for 38. The negotiations in WTO in Agriculture sector include
a period of 20 years from the general date of application
1. Substantial and effective reductions in overall
of the agreement.
trade-distorting domestic support by developed
Which of the statements given above are correct? countries.
(a) 1 and 3 (b) 2, 3 and 4 2. An operational and effective special safeguard
(c) 1, 2 and 4 (d) All of these mechanism.
188 Magbook ~ Indian Economy
Which of the statement(s) given above is/are correct? 2. Indonesia, Philippines, Malaysia, Singapore and
(a) Only 1 (b) Only 2 Thailand are the founding members of the ASEAN.
(c) Both 1 and 2 (d) Neither 1 nor 2 Which of the statement(s) given above is/are correct?
(a) Only 1 (b) Only 2
39. Consider the following regarding the functions of the
(c) Both 1 and 2 (d) Neither 1 nor 2
World Bank
1. The debtor nation has to repay either in reserve 45. Economic liberalisation in India started with
currencies or in the currency in which the loan is (a) substantial changes in industrial licensing policy
sanctioned. (b) the convertibility of Indian rupee
2. Generally, bank grants loan for a particular project only (c) doing away with procedural formalities for foreign
submitted to the bank by the member country. direct investment
Which of the statement(s) given above is/are correct? (d) significant reduction in tax rates
(a) Only 1 (b) Only 2
46. Consider the following statements
(c) Both 1 and 2 (d) Neither 1 nor 2
The Indian rupee is fully convertible
40. Identify the correct statement regarding Pradhan Mantri 1. In respect of current account of balance of payment.
Vaya Vandana Yojana (PMVVY) 2. In respect of capital account of balance of payment.
(a) PMVVY is a Pension Scheme under the Ministry of 3. Into gold.
Finance, exclusively for the senior citizens aged 60 years Which of the statement(s) given above is/are correct?
and above. (a) Only 1 (b) Only 3
(b) It provides social security to protect senior citizens (c) 1 and 2 (d) All of these
against a future fall in their interest income due to
uncertain market conditions. 47. The growth rate of per-capita income at current
(c) Life Insurance Corporation of India is the implementing prices is higher than that of per-capita income at
agency. constant prices, because the latter takes into
(d) All of these account the rate of
(a) growth of population (b) increase in price level
41. Consider the following regarding the Asian Clearing (c) growth of money supply (d) increase in the wage rate
Union
1. It was constituted to eliminate the difficulty of payments 48. Indian farmers insurance against crop failure is the
for trade in local currencies so as to solve the scarcity process of one of the major methods of risks
problem of foreign exchange. coverage by Indian farmers in
2. The headquarters of ACU is in Tehran (Iran). (a) Relay cropping (b) Dry farming
Which of the statement(s) given above is/are correct? (c) Mixed cropping (d) Share cropping
(a) Only 1 (b) Only 2 49. With reference to Indian economy consider the
(c) Both 1 and 2 (d) Neither 1 nor 2 following statements.
42. Consider the following statements about Payment Banks 1. The lead bank scheme was started towards the close
1. They can accept demand deposits. of 1980.
2. They will have a wide network of access points 2. The lead bank scheme was aimed to bring a balanced
particularly in remote areas. regional development in the country.
3. They can provide small loans. Which of the statement(s) given above is/are correct?
(a) Only 1 (b) Only 2
Which of the statements given above is/are correct?
(c) Both 1 and 2 (d) Neither 1 nor 2
(a) 2 only (b) 1 and 2 only (c) 2 and 3 only (d) 1, 2 and 3
43. Why are indirect taxes considered regressive taxing 50. The function of the Central Banks are
mechanisms? 1. Bankers to the bank.
2. Custodian of foreign exchange.
1. They are income irrespective.
3. Controller of credit.
2. They are lower than direct taxes.
4. Bank of issue.
Select the correct answer using the codes given below
5. Banker, agent and financial, advisor of government
(a) Only 1 (b) Only 2
6. Financial advisor of private banks.
(c) Both 1 and 2 (d) Neither 1 nor 2
Practice Set 2

Select the correct answer using the codes given below


44. Consider the following statements (a) 1, 2, 3 and 6 (b) 2, 3, 4 and 5
1. ASEAN was constituted on August 1967. (c) 1, 2, 3, 4 and 5 (d) All of these

Answers
1. (a) 2. (b) 3. (b) 4. (c) 5. (d) 6. (a) 7. (c) 8. (d) 9. (b) 10. (d)
11. (a) 12. (d) 13. (b) 14. (c) 15. (c) 16. (c) 17. (b) 18. (c) 19. (c) 20. (b)
21. (a) 22. (a) 23. (b) 24. (d) 25. (c) 26. (d) 27. (a) 28. (d) 29. (c) 30. (a)
31. (b) 32. (d) 33. (c) 34. (c) 35. (c) 36. (b) 37. (c) 38. (c) 39. (c) 40. (d)
41. (c) 42. (b) 43. (a) 44. (c) 45. (a) 46. (a) 47. (b) 48. (c) 49. (b) 50. (c)
Practice Set
1. Consider the following measures
ANALYSE YOURSELF 3
8. Under which of the following circumstances may
1. Repo rate. 2. Cash reserve requirement. ‘Capital gains’ arises?
3. Reverse repo rate. 1. When there is an increase in the sales of a product.
Which of the measures given above is/are major 2. When there is a natural increase in the value of the
instruments used in the Liquidity Adjustment Facility property owned.
(LAF)? 3. When you purchase a painting and there is a growth in it’s
(a) 1 and 2 (b) 1 and 3 (c) 2 and 3 (d) Only 2 value due to increase in it’s popularity.
Select the correct answer using the codes given below
2. Which one among the following is true about the repo
rate in the Indian Economy ? (a) Only 1 (b) 2 and 3 (c) Only 2 (d) All of these
(a) It is the rate at which banks borrow funds from the RBI 9. Consider the following statements in context with the
(b) It is the rate at which banks park their funds with the free market economy
Central Bank 1. The role of market forces and competition is eliminated by law.
(c) It is the percentage difference in the prices of 2. There is a high degree of competition in both commodity
repurchase of the bills from the original prices and factor markets.
(d) Both ‘a’ and ‘c’ 3. Private gains are the main motivating and guiding force
3. The Regional Comprehensive Economic Partnership for carrying out economic activities.
(RCEP), is a proposed free trade agreement between Which of the statements given above are correct?
the members of (a) 1 and 2 (b) 2 and 3 (c) All of these (d) None of these
(a) SAARC (b) ASEAN 10. The unique features of national level commodity
(c) BRICS (d) European Union exchanges include.
4. Arrange the following in chronological order. 1. They provide online platforms or screen based trading.
1. Nationalisation of Reserve Bank of India. 2. Investors have the option of taking delivery in physical
2. Foundation of Punjab National Bank. quantity.
3. Nationalisation of State Bank of India. 3. Global depository receipts is a unique feature of
4. Establishment of NABARD. commodity exchange.
Codes Which of the statements given above are correct ?
(a) 1, 2, 3, 4 (b) 1, 2, 4, 3 (c) 2, 1, 4, 3 (d) 2, 1,3, 4 (a) 1 and 2 (b) 1 and 3 (c) 2 and 3 (d) All of these

5. When the ratio of indirect taxes to total taxation 11. Which of the following pair (s) is/are correctly matched?
revenue is very high it leads to 1. Swabhiman : Financial Inclusion
1. rise in the price level. 2. Swavalamban : Financial Assistance
2. higher taxation burden on the rich. 3. Swadhar : To provide holistic and integrated
3. higher taxation burden on the poor. services to women
4. fall in tax revenues. Codes
Select the correct answer using the codes given below (a) 1 and 2 (b) 2 and 3 (c) Only 1 (d) All of these
(a) 1 and 2 (b) 1 and 3 (c)1, 2 and 4 (d) 1, 3 and 4 12. India continues to be dependent on import to meet the
6. Consider the following statements requirement of oilseeds in the country because
1. The Livestock Census in India is conducted every 5 1. farmers prefer to grow food grains with highly
years. remunerative support price.
Practice Set 3

2. The Livestock Census is conducted by the respective 2. most of the cultivation of oilseed crops depend on rainfall.
State Government. 3. oil from seeds of free origin and rice bran have remained
Which of the statement(s) given above is/are correct ? unexploited.
(a) Both 1 and 2 (b) Neither 1 nor 2 Which of the statements given above are correct?
(c) Only 1 (d) Only 2 (a) 1 and 2 (b) 1 and 3
7. Which of the following is not viewed as a national debt ? (c) All of these (d) None of these
(a) Life insurance policies (b) Provident fund 13. Mercosur, is a free trade agreement between several
(c) Long-term government bond countries on which of the following continents?
(d) National saving certificate (a) Africa (b) South America
(c) North America (d) Asia
190 Magbook ~ Indian Economy
14. Consider the following statements 22. Consider the following statements
1. S and P CNX Nifty is one of several leading stock market 1. Gilt-edged market means a market for bullions.
indices. 2. Gilt-edged securities can be issued by both National
2. Nifty is owned and managed as a joint venture company Governments and private firms.
between National Stock Exchange and Credit Rating and Which of the statement(s) given above is/are correct?
Information Services of India Limited. (a) Only 1 (b) Only 2
Which of the statement(s) given above is/are correct ? (c) Both 1 and 2 (d) Neither 1 nor 2
(a) Only 1 (b) Only 2
23. In the states of India, the State Financial Corporations
(c) Both 1 and 2 (d) Neither 1 nor 2
have given assistance mainly to develop
15. Consider the following statements 1. agricultural farms.
1. Most of the employment in India is in urban areas. 2. cottage industry.
2. Agricultural sector in India suffers from disguised 3. large-scale industries.
unemployment. 4. medium and small-scale industries.
Which of the statement(s) given above is/are incorrect? Select the correct answer using the codes given below
(a) Only 1 (b) Only 2 (a) Only 1 (b) 2 and 4 (c) Only 2 (d) Only 4
(c) Both 1 and 2 (d) Neither 1 nor 2
24. In India, rural incomes are generally lower than the
16. Which of the following is used for the measurement of urban incomes, which of the following reasons
distribution of income? account for this?
(a) Laffer Curve (b) Engel’s Law 1. A large number of farmers are illiterate and know little
(c) Gini-Lorenz Curve (d) Philip Curve about scientific agriculture.
2. Prices of primary products are lower than those of
17. Consider the following statements manufactured products.
1. All proceeds from the disinvestment of Central Public Sector 3. Investment in agriculture has been low when compared
Enterprises flow into the National Investment Fund (NIF). to investment in industry.
2. 75% of the annual income of the NIF is used to fund Social Select the correct answer using the codes given below
Sector schemes. (a) 2 and 3 (b) 1 and 2 (c) 1 and 3 (d) All of these
Which of the statement(s) given above is/are correct?
(a) Only 1 (b) Only 2 25. Which one of the following countries will hold the
(c) Both 1 and 2 (d) Neither 1 nor 2 presidency of the BRICS New Development Bank for
the first 6 years?
18. Which of the following scheme is designed and launched (a) India (b) China (c) South Africa (d) Russia
to improve the living standard of urban poor?
1. Accredited Social Health Activist (ASHA). 26. Regional Rural banks
2. Janani Suraksha Yojana (JSY). 1. have limited area of operation.
3. Valmiki Ambedkar Malin Basti AwaasYojna(VAMBAY). 2. have free access to liberal refinance facilities from
NABARD.
Select the correct answer using the codes given below
3. are required to lend only to weaker sections.
(a) 1 and 2 (b) 1 and 3 (c) 2 and 3 (d) AII of these
Select the correct answer using the codes given below
19. Consider the following statements (a) 1 and 3 (b) 2 and 3
1. Phillips curve is an inverse relationship between the rate (c) 1 and 2 (d) All of these
of unemployment and the rate of inflation in an economy.
27. Why is the increasing share of direct taxes relative to
2. Engel’s law observes that, as income rises the proportion indirect taxes considered a good thing?
of income spent on food falls even if the actual
1. Direct taxes are easier to collect.
expenditure on food rises.
2. Direct taxes are paid by well-off while indirect taxes are
Which of the statement(s) given above is/are correct ?
paid by everybody.
(a) Only 1 (b) Only 2 (c) Both 1 and 2 (d) Neither 1 nor 2
3. Economic activity can be promoted by lowering the
20. Which of the commodities are distributed under PDS indirect taxes.
1. Sugar 2. Pulses 3. Kerosene Select the correct answer using the codes given below
Select the correct answer using the codes given below (a) Only 3 (b) 1 and 2 (c) Only 2 (d) All of these
Practice Set 3

(a) 1 and 2 (b) 1 and 3 (c) 2 and 3 (d) All of these 28. Examine the following statements in the context of
21. Examine the following statements in the context Special Zero Based Budgeting (ZBB).
Products (SP) and Special Safeguard Mechanism (SSM) 1. ZBB was first taken up in India in the Union Budgeting
of the WTO. 1987.
1. Special products are limited to agricultural products only. 2. ZBB is based on prioritising all governing expenditure.
2. Special products are available to both developed countries 3. There is a cost benefit analysis of all schemes and the
and developing countries. most important ones are kept alive if they are working
3. SSM are used to defend against volatility in international well.
commodity prices. Which of the statements given above are correct?
Which of the statements given above are correct? (a) 1 and 3 (b) 1 and 2
(a) 2 and 3 (b) 1 and 2 (c) 1 and 3 (d) All of these (c) 2 and 3 (d) All of the above
Magbook ~ Practice Set 3 191
29. What are the purposes, for which the Government of 2. GATT was the precursor organisation to the WTO.
India disinvests its stake in the Public Sector 3. Uruguay round was the first round of WTO.
Enterprises? Which of the statements given above are correct?
1. To generate funds to increase the spending on Social (a) 1 and 2 (b) 1 and 3 (c) All of these (d) None of these
Sector Schemes.
36. The Small Industries Development Bank of India
2. To bring market discipline to the PSEs. (SIDBI) has been implementing two special schemes
3. To bridge the fiscal deficit of the Union Budget. for women viz;
Select the correct answer using the codes given below (a) Mahila Udyam Nidhi
(a) Only 2 (b) 1 and 3 (b) Mahila Vikas Nidhi
(c) 1 and 2 (d) All of these (c) Mahila Samridh Nidhi
30. Why does the Government of India allow the private (d) ‘a’ and ‘b’
sector to raise External Commercial Borrowings (ECB)? 37. Which of the following statement(s) is/are true
1. It helps to diversify the risk. regarding the market economy status under the WTO?
2. It helps with the Balance of Payments. 1. Market economy means that it is recognised that
3. Interest rates available outside India are more market forces of demand and supply are operating in
attractive. a particular country.
Select the correct answer using the codes given below 2. India has granted the market economy status to
(a) 1 and 3 (b) 1 and 2 China recently.
(c) Only 3 (d) All of these Select the correct answer using the codes given below
31. Consider the following statements. (a) Only 1 (b) Only 2
(c) Both 1 and 2 (d) Neither 1 nor 2
1. Deflation is a decrease in the general price level of goods
and services. 38. Consider the following statements in the context of
2. Deflation is the negative inflation rate which falls below Co-operative Banks in India.
zero per cent. 1. Co-operative Banks operate on no profit no loss basis.
Which of the statement(s) given above is/are correct? 2. Co-operative Banks are allowed to operate only in the
(a) Only 1 (b) Only 2 agriculture sector.
(c) Both 1 and 2 (d) Neither 1 nor 2 3. NABARD is a Co-operative Bank.
32. Consider the following statements in the context of an Which of the statement(s) given above is/are correct?
asset reconstruction company. (a) 1 and 3 (b) Only 1
1. Asset reconstruction companies buy bad loans from (c) 1 and 2 (d) All of these
banks and try to restructure them. 39. Which of the following steps will stop the
2. ARCIL is the first asset reconstruction company in India. depreciating the rupee, which has been happening
Which of the statement(s) given above is/are correct? recently?
(a) Only 2 (b) Only 1 1. Stopping the flow of Flls into India.
(c) Neither 1 nor 2 (d) Both 1 and 2 2. Buying up of foreign currency by the RBI.
33. Consider the following regarding National Agriculture 3. Reducing the CRR and other interest rates.
Insurance scheme Select the correct answer using the codes given below
1. It was introduced from rabi season of 1999-2000. (a) 2 and 3 (b) 1 and 3
2. The scheme covers all food crop and various (c) 1 and 2 (d) None of these
commercial crop. 40. Which among the following are true for Central Sales
Which of the statement(s) given above is/are correct ? tax?
(a) Only 1 (b) Only 2 1. It is levied on interstate trade.
(c) Both 1 and 2 (d) Neither 1 nor 2 2. It is also levied in the Union Territories.
34. Which of the following statement(s) is/are true regarding 3. It is levied in the SEZ.
the dispute settlement mechanism of the WTO? Select the correct answer using the codes given below
1. The losing nation must make changes in its law, if it is (a) 1 and 2
(b) Only 1
Practice Set 3

found to have violated a WTO provision.


2. If a nation does not follow the orders of the WTO after the (c) 2 and 3
completion of the dispute settlement process then a (d) All of the above
penalty can be imposed on that country. 41. Development expenditure of the Central Government
Select the correct answer using the codes given below does not include
(a) Only 1 (b) Only 2 1. Defence expenditure.
(c) Both 1 and 2 (d) Neither 1 nor 2 2. Expenditure on economic services.
35. Examine the following statements regarding the GATT 3. Expenditure on social and community services.
4. Grant to states.
and WTO.
Select the correct answer using the codes given below
1. India was not a founding member of GATT, but it later
(a) Only 1 (b) 1 and 3 (c) 1, 2 and 4 (d) Only 3
joined, when the WTO was formed.
192 Magbook ~ Indian Economy
42. Which of the following is true regarding the geographical Which of the statement(s) given above are correct?
indication provisions of the WTO? (a) 1 and 2 (b) 1 and 3
1. Gl is given only to a region or community for a particular (c) 2 and 3 (d) All of these
manufactured product. 47. Consider the following statements the functions
2. Gl remains valid for unlimited time as the geographical region of commercial banks in India include
is permanent in nature.
1. Purchase and sale of shares and securities on
3. India’s Gl registry is situated in Pune. behalf of customers.
Select the correct answer using the codes given below 2. Aeting as executers and trustees of wills.
(a) 1 and 3 (b) 2 and 3 (c) 1 and 2 (d) None of these Which of the statement(s) given above is/are correct
43. Consider the following statement(s) in the context of ?
development banks in India. (a) Only 1 (b) Only 2
1. Development banks are those, which provide long-term (c) Both 1 and 2 (d) Neither 1 nor 2
capital for developmental purposes. 48. Consider the following statements.
2. The first development bank constituted in India 1948 was 1. Green box subsidies relate to research and
IDBI. development and infrastructure like universities,
Which of the statement(s) given above is/are correct? roads in rural areas etc.
(a) Only 2 (b) Only 1 (c) Neither 1 nor 2 (d) Both 1 and 2 2. Green box subsidies don’t distort trade.
44. Consider the following regarding International Which of the statement(s) given above is/are correct
Development Association (IDA). ?
1. IDA is an associate institution of World Bank known as soft (a) Only 1 (b) Only 2
loan window of World Bank. (c) Both 1 and 2 (d) Neither 1 nor 2
2. The resources of IDA include subscribed capital by member 49. Consider the following regarding the objectives of
countries, general replenishments by developed countries, net International Finance Corporation (IFC).
income transferred by IBRD.
1. To provide loans to private sector.
Which of the statement(s) given above is/are correct? 2. To co-ordinate capital and management.
(a) Only 1 (b) Only 2 3. To induce capitalist countries to invest in developing
(c) Both 1 and 2 (d) Neither 1 nor 2 countries.
45. Consider the following regarding the Financial Stability Which of the statement(s) given above is/are
Board correct?
1. The Financial Stability Board, successor to the financial (a) Only 1 (b) 1 and 3
stability forum was established in 2009. (c) 2 and 3 (d) All of these
2. It comprises senior representatives of national financial 50. Which of the following is the specialised agency of
authorities (Central Banks, regulatory and supervisory the UN?
authorities and Ministry of Finance), international financial
1. International Bank for reconstruction and
institutions etc.
development.
Which of the statement(s) given above is/are correct ?
2. International Finance Corporation.
(a) Only 1 (b) Only 2 (c) Both 1 and 2 (d) Neither 1 nor 2
3. International Fund for Agricultural Development.
46. Consider the following regarding International 4. International Monetary Fund.
Development Association (IDA). Select the correct answer using the codes given
1. It was created in 1960. below
2. It is responsible for providing long-term, interest free loans to (a) 1 and 2 (b) 2 and 3
the world’s poorest countries. (c) Only 4 (d) All of these
3. IDA provides grants and credits with repayment periods of 35
to 40 years.

Answers
Practice Set 3

1. (b) 2. (d) 3. (b) 4. (d) 5. (b) 6. (c) 7. (d) 8. (b) 9. (b) 10. (a)
11. (d) 12. (c) 13. (b) 14. (c) 15. (b) 16. (c) 17. (c) 18. (c) 19. (c) 20. (b)
21. (c) 22. (d) 23. (d) 24. (d) 25. (a) 26. (d) 27. (d) 28. (b) 29. (c) 30. (a)
31. (c) 32. (d) 33. (c) 34. (a) 35. (d) 36. (d) 37. (a) 38. (b) 39. (d) 40. (a)
41. (a) 42. (d) 43. (b) 44. (c) 45. (c) 46. (d) 47. (c) 48. (c) 49. (c) 50. (c)
Practice Set ANALYSE YOURSELF 4
1. Consider the following regarding Trade Related Select the correct answer using the codes given below
Investment Measures (TRIMs) (a) Only 1 (b) Only 2
1. Trade related investment measures stipulate that all (c) Both 1 and 2 (d) Neither 1 nor 2
member nations must provide the same treatment to 6. Consider the following statements
foreign investors as to the domestic investors.
1. World Bank established IFC (International Finance
2. The agreement makes it obligatory for member
Corporation) in 1956.
countries to attract and invite foreign investment.
2. This corporation provides loan to private industries of
Which of the statement(s) given above is/are correct? developing nations without any government guarantee
(a) Only 1 (b) Only 2 and also promotes the additional capital investment in
(c) Both 1 and 2 (d) Neither 1 nor 2 these countries.
2. Consider the following regarding the objectives of Which of the statement(s) given above is/are correct?
G-20 forum (a) Only 1 (b) Only 2
1. Sustainable exchange rate regime supported by (c) Both 1 and 2 (d) Neither 1 nor 2
consistent exchange rate and monetary policy. 7. Prime Minister Narendra Modi launched the ‘Deen
2. Widespread implementation of codes and standards Dayal Upadhyaya Gram Jyoti Yojana’ (DDUGJY) in
including transparency data dissemination and financial Patna . The benefits of the scheme will be:
sector policy.
1. All villages and households shall be electrified
Which of the statement(s) given above is/are correct?
2. Increase in agriculture yield
(a) Only 1 (b) Only 2
3. Improvement in Health, Education, Banking (ATM)
(c) Both 1 and 2 (d) Neither 1 nor 2 services
3. What are the provisions under the Agreement on 4. Improvement in accessibility to radio, telephone,
Agriculture (AoA)? television, internet and mobile.
1. Reduction of domestic support by 21% by developed Select the correct answer using the codes given below
countries and 13% by developing countries. (a) Only 1 (b) 1 and 2 (c) 1,2 and 3 (d) 1, 2 , 3 and 4
2. Reduction of export subsidy by 36% by developed
8. The terms “Marginal Standing Facility Rate” and “Net
countries and 21% developing countries.
Demand and Time Liabilities,” sometimes appearing
Select the correct answer using the codes given below in news, are used in relation to
(a) Only 1 (b) Only 2
(a) banking operations (b) communication networking
(c) Both 1 and 2 (d) Neither 1 nor 2
(c) military strategies
4. With reference to the public sector undertakings in (d) supply and demand of agricultural products
India, consider the following statements
9. Which of the following bodies is preparing to launch
1. Minerals and Metals Trading Corporation of India an ‘inclusive economies’ initiative to combat
Limited is the largest non-oil importer of the country. economic inequality?
2. Project and Equipment Corporation of India Limited is
(a) APEC (b) European Union (c) SAARC (d) ASEAN
under the Ministry of Industry.
3. One of the objectives of Export Credit Guarantee 10. India is regarded as a country with ‘demographic
Corporation of India Limited is to enforce quality control dividend’. This is due to
Practice Set 4

and compulsory preshipment inspection of various (a) its high population in the age group below 15 years
exportable commodities. (b) its high population in the age group of 15-65 years
Which of the statement(s) given above is/are correct? (c) its high population in the age group above 65 years
(a) Only 1 (b) 1 and 2 (d) its high total population
(c) 2 and 3 (d) Only 3
11. Arrange the following in the chronological order.
5. Which of the following statements(s) is/are true 1. Kennedy round of negotiations.
regarding Green Revolution? 2. Uruguay round of negotiations.
1. Pulses weren’t targeted under Green Revolution. 3. Tokyo round of negotiations.
2. Then high yielding crops were dwarfed in stature. Codes
(a) 1, 2, 3 (b) 3, 1, 2 (c) 1, 3, 2 (d) 2, 1, 3
194 Magbook ~ Indian Economy
12. Consider the following statements Which of the statement(s) given above is/are correct?
1. Annual Survey of Industries (ASI) extends to the entire (a) Only 1 (b) 1 and 2
country. (c) 2 and 3 (d) All of these
2. ASI covers all factories registered under Sections 2m 19. Which of the following organisations were set-up
(i) and 2m (ii) of the Factories Act, 1948. under the Bretton Woods agreement?
Which of the statement(s) given above is/are correct? (a) World Bank
(a) Only 1 (b) Only 2 (b) IMF
(c) Both 1 and 2 (d) Neither 1 nor 2 (c) Both ‘a’ and ‘b’
13. Consider the following statements about the National (d) None of the above
Agricultural Insurance Scheme (NAIS) 20. Consider the following statement(s) regarding
1. The scheme has been implemented from Rabi 1999-2000 Financial Action Task Force
season. 1. It is an inter-governmental body responsible for
2. The scheme is available non-loanee farmers only. setting global standards for anti-money laundering
Which of the statement(s) given above is/are correct? and combating financing of terrorism.
(a) Only 1 (b) Only 2 2. India has become 34th country member of FATF.
(c) Both 1 and 2 (d) Neither 1 nor 2 Which of the statement(s) given above is/are correct?
14. Arrange the following items which are assigned weight (a) Only 1
in calculating wholesale price index in India. (b) Only 2
(c) Both 1 and 2
1. Primary articles
(d) Neither 1 nor 2
2. Fuel power light and lubricants
3. Food products 21. Which of the following statements are correct?
4. Chemicals and chemical products 1. RBI follows a proportional reserve system.
Codes 2. RBI follows a minimum reserve system.
(a) 1, 2, 3, 4 (b) 1, 2, 4, 3 3. RBI needs to maintain a gold reserve of atleast
(c) 4, 3, 2, 1 (d) 1, 3, 2, 4 115 crore.
Select the correct answer using the codes given below
15. Which of the following statements(s) is/are correct?
(a) 1 and 2 (b) 2 and 3
1. China consumes more gold than India.
(c) 1 and 3 (d) All of these
2. Bhutan and Nepal peg their currency to the Indian rupee.
Select the correct answer using the codes given below 22. Consider the following regarding the objectives of
(a) Only 1 (b) Only 2 UNCTAD
(c) Both 1 and 2 (d) Neither 1 nor 2 1. To promote the international trade especially with a
view to accelerating the economic development of
16. Consider the following statements- underdeveloped countries.
1. “Saubhagya” scheme was launched by the Prime Minister 2. To determine policies and principles for international
Narendra Modi to ensure electrification of all willing trade and economic development.
households in the country in rural as well as urban areas. 3. To assist economic and social council of United
2. Poor households would be provided electricity Nations Organisations (UNO).
connections free of cost. Which of the statements given above are correct?
3. Other households would also be provided electricity (a) 1 and 2 (b) 2 and 3
connections under the scheme on payment of ` 500 only (c) 1 and 3 (d) All of these
which shall be recovered by the DISCOMs.
Which of the above statements is/are correct? 23. With respect to NITI Aayog which of the following
(a) 1, 2 and 3 (b) Only 3 statements are true?
(c) Both 2 and 3 (d) Both 1 and 2 1. Sindhushree Khullar is the first CEO of NITI Aayog.
2. Arvind Panagariya is the first Vice-Chairman of the
17. Most international agencies which fund development NITI Aayog.
programmes on inter-governmental bilateral agreements 3. Economist Bibek Debroy and V K Saraswat are full
mainly provide. time members.
Practice Set 4

1. Technical assistance. Select the correct answer using the codes given below
2. Soft loans which are required to be paid back with (a) 1 and 2 (b) 2 and 3
interest. (c) 1 and 3 (d) All of these
3. Grants not required to be paid back.
24. Which of the following is most likely to cause current
Select the correct answer using the codes given below
account deficit in India?
(a) 2 and 3 (b) 1 and 3 (c) 1 and 2 (d) 2 and 3
1. Reduced excise duties on sports utility vehicles .
18. Consider the following statements 2. Reduced duties on gold.
1. The headquarters of the FAO is in Rome. 3. Ban on export of onions.
2. The FAO is an agency of United Nations Organisation. Select the correct answer using the codes given below
3. It came into existence 16th October, 1945. (a) 1 and 2 (b) 2 and 3
(c) 1 and 3 (d) All of the above
Magbook ~ Practice Set 4 195
25. Foreign Direct Investment (FDI) is investment directly Select the correct answer using the codes given below
into production in a country by a company located in (a) 1 and 2 (b) 2 and 3
another country then which one of the following (c) 1 and 3 (d) All of these
mode(s) is/are correct about the FDI?
33. Which one of the following statement(s) is correct
(a) Buying a company in the target country about the Agreement on Trade-Related Aspects of
(b) Expanding operations of an existing business in that Intellectual Property Rights (TRIPs) of the World
country Trade Organisation (WTO)?
(c) Investing in the shares and stocks of a company in the
(a) The patent right is covered under the TRIPS of the
target country
WTO
(d) Both ‘a’ and ‘b’
(b) Geographical Indication (GI) certification is
26. Consider the following items enforceable under the TRIPS of the WTO
1. Gems and jewellery (c) Copyright is the important component of the TRIPS of
2. Chemicals and related products WTO
3. Engineering goods (d) Both ‘a’ and ‘b’
4. Textiles
34. Kisan Credit Card was launched by Indian
Which of the items given above are the top 4 items in Government to facilitate the short-term loan and
India’s manufactured exports? cash delivery to farmers. Which one of the
(a) 1, 2 and 3 (b) 1, 2 and 4 following agencies is not included in the operation
(c) 2, 3 and 4 (d) All of these of the Kisan Credit Cards?
27. According to Reserve Bank of India, arrange the (a) Scheduled Commercial banks
following in increasing order of preferences to measure (b) Co-operative banks
inflation. (c) Regional Rural banks
1. Wholesale price index 2. Producer price index (d) NABARD
3. Consumer price index 35. Who among the following are major players in the
Codes government securities market?
(a) 3, 2, 1 (b) 2, 3, 1 (c) 1, 3, 2 (d) 1, 2, 3 1. Commercial Banks 2. Insurance Companies
28. Consider the following statements about Goods and 3. Individual Households
Services Tax (GST) Bill of India. 4. Co-operative and Regional Rural Banks
1. GST consists of 3 component such as Central GST, State Select the correct answer using the codes given below
GST and Local GST. (a) 1, 2 and 4 (b) 1, 3 and 4
2. Central GST and State GST will apply to all transactions (c) 1, 2 and 3 (d) All of these
of goods and services.
36. Indian Currency Printing Institutions are located in
Which of the statement(s) given above is/are correct? which of the following places?
(a) Only 1 (b) Only 2
1. Nasik 2. Hoshangabad 3. Hyderabad
(c) Both 1 and 2 (d) Neither 1 nor 2
4. Aurangabad 5. Devas
29. Which one of the following law states that bad money Select the correct answer using the codes given below
drives out good money if the exchange rate is set by law? (a) 1, 2 and 4 (b) 1, 2, 3 and 4
(a) Gresham’s law (b) Gilbert’s law (c) 1, 2, 3 and 5 (d) All of these
(c) Keynes’ law (d) Kuznet’s law
37. Which of the following statements regarding the
30. Consider the following statements Viability Gap Funding (VGF) scheme is incorrect?
1. India’s Trade Policy Review (TPR) is carried out every 4 (a) Under VGF, the Central Government meets up to 20%
years. of capital cost of a project being implemented in
2. WTO has a mechanism for regular review of the Trade Public Private Partnership (PPP) mode
Policies of member countries. (b) The scheme is administered by the Ministry of Finance
Which of the statement(s) given above is/are correct? (c) Sectors eligible for VGF are infrastructure, health and
(a) Only 1 (b) Only 2 education
(c) Both 1 and 2 (d) Neither 1 nor 2 (d) VGF is a force multiplier, enabling government to
Practice Set 4

leverage its resources more effectively


31. Global Competitiveness report is released by which of
the following organisations? 38. Since independence, both development and
(a) World Trade Organisation (b) European Union non-development expenditure have increased with
(c) World Bank (d) World Economic Forum more increase the latter. Non-development
expenditure involves
32. Which of the following activities fall under the purview 1. interest payments 2. subsidies
of Banking Ombudsman?
3. defence 4. irrigation
1. Internet banking 2. Credit cards Select the correct answer using the codes given
3. Debit cards below
4. Harassment by loan recovery agencies (a) 1 and 2 (b) Only 1
(c) 1, 2 and 3 (d) 2, 3 and 4
196 Magbook ~ Indian Economy
39. Which of the following statements(s) is/are the 45. Consider the following liquid assets
advantages of barter system? (a) Demand deposits with the banks
1. It is possible to trade without double coincidence of wants. (b) Time deposits with the banks
2. It promoted division of labour. (c) Savings deposits with the banks
Select the correct answer using the codes given below (d) Currency
(a) Only 1 (b) Only 2 The correct sequence of these assets in the decreasing
(c) Both 1 and 2 (d) Neither 1 nor 2 order of liquidity is
(a) 1, 4, 3, 2 (b) 4, 3, 2, 1
40. What will be the primary benefit of Inflation Indexed
(c) 2, 3, 1, 4 (d) 4, 1, 3, 2
Bonds?
(a) Curbing the fiscal deficit 46. Consider the following statements
(b) Decreasing the excessive gold consumption 1. Regarding the procurement of foodgrains, Government
(c) Reducing NPA of banks of India follows a procurement target rather than an
(d) Reducing inflation open ended procurement policy.
2. Government of India announces minimum support
41. Consider the following statements about the National prices only for cereals.
Development Council (NDC) of India
3. For distribution under Targeted Public Distribution
1. It discusses the progress of national plan.
System (TPDS), wheat and rice are issued by the
2. Suggest the ways to achieve goals of the national plan. Government of India at uniform Central issue prices to
3. It gives guidelines to formulate the national plan. the States/Union Territories.
4. It suggests plan allocations for the different sectors Which of the statement(s) given above is/are correct?
and states.
(a) 1 and 2 (b) Only 2
Which of the statements given above are correct? (c) 1 and 3 (d) Only 3
(a) 2, 3 and 4 (b) 1, 2 and 3 (c) 1, 3 and 4(d) All of these
47. With reference to agriculture sector of India, consider
42. Consider the following statements the following statements
1. NABARD is an apex institution handling matters concerning 1. Rural Infrastructure Development Fund disperses
policy, planning and operating the field of credit for loans to the states to complete the minor irrigation
agriculture and other’ developmental activities in rural India. projects.
2. NABARD operates through its head office at Delhi. 2. Rural Infrastructure Development Fund is managed
Which of the statement(s) given above is/are correct? by the public sector Commercial Banks.
(a) Only 1 (b) Only 2 Which of the statement(s) given above is/are correct?
(c) Both 1 and 2 (d) Neither 1 nor 2 (a) Only 1
43. Which among the following services are currently (b) Only 2
providing by post offices? (c) Both 1 and 2
1. Saving account 2. Recurring deposits (d) Neither 1 nor 2
3. Fixed deposits 48. An ad valorem duty is a tax on the basis of
4. Public Provident Fund Account (a) the value added
Select the correct answer using the codes given below (b) the units of the commodity
(a) 1, 2 and 3 (b) 2, 3 and 4 (c) the advertisement expenditure
(c) 1, 3 and 4 (d) All of these (d) the price of a commodity
44. Both Foreign Direct Investment (FDI) and Foreign 49. Poverty in less developed countries is largely due to
Institutional Investor (FII) are related to investment in a (a) lack of cultural activities
country. Which one of the following statement best (b) voluntary idleness
represent an important difference between the two? (c) income Inequality
(a) Fll helps bring better management skills and technology, (d) lack of the intelligence of the people
while FDI brings only capital
50. The modern economy of India is not characterised by
(b) FII helps in increasing capital availability in general, while
FDI only targets specific sectors (a) production for market
(b) development for money economy
Practice Set 4

(c) FDI flows only into the secondary market, while Fll targets
primary market (c) capital intensive mode of production
(d) FII is considered to be more stable than FDI (d) self-sufficient village system

Answers
1. (c) 2. (c) 3. (c) 4. (a) 5. (c) 6. (c) 7. (c) 8. (a) 9. (a) 10. (b)
11. (a) 12. (b) 13. (a) 14. (b) 15. (c) 16. (a) 17. (d) 18. (d) 19. (c) 20. (c)
21. (b) 22. (d) 23. (d) 24. (d) 25. (d) 26. (d) 27. (c) 28. (b) 29. (a) 30. (c)
31. (d) 32. (d) 33. (d) 34. (d) 35. (b) 36. (c) 37. (c) 38. (c) 39. (b) 40. (b)
41. (b) 42. (a) 43. (d) 44. (b) 45. (d) 46. (c) 47. (a) 48. (d) 49. (c) 50. (c)
Practice Set
1. Which among the following is/are the member
ANALYSE YOURSELF 5
6. Correct statements regarding Pradhan Mantri
countries of the Eurasian Economic Union (EEU)? LPG Panchayat -
1. Belarus 2. Russia (a) Pradhan Mantri LPG Panchayat’ will focus on
3. Kazakhstan 4. China spreading awareness among LPG users about the
Select the correct answer using the codes given below proper use and benefits of LPG.
(a) 1 and 3 (b) 1, 3 and 4 (b) This Panchayat will act as an interactive forum for
(c) 1, 2 and 3 (d) All of these Pradhan Mantri Ujjwala Yojana beneficiaries, officials,
LPG distributors and NGOs (Non-Governmental
2. Consider the following statements
Organisations).
1. Scheduled Banks are those banks which are included in (c) Under one Panchayat, around 100 LPG customers
the second Scheduld of the Reserve Bank Act, 1934. will be clubbed.
2. There are 10 Non-scheduled Commercial Banks (d) All are true
operating in the country.
3. Co-operative banks are organised and managed on the 7. Consider the following statements
principle of co-operation, self-help and mutual help. 1. The INC established National Planning commission
Which of the statement(s) given above is/are correct? under the chairmanship of JL Nehru.
(a) 1 and 3 (b) Only 2 2. A Plan of Economic Development of India was put
(c) 2 and 3 (d) All of these forward by leading businessmen and industrialists in
1944 was also called as the ‘Bombay plan’.
3. The Sex Ratio at Birth (SRB) saw a declined in 17 out of 3. Indicative plan is operated under and planned
21 Large State of the country, with ...... recording an economy, but not command economy.
alarming dip of 53 points, a report released by the NITI
Which of the statement(s) given above is/are correct?
Aayog.
(a) Only 2
(a) Bihar (b) Jharkhand (b) 1 and 3
(c) Gujrat (d) Haryana (c) 2 and 3
4. Consider the following statements (d) All of the above
1. The state is called the Laissez faire state in the Market 8. Arrange the events in chronological order regarding
Economy. The French word Laissez faire means ‘Let do’. Indian economy.
2. Command economy do not create wealth sustainably 1. 1st phase of nationalisation of 16 banks.
and are not conducive for innovation and efficiency. 2. Establishment of Narashimhan committee on the
3. The Bombay plan was made by J L Nehru. Banking system.
Which of the statement(s) given above are correct? 3. Enactment of FEMA Act in India.
(a) 1 and 2 (b) 2 and 3 4. Establishment of EXIM Bank in India.
(c) All of these (d) None of these Codes
5. In context of the present ceiling on investment for (a) 1, 3, 4, 2
categorisation of various enterprises as Micro, Small (b) 1, 4, 2, 3
and Medium Enterprises (MSMEs). (c) 1, 3, 2, 4
1. A micro-enterprise is an enterprise where the investment (d) 1, 4, 3, 2
in equipment does not exceed ` 25 lakh. 9. Which one of the following is the composition of
2. A small enterprise is an enterprise where the investment Liquidity aggregate-2 (L2) other than Liquidity
Practice Set 5

in equipment is more than ` 25 lakh, but does not exceed aggregate-1 (L1) in the Indian Monetary system?
` 5 crore. (a) Term deposits with Term Lending Institutions and
3. A medium enterprise is an enterprise where the Refinancing Institutions
investment in equipment is more than ` 5 crore but does (b) Term Borrowing by Refinancing Institutions
not exceed ` 10 crore. (c) Certificates of Deposit issued by Refinancing
Which of the statement(s) given above is/are correct? Institutions
(a) Only 1 (b) Only 2 (d) All of the above
(c) Only 3 (d) All of these
198 Magbook ~ Indian Economy
10. The Central Nodal Agency for implementing the price 4. MODVAT implies rationalisation of excise duty only
support operations for commercial crops is while VAT implies rationalisation of excise, sales and
(a) NAFED (b) NABARD turnover taxes etc.
(c) TRIFED (d) FCI Which of the statements given above are correct?
(a) 2, 3 and 4 (b) 1, 2 and 3
11. Which of the following are the provisions of SARFAESI (c) 2 and 3 (d) 3 and 4
Act which enables banks to reduce their
Non-Performing Assets (NPAs) ? 16. With regards to Scheduled Commercial banks
1. Enforcement of Security Interest by secured creditor consider the following statements about Regional
(Banks and Financial Institutions). Rural Banks (RRBs) in India.
2. Transfer of non-performing assets to asset reconstruction 1. RRBs are limited to a specific region comprising one
company, which will then dispose of those assets and or more districts of a State.
realise the proceeds. 2. RRBs grant direct loans and advances only to small
3. To provide a legal framework for securitisation of assets. and marginal farmers.
4. Assisting banks in making the credibility track record of 3. The lending rate of the RRBs are always less than the
customers under Credit Information Bureau of India prevailing rates of Indian Commercial banks.
Limited (CIBIL). 4. RRBs are functioned under the Supervision of
Select the correct answer using the codes given below Security Exchange Board of India.
(a) 1 and 2 (b) 1, 2 and 3 Which of the statement(s) given above are correct?
(c) 2, 3 and 4 (d) All of these (a) 3 and 4 (b) 1, 2 and 3
(c) 2 and 3 (d) All of these
12. The Laffer curve is the graphical representation of
(a) the relationship between tax rates and absolute revenue 17. Consider the following statements
these rates generate for the government 1. Integrated Rural Development Programme (IRDP)
(b) the inverse relationship between the rate of unemployment was launched to lift the target groups above the
and the rate of inflation in an economy below poverty line.
(c) the inequality in income distribution 2. The IRDP was started as an initiative of the Lead Bank
(d) the relationship between environmental quality and scheme to create additional employment
economic development opportunities in rural areas.
Which of the statement(s) given above is/are correct?
13. Consider the following statements (a) Only 1 (b) Only 2
1. Bulk of employment in India is in rural areas. (c) Both 1 and 2 (d) Neither 1 nor 2
2. The disguised unemployment in agricultural sector is
perennial. 18. Banks of India are required to maintain a certain
3. Industrialisation has rendered several people jobless in ratio between their risky assets and capital which is
India. known as
Which of the statements given above are correct? (a) Capital Adequacy Ratio (CAR)
(a) 1 and 2 (b) 1 and 3 (b) Statutory Liquidity Ratio (SLR)
(c) 2 and 3 (d) All of these (c) Reserve Bank of India (RBI)
(d) Capital to Risk Weighted Assets Ratio (CRAR)
14. The non-votable charges or ‘charged expenditure’
included in the Union Budget include 19. A financial intermediary is stronger than an
individual investor due to
1. salary and allowances of the Presiding Officers of the
Houses of Parliament. (a) economies of scale (b) asymmetry of information
2. salary and allowances of the Judges of the Supreme (c) portfolio diversification (d) All of the above
Court and High Courts. 20. Consider the following statements
3. salary and allowances of the CAG. 1. Swabhiman scheme provides banking facilities in
4. pension of the retired Judges of the Supreme Court. remote rural areas.
5. pension of the retired Judges of High Courts. 2. Swavalamban scheme provides subsidy on NPS
Which of the statements given abvoe are correct? accounts for the unorganised sector.
(a) 1 and 2 (b) 1, 2 and 3 Which of the statement(s) given above is/are correct?
Practice Set 5

(c) 1, 2, 3 and 4 (d) All of these (a) Only 1 (b) Only 2


15. MODVAT is different from VAT in the sense that (c) Both 1 and 2 (d) Neither 1 nor 2
1. under MODVAT, tax is levied only on value addition while 21. Which among the following statements is true about
under VAT, excise is imposed on final value and then the HRIDAY scheme?
rebate is given on inputs. (a) It is a scheme for conserving and preserving the
2. MODVAT is only at the production stage while VAT heritage characters of 12 cities
extends both to production and distribution stage. (b) It is a scheme for conserving the ancient medicines of
3. under MODVAT excise is levied on final value and then India
rebate is given on inputs while under VAT, tax is levied (c) It is a scheme for conserving the wetlands
only on value addition. (d) It is a scheme for rehabilitation of tribal people
Magbook ~ Practice Set 5 199
22. Consider the following statements Which of the credit controls given above are used by
1. Certificates of deposits are the very important RBI as a selective credit control measures?
instrument of the Indian money market. (a) 1 and 2 (b) 1 and 3 (c) 2 and 3 (d) All of these
2. Certificates of deposits are generally issued by the Union 28. With reference to Indian Capital Market, consider the
Government with collaboration with certain states. following statements
Which of the statement(s) given above is/are correct? 1. CRISIL was set-up in the eighth Five Year Plan.
(a) Only 1 (b) Only 2 2. CRISIL rates the debt instruments of the public
(c) Both 1 and 2 (d) Neither1 nor 2 sectors.
23. Which one of the following is correct about the Which of the statement(s) given above is/are correct?
Commercial Papers (CPs) in the Indian money market? (a) Only 1 (b) Only 2
(a) It is issued by Indian Commercial banks (c) Both 1 and 2 (d) Neither 1 nor 2
(b) It is issued by the Central Government 29. Consider the following about the Lead Banks role in
(c) It is issued by the companies with a net worth more than the Lead Bank scheme.
` 4 crore
1. To co-ordinate the co-operative banks. Commercial
(d) It is issued by the companies with a net worth less than
banks and other financial institutions in their allotted
` 50 lakh districts.
24. Consider the following about the Banking Financial 2. To survey and identify the unbanked districts and take
Supervision (BFS) in India. initiatives to open bank branches.
1. The RBI performs financial supervision function under 3. To estimate the deposits potential and the credit gaps
the guidance of the Board for Financial Supervision and fill the vacuum in the districts.
(BFS). Which of the roles given above is/are correct?
2. Primary objective of BFS is to undertake consolidate (a) Only 1 (b) 1 and 3 (c) 2 and 3 (d) All of these
supervision of the financial sector comprising
30. Which of the following union ministries have jointly
commercial banks, financial institutions and
launched a new scheme “SAATHI”?
non-banking finance companies.
Select the correct answer using the codes given below (a) Ministries of Finance and Defense
(b) Ministries of Power and Science & Technology
(a) Only 1 (b) Only 2
(c) Ministries of Minority Affairs and Human Resource
(c) Both 1 and 2 (d) Neither 1 nor 2
Development
25. The G-8 countries include (d) Ministries of Power and Textiles
1. France 2. Germany 31. Which one of the following committee was
3. Canada 4. Italy constituted by the RBI to study issues and concerns
5. Netherlands 6. UK in the micro-finance sector?
7. US 8. Russia (a) M Narasimhan committee
9. Japan (b) S Janikiraman committee
Select the correct answer using the codes given below (c) Chakravarti committee
(a) 1, 2, 3, 4, 5, 6, 8 and 9 (d) YH Malegam committee
(b) 1, 2, 4, 5, 6, 7, 8 and 9
(c) 1, 2, 3, 4, 6, 7, 8 and 9
32. Consider the following statements
(d) 1, 2, 3, 4, 5, 6, 7 and 9 1. Money supply measures M0 , M1 and M3 are widely used
in India.
26. Choose the correct statements 2. Money supply measures M2 and M4 are rarely used in
1. The Reserve Bank of India (RBI) announced that Masala India.
bonds will be treated as External Commercial Which of the statement(s) given above is/are correct?
Borrowings (ECB). (a) Only 1 (b) Only 2
2. Considering Masala bonds as ECBs will free up more (c) Both 1 and 2 (d) Neither 1 nor 2
investments by FPIs (Foreign Portfolio Investors). 33. Which of the following are among the taxes levied
3. Masala bonds are rupee-denominated overseas bonds. exclusively by the Central Government and are
Which of the above statement(s) is/are correct? mentioned in the Union list of the seventh Schedule
Practice Set 5

(a) 1 , 2 and 3 (b) Only 3 of the Constitution of India?


(c) Both 2 and 3 (d) Both 1 and 2 1. Corporation tax.
27. Consider the following kinds of credit controls 2. Taxes on advertisement in the newspapers.
1. Minimum margins for lending against specific 3. Taxes on agricultural income.
securities. 4. Taxes on consumption or sale of electricity.
2. Ceiling on the amount of credit for certain purposes. Select the correct answer using the codes given below
3. Discriminatory rate of interest charged on certain types (a) Only 2 (b) Only 4
of advances. (c) 1 and 2 (d) 3 and 4
200 Magbook ~ Indian Economy
34. Consider the following statements 40. Consider the following types of income
1. The Direct Taxes code proposes a Corporate rate tax of 1. Personal income 2. Private income
30% as against the current effective rate of 33.2%. 3. Disposable income 4. National income
2. The direct taxes code incorporates taxation mechanisms The correct sequence in descending order of their
to deal with the international issue of ‘transfer pricing’. magnitude is
Which of the statement(s) given above is/are correct? (a) 4, 2, 3, 1 (b) 2, 4, 3, 1
(a) Only 1 (b) Only 2 (c) 4, 2, 1, 3 (d) 2, 4, 1, 3
(c) Both 1 and 2 (d) Neither 1 nor 2
41. Micro-finance of the provision of financial services
35. The government has powers under the Essential to people of low income groups. This includes both
Commodities Act, 1955 (EC Act) to declare a commodity the consumers and the self-employed.
as an essential commodity to ensure its availability to The service/services rendered under micro-finance
people at fair price. The commodities covered under the is/are
act include 1. credit facilities 2. saving facilities
1. paper 3. insurance facilities 4. fund transfer facilities
2. hank Yarn Select the correct answer using the codes given below
3. fertilizers (a) Only 1 (b) 1 and 4
4. petroleum and petroleum products (c) 2 and 3 (d) All of these
5. cotton and jute seeds
42. Economic development may be defined as a process
Select the correct answer using the codes given below whereby the real per-capita income of a country
(a) 1, 2 and 3 (b) 1, 3 and 4 increases
(c) 3, 4 and 5 (d) 2, 3, 4 and 5
(a) subject to the condition that the number below an
36. Global capital flows to developing countries increased absolute poverty line does not increase
significantly during the 90s. In view of the East Asian (b) over a long period of time
financial crisis and Latin American experience, which (c) object to the condition that the stribution of income
type of inflow is good for the host country? does not become more unequal
(a) Commercial loans (d) All of the above
(b) Foreign Direct investment 43. Consider the following objectives
(c) Foreign Portfolio investment
1. To restrict the expansion of bank credit.
(d) External Commercial borrowings
2. To augment the investment of the banks in
37. Why is Reserve Bank of India (RBI) also known as Government decurities.
Bankers' Bank and lender of last resort? 3. To ensure solvency of banks.
(a) Reserve Bank of India maintains the security of the Which of the objectives given above are the main
commercial banks and appointment of its officials objectives of the Statutary-Liquidity Requirements
(b) Commercial banks need to keep some amount in the (SLRs)?
custody of the RBI and can borrow from the RBI at the (a) 1 and 2 (b) 1 and 3
times of need and stringency (c) 2 and 3 (d) All of these
(c) RBI acts as an agent of the government to regulate the
day-to-day functions of the commercial banks and
44. Consider the following components
maintain the liquidity of the banks 1. New Broad Money (NM3 ).
(d) Both ‘a’ and ‘c’ 2. New Narrow Money.
3. All Deposits with the post office savings banks.
38. Which one of the following agencies of the Indian
4. National Savings Certificates.
Government publishes the narrow Money (M1) and Broad
Which of the components given above is/are correct
money (M3 ) on fortnightly basis?
included in the Liquidity Aggregates (L1) in the Indian
(a) State Bank of India
economy?
(b) Security Exchange Board of India
(a) 2 and 4 (b) 2 and 3
(c) Reserve Bank of India
(c) 1 and 3 (d) All of these
(d) Central Statistical Organisation
45. Consider the following statements
Practice Set 5

39. Which of the following situations does not result due to


the depreciation of the rupee with respect to foreign 1. Housing Development Finance Corporation Limited
(HDFC) is a National Level Housing Finance
currencies?
institution set-up in 1977.
(a) Exports become competitive in the global market
2. HDFC was established as a subsidiary company of
(b) Import substitution becomes impossible
the ICICI.
(c) Costlier imports results into higher prices propelling
Which of the statement(s) given above is/are correct?
further depreciation
(a) Only 1 (b) Only 2
(d) Both ‘a’ and ‘b’
(c) Both 1 and 2 (d) Neither 1 nor 2
Magbook ~ Practice Set 5 201
46. Consider the following statements 48. With reference to the institution of Banking
1. A Commercial bank is a type of financial intermediatory Ombudsman in India, which one of the statements is
because it mediates between the savers and borrowers. not correct?
2. Commercial bank and assist companies in raising funds (a) The Banking Ombudsman is appointed by the Reserve
in the capital markets (both equity and debt). Bank of India
3. Investment bank is also known as the merchant bank. (b) The Banking Ombudsman can consider complaints
Which of the statement(s) give above is/are correct? from non-resident Indians having accounts in India
(a) Only 2 (c) The orders passed by the Banking Ombudsman are
(b) 1 and 3 final and binding on the parties concerned
(c) Only 1 (d) The service provided by the Banking Ombudsman is
(d) All of the above free of any fee

47. Which of the following states is/are have 100% bank 49. Changes in the prices of goods in India is measured
accounts in every household? by which of the following index numbers ?
1. Gujarat (a) Wholesale Price Index (WPI)
2. Kerala (b) Consumer Price Index (CPI)
3. Madhya Pradesh (c) Both ‘a’ and ‘b’
4. Goa (d) None of the above
Select the correct answer using the codes given below 50. Which of the following remedies are adopted to
(a) 2 and 4 control price rise in the economy?
(b) 2 and 3 (a) Monetary measures
(c) 2, 3 and 4 (b) Fiscal measures
(d) All of the above (c) Administered price mechanism
(d) All of the above

Answers
1. (c) 2. (c) 3. (c) 4. (a) 5. (d) 6. (d) 7. (d) 8. (b) 9. (d) 10. (a)
11. (b) 12. (a) 13. (a) 14. (d) 15. (a) 16. (b) 17. (c) 18. (d) 19. (d) 20. (c)
21. (a) 22. (a) 23. (c) 24. (c) 25. (c) 26. (a) 27. (d) 28. (b) 29. (d) 30. (d)
31. (d) 32. (c) 33. (c) 34. (c) 35. (d) 36. (b) 37. (b) 38. (c) 39. (b) 40. (c)
41. (d) 42. (d) 43. (d) 44. (c) 45. (c) 46. (b) 47. (d) 48. (c) 49. (c) 50. (d)

Practice Set 5
PREVIOUS YEARS' Questions
SOLVED PAPERS
Set 1
1. Atal Innovation Mission is setup under the [IAS 2019] Exp. (a) Purchasing Power Parity (PPP) exchange rate is the
(a) Department of Science and Technology rate at which the currency of one country would have to be
(b) Ministry of Labour and Employment converted into that of another country to buy the same amount
(c) NITI Aayog of goods and services in each country.
(d) Ministry of Skill Development and Entrepreneurship According to this concept, two currencies are in equilibrium,
known as the currencies being at par, when a basket of goods
Exp. (c) The Atal Innovation Mission (AIM) is a flagship initiative is priced the same in both countries, taking into account the
setup by NITI Aayog to promote innovation and exchange rates. In terms of PPP dollars, India is the third largest
entrepreneurship across the length and breadth of the country. economy in the world.
Atal Innovation Mission including Self-Employment and Talent
Utilisation (SETU) is government of India’s endeavour to 5. Among the agricultural commodities imported by India,
promote a culture of innovation and entrepreneurship. Its which one of the following accounts for the highest
objective is to serve as a platform for promotion of worldclass imports in terms of value in the last five years?[IAS 2019]
innovation Hubs particularly in technology driven areas. (a) Spices (b) Fresh fruits
(c) Pulses (d) Vegetable oils
2. The Global Competitiveness Report is published by the
[IAS 2019] Exp. (d) In terms of value of imports, vegetable oils constitute
(a) International Monetary Fund the highest imported commodity in last five years.
India imports huge amount of various types of vegetable oil
(b) United Nations Conference on Trade and Development
from many countries. India relies on imports for almost 70% of
(c) World Economic Forum
its vegetable oil consumption.
(d) World Bank
Exp. (c) The Global Competitiveness Report is an annual 6. Which one of the following is not the most likely
report published by the World Economic Forum since 2004. It measure the government/RBI takes to stop the slide of
ranks countries based on Global Competitiveness Index and
Indian rupee? [IAS 2019]
assess their ability to provide high levels of prosperity to their (a) Curbing imports of non-essential goods and promoting
citizens. exports.
(b) Encouraging Indian borrowers to issue rupee
3. In a given year in India, official poverty lines are higher in denominated Masala Bonds.
some states than in others because [IAS 2019] (c) Easing conditions relating to external commercial
(a) poverty rates vary from state to state borrowing.
(b) price levels vary from state to state (d) Following an expansionary monetary policy.
(c) gross state product varies from state to state Exp. (d) In order to stop the slide of Indian rupee, following
(d) quality of public distribution varies from state to state measures can be taken
Exp. (b) The levels of poverty vary from state to state because
l
Curb imports and promote exports.
there are interstate price variations. The Planning Commission
l
Mandatory hedging conditions for infrastructure loans
estimates poverty in the country on the basis of consumer External Commercial Borrowing (ECB) route needs to be
expenditure surveys conducted by NSSO (National Sample reviewed.
Survey Office). So, price difference is bound to create
l
Removal of 20% exposure limit on investments by
variations in poverty lines of states across India. foreign portfolio investors in debt to a single corporate
group.
4. Consider the following statements. [IAS 2019] l
Permit manufacturing sector to access ECBs upto $ 50
1. Purchasing Power Parity (PPP) exchange rates are million with residual maturity of one year instead of three
calculated by comparing the prices of the same basket of years.
goods and services in different countries. l
Exempt Masala Bonds from withholding tax.
2. In terms of PPP dollars, India is the sixth largest economy By following an expansionary monetary policy economy can
in the world. be stimulated. It increases money supply, lowers the interest
Which of the statements given above is/are correct? rate and increases the aggregate demand. It cannot stop the
(a) Only 1 (b) Only 2 slide of rupee but lowers the value of the currency and
(c) Both 1 and 2 (d) Neither 1 nor 2 decreases exchange rate.
206 Magbook ~ Indian Economy

7. Consider the following statements. Exp. (b) Service Area Approach is an alternative and
The Reserve Bank of India’s recent directives relating to improved method of Lead Bank Scheme for deployment of
‘Storage of Payment System Data’, popularly known as bank credit for rural development. According to it, every
Data Diktat, command the payment system providers that village in the country would have an access to banking
[IAS 2019] facilities. The concerned bank should meet banking needs of
1. they shall ensure that the entire data relating to payment service area by creating link between bank credit-production
systems operated by them are stored in a system only in and productivity and income expansion.
India.
10. Consider the following statements. [IAS 2019]
2. they shall ensure that the systems are owned and operated
1. Most of India’s external debt is owed by governmental
by public sector enterprises.
entities.
3. they shall submit the consolidated system audit report to
2. All of India’s external debt is denominated in US dollars.
the Comptroller and Auditor General of India by the end of
the calendar year.
Which of the statements given above is/are correct?
(a) Only 1 (b) Only 2
Which of the statements given above is/are correct?
(c) Both 1 and 2 (d) Neither 1 nor 2
(a) Only 1 (b) 1 and 2 (c) Only 3 (d) 1, 2 and 3
Exp. (d) Both options and incorrect. External debt is the
Exp. (a) In April 2018, RBI issued following directives related to
money that borrowers in a country owe to foreign lenders. At
‘Storage of Payment System Data’, popularly known as Data
the end of March 2018, India’s external debt was placed at
Diktat: US $ 529.7 billion. Most of it was owed by private businesses
l
All system providers shall ensure that the entire data which borrowed at attractive rates from foreign lenders.
relating to payment systems operated by them are stored External debt may be denominated in either the rupee or a
in a system only in India. This data should include full foreign currency like US dollar, most of India’s external debt
end-to-end transaction details/information is linked to the dollar.
collected/carried/processed as part of the message/
payment instruction. For the foreign transaction, if any, 11. Which of the following is not included in the assets of a
data can also be stored in foreign country, if required. commercial bank in India? [IAS 2019]
l
System providers shall ensure compliance of data storage (a) Advances
within a period of six months and report compliance of same (b) Deposits
to RBI. (c) Investments
l
System providers shall submit the System Audit Report (d) Money at call and short notice
(SAR) on completion of monitoring of data storage. The Exp. (b) Assets of commercial banks include cash in hand
audit should be conducted by CERT-IN empaneled auditors. and with RBI, money at call and short notice, investments,
The SAR duly approved by Board of System Providers loans, advances and bills discounted and purchased.
should be submitted to RBI. Liabilities of commercial banks include paid-up capital and
reserves, time deposits, demand deposits and borrowing.
8. The money multiplier in an economy increases with
Deposits are a liability on a commercial bank, since it must
which one of the following? [IAS 2019]
return its lenders the amount it owes them.
(a) Increase in the Cash Reserve Ratio.
(b) Increase in the banking habit of the population. 12. In the context of India, which of the following factors
(c) Increase in the Statutory Liquidity Ratio. is/are contributor/contributors to reducing the risk of a
currency crisis? [IAS 2019]
(d) Increase in the population of the country.
1. The foreign currency earnings of India’s IT sector.
Exp. (b) Money multiplier is defined as ratio of the stock of
2. Increasing the government expenditure.
money to the stock of high powered money in an economy.
3. Remittances from Indians abroad.
Clearly, its value is greater than 1.
M 1 + cdr Select the correct answer by using the codes given
= below.
H edr + rdr
(a) Only 1 (b) 1 and 3 (c) Only 2 (d) 1, 2 and 3
Where, M = Stock of money, H = High-powered money stock
Exp. (b) Currency crisis refers to decline in value of a
cdr = Currency Deposit Ratio, rdr = Reserve Deposit Ratio country’s currency. The decline in value creates instabilities in
A high (or low) value of CRR (Cash Reserve Ratio) or SLR exchange rates. The foreign exchange earnings and
(Statutory Liquidity Ratio) helps to increase (or decrease) the remittances arrest downfall of currency and will bring foreign
value of Reserve Deposit Ratio, thus diminishing (or increasing) currency. This will contribute in reducing the risk of a
value of money multiplier. Increase in banking habit of the currency crisis. Increasing the government expenditure will
population and also increases money multiplier. not reduce the risk of currency crisis.
9. The Service Area Approach was implemented under the 13. Which of the following is issued by registered Foreign
purview of [IAS 2019]
Institutional Investors to overseas investors who want
(a) Integrated Rural Development Programme. to be part of the Indian stock market without
(b) Lead Bank Scheme. registering themselves directly? [IAS 2019]
(c) Mahatma Gandhi National Rural Employment Guarantee (a) Certificate of Deposit (b) Commercial Paper
Scheme. (c) Promissory Note (d) Participatory Note
(d) National Skill Development Mission.
Magbook ~ Solved Paper 1 207
Exp. (d) A Participatory Note, commonly known as P-note or 16. With reference to Asian Infrastructure Investment Bank
PN, is an instrument issued by a registered Foreign (AIIB), consider the following statements. [IAS 2019]
Institutional Investor (FII) to an overseas investors who wishes 1. AIIB has more than 80 member nations.
to invest in Indian stock markets without registering 2. India is the largest shareholder in AIIB.
themselves with the market regulator, the Securities and 3. AIIB does not have any members from outside Asia.
Exchange Board of India (SEBI).
Which of the statements given above is/are correct?
14. Consider the following statements. [IAS 2019] (a) Only 1 (b) 2 and 3 (c) 1 and 3 (d) 1, 2 and 3
1. The United Nations Convention Against Corruption Exp. (a) Asian Infrastructure Investment Bank (AIIB) is a
(UNCAC) has a ‘Protocol against the Smuggling of multilateral development bank with a mission to improve social
Migrants by Land, Sea and Air’. and economic outcomes in Asia. Its headquarter is in Beijing,
2. The UNCAC is the first-ever legally binding global China. The bank was proposed by China in 2013 and the
anti-corruption instrument. initiative was launched at a ceremony in Beijing in October
3. A highlight of the United Nations Convention against 2014. It has now 97 approved members (As of April, 2019)
Transnational Organised Crime (UNTOC) is the inclusion worldwide. China has single largest voting share at 26.6%. It
of a specific chapter aimed at returning assets to their has various non-Asian members.
rightful owners from whom they had been taken illicitly.
4. The United Nations Office on Drugs and Crime (UNODC)
17. What was the purpose of Inter-Creditor Agreement signed
by Indian banks and financial institutions recently?
is mandated by its member states to assist in the
[IAS 2019]
implementation of both UNTOC and UNCAC.
(a) To lessen the Government of India’s perennial burden of
Which of the statements given above are correct?
fiscal deficit and current account deficit.
(a) 1 and 3 (b) 2, 3 and 4 (c) 2 and 4 (d) 1, 2, 3 and 4
(b) To support the infrastructure projects of Central and State
Exp. (b) The protocol against the smuggling of migrants by Governments.
land, sea and air is protocol to United Nations Convention (c) To act as independent regulator in case of applications for
Against Transnational Organised Crime (UNTOC). The loans of ` 50 crore or more.
convention UNTOC is supplemented by three protocols, (d) To aim at faster resolution of stressed assets of ` 50 crore
which target specific areas and manifestations of organised or more which are under consortium lending.
crime. The United Nations Convention Against Corruption Exp. (d) The agreement, known as Inter-Creditor Agreement
(UNCAC) is the only legally binding universal anti-corruption (ICA) was framed under the aegis of the Indian Banks
instrument. The vast majority of UN member states are parties Association and follows the recommendations of the Sunil
to the convention. A highlight of UN Convention Against Mehta Committee on stressed asset resolution. ICA is an
Corruption is the inclusion of a specific chapter on asset agreement among banks that have dues from a borrower in the
recovery which aimed at returning assets to their rightful stress. Banks and Financial institutions have signed
owners including countries from which they had been taken Inter-Creditor Agreement (ICA) aimed at faster resolution of
illicitly. stressed assets of ` 50 crore or more which are under
UN Office on Drugs and Crime (UNODC) is available to consortium lending. It has been signed by 22 public sector
provide assistance in implementation of both UNTOC and banks, 19 private sector banks and 32 foreign banks and 12
UNCAC. major financial intermediaries like LIC, HUDCO etc. This is a
part of project ‘Sashakt’.
15. Consider the following statements. [IAS 2019]
As per the Industrial Employment (Standing Orders) 18. The Chairmen of public sector banks are selected by the
Central (Amendment) Rules, 2018 [IAS 2019]
1. if rules for fixed-term employment are implemented, it (a) Banks Board Bureau
becomes easier for the firms/companies to lay off (b) Reserve Bank of India
workers. (c) Union Ministry of Finance
2. no notice of termination of employment shall be (d) Management of concerned bank
necessary in the case of temporary workman. Exp. (a) The government has set up an autonomous Banks
Which of the statements given above is/are correct? Board Bureau (BBB) for selection of heads of Public Sector
(a) Only 1 (b) Only 2 Banks (PSBs) and financial institutions and help banks in
(c) Both 1 and 2 (d) Neither 1 nor 2 developing strategies and capital raising plans. BBB was part
Exp. (c) According to the Industrial Employment (Standing of Indradhanush Plan of government. It is housed in RBI’s
Orders) Central (Amendment) Rules, 2018, the government central office in Mumbai. It works towards governance reforms
has notified fixed-term employment for all sectors. Fixed term in PSBs as recommended by PJ Nayak Committee.
employment for all sectors will make it easier for companies 19. Consider the following statements. [IAS 2019]
to hire-and-fire workers along with reducing role of
1. Petroleum and Natural Gas Regulatory Board (PNGRB)
middlemen.
is the first regulatory body set up by the Government of
Also, no notice of termination of employment shall be India.
necessary in the case of temporary workman whether
2. One of the tasks of PNGRB is to ensure competitive
monthly rated, weekly rated or piece rated and probationers
markets for gas.
or badli workmen.
3. Appeals against the decisions of PNGRB go before the
Appellate Tribunals for Electricity.
208 Magbook ~ Indian Economy

Which of the statements given above are correct? Exp. (c) Economic cost of foodgrains to the Food Corporation
(a) 1 and 2 (b) 2 and 3 (c) 1 and 3 (d) 1, 2 and 3 of India has three main components i.e. procurement cost,
Exp. (b) The Petroleum and Natural Gas Regulatory Board procurement price and distribution cost.
(PNGRB) was constituted under the Petroleum and Natural The procurement incidentals (costs) are the initial costs
Gas Regulatory Board Act, 2006. The Act provide for the incurred during procurement of foodgrains in the
establishment of Petroleum and Natural Gas Regulatory markets/yards/centres. It includes statutory charges, labour
Board (PNGRB) to protect interests of consumers and entities charges, amount paid to the state agencies for establishment,
engaged in specified activities relating to petroleum, storage and interest for stocks etc.
petroleum products and natural gas and to promote FCI buys foodgrains from farmers at pre-announced MSP. The
distribution costs include freight, handling, storage, interest
competitive markets. Tribunal established under Electricity
and transit charges and establishment cost.
Act, 2003 is the Appellate Tribunal for purposes of Petroleum
and Natural Gas Regulatory Board Act, 2006. 23. Consider the following. [IAS 2018]
20. Which one of the following is not a sub-index of the 1. Areca nut 2. Barley
World Bank’s ‘Ease of Doing Business Index’? [IAS 2019] 3. Coffee 4. Finger millet
5. Groundnut 6. Sesamum
(a) Maintenance of law and order
7. Turmeric
(b) Paying taxes
The Cabinet Committee on Economic Affairs has announced
(c) Registering property the Minimum Support Price for which of the above?
(d) Dealing with construction permits (a) 1, 2, 3 and 7 (b) 2, 4, 5 and 6
Exp. (a) Ease of Doing Business is an Index created by (c) 1, 3, 4, 5 and 6 (d) 1, 2, 3, 4, 5, 6 and 7
Bulgarian economist Simeon Djankov. It is released by World
Exp. (b) Minimum Support Price (MSP) is a form of market
Bank ranks 190 countries based on 10 sub-indices across
intervention by the Government of India to insure agricultural
lifecycle of a business, from starting a business to resolving
producers against any sharp fall in farm prices. The minimum
insolvency. These ten sub-indices are:
support prices are announced by the Cabinet Committee of
(i) Starting a business (ii) Dealing with construction Economic Affairs at the beginning of the sowing season for
permits certain crops on the basis of the recommendations of the
(iii) Getting electricity (iv) Registering property Commission for Agricultural Costs and Prices (CACPs). 
In total MSP is announced for 26 commodities i.e. 7 cereal, 8
(v) Getting credit (vi) Protecting investors
oil seeds, 5 pulses and 6 other crops. Areca nut and coffee are
(vii) Paying taxes (viii) Trading across borders out of MSP ambit.
(ix) Enforcing contracts (x) Resolving insolvency
24. With reference to the governance of the public sector
Each of these sub-indices carry equal weightage. In case of banking in India, conside the following statements.
India, Delhi and Mumbai are only two cities surveyed by World 1. Capital infusion into public sector banks by the
Bank for this rankings. Government of India has steadily increased in the last
21. In India, ‘Extended Producer Responsibility’ was decade.
introduced as an important feature in which of the 2. To put the public sector banks in order, the merger of
following? [IAS 2019] associate banks with the parent State Bank of India has
(a) The Bio-medical Waste (Management and Handling) been affected. [IAS 2018]
Rules, 1998 Which of the statements given above is/are correct?
(b) The Recycled Plastic (Manufacturing and Usage) Rules, (a) Only 1 (b) Only 2
1999 (c) Both 1 and 2 (d) Neither 1 nor 2
(c) The e-Waste (Management and Handling) Rules, 2011 Exp. (b) Most of the public sectors banks in India are suffering
(d) The Food Safety and Standard Regulations, 2011 from the issues of bad loans (NPAs) and poor management
Exp. (c) E-waste (Management and Handling) Rules, 2011 practices, so to set in order some of these banks (SBI
place main responsibility of e-waste management on associates banks) government decided to merge them with
producers of the electrical and electronic equipment by SBI. This step will allow the government to infuse capital in
introducing concept of ‘Extended Producer Responsibility’ PSBs in an easy manner and under a bigger professionally
(EPR). EPR is a responsibility of any producer of electrical or managed bank these small banks can improve their efficiency
electronic equipment for their products beyond manufacturing as well.
until environmentally sound management of their end of life
25. Consider the following items. [IAS 2018]
products.
1. Cereal grains hulled
22. The economic cost of foodgrains to the Food 2. Chicken eggs cooked
Corporation of India is Minimum Support Price and 3. Fish processed and canned
bonus (if any) paid to the farmers plus [IAS 2019] 4. Newspapers containing advertising material
(a) transportation cost only. Which of the above items is/are exempted under GST
(b) interest cost only. (Goods and Services Tax)?
(c) procurement incidentals and distribution cost. (a) Only 1 (b) 2 and 3
(d) procurement incidentals and charges for godowns. (c) 1 and 4 (d) 1, 2, 3 and 4
Magbook ~ Solved Paper 1 209
Exp. (c) Certain goods are kept under nil or 0% GST rate to 28. In 1920, which of the following changed its name to
benefit the masses. Previously, GST council imposed 5% rate Swarajya Sabha? [IAS 2018]
of GST on hulled grains. However, later on 11th June 2017, the (a) All India Home Rule League
rate of GST applicable on hulled grains is 0%. (Nil rate) falls
(b) Hindu Mahasabha
under GST HSN (Harmonised System of Nomenclature) code
(c) South Indian Liberal Federation
number 1104.
(d) The Servants of India Society
Chicken eggs cooked and fish processed and canned are
under 5% GST rate. Newspapers containing advertising Exp. (a) The  Indian Home Rule movement  was a movement
material are under 0% GST rate. in  British India  on the lines of  Irish Home Rule
movement  and other  home rule  movements. The movement
26. Consider the following countries. [IAS 2018] lasted around two years between 1916–1918 and is believed
1. Australia 2. Canada to have set the stage for the  independence movement  under
3. China 4. India the leadership of  Annie Besant  all over India whereas  B. G.
5. Japan 6. USA Tilak  participation was limited to Western India only. In 1920,
Which of the above are among the ‘free-trade partners’ of All India Home Rule League changed its name to Swarajya
ASEAN ? Sabha.
(a) 1, 2, 4 and 6 (b) 3, 4, 5 and 6
29. The Partnership for Action on Green Economy (PAGE), a
(c) 1, 3, 4 and 5 (d) 2, 3, 4 and 6
UN mechanism to assist countries transition towards
Exp. (c) Apart from the ASEAN Free Trade Area (AFTA) greener and more inclusive economies, emerged at
between ASEAN member states, the regional trade bloc has [IAS 2018]
signed several FTAs with some of the major economies in the (a) The Earth Summit on Sustainable Development 2002,
Asia-Pacific region. These include the ASEAN-Australia-New Johannesburg.
Zealand FTA, the ASEAN-China FTA, the ASEAN-India FTA, (b) The United Nations Conference on Sustainable
the ASEAN-Korea FTA, and the ASEAN-Japan Comprehensive Development 2012, Rio de Janeiro.
Economic Partnership. (c) The United Nations Framework Convention on Climate
Alongwith these FTAs, ASEAN and other Asia pacific countries Change 2015, Paris.
are moving towards Regional Comprehensive Economic (d) The World Sustainable Development Summit 2016, New
Partnership  (RCEP), which is a proposed  Free Trade Delhi.
Agreement  (FTA) between the ten member states of Exp. (b) The Partnership for Action on Green Economy
the  Association of South East Asian Nations  (ASEAN) and the (PAGE) was launched in 2013 as a response to the call at
six states with which ASEAN has existing free trade Rio+20 in 2012 in Rio de Janeiro to support those countries
agreements (Australia,  China,  India,  Japan,  South wishing to embark on greener and more inclusive growth
Korea  and  New Zealand). trajectories.
27. With the reference to the Pradhan Mantri Kaushal Vikas The Partnership for Action on Green Economy (PAGE) seeks
Yojana, consider the following statements. to put sustainability at the heart of economic policy making.
[IAS 2018] The partnership supports nations and regions in reframing
1. It is flagship scheme of the Ministry of Labour and economic policies and practices around sustainability to foster
Employment. economic growth, create income and jobs, reduce poverty
2. It, among other things, will also impart training in soft and inequality, and strengthen the ecological foundations of
skills, entrepreneurship, financial and Digital Literacy. their economies. PAGE is a direct response to the Rio+20
3. It aims to align the competencies of the unregulated Declaration, The Future We Want, which called upon the
workforce of the country to the National Skill Qualification United Nations System and the international community to
Framework. provide assistance to interested countries in developing,
Which of the statements given above is/are correct? adopting and implementing green economy policies and
(a) 1 and 3 (b) Only 2 strategies.
(c) 2 and 3 (d) 1, 2 and 3
30. Which one of the following statements correctly
Exp. (c) Pradhan Mantri Kaushal Vikas Yojana (PMKVY) is the describes the meaning of legal tender money? [IAS 2018]
flagship scheme of the Ministry of Skill Development and (a) The money which is tendered in courts of law to defray the
Entrepreneurship (MSDE). The objective of this Skill fee of legal cases.
Certification Scheme is to enable a large number of Indian (b) The money which a creditor is under compulsion to
youth to take up industry-relevant skill training that will help accept in settlement of his claims.
them in securing a better livelihood. (c) The bank money in the form of cheques, drafts, bill of
The Short Term Training imparted at PMKVY Training Centres exchange, etc.
(TCs) is expected to benefit candidates of Indian nationality (d) The metallic money in circulation in a country.
who are either school/college dropouts or unemployed. Apart
Exp. (b) Legal tender is any official medium of payment
from providing training according to the National Skills
recognised by law that can be used to extinguish a public or
Qualification Framework (NSQF), TCs shall also impart training
private debt, or meet a financial obligation. The national
in Soft Skills, Entrepreneurship, Financial and Digital
currency is legal tender in practically every country. A creditor
Education.
is obligated to accept legal tender towards repayment of a
debt.
210 Magbook ~ Indian Economy

31. If a commodity is provided free to the public by Exp. (d) Capital Output Ratio (COR) is the amount of
government, then [IAS 2018] capital needed to produce one unit of output. A higher COR
(a) the opportunity cost is zero. value is not preferred because it indicates that the entity's
(b) the opportunity cost is ignored. production is inefficient, e.g. suppose that investment in an
(c) the opportunity cost is transferred from the consumers of the economy is 32% (of GDP) and the economic growth
product to the tax paying public. corresponding to this level of investment is 8%. Capital
(d) the opportunity cost is transferred from the consumers of the Output Ratio is 32/8 or 4. In other words, to produce one
unit of output, 4 unit of capital is needed. Thus, in case of
product to the government.
high Capital Output Ratio, despite high savings, capital
Exp. (c) In  microeconomic theory, the  opportunity cost, also formation may not result in significant increase in output.
known as  alternative cost, is the  value  (not a benefit) of the
choice of a best alternative cost while making a decision. A 35. Consider the following statements. [IAS 2018]
choice needs to be made between several  mutually 1. The Reserve Bank of India manages and services
exclusive  alternatives; assuming the best choice is made, it is Government of India securities but not any State
the "cost" incurred by not enjoying the  benefit  that would have Government securities.
been had by taking the second best available choice. 2. Treasury bills are issued by the Government of India
Opportunity cost = return of most lucrative option not chosen – and there are no treasury bills issued by the State
return of chosen option According to the above explanation when Governments.
a commodity is provided free to the public by the government 3. Treasury bills offers are issued at a discount from the
then the opportunity cost is transferred to the people who have par value.
paid for this commodity i.e tax paying individuals. Which of the statements given above is/are correct?
(a) 1 and 2 (b) Only 3 (c) 2 and 3 (d) 1, 2 and 3
32. Increase in absolute and per capita real GNP do not
connote a higher level of economic development, if Exp. (c) RBI manages securities of both Central and State
Governments. Thus, this statement is incorrect.
(a) industrial output fails to keep pace with agricultural output
Treasury bills are only issued by Central Government. Thus,
(b) agricultural output fails to keep pace with industrial output
this statement is incorrect.
(c) poverty and unemployment increases
Treasary bills are money market and short-term debt
(d) imports grow faster than exports [IAS 2018]
instruments issued by the Government. of India and
Exp. (c) GNP is quantitative but development means a presently issued in three tenors, namely 91, 182 and 364
qualitative change which is always value positive. This means days. Treasury bills are zero coupon securities and pay no
that development cannot take place unless there is an increment interest. Rather, they are issued at a discount (at a reduced
or addition to the existing conditions. Development occurs when amount) and redeemed (given back money) at the face
positive growth takes place. Yet, positive growth does not always value at maturity.
lead to development. This happens when poverty and
unemployment increases. 36. Consider the following statements. [IAS 2018]
1. Capital Adequacy Ratio (CAR) is the amount that the
33. Consider the following statements: Human capital banks have to maintain in the form of their own funds
formation as a concept is better explained in terms of a to offset any loss that bank incur if the account holder
process which enables [IAS 2018] fail to repay any dues.
1. Individuals of a country to accumulate more capital. 2. CAR is decided by each individual bank.
2. Increasing the knowledge, skills level and capacities of the Which of the statements given above is/are correct?
people of the country. (a) Only 1 (b) Only 2
3. Accumulation of tangible wealth. (c) Both 1 and 2 (d) Neither 1 nor 2
4. Accumulation of intangible wealth. Exp. (a) Capital Adequacy Ratio (CAR) is the ratio of a
Which of the statements given above is/are correct? bank’s capital in relation to its risk weighted assets and
(a) 1 and 2 (b) Only 2 (c) 2 and 4 (d) 1, 3 and 4 current liabilities. It is decided by Central Banks and bank
Exp. (c) Human capital is a collection of intangible collective regulators to prevent Commercial Banks from taking excess
resources possessed by individuals and groups within a given leverage and becoming insolvent in the process.
population. These resources include all the knowledge, talents, Capital Adequacy Ratio
Tier I + Tier II + Tier III (Capital funds))
skills, abilities, experience, intelligence, training, judgement and =
wisdom possessed individually and collectively, the cumulative Risk weighted assets
total of which represents a form of wealth available to nations and
organisations to accomplish their goals. Thus, statements 1 and 37. The identity platform Aadhar provides open
3 are incorrect. Statements 2 and 4 are correct. Application Programming Interfaces (APIs). What
does it imply?
34. Despite being a high saving economy, capital formation 1. It is integrated into an electronic device. [IAS 2018]
may not result in significant increase in output due to 2. Online authentication using iris is possible.
(a) weak administrative machinery [IAS 2018] Which of the statements given above is/are correct?
(b) Illiteracy (a) Only 1 (b) Only 2
(c) high population density (c) Both 1 and 2 (d) Neither 1 nor 2
(d) high capital-output ratio
Magbook ~ Solved Paper 1 211
Exp. (c) Aadhaar authentication provides several ways in 40. India enacted the Geographical Indications of Goods
which an Aadhaar holder can authenticate himself using the (Registration and Protection) Act, 1999 in order to
system. At a high level, authentication can be done using comply with the obligations to [IAS 2018]
demographics data and/or biometric data and/or OTP. Face (a) ILO (b) IMF
authentication is currently not supported. It is proposed to start (c) UNCTAD (d) WTO
from July, 2018. The API can be integrated into any electronic
device. Thus, both statements 1 and 2 are correct. Exp. (d) Article 22 of the TRIPS Agreements (WTO
Agreement) defines Geographical Indication as “indications
38. Consider the following statements [IAS 2018] which identify a good as originating in the territory of a
1. The Food Safety and Standards Act, 2006 replaced the member or a region or locality in that territory, where a given
Prevention of Food Adulteration Act, 1954. quality, reputation or other characteristic of the good is
2. The Food Safety and Standard Authority of India (FSSAI) essentially attributable to its geographical origin”.
is under the charge of Director General of Health Services Consequently sui-geneis legislation for the protection of
in the Union Ministry of Health and Family Welfare. geographical indication was enacted in 1999. When India
Which of the following statements is/are correct? joinned as a member state of the TRIPS as a member state
(a) Only 1 (b) Only 2 agreement.
(c) Both 1 and 2 (d) Neither 1 nor 2 41. As per the NSSO 70th round situation assessment
Exp. (a) Statement 1 is correct. Food Safety and Standards survey of technical households, consider the following
Act, 2006 replaced Prevention of Food Adulteration Act, 1954. statements [IAS 2018]
FSSAI has beencreated for laying down science based 1. Rajasthan has the highest percentage share of
standards for articles of food and to regulate their manufacture agricultural households among its rural households.
storage, distribution, sale and import to ensure availability of 2. Out of the total agricultural households in the country, a
safe and wholesome food for human consumption. little over 60 percentage belongs to OBCs.
Statement 2 is incorrect: FSSAI is not under Director General 3. In Kerala, a little over 60 percent agricultural households
of Health Services. It is an independent authority under reported to have maximum income from sources other
Ministry of Health and Family Welfare. than agricultural activities.
Which of the statements given above is/ are correct?
39. With reference to the provisions made under the
National Food Security Act, 2013, consider the following (a) 2 and 3 (b) Only 2
statements. [IAS 2018] (c) 1 and 3 (d) 1, 2 and 3
1. The families coming under the category of Below Poverty Exp. (c) The results of the survey are based on the Central
Line (BPL) only are eligible to receive subsidised food sample Caudated by NSSO, consisting of 4,529 villages
grains. spread over rural areas of all states and union territories. A
2. The eldest woman in a household, of age 18 years or total number of 35,200 households were surveyed in first visit
above, shall be the head of the household for the purpose and 34,907 of them could be re-surveyed in second visit.
of issuance of a ration card. Some salient findings of the survey regarding situation of
3. Pregnant women and lactating mothers are entitled to a agricultural households in the country are as follows
take-home ration' of 1600 calories per day during l
During the agricultural year July 2012- June 2013, rural
pregnancy and for six months thereafter. India had an estimated total of 90.2 million agricultural
Which of the following statements given above is/are households, which constituted about 57.8 percent of the
correct? total estimated rural households of the country during the
(a) 1 and 2 (b) Only 2 (c) 1 and 3 (d) Only 3 same period.
Exp. (b) Some of salient features of NFSA (National Food
l
Uttar Pradesh, with an estimate of 18.05 million
Security Act) 2013 are agricultural households, accounted for about 20 percent
of all agricultural households in the country.
l
Coverage and Entitlement under Targeted Public
Distribution System (TPDS) Upto 75% of the rural
l
Rajasthan had highest percentage of agricultural
population and 50% of the urban population will be households (78.4 percent) among its rural households
covered under TPDS, with uniform entitlement of 5 kg per and Kerala had the least percentage share of agricultural
person per month. Not restricted to only BPL families. households (27.3 percent) in its rural households.
l
Subsidised Prices under TPDS and their Revision
l
Out of the total estimated agricultural households in the
Foodgrains under TPDS will be made available at country, about 45 percent belonged to Other Backward
subsidised prices of ` 3/2/1 per kg for rice, wheat and Classes (OBCs).
coarse grains for a period of three years from the date of 42. Which of the following best describes the term
commencement of the act. ‘Merchant Discount Rate’? [IAS 2018]
l
Maternity Benefit Pregnant women and lactating mothers (a) The incentive by a bank given to a merchant for accepting
will also be entitled to receive maternity benefit of not less payments through debit cards for financial transactions
than ` 6,000. for purchasing goods and services
l
Women Empowerment Eldest woman of the household of (b) The amount paid by bank to their customers when they
age 18 years or above to be the head of the household pay by debit cards for financial transactions for
for the purpose of issuance of ration cards. purchasing goods and services
212 Magbook ~ Indian Economy

(c) The charge to a merchant by a bank for accepting payments 45. Consider the following statements [IAS 2018]
from his customers through the bank’s debit card.
1. The quality of imported edible oil is more than the
(d) The incentive given by the Government to merchants for domestic production of edible oils in the last five years.
promoting digital payments by their customers through
2. The government does not impose any custom duty on all
POS (Point of Sale) machines and debit cards. the imported edible oils as a special case.
Exp. (c) MDR is a fee charged from a merchant by a bank for Which of the statements given above is/are correct?
accepting payments from customers through credit and debit (a) Only 1 (b) Only 2
cards in their establishments. MDR compensates the card (c) Both 1 and 2 (d) Neither 1 nor 2
issuing bank, the lender which puts the POS terminal and
payment gateways such as Mastercard or Visa for their
Exp. (a) Statement 1 is correct. The quantity of imported
edible oil has been in the range of 11 - 15 metric tonnes,
services. MDR charges are usually shared in pre-agreed
while domestic production have been in the range of 5-8
proportion between the bank and a merchant and is expressed
metric tonnes.
in percentage of transaction amount.
Statement 2 is incorrect. As government imposes customs
43. With reference to India’s decision to levy an equalisation duty on imported oil. At present, the import duty on crude
tax of 6% on online advertisement services offered by soyabean oil is 3.0 percent, crude sunflower oil 25 percent
non-resident entities, which of the following statement and crude rapeseed oil 25 percent.
is/are correct? [IAS 2018]
46. The term ‘Domestic Content Requirement’ is
1. It is introduced as a part of Income Tax Act.
sometimes seen in the news with reference to
2. Non resident entities that offer advertisement services in [IAS 2017]
India can claim a tax credit in their home country under the
(a) Developing solar power production in our country
Double Taxation Avoidance commission.
(b) Granting licences to foreign TV channels in our country
Select the correct answer using the code given below.
(c) Exporting our food products to other countries.
(a) Only 1 (b) Only 2
(d) Permitting foreign educational institutions to set-up their
(c) Both 1 and 2 (d) Neither 1 nor 2 campuses in our country
Exp. (d) Equalisation Levy is a tax outside on business Exp. (a) The World Trade Organisation’s appellate body has
transaction for online marketing in which any Indian pays a sum declared Domestic Content Requirements (DCRs) in India’s
of more than ` 1 lakh to non-residents entities such Google and Jawaharlal Nehru National Solar Mission (JNNSM) as illegal.
Facebook, etc. Since, equalisation levy is outside the scope of
The WTO disputes panel also ruled that India’s subsidies for
tax treaties entered into by India with other countries, the foreign
solar power contravene WTO trade rules and India must
company cannot claim a tax credit in its home country.
remove the subsidies or face trade sanction. In 2013, the US
44. Consider the following Statements. [IAS 2018] brought a complaint before the WTO arguing that the
1. The Fiscal Responsibility and Budget Management Review Domestic Content Requirement imposed under India’s
(FRBM) Committee report has recommended a debt to GDP national solar programme is in violation of the global trading
ratio of 60% for the general (combined) Government by rules.
2023, comprising 40% for the Central Government and 20% Specifically, it said, India has violated its ‘national treatment’
for the State Government. obligation by unfavourably discriminating against imported
2. The Central Government has domestic liabilities of 21% of solar cells and modules.
GDP as compared to 49% of GDP of the State Government.
47. Who among the following can join the National Pension
3. As per the Constitution of India, it is mandatory for the state
System (NPS)? [IAS 2017]
to take Central Government’s consent for raising any loans
if the former owes any liabilities to the latter. (a) Resident Indian citizens only
Which of the given statement is/ are correct? (b) Persons of age from 21 to 55 only
(a) Only 1 (b) 2 and 3 (c) 1 and 3 (d) 1, 2 and 3 (c) All state government employees joining the services
after the date of notification by the respective state
Exp. (c) Statement 1 is correct. According to the FRBM governments
committee review committee report headed by NV. Singh. The (d) All central government employees including those of
combined debt to GDP ratio of the central and states should be armed forces joining the services on or after 1st April,
brought down to 60 percent by 2023 (comprising of 40 per cent 2004.
for the centre and 20 percent for states) as against the existing
Exp. (c) Any citizen of India, whether resident or
49.4 percent, and 21 per cent respectively.
non-resident, Individuals who are aged between 18 – 60
Statement 2 is incorrect. The Union Government, which has
years age can join the National Pension Scheme. The
larger domestic liabilities of 49.23% of GDP as compared to that
National Pension System (NPS) is a voluntary defined
of the states (21% of GDP) Statement 3 is correct. Under Article
contribution pension system administered and regulated by
293, a state may not without the consent of the Government of
the Pension Fund Regulatory and Development Authority
India raise any loan if there is still outstanding any part of a loan
(PFRDA), created by an Act of the Parliament of India. The
which has been made to the state by the Government of India
NPS started with the decision of the Government of India to
or by its predecessor government, or in respect of which a
stop defined benefit pensions for all its employees who
guarantee has been given by the Government of India or by its
joined after January 1, 2004. NPS is an attempt by the
predecessor government.
government to create a pensioned society in India.
Magbook ~ Solved Paper 1 213

48. Consider the following statements: [IAS 2017] research and development organisations, educational
1. The standard mark of Bureau of Indian Standards (BIS) is institutions, corporate entities including MSMEs, startups and
mandatory for automotive tyres and tubes. other stakeholders in the creation of an innovation-conducive
2. AGMARK is a quality certification mark issued by the Food environment,  which stimulates creativity and innovation
and Agriculture Organisation (FAO). across sectors, as also facilitates a stable, transparent and
Which of the statements given above is/are correct? service-oriented IPR administration in the country.
(a) Only 1 (b) Only 2 51. With reference of ‘Quality Council of India (QCI)’,
(c) Both 1 and 2 (d) Neither 1 nor 2 consider the following statements : [IAS 2017]
Exp. (a) AGMARK is a  certification mark employed on 1. QCI was set-up jointly by the government of India and
agricultural products in  India, assuring that they conform to a the Indian Industry.
set of standards approved by the  Directorate of Marketing and 2. Chairman of QCI is appointed by the Prime Minister on
Inspection, an agency of the government of India. the recommendations of the industry to the government .
The  AGMARK is legally enforced in India by the  agricultural Which of the above statements is/are correct?
produce (grading and marking) Act of 1937 (and amended in (a) Only 1 (b) Only 2
1986). The present AGMARK standards cover quality guidelines (c) Both 1 and 2 (d) Neither 1 nor 2
for 213 different commodities spanning a variety of pulses, Exp. (c) The Quality Council of India (QCI) is a pioneering
cereals, essential oils, vegetable oils, fruits and vegetables and experiment of the government of India in setting up
semi-processed products like vermicelli. organisations in partnership with the Indian industry. The
49. What is/are the advantage/advantages of implementing work was coordinated by the then Department of Industries
the ‘National Agriculture Market’ scheme? [IAS 2017] (Department of Industrial Policy and Promotion) and the
recommendations were submitted to the Cabinet in 1996.
1. It is a pan-India electronic trading portal for agricultural
commodities. Key recommendations included the need for establishing an
organisations jointly by the government and the industry and
2. It provides the farmers access to nationwide market, with
prices commensurate with the quality of their produce. the need for the organisations to be self-sustaining and be
away from the government. Both statements are explanatory
Select the correct answer using the code given below :
and correct.
(a) Only 1 (b) Only 2
(c) Both 1 and 2 (d) Neither 1 nor 2 52. What is the purpose of setting up of Small Finance
Exp. (c) National Agriculture Market (NAM) is a pan-India Banks (SFBs) in India? [IAS 2017]
electronic trading portal which networks the existing APMC 1. To supply credit to small business units
mandis to create a unified national market for agricultural 2. To supply credit to small and marginal farmers
commodities. Its a national e-market platform for transparent sale 3. To encourage young entrepreneurs to set-up business
transactions and price discovery initially in regulated markets. particularly in rural areas.
The NAM portal provides a single window service for all APMC Select the correct answer using the code given below :
related information and services. This includes commodity arrivals (a) 1 and 2 (b) 2 and 3
& prices, buy & sell trade offers, provision to respond to trade (c) 1 and 3 (d) 1, 2 and 3
offers, among other services. While material flow (agriculture Exp. (a) The objective of Small Finance Banks is to further
produce) continues to happen through mandis, an online market financial inclusion by providing: Basic banking facilities to the
reduces transaction costs and information asymmetry. unbanked and thereby boosting saving habits and supply of
50. With reference to the ‘National Intellectual Property Rights credit to small business units, small and marginal farmers,
Policy’, consider the following statements : [IAS 2017] micro and small industries and other unorganised sector
entities, through high technology-low cost operations.
1. It reiterates India’s commitment to the Doha Development
Agenda and the TRIPS Agreement. For the first three years, 25% of branches should be in
2. Department of Industrial Policy and Promotion is the nodal unbanked rural areas.
agency for regulating intellectual property rights in India. Of the loans issued by Small Banks, 75% should be to the
Which of the above statements is/are correct? so-called priority sector which includes agriculture and small
businesses. And half the loan portfolio of the banks should
(a) Only 1 (b) Only 2
be loans and advances of up to ` 25 lakh to micro finance
(c) Both 1 and 2 (d) Neither 1 nor 2
businesses.
Exp. (c) The Policy recognises that India has a well-established
TRIPS compliant legislative, administrative and judicial 53. Which of the following is a most likely consequence of
framework to safeguard IPRs, which meet its international implementing the ‘Unified Payments Interface (UPI)’?
obligations and reiterates India’s commitment to the Doha [IAS 2017]
Development Agenda and the TRIPS agreement. The National (a) Mobile wallets will not be necessary for online payments.
IPR Policy aims to create and exploit synergies between all (b) Digital currency will totally replace the physical currency
forms of Intellectual Property (IP), concerned statutes and in about two decades.
agencies. It sets in place an institutional mechanism for (c) FDI inflows will drastically increase.
implementation, monitoring and review. It aims to incorporate (d) Direct transfer of subsides to poor people will become
and adapt global best practices to the Indian scenario. very effective.
This policy shall weave in the strengths of the Government,
214 Magbook ~ Indian Economy

Exp. (a) Unified Payments Interface (UPI) is a system that (a) 1 and 4 (b) 2, 3 and 4
powers multiple bank accounts into a single mobile application (c) 2 and 3 (d) 1, 2, 3 and 4
(of any participating bank), merging several banking features, Exp. (b)
seamless fund routing and merchant payments into one hood.
1. In 1991, Agriculture was contributing 32% to GDP but
It also caters to the “Peer to Peer” collect request which can
now it has declined to 18%.
be scheduled and paid as per requirement and convenience.
2. Share of exports has gone up from below 1% to above 1%.
The key objective of a unified system is to offer a complete
architecture to facilitate next generation online immediate 3. FDI has gone up because of opening of economy.
payments leveraging trends such as increasing smartphone 4. From $3 billion in 1991, now we have comfortable $350
adoption, Indian language interfaces, and universal access to billion forex.
Internet and data. 57. Consider the following statements : [IAS 2017]
It cannot replace physical currency in two decades and has no
1. National Payments Corporation of India (NPCI) helps in
clear relation with FDI and Direct Benefit Transfer. promoting the financial inclusion in the country.
54. Which of the following statements is/are correct regarding 2. NPCI has launched RuPay, a card payment scheme.
the Monetary Policy Commitee (MPC)? [IAS 2017] Which of the statements given above is/are correct?
1. It decides the RBI’s benchmark interest rates. (a) Only 1 (b) Only 2
2. It is a 12-member body including the Governor of RBI and (c) Both 1 and 2 (d) Neither 1 nor 2
is reconstituted every year. Exp. (c) The core objective was to consolidate and integrate
3. It functions under the chairmanship of the Union Finance the multiple systems with varying service levels into
Minister. nation-wide uniform and standard business process for all
Select the correct answer using the code given below : retail payment systems. The other objective was to facilitate an
(a) Only 1 (b) 1 and 2 (c) Only 3 (d) 2 and 3 affordable payment mechanism to benefit the common man
Exp. (a) Statement 1 The MPC would be entrusted with fixing across the country and help financial inclusion.
the benchmark policy rate (Repo Rate). RuPay, a new card payment scheme launched by the National
Statement 2 The MPC is a 6 member body who shall hold Payments Corporation of India (NPCI).
office for a period of four years.
58. What is/are the most likely advantages of implementing
Statement 3 The Governor of the RBI is the ex-officio ‘Goods and Services Tax (GST)’? [IAS 2017]
chairperson of the MPC.
1. It will replace multiple taxes collected by multiple
55. Which of the following are the objectives of ‘National authorities and will thus create a single market in India.
Nutrition Mission’? [IAS 2017] 2. It will drastically reduce the ‘Current Account Deficit’ of
1. To create awareness relating to malnutrition among India and will enable it to increase its foreign exchange
pregnant women and lactating mothers. reserves.
2. To reduce the incidence of anaemia among young 3. It will enormously increase the growth and size of
children, adolescent girls and women. economy of India and will enable it to overtake China in
the near future.
3. To promote the consumption of millets, coarse cereals
and unpolished rice. Select the correct answer using the code given below :
4. To promote the consumption of poultry eggs. (a) Only 1 (b) 2 and 3
Select the correct answer using the code given below : (c) 1 and 3 (d) 1, 2 and 3
(a) 1 and 2 (b) 1, 2 and 3 Exp. (a)
(c) 1, 2 and 4 (d) 3 and 4 1. Indeed one nation one market is the main theme on
Exp. (a) The key objectives of National Nutrition Mission are which GST is based.
as under : 2. GST may improve effeciency, may reduce domestic cost
l
To create awareness relating to malnutrition amongst of manufacturing and thereby will help export, but idea of
pregnant women, lactating mothers, promote healthy drastic reduction is not correct.
lactating practices and importance of balanced nutrition;
3. China is also competing the proceess of integrating
l
To improve maternal and child under-nutrition in 200 high indirect taxes at same time, so the competitive effect will
burdened districts and to prevent and reduce the be nullified. Hence overtaking China will require much
under-nutrition prevalent among children below 3 years; more reforms.
To reduce incidence of anaemia among young children,
adolescent girls and women. 59. Consider the following statements : [IAS 2017]
1. India has ratified the Trade Facilitation Agreement (TFA)
56. Which of the following has/have occurred in India after of WTO.
its liberalisation of economic policies in 1991?
2. TFA is a part of WTO’s Bali Ministerial Package of 2013.
[IAS 2017]
3. TFA came into force in January, 2016.
1. Share of agriculture in GDP increased enormously.
Which of the statements given above is/are correct?
2. Share of India’s exports in world trade increased.
3. FDI inflows increased. (a) 1 and 2 (b) 1 and 3 (c) 2 and 3 (d) 1, 2 and 3
4. India’s foreign exchange reserves increased enormously. Exp. (a) WTO members concluded negotiations at the 2013
Select the correct answer using the codes given below : Bali Ministerial Conference on the landmark Trade Facilitation
Magbook ~ Solved Paper 1 215
Agreement (TFA), which entered into force on February 22, 62. Consider the following statements : [IAS 2017]
2017 following its ratification by two-thirds of the WTO 1. Tax revenue as a percent of GDP of India has steadily
membership. increased in the last decade.
India has ratified the Trade Facilitation Agreement (TFA) of 2. Fiscal deficit as a percent or GDP of India has steadily
the World Trade Agreement (WTO) and the instrument of increased in the last decade.
Acceptance for Trade Facilitation Agreement was handed Which of the statements given above is/are correct?
over to WTO Director-General by India on April 22, 2016. (a) Only 1 (b) Only 2
60. What is the purpose ‘Vidyanjali Yojana’? [IAS 2017] (c) Both 1 and 2 (d) Neither 1 nor 2
1. To enable the famous foreign educational institutions Exp. (d) Tax revenue to GDP ratio (2007-11.89%,2008-10.75%,
to open their compuses in India. 2009-9.64%, 2010-10.19%, 2011-8.98%, 2012-10.79%), In the
2. To increase the quality of education provided in recent budget (2016-2017) total tax revenue was calculated to
government schools by taking help from the private be 11.3% of GDP ,this data shows that tax revenue in India has
sector and the community. not increased in the last decade So, the 1st statement is wrong.
3. To encourage voluntary monetary contributions from Fiscal deficit in India was around 6% in 2008-09 in the wake of
private individuals and organisationss so as to improve global financial crisis after that it came down to 4.5% in 2013-14
the infrastructure facilities for primary and secondary and in the recent budget it was calculated to be 3.2%. So the
schools. 2nd statement is also wrong.
Select the correct answer using the code given below : 63. With reference to ‘National Investment and Infrastructure
(a) Only 2 (b) Only 3 Fund’, which of the following statements is/are correct?
(c) 1 and 2 (d) 2 and 3 [IAS 2017]
Exp. (a) Statement 1 and 3 are not described as being a 1. It is an organ of NITI Aayog.
part of Vidyanjali initiative. 2. It has a corpus of ` 4,00,000 crore at present.
The programme is designed to involve volunteers from Select the correct answer using the code given below :
different walks of life to strengthen the co-scholastic (a) Only 1 (b) Only 2
activities in government schools. Vidyanjali, which is being (c) Both 1 and 2 (d) Neither 1 nor 2
implemented under the overall aegis of the Sarva Shiksha
Exp. (d)
Abhiyan, will enhance the community involvement in
1. National Investment and Infrastructure Fund (NIIF) is a fund
government run elementary schools and effectively engage
created by the government of India for enhancing
children in reading, creative writing, public speaking, play
infrastructure financing in the country. This is different from
acting, preparing story books etc.
the National Investment Fund. It is not an organ of NITI
61. What is the aim of the programme ‘Unnat Bharat Aayog.
Abhiyan’? [IAS 2017] 2. The proposed corpus of NIIF is ` 40,000 crore (around USD
(a) Achieving 100% literacy by promoting collaboration 6 Billion).
between voluntary organisationss and government’s 64. The Global Infrastructure Facility is a/an [IAS 2017]
education system and local communities.
(a) ASEAN initiative to upgrade infrastructure in Asia and
(b) Connecting institutions of higher education with local
financed by credit from the Asian Development Bank.
communities to address development challenges
(b) World Bank collaboration that facilitates the preparation and
through appropriate technologies.
structuring of complex infrastructure Public-Private
(c) Strengthening India’s scientific research institutions in
Partnerships (PPPs) to enable unbilisation of private sector
order to make India a scientific and technological
and institutional investor capital.
power.
(c) Collaboration among the major banks of the world working
(d) Developing human capital by allocating special funds
with OECD and focused an expanding the set of
for health care and education of rural and urban poor,
infrastructure projects that have the potential to mobilise
and organizing skill development programmes and
private investment.
vocational training for them.
(d) UNCTAD funded initiative that seeks to finance and facilitate
Exp. (b) Connecting institutions of higher education with infrastructure development in the world.
local communities to address development challenges
Exp. (b) The World Bank (WB) has launched the GIF to
through appropriate technologies.
specifically cater to the infrastructure needs of the emerging
The Ministry of Human Resource Development (MHRD) has economies and developing countries. The GIF will channel
launched a programme called Unnat Bharat Abhiyan with money towards bankable infrastructure project in such countries.
an aim to connect institutions of higher education, including GIF also places importance on sustainable development. Its key
Indian Institutes of Technology (IITs), National Institutes of focus will be on climate friendly infrastructure investments and
Technology (NITs) and Indian Institutes of Science projects that will boost trade. The GIF will collaborate with other
Education & Research (IISERs) etc. with local communities international and multilateral agencies which provide loans and
to address the development challenges through appropriate financial assistance to countries across the globe. It will also help
technologies. these agencies with its expertise in financing, supervising and
implementing projects.
216 Magbook ~ Indian Economy

65. Which of the following statements best describes the BEPS refers to tax planning strategies that exploit gaps and
term ‘Scheme for Sustainable Structuring of Stressed mismatches in tax rules to artificially shift profits to low or no-tax
Assets (S4A)’, recently seen in the news? [IAS 2017] locations where there is little or no economic activity. Although
(a) It is a Procedure for considering ecological costs of some of the schemes used are illegal, most are not.
developmental schemes formulated by the government. This undermines the fairness and integrity of tax systems
(b) It is a scheme of RBI for reworking the financial structure of because businesses that operate across borders can use
big corporate entities facing genuine difficulties. BEPS to gain a competitive advantage over enterprises that
(c) It is a disinvestment plan of the government regarding operate at a domestic level. Moreover, when taxpayers see
Central Public Sector undertakings. multinational corporations legally avoiding income tax, it
(d) It is a disinvestment plan of the government regarding undermines voluntary compliance by all taxpayers.
Private Sector undertakings.
68. With reference to ‘Bitcoins’, sometimes seen in the news,
Exp. (b) Reserve Bank of India (RBI), after due consultation
which of the following statements is/are correct?
with banks introduced a scheme to weigh down bad loans and
[IAS 2016]
their stressed corporate clients and at the same time, ease the
1. Bitcoins are tracked by the Central Banks of the
pressure on company balance sheets. With this motive RBI has
countries.
introduced, on June 13, 2016 the Scheme for Sustainable
2. Anyone with a Bitcoins address can send and receive
Structuring of Stressed Assets (S4A Scheme) to offer adequate
Bitcoins from anyone else with a Bitcoin address.
deep financial restructuring opportunities to large borrowers .
3. Online payments can be sent without either side knowing
66. With reference to ‘Financial Stability and Development the identity of the other.
Council’, consider the following statements [IAS 2016] Select the correct answer using the codes given below
1. It is an organ of NITI Aayog. (a) 1 and 2 (b) 2 and 3
2. It is headed by the Union Finance Minister. (c) Only 3 (d) 1, 2 and 3
3. It monitors macroprudential supervision of the economy.
Exp. (b) Bitcoin is digital currency which was founded in
Which of the statements given above is/are correct?
2008. It is designed for secure financial transactions that
(a) 1 and 2 (b) Only 3
require no central authority, no banks and no government
(c) Only 2 and 3 (d) 1, 2 and 3 regulators. Bitcoin would let transacting parties remain
Exp. (c) The Central Government is at consensus to establish anonymous, keep transactions very secure, and eliminate
Financial Stability and Development Council (FSDC) to middlemen fees.
strengthen and institutionalise the mechanism for maintaining What drove its initial development was its purely digital
financial stability and development. existence, away from the control of government regulators.
FSDC will perform following roles The values of other currencies can rise and fall when a
l
To engage in macroprudential supervision of the economy, Central Bank decides to print more paper money. But since
including the functioning of large financial conglomerates Bitcoin is digital and there is a limited number of them, the
and address inter-regulatory coordination issues. expectation is that it won't be prone to such devaluation.
l
To focus on financial literacy and financial inclusion.
l
To periodically look into issue relating to financial
69. What is/are the purpose/purposes of Government’s
‘Sovereign Gold Bond Scheme’ and ‘Gold Monetisation
development.
Scheme? [IAS 2016]
The Council would have one Sub-Committee which would be
1. To bring the idle gold lying with Indian households into
headed by Governor, RBI. The Secretariat of the Council would
the economy.
be in the Department of Economic Affairs, Ministry of Finance.
2. To promote FDI in the gold and jewellery sector.
67. The term ‘Base Erosion and Profit Shifting’ is sometimes 3. To reduce India’s dependence on gold imports.
seen in the news in the context of [IAS 2016] Select the correct answer using the codes given below
(a) mining operation by multinational companies in (a) Only 1 (b) 2 and 3
resource-rich but backward areas. (c) 1 and 3 (d) 1, 2 and 3
(b) curbing of the tax evasion by multinational companies.
Exp. (c) Prime Minister Narendra Modi on 5th November,
(c) exploitation of genetic resources of a country by
2015 launched 3 gold related schemes. The schemes are -
multinational companies.
Gold Monetisation Scheme, Sovereign Gold Bond Scheme
(d) lack of consideration of environmental costs in the
and Indian Gold Coins.
planning and implementation of developmental projects.
The primary purpose of the schemes is to reduce
Exp. (b) Base Erosion and Profit Shifting (BEPS) is of major dependence on imported gold, recycle the unutilised gold in
significance for developing countries due to their heavy reliance the country and most significantly, streamline the gold
on corporate income tax, particularly from multinational business within the country. At present, India is the largest
enterprises. Engaging developing countries in the international consumer of gold within the world. Gold Monetisation
tax agenda is important to ensure that they receive support to Scheme, which would replace both the present Gold Deposit
address their specific needs. The inclusive framework brings and Gold Metal Loan Schemes, is intended to mobilise gold
together over 100 countries and jurisdictions to collaborate on held by households and institutions of the country.
the implementation of the OECD/ G20 BEPS Package.
Magbook ~ Solved Paper 1 217
The purpose of the Sovereign Gold Bond Scheme is to recommendations, it recommended the formation of a new
reduce the demand for physical gold and to shift part of the category of bank called Payment Bank. On 17th July, 2014, the
estimated three hundred tonnes of physical bars and coins RBI released the draft guidelines for Payment Banks, seeking
purchased per annum for investment into Demat comments for interested entities and the general public. On
(Dematerialised) gold bonds. 27th November, RBI released the final guidelines for Payment
Banksw.
70. There has been a persistent deficit budget year after
The key aspirants to payment banking business include
year. Which action/actions of the following can be
telecom firms, prepaid payment instruments / payment solution
taken by the government to reduce the deficit?
[IAS 2016]
providers, retail chains, large business correspondents and
business conglomerates. Out of them, Telecom firms have an
1. Reducing revenue expenditure
advantage over others mainly because they already have a
2. Introducing new welfare schemes
distribution network in rural areas. Most of the pre-paid
3. Rationalising subsidies
payment instruments / payment solution providers are tech
4. Reducing import duty
savvy and already working in the field of mobile payments.
Select the correct answer using the codes given below
Scope of activities of Payment Banks
(a) Only 1 (b) 2 and 3 l
Payment Banks can accept demand deposits. This implies
(c) 1, 2 and 3 (d) 1, 2, 3 and 4
that customers can open savings accounts as well as
Exp. (c) A deficit is the amount by which a sum falls short of current accounts; but NO time deposits (such as FDs).
some reference amount. In economics, a deficit is an excess l
An account balance cannot exceed ` 1 lakh for an
of expenditures over revenue in a given time period; in more individual customer.
specific cases it may refer to: l
Payment Banks can issue ATM/ debit cards but not credit
Balance of Payments (BOP) deficit, when the Balance of
cards.
Payments is negative. l
Payment Banks can not give loans.
Government budget deficit that is deficit spending.
l
Payments and remittance services through various
Primary deficit -the pure deficit derived after deducting the
interest payments and structural and cyclical deficit, parts of channels.
the public sector deficit.
l
A Payment Bank can become Banking Correspondent
Income deficit (the difference between family income and the (BC) of another bank and offer all products / services
poverty threshold). Trade deficit ( when the value of imports which a BC can offer.
exceed the value of exports).
l
Payment Banks can distribute non-risk sharing financial
Introduction of new schemes would entail more spending products like mutual fund units and insurance products,
and it goes just opposite to what we are trying to do, i.e. etc.
reduce deficit. Import duty is a tax collected on imports and
l
The revenue of these banks would come mainly from the
some exports by the customs authorities of a country. It is transaction fees.
usually based on the value of the goods that are imported. 72. With reference of ‘IFC Masala Bonds’, sometimes seen in
There are two distinct goals to import duties: to raise income the news, which of the statements given below is/are
for Local Government, and to give a market advantage to correct? [IAS 2016]
locally grown or produced goods that are not subject to 1. The International Finance Corporation, which offers these
import duties. bonds, is an arm of the World Bank.
71. The establishment of ‘Payment Banks’ is being allowed 2. They are the rupee-denominated bonds and are a source of
in India to promote financial inclusion. Which of the debt financing for the public and private sector.
following statements is/are correct in this context? Select the correct answer using the codes given below
[IAS 2016] (a) Only 1 (b) Only 2
1. Mobile telephone companies and supermarket chains (c) Both 1 and 2 (d) Neither 1 nor 2
that are owned and controlled by residents are eligible to Exp. (c) Masala bonds are rupee denominated overseas
be promoters of Payment Banks. bonds. Masala bonds will help to internationalise the Indian
2. Payment Banks can issue both credit cards and debit rupee and also deepen the Indian financial system( Public and
cards. Private Sector).
3. Payment Banks cannot undertake lending activities. By issuing bonds in rupees, an Indian company is shielded
Select the correct answer using the codes given below against the risk of currency fluctuation, typically associated with
(a) 1 and 2 (b) 1 and 3 borrowing in foreign currency. Besides helping diversify funding
(c) Only 2 (d) 1, 2 and 3 sources, the cost of borrowing could also turn out to be lower
Exp. (b) On 23rd September, 2013, Committee on than domestic markets.
Comprehensive Financial Services for Small Businesses and In 2013, the first masala bonds were issued by the International
Low Income Households, headed by Nachiket Mor, was Finance Corporation (IFC), an arm of the World Bank. IFC then
formed by the RBI. On 7th January, 2014, the Nachiket Mor named them Masala bonds to give a local flavour by calling to
Committee submitted its final report. Among its various mind Indian culture and cuisine.
218 Magbook ~ Indian Economy

The Reserve Bank of India has issued guidelines allowing exporters and the importing country. Import cover is an
Indian companies, non-banking finance companies (HDFC, important indicator of the stability of the currency. During the
Indiabulls Housing Finance are examples of such companies) currency crisis of 2013, when foreign exchange reserves fell to
and infrastructure investment trusts and real investment trusts around $275 billion, import cover dipped to around seven
(investment vehicles that pool money from various investors months. According to currency explits, eight to ten months of
and invest in infrastructure and real estate sectors) to issue import cover is essential for the stability of the currency.
rupee-denominated bond overseas.
75. The term ‘Core Banking Solution’ is sometimes seen in
Masala bond will help the Indian corporates to reduce its
the news. Which of the following statements best
interest cost burden on the debt amount on its balance sheet. describes/describe this term? [IAS 2016]
The more of foreign funds can be used for infrastructural
1. It is a networking of a bank’s branches which enables
development in the country. Overall, the development of a
customers to operate their accounts from any branch of
Masala bond market would be positive for Indian firms,
the bank on its network regardless of where they open
opening up potentially significant new sources of funding over
their accounts.
External Commercial Borrowings (ECBs).
2. It is an effort to increase RBI’s control over commercial
73. Which of the following is/are included in the capital banks through computerisation.
budget of the Government of India? [IAS 2016] 3. It is a detailed procedure by which a bank with huge
1. Expenditure on acquisition of assets like roads, buildings, non-performing assets is taken over by another bank.
machinery, etc Select the correct answer using the codes given below
2. Loans received from foreign governments (a) Only 1 (b) 2 and 3
3. Loans and advances granted to the States and Union (c) 1 and 3 (d) 1, 2 and 3
Territories Exp. (a) Core banking is a banking service provided by a
Select the correct answer using the codes given below group of networked bank branches where customers may
(a) Only 1 (b) 2 and 3 access their bank account and perform basic transactions
(c) 1 and 3 (d) 1, 2 and 3 from any of the member branch offices. Core banking is often
Exp. (d) Capital budgeting, or investment appraisal, is the associated with retail banking and many banks treat the retail
planning process used to determine whether an organisation’s customers as their core banking customers. Businesses are
long-term investments such as new machinery, replacement usually managed via the Corporate banking division of the
of machinery, new plants, new products, and research institution. Core banking covers basic depositing and lending
development projects are worth the funding of cash through of money.
the firm’s capitalisation structure (debt, equity or retained Normal Core Banking functions will include transaction
earnings). It is the process of allocating resources for major accounts, loans, mortgages and payments. Banks make these
capital or investment expenditures. One of the primary goals services available across multiple channels like ATMs, Internet
of capital budgeting investments is to increase the value of the banking, mobile banking and branches. The advancement in
firm to the shareholders. technology, especially Internet and information technology has
Capital budget includes capital receipts and payments of the led to new ways of doing business in banking. These
government. Loans from public, foreign governments and RBI technologies have reduced manual work in banks and
form a major part of the government’s capital receipts. Capital increasing efficiency.
expenditure is the expenditure on development of machinery, 76. What is/are the purpose/purposes of the ‘Marginal Cost
equipment, building, health facilities, education, etc. Fiscal of Funds based Lending Rate (MCLR)’ announced by
deficit is incurred when the government’s total expenditure RBI?
exceeds its total revenue. 1. These guidelines help to improve the transparency in the
74. Which of the following best describes the term ‘import methodology followed by banks for determining the
cover’, sometimes seen in the news? [IAS 2016] interest rates on advances.
2. These guidelines help to ensure availability of bank credit
(a) It is the ratio of value of imports to the Gross Domestic
at interest rates which are fair to the borrowers as well as
Product of a country.
the banks. [IAS 2016]
(b) It is the total value of imports of a country in a year.
Select the correct answer using the codes given below
(c) It is the ratio between the value of exports and that of
(a) Only 1 (b) Only 2
imports between two countries.
(d) It is the number of months of imports that could be paid (c) Both 1 and 2 (d) Neither 1 nor 2
for by a country’s international reserves. Exp. (c) The Reserve Bank of India has brought a new
Exp. (d) The stock of foreign exchange reserves in terms of methodology of setting lending rate by commercial banks
months of retrained imports of goods as at end of year. It under the name Marginal Cost of Funds based Lending Rate
measures the number of months of money available in the (MCLR). It has modified the existing base rate system from
national bank to cover the cost of imports. It is an operational April 2016 onwards.
definition of import cover. An import coverage ratio is the As per the new guidelines by the RBI, banks have to prepare
share (or percentage) of a country’s own imports that is Marginal Cost of Funds based Lending Rate (MCLR) which will
subject to a particular non-tariff barrier, or any one of a be the internal benchmark lending rates.
specified group of non-tariff barriers. They are calculated by Based upon this MCLR, interest rate for different types of
attaching actual values to bilateral trade flows between various customers should be fixed in accordance with their riskiness.
Magbook ~ Solved Paper 1 219
The base rate will be now determined on the basis of the MCLR and Remunerative Price (FRP) of sugarcane for 2009-10 and
calculation. The MCLR should be revised monthly by subsequent sugar seasons. The cane price announced by
considering some new factors including the repo rate and other the Central Government is decided on the basis of the
borrowing rates. Specifically the repo rate and other borrowing recommendations of the Commission for Agricultural Costs
rates that were not explicitly considered under the base rate and Prices (CACP) after consulting the State Governments
system. and associations of sugar industry, and approved by the
Cabinet Committee on Economic Affairs.
77. With reference to Indian economy, consider the following
statements [IAS 2015] 80. A decrease in tax to GDP ratio of a country indicates
1. The rate of growth of Real Gross Domestic product has which of the following? [IAS 2015]
steadily increased in the last decade. 1. Slowing economic growth rate.
2. The Gross Domestic Product at market prices (in rupees) 2. Less equitable distribution of national income.
has steadily increased in the last decade. Select the correct answer using the codes given below
Which of the statements given above is/are correct? (a) Only 1 (b) Only 2
(a) Only 1 (b) Only 2 (c) Both 1 and 2 (d) Neither 1 nor 2
(c) Both 1 and 2 (d) Neither 1 nor 2 Exp (b) A decrease in tax to GDP ratio of a country means
Exp (b) Real Gross Domestic Product (Real GDP) is a less equitable distribution of national income. The ratio of tax
macro-economic measure of the value of economic output collection against the national Gross Domestic Product
adjusted for price changes (i.e., inflation or deflation). This (GDP) is tax to GDP ratio. This ratio is the total government
adjustment transforms the money-value measure, nominal tax collections divided by the country’s GDP. An economic
GDP, into an index for quantity of total output. The Economy of slowdown occurs when the rate of economic growth slows in
India is the seventh-largest in the world by nominal GDP and an economy. Countries usually measure economic growth in
the third-largest by Purchasing Power Parity (PPP). GDP is the terms of GDP, which is the total value of goods and services
sum of consumer spending, investment made by industry, produced in an economy during a specific period of time.
Hence, option (b) is correct.
excess of exports over imports, and government spending. The
GDP at market price of India has steadily increased in the last 81. With reference to Indian economy, consider the
decades. Hence, option (b) is correct. following : [IAS 2015]
1. Bank rate 2. Open market operations
78. There has been a persistent deficit budget year after year.
Which of the following actions can be taken by the 3. Public debt 4. Public revenue
government to reduce the deficit? [IAS 2015] Which of the above is/are component(s) of Monetary
policy?
1. Reducing revenue expenditure
(a) Only 1 (b) 2, 3 and 4 (c) 1 and 2 (d) 1, 3 and 4
2. Introducing new welfare schemes
3. Rationalising subsidies
Exp (c) An Open Market Operation (OMO) is an activity to
buy or sell government bonds on the open market by a
4. Expanding industries
Central bank (Reserve Bank of India). A Central bank uses them
Select the correct answer using the codes given below.
as the primary means of implementing Monetary policy.
(a) 1 and 3 (b) 2 and 3
Bank rate is the rate of interest which a Central bank charges on
(c) Only 1 (d) 1, 2, 3 and 4
the loans and advances to a Commercial bank. Whenever a
Exp (a) A budget deficit is caused when a government spends bank has a shortage of funds, they can typically borrow from the
more than its collects revenue. Government revenue includes Central bank based on the monetary policy of the country. Both
all amounts of money (i.e., taxes and/or fees) received from OMO and Bank Rate are the components of Monetary policy.
sources outside the government entity. A subsidy is a form of Public revenue and public debt are the part of public finance.
financial aid or support, generally with the aim of promoting Under the theory of public revenue , we study alternative
economic and social policy. sources of state income. Theory of public debt deals with all
So by the reducing revenue expenditure and rationalising the loans and other liabilities of the government.
subsidies, the government can reduce the deficit. Introducing Hence, option (c) is correct.
new welfare scheme and expenditure industries are the major
82. With reference to inflation in India, which of the
government expenditure.
following statements is correct? [IAS 2015]
Hence, option (a) is correct
(a) Controlling the inflation in India is the responsibility of the
79. The Fair and Remunerative Price (FRP) of sugarcane is government of India only
approved by the [IAS 2015] (b) The Reserve Bank of India has no role in controlling the
(a) Cabinet Committee on Economic Affairs inflation
(b) Commission for Agricultural Costs and Prices (c) Decreased money circulation helps in controlling the
(c) Directorate of Marketing and Inspection, Ministry of inflation
Agriculture (d) Increased money circulation helps in controlling the
(d) Agricultural Produce Market Committee inflation
Exp (a) With the Amendment of the sugarcane (Control) Order, Exp (d) In economics, inflation refers to the rise in general
1966 on 22nd October, 2009, the concept of Statutory price level of goods and services in a country over a period
Minimum Price (SMP) of sugarcane was replaced with the Fair of time. The term ‘inflation’ originally referred to increases in
220 Magbook ~ Indian Economy

the amount of money in circulation and some economists still incorrect. It is not a recently formed organisation, but dates
use the word in this way. So, decreased money circulation back to 1997. So, statement 1 is also false.
helps in controlling the inflation. However, most economists
today use the term ‘inflation’ to refer to a rise in the price
86. ‘Basel III Accord’ or simply ‘Basel III’, often seen in the
news, seeks to [IAS 2015]
level. Controlling the inflation in India is major responsibility of
Indian Government and the Reserve Bank of India (RBI). (a) develop national strategies for the conservation and
sustainable use of biological diversity
83. The substitution of steel for wooden ploughs in (b) improve banking sector’s ability to deal with financial and
agricultural production is an example of [IAS 2015] economic stress and improve risk management
(a) labour-augmenting technological progress (c) reduce the Green House Gas emission but places a
(b) capital-augmenting technological progress heavier burden on developed countries
(c) capital-reducing technological progress (d) transfer technology from developed countries to poor
(d) None of the above countries to enable them to replace the use of
chlorofluorocarbons in refrigeration with harmless chemicals
Exp (b) Capital augmenting technological progress results in
more productive use of existing capital goods. The best
Exp (b) According to Basel Committee on Banking
Supervision, “Basel III is a comprehensive set of reform
example for capital augmentation technology is the
measures, developed by the Basel Committee on Banking
substitution of steel for wooden ploughs in agricultural
Supervision, to strengthen the regulation, supervision and risk
production.
management of the banking sector”. Basel III or Basel 3
84. In the ‘Index of Eight Core industries’, which one of the released in December, 2010 is the third in the series of Basel
following is given the highest weight? [IAS 2015] Accords. These accords deal with risk management aspects for
(a) Coal production (b) Electricity generation the banking sector.
(c) Fertilizer production (d) Steel production 87. India is a member of which among the following?
Exp (b) The Eight Core industries comprise nearly 38% of the [IAS 2015]
weight of items included in the Index of Industrial Production 1. Asia-Pacific Economic Co-operation
(IIP). 2. Association of South-East Asian Nations
Core Industry Weight 3. East Asia Summit
Coal Production 4.38% Select the correct answer using the codes given below
Crude Oil 5.22% (a) 1 and 2
Natural Gas 1.71% (b) Only 3
Refinery Products 0.93% (c) 1, 2 and 3
Fertilizer Production 1.25% (d) India is a member of none of them
Steel Production 6.68% Exp (b) At present, India is not a member of Asia-Pacific
Cement 2.41% Economics Co-operation (APEC). It does not belong to the
Association of South-East Asian Nations (ASEAN) group as
85. With reference to ‘Indian Ocean Rim Association for well. The East Asia Summit (EAS) is a forum held annually by
Regional Co-operation (IOR-ARC)’, consider the leaders of, initially, 16 countries (included India) in the East
following statements [IAS 2015] Asian region. Membership expanded to 18 countries including
1. It was established very recently in response to incidents the United States and Russia at the 6th EAS in 2011.
of piracy and accidents of oil spills. Hence, option (b) is correct.
2. It is an alliance meant for maritime security only.
88. Consider the following statements [IAS 2015]
Which of the statements given above is/are correct?
1. The Accelerated Irrigation Benefits programme was
(a) Only 1 (b) Only 2
launched during 1996-97 to provide loan assistance to
(c) Both 1 and 2 (d) Neither 1 nor 2 poor farmers.
Exp (d) Objectives of IOR-ARC is to promote the sustained 2. The Command Area Development programme was
growth and balanced development of the region and of the launched in 1974-75 for the development of water use
Member States, and to create common ground for regional efficiency.
economic co-operation. Which of the statement given above is/are correct?
6 priority areas are as following: (a) Only 1 (b) Only 2
1. Maritime Safety and Security (c) Both 1 and 2 (d) Neither 1 nor 2
2. Trade and Investment Facilitation Exp (b) Accelerated Irrigation Benefits Programme (AIBP) was
3. Fisheries Management launched by government of India during 1996-97 to assist states
4. Disaster Risk Management for early and time bound completion of major and medium
5. Academic, Science and Technology irrigation projects and extension, renovation and modernisation
6. Tourism and Cultural Exchanges of completed major medium projects. Under the programme,
Central assistance to the states was in the form of loan.
Hence, option (d) is correct
The Command Area Development Programme (CADP) started
Providing security is not the only objective of IOR-ARC. It also
in 1974-75 as a Centrally-sponsored scheme, envisaged
works in the field on economic co-operation, including trade,
execution of on- farm development works like field channels,
tourism, education, ICT etc. Therefore, statement 2 is
Magbook ~ Solved Paper 1 221
land levelling, field drains and conjunctive use of ground and (b) gold and silver
surface water; the introduction of Warabandi, or the rotational (c) dollars and other hard currencies
system of water distribution to ensure equitable and timely (d) exportable surplus
supply of water to each holding; and evolving and propagating
Exp (c) The Concept of International Liquidity is associated
crop patterns and water management practices appropriate to
with international payments. These payments arise out of
each command area.
international trade in goods and services and also in connection
Hence, option (b) is correct
with capital movements between one country and another.
89. Which of the following brings out the ‘Consumer Price International liquidity is foreign currency like dollars, sterling or
Index Number for Industrial Workers’? [IAS 2015] gold in the reserve of any country. It is very useful to pay the
(a) The Reserve Bank of India amount of imported goods and reduce balance of payment
(b) The Department of Economic Affairs deficit. Every country should increase exports for reducing
(c) The Labour Bureau international liquidity shortage.
(d) The Department of Personnel and Training
93. Convertibility of rupee implies [IAS 2015]
Exp (c) The Labour Bureau brings out the Consumer Price (a) being able to convert rupee notes into gold
Index Number for industrial workers. The new series on base
(b) allowing the value of rupee to be fixed by market forces
2001=100 has been released on 9th March, 2006 with
(c) freely permitting the conversion of rupee to other
January, 2006 index which has replaced the earlier
currencies and vice- versa
(1982=100) series. Labour Bureau is the competent authority
(d) developing an international market for currencies in India
under the Minimum Wages Act, 1948 to ascertain, from
time-to-time, the Consumer Price Index Numbers applicable to Exp (c) Convertibility of a currency means, currency of a
employees employed in the Scheduled employments in country can be freely converted into foreign exchange at
respect of all the undertakings in the Central Sphere and the market determined rate of exchange that is, exchange rate as
Union Territories. determined by demand for and supply of a currency.
Convertibility of rupee means that those who have foreign
90. Which one of the following issues the ‘Global Economic exchange (e.g. US Dollars, Pound Sterlings etc) can get them
Prospects’ report periodically? [IAS 2015] converted into rupees and vice-versa at the market determined
(a) The Asian Development Bank rate of exchange.
(b) The European Bank for Reconstruction and Development
94. Which one of the following best describes the main
(c) The US Federal Reserve Bank objective of ‘Seed Village concept’? [IAS 2015]
(d) The World Bank (a) Encouraging the farmers to use their own farm seeds and
Exp (d) The World Bank issues the Global Economic discouraging them to buy the seeds from others
Prospects’ report. The World Bank’s twice-yearly (January and (b) Involving the farmers for training in quality seed
June) Global Economic Prospects examines growth trends for production and thereby to make available quality seeds to
the global economy and how they affect developing countries. others at appropriate time and affordable cost
The reports include 3 years forecasts for the global economy (c) Earmarking some villages exclusively for the production of
and long-term global scenarios which look 10 years into the certified seeds
future. Topical annexes in this online publication cover (d) Identifying the entrepreneurs in villages and providing
financial markets, trade, commodities and inflation. them technology and finance to set-up seed companies
Exp (b) A village, wherein trained group of farmers are
91. When the Reserve Bank of India reduces the Statutory
involved in production of seeds of various crops and cater to
Liquidity Ratio by 50 basis points, which of the following
the needs of themselves, fellow farmers of the village and
is likely to happen? [IAS 2015]
farmers of neighbouring villages in appropriate time and at
(a) India’s GDP growth rate increases drastically
affordable cost is called ‘a seed village’.
(b) Foreign Institutional Investors may bring more capital into
our country 95. If the interest rate is decreased in an economy,[IAS
it will
2014]
(c) Schedule Commercial Banks may cut their lending rates (a) decrease the consumption expenditure in the economy
(d) It may drastically reduce the liquidity to the banking system (b) increase the tax collection of the government
Exp (c) Statutory Liquidity Ratio (SLR) is the amount of money (c) increase the investment expenditure in the economy
that is invested in certain specified securities predominantly (d) increase the total savings in the economy
Central and State Government securities. This percentage is of
Exp (c) A decreased interest rate would lead to cheaper
the percentage of the total bank deposits available as far as
loans, which would further help in increasing the investment
the particular bank is concerned. The SLR is maintained in
expenditure by private players, as the loans would be cheaper.
order to control the expansion of Bank Credit. By changing the
level of SLR, Reserve Bank of India can increase or decrease Thus, decreasing the interest rate in an economy would lead
bank credit expansion. When the RBI reduces the SLR by 50 to an upward shift in the investment expenditure of the
basis points, then scheduled Commercial banks may cut their economy.
lending rates. 96. Which of the following organisations brings out the
92. The problem of international liquidity is related to the publication known as ‘World Economic Outlook’?
non-availability of [IAS 2015] (a) The International Monetary Fund [IAS 2014]
(a) goods and services (b) The United Nations Development Programme
222 Magbook ~ Indian Economy

(c) The World Economic Forum 100.The terms ‘Marginal Standing Facility (MSF) Rate’
(d) The World Bank and ‘Net Demand and Time Liabilities (NDTL)’,
Exp (a) World Economic Outlook (WEO) is a survey conducts sometimes appearing in news, are used in relation to
and published by the International Monetary Fund. It is published [IAS 2014]
twice and partly updated 3 times a year. WEO forecasts include (a) banking operations
the macro-economic indicators such as GDP, inflation, current (b) communication networking
account and fiscal balance of more than 180 countries around the (c) military strategies
globe. (d) supply and demand of agricultural products

97. With reference to Union budget, which of the following Exp (a) MSF is the rate at which banks can borrow
overnight from RBI. This was introduced in the Monetary
is/are covered under non-plan expenditure? [IAS 2014]
Policy of RBI for the year 2011-2012. The MSF is pegged
1.Defence expenditure 2. Interest payments
100 bps or a % above the repo rate. Banks can borrow
3.Salaries and pensions 4. Subsidies
funds through MSF when there is a considerable shortfall
Select the correct answer using the codes given below
of liquidity.
(a) Only 1 (b) 2 and 3
This measure has been introduced by RBI to regulate
(c) All of these (d) None of these
short-term asset liability mismatches more effectively been
Exp (c) Plan expenditure includes money given to union’s own introduced by RBI to regulate short-term asset liability
Five Year Plans and money given to states Five Year Plans and mismatches more effectively.
any expenditure other than that of plan expenditure, comes under NDTL, on the other hand, refers to the sum of demand and
non-plan expenditure. time liabilities of banks with public and other banks. The
So, all the 4 options mentioned in the question above comes deposits with the bank are its liabilities, which are payable
under non-plan expenditure. on demand. It includes current deposits, demand liabilities,
Hence, the correct answer is (c). demand drafts etc.
98. What does venture capital mean? [IAS 2014] 101. With reference to the usefulness of the by products
(a) A short-term capital provided to industries of sugar industry, which of the following statement(s)
(b) A long-term start-up capital provided to new entrepreneurs is/are correct? [IAS 2013]
(c) Funds provided to industries at times of incurring losses 1. Bagasse can be used as biomass fuel for the
(d) Funds provided for replacement and renovation of industries generation of energy.
2. Molasses can be used as one of the feed stocks for the
Exp (d) Money provided by investors to start up firms and small
production of synthetic chemical fertilizers.
businesses with perceived long-term growth potential. This is a
3. Molasses can be used tor the production of ethanol.
very important source of funding for startups that do not have
Select the correct answer using the codes given below
access to capital markets. It typically entails high risk for the
(a) Only 1 (b) 2 and 3
investor, but it has the potential for above-average returns. This
(c) 1 and 3 (d) All of these
form of raising capital is popular among new companies or
ventures with limited operating history, which cannot raise funds Exp (c) Bagasse is the fibrous material, which is left over
by issuing debt. after juice is extracted from sugarcane. It is a good source
of energy and usually produces enough electricity to power
99. The main objective of the Twelfth Five Year Plan is all of the mill’s operations. Molasses are organic
(a) inclusive growth and poverty reductions [IAS 2014] substances and can be used as organic fertilizer and not
(b) inclusive and sustainable growth synthetic fertilizer. Sugarcane ethanol is produced by the
(c) sustainable and inclusive growth to reduce unemployment fermentation of sugarcane and molasses. It is emerging as
(d) faster, sustainable and more inclusive growth a leading additive to petrol based fuels in the
Exp (d) Planning Commission Government in plans mentions the transportation sector.
objective of Twelfth Five Year Plan as faster, more inclusive and
sustainable growth.
PREVIOUS YEARS' Questions
SOLVED PAPERS
Set 2
1. When the annual Union Budget is not passed by the Lok 3. India is regarded as a country with ‘demographic
Sabha? [IAS 2012] dividend‘. This is due to [IAS 2012]
(a) The budget is modified and presented again (a) its high population in the age group below 15 years
(b) The budget is referred to the Rajya Sabha for suggestions (b) its high population in the age group of 15-64 years
(c) The Union Finance Minister is asked to resign (c) its high population in the age group above 65 years
(d) The Prime Minister submits the resignation of Council of (d) its high total population
Ministers Exp (b) India has a majority population in the age brake 15-64
Exp (a) This is perhaps the most contentious question given years and hence, benefits from demographic dividend.
this time. Option (d), though favoured by many, is not tenable
as the only single way for the resignation of Prime Minister and 4. Regarding ‘carbon credits’, which one of the following
Union Council of Ministers is a defeat in the Lok Sabha through statements is not correct? [IAS 2012]
the no-confidence motion. Unless that is passed, it is deemed (a) The carbon credit system was ratified in conjunction with
that the Council of Ministers enjoy the confidence of the house. the Kyoto Protocol
Referring to Rajya Sabha may seem correct, but the (b) Carbon credits are awarded to countries of group that have
significance of Rajya Sabha is very low in financial matters. reduced greenhouse gases below their emission quota
The correct answer is option (a). The logic behind this is that if (c) The goal of the carbon credit system is to limit the
the budget has not been passed owing to some policy increase of carbon dioxide emission
disagreements by legislators over taxation or expenditure and (d) Carbon credits are traded at a price fixed from time-to-time
the government intend to modify them and bring in a modified by the United Nations Environment Programme
budget acceptable to all, it is completely valid for the Exp (d) Carbon credits are not traded at fixed prices under
government to withdrew the budget and pass a modified 1. the aegis of the UN Environment Programme.

2. A rapid increase in the rate of inflation is sometimes 5. Consider the following actions which the government
attributed to the ‘base effect’. What is ‘base effect’? can take. [IAS 2012]
[IAS 2012] 1. Devaluing the domestic currency.
(a) It is the impact of drastic deficiency in supply due to failure 2. Reduction in the export subsidy.
of crops 3. Adopting suitable policies which attract greater FDI and
(b) It is the impact of the surge in demand due to rapid more funds from FIIs.
economic growth Which of the above action/actions can help in reducing the
(c) It is the impact of the price levels of previous year on the current account deficit?
calculation of inflation rate (a) 1 and (b) 2 and 3 (c) Only 3 (d) 1 and 3
(d) None of the above
Exp (d) All options are seemingly correct. Devaluation of
Exp (c) Base effect is the impact of the price level of previous currency helps to overcome large current account deficits as
years on the calculation of inflation rate. If inflation is they help boost exports. Reduction in export subsidy may not
calculated from a base year, in which price index is assigned necessarily mean that exports are not performing well and
the number 100. So, if price index in 2010 was 100 and the may also means by withdrawing the subsidies fiscal deficit is
price index in 2011 rose to 110, then inflation rate would be being reined and consequently, improving the economic
10%. If the price index rose to 115 in 2012, then we will say outlook. FDI and FII strictly come under capital account, but
prices have risen by 5% over the previous year, but in reality, their promotion do bridge the current account deficit not
prices have increased by 15% since 2010. So, high inflation in directly, but through make up for balance of payments.
2011 makes inflation rate in 2012 look relatively small and Since, there are differing views over the context of the
doesn't really provide an accurate picture. This distortion is the question, it remains to be seen how the UPSC evaluates the
base effect. question. Hence, the correct option might be (d).
224 Magbook ~ Indian Economy

6. Which one of the following statements appropriately (c) It is a tax on the final consumption of goods or services
describes the ‘fiscal stimulus’? [IAS 2012] and must ultimately be borne by the consumer
(a) It is a massive investment by the government in (d) It is basically a subject of the Central Government and the
manufacturing sector to ensure the supply of goods to State Governments are only a facilitator for its successful
meet the demand surge caused by rapid economic implementation
growth Exp (d) Option (a) may seem correct as VAT is value based
(b) It is an intense affirmative action of the government to and not destination based. But one has to carefully read the
boost economic activity in the country statement. It says VAT is multi-point destination based system
(c) It is government’s intensive action of financial institutions of taxation, which is understood as multiple chains of
to ensure disbursement of loans to agriculture and allied consumers paying their respective tax portions according to
sectors to promote greater food production and contain value addition.
food inflation
VAT is a subject of legislation for the states and the state
(d) It is an extreme affirmative action by the government to
sales tax is actually a VAT, which is major source of revenue
pursue its policy of financial inclusion
for the State Governments.
Exp (b) Fiscal stimulus involves affirmative action like tax
breaks, boosting public investments etc to overcome the 10. A ‘closed economy’ is an economy in which [IAS 2012]
downward effect of business contraction. (a) the money supply is fully controlled
(b) deficit financing takes place
7. India has experienced persistent and high food inflation
(c) only exports take place
in the recent past. What could be the reasons? [IAS 2012]
(d) neither exports nor imports take place
1. Due to a gradual switch over to the cultivation of
commercial crops, the area under the cultivation of food Exp (d) Closed economy involve no export nor import as the
grains has steadily decreased in the last 5 years by about country is deemed to be self-sufficient. Some say, North Korea
30%. is an example of closed economy, although the government
2. As a consequence of increasing incomes, the over their do purchase arms from other countries clandestinely.
consumption patterns of the people have undergone a 11. With reference to India, consider the following Central Acts
significant change. 1. Import and Export (Control) Act, 1947. [IAS 2012]
3. The food supply chain has structural constraints. 2. Mining and Mineral Development (Regulation) Act, 1957.
Which of the statements given above are correct? 3. Customs Act, 1962.
(a) 1 and 2 (b) 2 and 3 4. Indian Forest Act, 1927.
(c) 1 and 3 (d) All of the above
Which of the above acts have relevance to/ bearing on the
Exp (b) Statement 1 is partially correct as there is infact a biodiversity conservation in the country?
decline in the area of cultivation of food crops, but is not as (a) 1 and 3 (b) 2, 3 and 4
drastic as 30%. Indeed, the acreage of pulses and other coarse (c) All of these (d) None of these
cereals have marginally increased whereas that of rice and
Exp (c) All 4 acts are related to biodiversity. Import and export
wheat have decline. In any case, the decline of 30% is incorrect.
of GMOs or exotic species is prohibited whereas certain
8. Both Foreign Direct Investment (FDI) and Foreign medicinal plants are subjected to high custom duties to
Institutional Investor (FII) are related to investment in a regulate their trade. Mines regulation and forest conservation
country. Which one of the following statements best anyway involve provisions related to biodiversity conservation.
represents an important difference between the two?
[IAS 2012] 12. Karl Marx explained the process of class struggle with
(a) FII helps bring better management skills and technology,
the help of which one of the following theories?
while FDI only brings in capital (a) Empirical liberalism [IAS 2012]
(b) FII helps in increasing capital availability in general, while (b) Existentialism
FDI only targets specific sectors (c) Darwin‘s theory of evolution
(c) FDI flows only into the secondary market, while FII targets (d) Dialectical materialism
primary market Exp (d) Dialectical materialism is the major theme of Marxism.
(d) FII is considered to be more stable than FDI
Exp (b) FII is like hot money, it pours in very quickly and
13. Which of the following can aid in furthering the
government’s objective of inclusive growth? [IAS 2012]
leaves off also very quickly whereas FDI is more stable and
long-term. So, in short-term, FII helps in increasing capital 1. Promoting self-help groups.
availability. But attracting FDI is a tough work and since they 2. Promoting micro, small and medium enterprises.
are long-term, therefore FDI targets only specific sectors of 3. Implementing the Right to Education Act.
economy. Select the correct answer using the codes given below
(a) Only 1 (b) 1 and 2 (c) 2 and 3 (d) All of these
9. Which one of the following is not a feature of ‘Value
Added Tax’ (VAT)’? [IAS 2012] Exp (d) All the 3 are essential for inclusive growth as the
(a) It is a multi-point destination based system of taxation inclusive growth concept itself is based on both economic
(b) It is a tax levied on value addition at each stage of and social empowerment of ordinary and underprivileged
transaction in the production-distribution chain individuals.
Magbook ~ Solved Paper 2 225

14. Regarding the International Monetary Fund (IMF), which 17. Consider the following statements in the context of
one of the following statements is correct? Indian economy. [IAS 2012]
(a) It can grant loans to any country [IAS 2012] 1. The growth rate of GDP has steadily increased in the last
(b) It can grant loans to only developed countries 5 years.
(c) It grants loans to only member countries 2. The growth rate in per-capita income has steadily
(d) It can grant loans to the Central Bank of a country increased in the last 5 years.
Exp (c) IMF grants loans only to its member countries if there Which of the statement(s) given above is/are correct?
is any BoP problem in the concerned country. IMF grants (a) Only 1 (b) Only 2
loans only to its member countries if any member country is (c) Both 1 and 2 (d) Neither 1 nor 2
facing its balance of payments account related problem, then Exp (b) Except for the slowdown in the year 2008-09, the Indian
it can take loan from IMF to achieve balance of payments economy has consistently grown and an incrementing rate.
equilibrium. At present, IMF has 188 member countries and
was founded by 29 member countries. 18. What is the difference between ‘vote-on-account’ and
‘interim budget’? [IAS 2012]
15. Recently, the USA decided to support India’s membership 1 The provision of a ‘vote-on-account’ is used by a regular
in multilateral export control regimes called the ‘Australia government, while ‘interim budget’ is a provision used by
Group’ and the ‘Wassenaar Arrangement’. What is the
a caretaker government.
difference between them? [IAS 2012]
2. A ‘vote-on-account’ only deals with the expenditure in
1. The ‘Australia Group’ is an informal arrangement which
government’s budget while an ‘interim budget’ includes
aims to allow exporting countries to minimise the risk
both expenditure and receipts.
of assisting chemical and biological weapons
Which of the statement(s) given above is/are correct?
proliferation, whereas the ‘Wassenaar Arrangement’ is
(a) Only 1 (b) Only 2
a formal group under the OECD holding identical
(c) Both 1 and 2 (d) Neither 1 nor 2
objectives.
2. The ‘Australia Group’ comprises predominantly of Exp (c) The difference between vote-on-account and interim
Asian, African and North American countries whereas budget is that vote-on-account is used by a regular
the member countries of ‘Wassenaar Arrangement’ a government and only deal with expenditure in government’s
pre-dominantly from the European Union and American budget while the interim budget is used by a caretaker
continents. government only and deal with both expenditure and receipts.
Which of the statement(s) given above is/are correct? 19. Why is the Government of India disinvesting its equity in
(a) Only 1 Central Public Sector Enterprises (CPSEs)? [IAS 2012]
(b) Only 2 1. The government intends to use the revenue earned
(c) Both 1 and 2 from the disinvestment, mainly to pay back the external
(d) Neither 1 nor 2 debt.
Exp (d) The difference between Australian group and 2. The government no longer intends to retain the
Wassenaar Arrangement is that, the Australian group is an management control of the CPSEs.
informal group of countries established in 1985 to control the Which of the statement(s) given above is/are correct?
exports which may lead to spread of chemical and biological (a) Only 1 (b) Only 2
weapons. Whereas Wassenaar Arrangement is a multilateral (c) Both 1 and 2
export control regime with 41 states to control for exports of (d) Neither 1 nor 2
conventional arms and dual use goods and technologies not
Exp (d) Government of India is disinvesting its equity because
chemical and biological weapons which are not conventional.
these units have proved to be inefficient and have been
16. Economic growth is usually coupled with [IAS 2012] incurring loses.
(a) deflation 20. The lowering of bank rate by the Reserve Bank of India
(b) inflation leads to [IAS 2012]
(c) stagflation (a) more liquidity in the market
(d) hyper-inflation (b) less liquidity in the market
Exp (b) As an economy grows, income of the people goes (c) no change in the liquidity in the market
up as a result they consume more and rise in demand leads (d) mobilisation of more deposits by Commercial banks
to increase a normal increase in price levels. Economic Exp (a) Bank rate is the rate at which the RBI lends to the
growth is usually coupled with inflation. Commercial banks. So, when the rate is reduced, banks
This happens because as an economy grows, the income borrow more and lend more to retail loan seekers. Thus,
level of its people increase, which leads to increase in infusing more liquidity, meaning higher off take of currency
consumption expenditure, which means increase in demand supply in the system.
for consumer goods which leads to mild inflation.
226 Magbook ~ Indian Economy

21. Why is the offering of ‘teaser loans’ by Commercial 24. Consider the following specific stages of demographic
banks a cause of economic concern? [IAS 2012] transition associated with economic development and
1. The ‘teaser loans’ are considered to be an aspect of arrange the correct order of the following. [IAS 2011]
sub-prime lending and banks may be exposed to the risk 1. Low birth rate with low death rate.
of defaulters in future. 2. High birth rate with high death rate.
2. In India, the ‘teaser loans’ are mostly given to 3 High birth rate with low death rate.
inexperienced entrepreneurs to set-up manufacturing Codes
or export units. (a) 1, 2, 3 (b) 2, 1, 3
Which of the statement(s) given above is/are correct? (c) 2, 3, 1 (d) 3, 2, 1
(a) Only 1
Exp. (c) In the demographic transition, the country
(b) Only 2 characterise firstly by high birth rate due to illiteracy and low
(c) Both 1 and 2 survival rate and high death rate due to lack of health facilities.
(d) Neither 1 nor 2 This stage is followed by high birth rate due to low level of
Exp. (a) ‘Teaser loans’, which has a feature of lending at literacy and low death rate due to better health facilities. In the
lesser premium interest rates in the beginning and later last phase, low birth rate reflects high literacy rates of the
incrementing them heftily after a certain period, are at present population and low death rate show better health facilities.
given to credit worthy home buyers.
25. How does National Biodiversity Authority help in
22. Under which of the following circumstances may protecting the Indian agriculture? [IAS 2011]
‘capital gains’ arise? [IAS 2011] 1. NBA checks the biopiracy and protects, indigenous and
1. When there is an increase in the sales of a product. traditional genetic resources.
2. When there is a natural increase in the value of the 2. NBA directly monitors and supervises the scientific
property owned. research on genetic modification of crop plants.
3. When you purchase a painting and there is a growth in 3. Application for Intellectual Property Rights (IPR) related
its value due to increase in its popularity. to genetic/biological resources cannot be made without
Select the correct answer using the codes given below the approval of NBA.
(a) Only 1 Select the correct answer using the codes given below
(b) 2 and 3 (a) Only 1 (b) 2 and 3
(c) Only 2 (c) 1 and 3 (d) All of the above
(d) All of the above Exp. (c) The NBA set-up under the Biodiversity Act, 2002
Exp. (b) Capital gains arise when the value of a capital asset imposes conditions which secure equitable sharing to be
held by a person increases over a period of time for any benefits arising out of the use of biological resources and
reason. The asset can be anything like shares, real estate and associated knowledge from indigenous and traditional sources
even collectible items like paintings, baseball, cards etc. There this includes powers to grant IPR to domestic and foreign
is also a concept of asset depreciation on which tax deduction persons as well as checking biopiracy. It does not however,
can be claimed. monitor scientific research on genetic modification of crops.

23. The price of any currency in international market is 26. With reference to National Rural Health Mission which of
decided by the [IAS 2011]
the following are the jobs of ‘ASHA’, a trained immunity
health worker? [IAS 2011]
1. World Bank.
1. Accompanying women to the health facility for
2. demand for goods/services provided by the country
anti-natal care check-up.
concerned.
2. Using pregnancy test kits for early detection of
3. stability of the government of the concerned country. pregnancy.
4. economic potential of the country in question. 3. Providing information on nutrition and immunisation.
Which of the statements given above are correct? 4. Conducting the delivery of baby.
(a) 1 and 4 (b) 2 and 3 Select the correct answer using the codes given below
(c) 3 and 4 (d) All of these (a) 1, 2 and 3
Exp. (b) Price of currency in case of floating currency is (b) 2 and 4
decided by the inflow and outflow of foreign exchange from a (c) 1 and 3
country. This depends on the demand for goods/services of (d) All of the above
the country which bring in additional foreign exchange as also
Exp. (a) ASHA stands for ‘Accredited Social Health Activist’.
the stability of the government which can attract or repel
She provides information to the community on determinants of
foreign investors. The World Bank is a development institution
health such as nutrition, basic sanitation and hygienic
and it has no role in pricing of currency. Economic potential of
practices, healthy living and working conditions, information
a country does not tell us anything about inflow and outflow of
on existing health services and the need for timely utilisation of
foreign funds. World Bank is a development institution and it health and family welfare services. She receives incentives for
has no role in currency pricing. promoting immunisation.
Magbook ~ Solved Paper 2 227
She will counsel women on birth preparedness, importance of 29. Which of the following is/are treated as artificial
safe delivery, breast-feeding and complementary feeding, currency? [IAS 2010]
immunisation, contraception and prevention of common (a) ADR
infections including Reproductive Tract Infections/Sexually
(b) GDR
Transmitted Infections (RTIs/STIs) and care of the young child.
(c) SDR
ASHA is not responsible for delivery, but she may accompany
(d) Both ‘a’ and ‘c’
women for antenatal check up and for institutional delivery.
Exp. (c) SDR is the currency of IMF while ADR/GDR are
27. If National Water Mission is properly and completely financial instruments.
demented, how will it impact the country? [IAS 2011]
1. Part of the water needs of urban areas will be met 30. An objective of the National Food Security Mission is to
through recycling of waste-water. increase the production of certain crops through area
expansion and productivity enhancement in a
2. The water requirements of coastal cities with inadequate
sustainable manner in the identified districts of the
alternative sources of water will be met by adopting
country. What are those crops? [IAS 2010]
appropriate technologies that allow for the use of ocean
(a) Rice and wheat
water.
(b) Rice, wheat and pulses
3. All the rivers of Himalayan origin will be linked to the
(c) Rice, wheat, pulses and oil seeds
rivers of peninsular India.
(d) Rice, wheat, pulses, oil seeds and vegetables
4. The expenses incurred by farmers for digging bore-wells
and for installing motors and pump-sets to draw Exp. (b) The target is to increase production of rice, wheat
groundwater will be completely reimbursed by the and pulses by 108 and 2 million tones respectively.
government.
31. In India, the interest rate on savings accounts in all the
Select the correct answer using the codes given below nationalised Commercial banks is fixed by [IAS 2010]
(a) Only 1
(a) Union Ministry of Finance
(b) 1 and 2
(b) Union Finance Commission
(c) 3 and 4
(c) Indian Banks’ Association
(d) All of these
(d) None of the above
Exp. (b) The National Water Mission one of the key missions of Exp. (d) Interest rate on saving bank account is fixed by the RBI.
the National Action Plan on climate change aims at the
‘conservation of water, minimising wastage and ensuring its 32. In the context of bilateral trade negotiations between
more equitable distribution both across and within states through India and European Union, what is the difference
integrated water resources’, development and management. between European Commission and European Council?
The mission has five goals are as follows : [IAS 2010]
1. Comprehensive water database in public domain and 1. European Commission represents the EU in trade
assessment of impact of climate change on water negotiations whereas European Council participates in
resources. the legislation of matters pertaining to economic
2. Promote citizen and state action for water conservation, policies of the European Union.
augmentation and preservation. 2. European Commission comprises the heads of state or
3. Focused attention on over-exploited areas. government of member countries whereas the
4. Increasing water use efficiency by 20%. European Council comprises of the persons nominated
5. Promote basin-level integrated water resources by European Parliament. [IAS 2010]
management. Recycling of waste water will ensure water. Which of the statement(s) given above is/are correct ?
(a) Only 1
28. In the context of Indian economy which of the following (b) Only 2
pair(s) is/are correctly matched? [IAS 2010] (c) Both 1 and 2
Term Most Appropriate Description (d) Neither 1 nor 2
1. Melt down : Fall in stock prices Exp. (d) The European Council is the institution of the
2. Recession : Fall in growth rate European Union (EU) responsible for defining the general
3. Slow down : Fall in GDP political direction and priorities of the union. It comprises of
heads of state or government of European Union member
Codes
states, alongwith its President and the President of the
(a) Only 1 (b) 2 and 3 commission. The European Commission is the executive
(c) 1 and 3 (d) All of these body of the European Union. The body is responsible for
Exp. (a) Recession means negative growth is fall in GDP. Slow proposing legislation. The hot climates, the virus cannot
down means reduction in the growth rate of GDP. Melt down is survive outside for long implementing decisions, upholding
the term used for weakness in the financial markets which the Union’s treaties and the general day-to-day running of the
implies security prices. union.
228 Magbook ~ Indian Economy

33. Consider the following statements with reference to the Exp. (c) With reference to the institution of Banking
National Investment Fund to which the disinvestment Ombudsman in India, if one is not satisfied with the decision
proceeds are routed [IAS 2010] passed by the Banking Ombudsman, one can approach the
1. The assets in the National Investment Fund are appellate authority against the banking decision. Hence, option
managed by the Union Ministry of Finance. (c) is correct answer.
2. The National Investment Fund is to be maintained The Banking Ombudsman is appointed by the RBI and
within the Consolidated Fund of India. appellate authority is vested with a Deputy Governor of the RBI.
3. Certain Asset Management Companies are appointed One can also explore any other resource and remedies
as the fund managers. available to him as per the law. The bank also has the option to
file an appeal before the appellate authority under the
4. A certain proportion of annual income is used for
scheme.The Banking Ombudsman can consider complains
financing select social sectors.
from Non-Resident Indians having accounts in India and the
Which of the statement(s) given above is/are correct ?
service provided by the Banking Ombudsman is free of any
(a) 1 and 2
fee.
(b) Only 2
(c) 3 and 4 37. With reference to India, consider the following [IAS 2010]
(d) Only 3 1. Nationalisation of banks.
Exp. (c) State Bank of India (SBI), Unit Trust of India (UTI), 2. Formation of regional rural banks.
LIC, AMCs to act as fund managers. These are the assets 3. Adoption of villages by bank branches.
management companies appointed as the fund managers by Which of the above can be considered as steps taken to
the Government of India. About 75% of Income from the fund achieve the ‘financial inclusion’ in India?
will go to social sector. (a) 1 and 2 (b) 2 and 3
(c) Only 3 (d) All of these
34. In India, which of the following is regulated by the
Forward Markets commission? [IAS 2010] Exp. (c) All of these can be considered as steps taken to
achieve the ‘financial inclusion’ in India. Nationalisation
(a) Currency futures trading
targeted expansion of bank branches in rural area and also a
(b) Commodities futures trading
control on concentration of economic powers in the hands of
(c) Equity futures trading
few. RRBs were planned to cater to the needs of the rural
(d) Both commodities futures and financial futures trading
areas.
Exp. (b) Commotities future trading is regulated by the
Forward Markets commission. Forward Market commission is 38. Consider the following statements [IAS 2010]
the regulator of commodity future market. For financial The functions of commercial banks in India include
markets, the role is performed by the SEBI. 1. Purchase and sale of shares and securities on behalf of
customers.
35. Which one of the following is not a feature of limited
liability partnership firm? [IAS 2010]
2. Acting as executors and trustees of wills.
Which of the statement(s) given above is/are correct ?
(a) Partners should be less than 20
(a) Only 1 (b) Only 2
(b) Partnership and management need not be separate
(c) Internal governance may be decided by mutual (c) Both 1 and 2 (d) Neither 1 nor 2
agreement among partners Exp. (b) Commercial banks are not directly engage in capital
(d) It is corporate body with perpetual succession market. Capital market is the primary concern of investment
Exp. (a) Maximum limit of partners is 20 in case of banks. They act as executors and trustees of wills of their
partnership firms. In LLP firm, partnership and management clients.
need not be separate and internal government may be 39. In India, the tax proceeds of which one of the following as
decided by mutual agreement among partners. It is also a percentage of gross tax revenue has significantly
corporate body with perpetual succession. declined in the last 5 years? [IAS 2010]
36. With reference to the institution of Banking (a) Service tax
Ombudsman in India, which one of the statements is (b) Personal income tax
incorrect ? [IAS 2010] (c) Excise duty
(a) The Banking Ombudsman is appointed by the Reserve (d) Corporation tax
Bank of India Exp. (c) Excise duty rates have been gradually lowered in the
(b) The Banking Ombudsman can consider complaints from past few years as a measure to help the produce more
Non-Resident Indians having accounts in India competitive in the world market by reduction in the price. At the
(c) The orders passed by the Banking Ombudsman are final same time with expansion of services in the service tax net and
and binding on the parties concerned growing economy and income, other tax revenues have
(d) The service provided by the Banking Ombudsman is free improved.
of any fee
Magbook ~ Solved Paper 2 229

40. Which one of the following authorities makes Exp. (d) Statutory Liquidity Ratio (SLR) is the ratio used by the
recommendation to the Governor of a State as to the RBI as a monetary tool. Banks prefer to keep a part of the
principles for determining the taxes and duties which prescribed ratio in the form of treasury bills, thus, financing the
may be appropriated by the Panchayats in that particular government short-term borrowing.
state? [IAS 2010]
44. In order to comply with TRIPS Agreement, India enacted
(a) District Planning Committees the Geographical Indications of Goods (Registration and
(b) State Finance Commission Protection) Act, 1999. The difference/ differences
(c) Finance Ministry of that State between a trade mark’ and a geographical indications
(d) Panchayati Raj Ministry of that State is/are [IAS 2010]
Exp. (b) State Finance Commission makes recommendation 1. a trade mark is an individual or a company’s right
to the Governor of a state as to the principles for determine the whereas a geographical indication is a community’s
taxes and duties which may be appropriated by the right.
Panchayats in that particular state. According to Article 243-I 2. a trade mark can be licensed whereas a geographical
of Indian Constitution, the Governor of a state shall constitute indication cannot be licensed.
a Finance Commission to review the financial position of the 3. a trade mark is assigned to the manufactured goods
Panchayats and to make recommendations to the Governor whereas the geographical indication is assigned to the
as to: (a) The principles which should govern the distribution agricultural goods/products and handicraft only.
between the state and the Panchayat of the net proceeds of Which of the statement(s) given above is/are correct ?
the taxes, duties, toll and fees leviable by the state. (b) The (a) Only 1 (b) 1 and 2
determination of the taxes, duties, tools and fees which may (c) 2 and 3 (d) All of the above
be assigned to or appropriated by the Panchayat. Exp. (b) Trade mark is a right of a company or manufacturer
41. Consider the following statements while geographical indication is related with a particular area. It
In India, taxes on transactions in stock exchanges and applies to the products which are area specific like particular
futures markets are [IAS 2010]
variety of mango produced in specific area in a country.
Geographical Indications (GI) are like trade marks, expect that
1. levied by the union.
they are indicators of a geographical area. They can belong to
2. collected by the states.
any goods or products whether natural, agricultural or
Which of the statement(s) given above is/are correct? manufacturing. Example of GIs are Basmati rice, champagne,
(a) Only 1 (b) Only 2 Swiss watches etc. GIs are area specific as an identity, so it
(c) Both 1 and 2 (d) Neither 1 nor 2 cannot be licensed.
Exp. (a) The tax was introduced as STT and CTT which are
central taxes. Taxes on transactions in stock exchanges and
45. The SEZ Act, 2005 which came into effect in February
2006 has certain objectives. In this context, consider the
futures markets are levied by the Union Government and also
following [IAS 2010]
collected by the Union Government. Hence, only Statement (1)
is true. 1. Development of infrastructure facilities.
2. Promotion of investment from foreign sources.
42. Consider the following statements [IAS 2010] 3. Promotion of exports of services only.
1. The Union Government fixes the Statutory Minimum Which of the statement(s) given above is/are the objectives
Price of sugarcane for each sugar season. of this act?
2. Sugar and sugarcane are essential commodities under (a) 1 and 2 (b) Only 3
the Essential Commodities Act. (c) 2 and 3 (d) All of the above
Which of the statement(s) given above is/are correct ? Exp. (a) The objective of the SEZ Act and SEZ policy is to
(a) Only 1 (b) Only 2 create infrastructure for manufacturing as well as services and
(c) Both 1 and 2 (d) Neither 1 nor 2 to promote export. It also promotes FDI in these areas.
Exp. (a) The Union Government fixes the Statutory Minimum
46. Which one of the following statements is an appropriate
Price of sugarcane for each sugar season. Sugar is an
description of deflation? [IAS 2010]
essential commodity under the Essential Commodities Act,
but not sugarcane. Hence, Statement (1) is true. (a) It is a sudden fall in the value of a currency against other
currencies
43. Which of the following terms indicates a mechanism (b) It is persistent recession in both the financial and real
used by Commercial banks for providing credit to the sectors of economy
government? [IAS 2010] (c) It is a persistent fall in the general price level of goods and
(a) Cash Credit Ratio services
(b) Debt Service Obligation (d) It is a fall in the rate of inflation over a period of time
(c) Liquidity Adjustment Facility Exp. (c) Inflation denotes rise in general price level in an
(d) Statutory Liquidity Ratio economy. Deflation is just opposite of inflation which denotes
fall in general price level.
230 Magbook ~ Indian Economy

47. With reference to the Non-Banking Financial Companies Exp. (c) In the FRBM Act, the target has been set for
(NBFCs) in India, consider the following statements reduction in revenue deficit and fiscal deficit only and not for
[IAS 2010] primary deficit.
1. They cannot engage in the acquisition of securities
49. In the parlance of financial investments, the term ‘bear’
issued by the government. denotes [IAS 2010]
2. They cannot accept demand deposits like savings
(a) an investor who feels that the price of a particular security
account.
is going to fall
Which of the statement(s) given above is/are correct ? (b) an investor who expects the price of particular shares to
(a) Only 1 (b) Only 2 rise
(c) Both 1 and 2 (d) Neither 1 nor 2 (c) a shareholder on a bondholder who has an interest in a
Exp. (b) Non-Banking Finance Company (NBFC) is a company, financial or otherwise
company registered under the Companies Act, 1956 and is (d) any lender whether by making a loan or buying a bond
engaged in the business of loans and advances, acquisition at Exp. (a) Bears are the investors who expect the prices of
shares/stock/ bonds/debentures/securities issued by securities to fall and they sell the securities and buy again
government or local authority or other securities of marketable when the prices actually fall.
nature, leasing, hire purchase, insurance, business, chit
business. NBFCs are finance companies, but not banks and 50. A great deal of Foreign Direct Investment (FDI) to India
are thus, not allowed to function as bank, but to take deposits comes from Mauritius than from many major and
in other forms like fixed deposit. They are not allowed to open mature economies like UK and France. Why? [IAS 2010]
savings bank accounts and are not members of the clearing (a) India has preference for certain countries as regards
houses. receiving FDI
(b) India has double taxation avoidance agreement with
48. Which one of the following was not stipulated in the Mauritius
Fiscal Responsibility and Budget Management Act,
(c) Most citizens of Mauritius have ethnic identity with India
2003? [IAS 2010]
and so, they feel secure to invest in India
(a) Elimination of revenue deficit by the end of the fiscal year (d) Impending dangers of global climatic change prompt
2007-08 Mauritius to make huge investments in India
(b) Non-borrowing by the Central Government from Reserve
Bank of India except under certain circumstances
Exp. (b) India has a double tax avoidance treaty with
Mauritius. Investors from Mauritius are exempted from tax in
(c) Elimination of primary deficit by the end of the fiscal year
India and the tax rates are very low in Mauritius. So, it is
2008-09
favourable for investors to get registered in Mauritius and than
(d) Fixing government guarantees in any financial year as a
invest in India as an entity from Mauritius. This investment is
percentage of GDP
not from Mauritius, but through Mauritius.

You might also like